Capsules DR Mohammed Eid

Download as pdf or txt
Download as pdf or txt
You are on page 1of 246

P ag e |1

Capsules
OF
General surgery

2nd Edition (2021)

PLEAZE, ANY NOTE OR CORRECTION OR ADDITIN SEND: [email protected]


‫نسألكم الدعاء بظهر الغيب‬
‫‪P ag e |2‬‬

‫عل ْمتْنْا‬ ‫ال ما‬ ‫ْ حا ك ال ع ْل‬ ‫قْالْوا‬


‫ْم‬ ‫ب نْ‬
‫لْ‬ ‫س‬
‫نْا‬
‫ح ْكي ْم‬ ‫ا م‬
‫ْ‬ ‫نك أ ْن‬
‫ْ ا ْل‬
‫لت ْلي‬

‫‪ ) (32‬اآلية‪:‬سورة البقرة‬

‫‪PLEAZE, ANY NOTE OR CORRECTION OR ADDITIN SEND:‬‬ ‫‪[email protected]‬‬


‫نسألكم الدعاء بظهر الغيب‬
P ag e |3

PLEAZE, ANY NOTE OR CORRECTION OR ADDITIN SEND: [email protected]


‫نسألكم الدعاء بظهر الغيب‬
P ag e |4

INDEX
M.C.Q .......................................................... 5
CAPSULES ................................................. 142
ARABIC ETHICS ........................................ 179

‫تم وضع خط تحت االجابة الصحيحة وكتابته بخط كبير‬

PLEAZE, ANY NOTE OR CORRECTION OR ADDITIN SEND: [email protected]


‫نسألكم الدعاء بظهر الغيب‬
P ag e |5

M.C.Q
1. 90 yr. old female, HTN and DM, admitted to the hospital as a case of HF and was
treated withmedication, there was incidental finding of uterine procidentia, when taking hx
from pt., she said it has been for 10 year and never bothered her, she used to push her
uterus back when urinate and pass stool. What is your management?
A- Vaginal pessary B- Vaginal hysterectomy
C- Radical hysterectomy D-Lap hysterectomy with Sacro-spinous suspension
2. You did fibroadenoma excision for one of your pts and send it to histopathology then
histopathology reported: lobular carcinoma in situ with free margins. What is your
management?
A- Mastectomy B- Chemo best observation
C-Radiotherapy D- Follow up
The patient has critical limb ischemia, what is the golden standard investigation to be done?
A- B- Conventional angiography
3. TSH high, T4 normal want to get pregnant??
A- Give thyroxin now B- Give thyroxine during pregnancy
C- F/U D- Proceed to pregnancy without any management
4. 24 weeks preterm delivered, mother she is a doctor and she asked not to Resuscitate
herbaby. What you will do?
A- Ignore the mother and Resuscitate B- Tell social services C- respect her wishes
5. Woman brought with the police with bruises. How to deal with her condition
Call the social service of the hospital
6. UTI> 14day, most probably cause pyelonephritis?
A- 0.05% B- 0.5% C- 5% D- 50%
7. Patient complaining of pain along median nerve distribution, and positive tannel
signtreatment include casting of both hand in what position?
A- Dorsiflexion B. plantar flexion C. extension D. Adduction
8. Patient is presented with hand cellulitis and red streaks in the hand and tender
axillarylymphadenopathy. This condition is more likely to be associated with?
A- Malignancy B- Pyoderma C- Neuropathy D- Lymphangitis
9. 2-month infant with vomiting after each meal, he is in 50 centiles, He passed
meconiumearly and stool, diagnosis is
A- Midgut volvulus B- Meconium ileus (not passing early meconium and stool)
C- Hirschsprung disease (not passing early meconium and stool)

PLEAZE, ANY NOTE OR CORRECTION OR ADDITIN SEND: [email protected]


‫نسألكم الدعاء بظهر الغيب‬
P ag e |6
10. Subarachnoid hemorrhage but on CT everything normal. What would you do next?
A- Lumbar puncture B- MRI
11. RLQ pain with abdominal Guarding, Pt very toxic 22000 WBCs febrile. what is ttt?
A- Exploration laparotomy B- CT abd C- MRI abd D- Conservative
Patient post Hartman procedure. urine output is 20ml/h. and O2 venous pulse pressure 10.
What to do?
A- CT abdomen B- US abdomen C- Venous duplex D- Exploration
12. Normal delivery and she did episiotomy the developed retroperitoneal collection,
wasblueish painfully, what is the treatment?
A. packing B. aspiration C. surgical evacuation
13. 70 years old patient present with sudden abdominal tenderness did U/S normal
and normalexamination. HB low. What is the next?
A- Occult stool B-Endoscopy and colonoscopy
14. Trauma with hypotension, x-ray trachea shifted to the right, expanded lungs and
widenedmediastinum. What is the dx?
A- Massive hemothorax B- Pneumothorax
C- Thoracic aorta rupture D- Spontaneous pneumothorax
15. Elderly with massive lower GI bleeding, most common dx?
A- Angiodysplasia B- Diverticulosis C- Colon ca
16. Child with GI symptoms and positive secretin test, Dx?
A- VIPoma B- Gastrinoma C- Glucagonoma
17. RTA lumbar spine pain, initial most imp step before neurosurgery arrive to assess?
A- Pelvic binder B- CT C- Restrict spine mobilization
18. 30 years old female breastfeeding her baby present with painful lump on RT breast
O/E: redness hotness, tenderness, no lymph node enlargement, fever 37.1, she starts on
antibiotics since period of time, US report show: thick cyst with fluid collection, dx
abscess, cyst for clinical correlation, what is the next appropriate management?
A- observe B- us guide aspiration C- Incision and drainage
19. Pt with repeated vomiting with a neck emphysema and opacity or fluid in pleural space…?
A- Spontaneous esophageal rupture B- Lung bullae rupture
20. Enlarged pathological LNs. next step in management?
A- Core needle bx B- Excisional bx
C- Modified radical mastectomy D- Ct scan of bone and chest
21. Patient has no signs or symptoms of cirrhosis all labs normal presented
hematemesis,Endoscopy done showed gastric fundus varices, imaging enlarged spleen
(no thrombus).

PLEAZE, ANY NOTE OR CORRECTION OR ADDITIN SEND: [email protected]


‫نسألكم الدعاء بظهر الغيب‬
P ag e |7
A- gastritis B- mallory weiss syndrome C- left portal
hypertension(splenic vein thrombosis)

PLEAZE, ANY NOTE OR CORRECTION OR ADDITIN SEND: [email protected]


‫نسألكم الدعاء بظهر الغيب‬
P ag e |8
22. A 56-year-old man presented to the Emergency Department with intermittent lower
abdominal cramps typically coming three or four hours after a meal for the last six weeks.
His bowel habit had become more constipated than usual and he was forced to strain
hard to achieve evacuation. On examination there were no significant clinical findings.
Which of the following tests would most useful to establish a management plan?
A. FOBT (fecal occult blood test) B. Colonoscopy
C. Flexible sigmoidoscopy D. Double contrast Ba enema
23. A 58 years old woman with severe constipation, presents with colicky central
abdominal pain, loss of appetite and malaise. On admission and investigation her total
colon transit time was found to be 140 hours (see lab results and reports).
Double Contrast Ba Enema: No abnormal findings
Colonoscopy: No abnormal finding

Test Result Normal Values


Free thyroxin T4 51 50-140 nmol/L
Free Triiodothyronine T3 2.6 2.0-4.0 nmol/L
TSH 6.5 0.4-6.5 mU/L

What action should be given?


A- Anorectal manometry B- Increase fiber and fluids
C- Repeat thyroid function tests D- Investigate secret self-medications
24. 65-year-old patient is intubated in the Intensive Care Unit because of septic shock
Which ofthe following will best indicate adequate systemic perfusion?
A- Cardiac index B- CVP
C- Mixed venous oxygen saturation D- Pulmonary capillary wedge pressure
25. Pediatric patient came to ER with projectile vomiting, what is the initial management to
givethis case?
A- NS B- Ringer lactate C- D5NS D- 1/5 NS
26. A patient with pelvic fracture who was resuscitated and is currently unstable, BP
86/50 HR 120 What is the next step?
A- Surgical intervention B- Pelvic binder
27. A24 year old lady presents with a hard, mobile, well-circumscribed painless left
breast mass that has been increasing in size from the past few months, not related her
menstrual cycle. The most likely Dx is?
A- Fat cyst B- Fibroadenoma C- Fibrocystic changes D- Intraductal papilloma

PLEAZE, ANY NOTE OR CORRECTION OR ADDITIN SEND: [email protected]


‫نسألكم الدعاء بظهر الغيب‬
P ag e |9
28. Diabetic patient with bad smelling ulcer, mx?
A- Debridement B- Advanced wound care C- Vacuum assist
29. A male patient, a known case of GERD on PPI, symptoms only mild improvement.
Endoscopy was done and showed esophagitis. What is the most important next step?
A- Repeat Endoscopy in 6 months. B- Nissen fundoplication.
C- Esophageal manometry. D- Ambulatory PH monitoring.
30. A 50 yr. old, with IHD and DM, Admitted to ICU with severe pneumonia and was treated
with abx. After 3 days of admission, he developed hypotension and treated with hydration
and inotrope. On admission lab was normal, after 3 days LFT was abnormal, Total bil 20 &
very high AST and ALT (1000), mild increase in LDH, us: unremarkable, what is dx?
A- Ischemic hepatitis B- Intravascular hemolysis
C- ICU related jaundice D- Acalculous cholecystitis
31. Patient with weight loss and dysphasia diagnosed by EGD and biopsy as a case of
adenocarcinoma. how to know the T stage of cancer?
A- abdominal US B- CT chest abdomen pelvis C- endoscopic US
32. Para 1 or 3, has not been able to get pregnant for 3 years, she has symptom (not sure-
what it was exactly but i remember fatigue, weight gain, constipation) Labs showed: High
TSH (T3, T4 not given) Normal FSH, LH, Prolactin very high, what is the diagnosis?
A- Hyperprolactinemia B- Hypothyroidism
33. Case about a child with increased water consumption and going to the toilet, not to
mention Urine and serum osmolality, not mention any electrolyte, with no response to
water deprivationtest, no other complains?
A- central DI B- peripheral DI C- SIADH
34. A patient who underwent a major pelvic operation. How to prevent DVT?
A- Mechanical prophylaxis B- Enoxaparin C- Heparin D- Warfarin
35. A 45 years old male or female, has a mass 5 cm in right upper limb, (MRI shows a mass
fromtriceps) what to do next?
A- Incisional biopsy B- Excisional biopsy C- Core needle biopsy D- PET Scan
36. Pediatric patient suddenly developed SOB. X ray showed coiled orogastric tube in the
esophagus. The baby will have a great risk developing which of the following?
A- GERD B- Chylothorax C- Phrenic nerve injury D- Aspiration
37. An IBD patient does resection of the terminal ileum. What is the most
important nutrient tobe replaced? A- Bile salts B- B12
38. Male victim of gunshot to the thigh, pale and unconscious, BP 90/60 pulse 130, next?
A- Orotracheal intubation B- Blood transfusion C- Shift to OR D- RL infusion

PLEAZE, ANY NOTE OR CORRECTION OR ADDITIN SEND: [email protected]


‫نسألكم الدعاء بظهر الغيب‬
P a g e | 10
39. Postoperatively, a patient develops distension and abdominal pain, US shows mild
ascitesWhat is the management?
A- Laparotomy B- CT scan
40. RTA with hypovolemic shock signs, Hb low, what to give initially?
A- Ringer lactate B- Packed RBC C- Whole blood transfusion
41. Male patient present in the ER after RTA with tibial fracture and he was stable What is
our priority in regard to his management?
A. control the pain B. control the bleeding C. limit the soft tissue injury
42. Old man came complain of tender abdominal bloody stool and history of take
amoxicillin from 3 wks. for UTI, sigmoidoscopy inflamed mucosa with plaque like lesions?
A- Ischemic colitis B- Pseudomembranous colitis
43. A 50 yo Female came back from Egypt, known case of penicillin allergy, presented to
the ER because of painful red lesions on the anterior shin of the right leg, she received
antibiotics, next day developed abdominal pain, fever and diarrhea. What's the cause of her
symptoms?
A- AB allergy. B- Clostridium difficile (pseudomembranous colitis) C- Salmonellosis
Signs and symptoms usually develop within five to 10 days after starting a course of antibiotics, but may
Occur as soon as the first day or up to two months later
What is most commonly Antibiotic can causes C difficile? implicated agents include the cephalosporins
(especially second and third generation), the fluoroquinolone, ampicillin/amoxicillin, and clindamycin.
44. 32 yo male with neck mass 2*3. Labs showed high calcitonin. most likely diagnosis?
A. Papillary B. Anaplastic C. Medullary D. Lymphoma
45. Non-Hodgkin lymphoma on Tx developed tumor lysis syndrome?
A- Hypokalemia hypocalcemia B- Hyperkalemia hypocalcemia

C- Hypokalemia hypercalcemia D- Hyperkalemia hypercalcemia


46. A chest stab wound, fast was done and showed fluid in peritoneum, the best next step
(Theydid not mention the vitals)
A- Thoracostomy B- Pneumonectomy C- Exploration
47. Female with multiple liver masses, k/c of hepatitis c, what is your further management?
A- Liver biopsy B- Us C- CT
48. Lymph Node swelling and was clear lymphatic fluid in this swelling. What to do?
A- Surgery B- Sclerotherapy C- Radiotherapy D- chemotherapy
49. Pt has chemical burn all back area what to do in ER?
A- sweep chemical powder B- wash by clean water for 30 min (if liquid)
50. A 42 yr. male, with rectal bleeding, a biopsy from sigmoid showed "adenoma", what
intervalshall you do colonoscopy screening?

PLEAZE, ANY NOTE OR CORRECTION OR ADDITIN SEND: [email protected]


‫نسألكم الدعاء بظهر الغيب‬
P a g e | 11
A- 3- & 6-months B- 3 years B- 10 years C- No need

PLEAZE, ANY NOTE OR CORRECTION OR ADDITIN SEND: [email protected]


‫نسألكم الدعاء بظهر الغيب‬
P a g e | 12
51. The government has decided to set up campaigns to help increase the awareness of
hypertension to the public, as well as educate them about the risk factors of hypertension
andencourage low salt diets. What kind of prevention is this?
A- Primordial B- Primary C- Secondary D- Tertiary
52. RTA patient presented with multiple chest bruises (I think they multiple rib fractures)
unable to breath or cough and pain started to develop hypotension, tachycardia. what is
the management
A- Analgesia B- Surgical thoracostomy C- Chest Tube
53. Best way to clear the cervical spine?
A- CT B- MRI C- Lateral view XRAY OF SPINE
54. Which of the following strategy to avoid air bag injury under 12 yrs. child?
A- Restrain in back seat B- Restrain in front seat
C- Rear facing in front seat D- Booster seats in front seat
55. Pt falls on his elbow. what will you see in lateral x ray?
A- radial line anterior to acetabulum B- some line bisects with acetabulum
C- anterior fat D- posterior fat
56. A woman presents with a long-standing 4 cm irregular breast mass, which is growing
in size. She also has swollen axillary lymph nodes and thickened skin overlying the mass.
Mammogram shows an ill-defined mass with calcifications. What is the next step?
A- Core biopsy B- Excisional biopsy
57. If there is open fracture, which first thing to do

A- Abx B- Pain control C- Stop bleeding D- Debridement


58. 55 yrs. female with breast ulceration scaling Oozing, no mass in examination,
Mammogramshow no mass nothing What is the appropriate next step?
A- Derma consult B- F/u within 6-month C- Nipple biopsy
59. Woman with heartburn and regurgitation. On endoscopy she was found to have distal
esophagitis. What would be the management? (she's already on omeprazole 20mg)
A- Add anti acid B- Increase the dose
60. If we have MVC or RTA, which first thing to make control for it as first
A- Pain B- Bleeding C- Airways
61. Flail chest, which control first?
A- Airways B- Pain
62. A young female presents with a vesicular rash around the anus. What is the diagnosis?
A- Condylomata lata B- Condylomata acuminate C- Anogenital herpes D- Candida

PLEAZE, ANY NOTE OR CORRECTION OR ADDITIN SEND: [email protected]


‫نسألكم الدعاء بظهر الغيب‬
P a g e | 13
63. Patient with signs of acute limb ischemia. Peripheral pulses absent in right leg
(dorsalis,popliteal) and dorsalis pulse absent in left leg.
A- heparin and observe B- femoral thrombectomy C- catheter thrombolysis
64. 32 years old Pt came after 3 days with perianal pain, swelling and tenderness no fever
Whatis the diagnosis?
A- Anal fissure B- Prolapse pile C- hematoma D- abscess
65. A patient starts to have dysphagia to liquids What is the investigation of choice?
A- Upper Gl endoscopy B- Barium swallow C- CT scan D- X-ray
66. A year back Hx of trauma and patient is paraplegic came now with SOB and
coughdiagnosed as PE with DVT and patient is HemoDynamic unstable
A- Enoxaparin B- Tissue plasminogen activator C- IVC
67. Patient developed DVT, she had C-section 10 days ago. What to give?
A- Thrombolytics B- Heparin C- IVC D- Embolectomy
68. History of hemorrhoidectomy surgery 3 weeks ago, presents with tachycardia only.
What'sthe highest diagnostic value?
A) D dimer B) CTA
69. Old patient came with subdural hematoma with signs of lateralization imaging
revealed 13mm shifting. his GCS 7/15 then was intubated and resuscitated. what to do
next?
A- iv mannitol B- admit ICU and observe C- craniotomy
70. Most important factor for staging "T" component of gastric adenocarcinoma?
A- endoscopic US B- transabdominal US C- Gastrographin
71. Cause of biliary colic?
A. Choledocholithiasis B. Gallbladder stone C. Gallbladder sludge
72. Female healthy with regular cycle, never had sex. History of bilateral breast pain 3
days before her cycle, no family history of cancers. She said she wants to get pregnant
within 2 years. What will you screen her for?
A- US breast B- PAP Smear C- human papilloma virus D- Gram stain for streptococcus
73. Female came with multiple injury, on examination detect she hitted by her husband
what isthe action
A- inform ethical committee B- inform the authorities C- Call violence
74. Case with S&S of dehydration asking about a level in serum
A- Low B- Normal C- High D- low normal
75. How to detect renal scarring in a child
A- DMSA B- MCUG
76. hypothyroidism symptoms TSH high and T4 normal what is the diagnosis (hashimoto

PLEAZE, ANY NOTE OR CORRECTION OR ADDITIN SEND: [email protected]


‫نسألكم الدعاء بظهر الغيب‬
P a g e | 14
thyroiditis)

PLEAZE, ANY NOTE OR CORRECTION OR ADDITIN SEND: [email protected]


‫نسألكم الدعاء بظهر الغيب‬
P a g e | 15
77. Old pt. after rectal surgery he is not doing well after in the recovery he starts to have
leg pain he developed DVT from the popliteal to the femoral?
A. Enoxaparin B. Heparin C. Warfarin D. IVC
78. Pt confirmed to have plural effusion Plural tap is obtained What suggest exudative?
A- pleural/serum protein <5 B- pleural/serum LDH <6
C- pleural fluid LDH <1/3 D- pleural fluid LDH>2/3
79. A 45-year-old male smoker had an adenoma removed from his colon. Pathology
report shows a benign lesion. What advice should you give this patient to prevent him
from getting colon cancer?
A- Ear a low-fiber diet B- Eat a high-protein diet
C- Colonoscopy every year D- Stop smoking and start exercising
80. Patient with diagnosis of thyroglossal duct cyst. How to treat
A- surgery B- follow up
81. 55-year-old female that had an episode of visual loss for 20 minutes and it came back.
Whatis your diagnosis (literally that was the only given)?
A- transient ischemic attack B- multiple sclerosis
82. A 12-year-old girl presents with severe right lower-quadrant abdominal pain and
marked tenderness and is found to have acute appendicitis. The child is at a sleep away
camp. You are not able to locate her parents. They are not at home and you cannot reach
them on the cell phone. The camp counselor and the director of the camp bring in the
child. What should you do?
A- Do not do the surgery without parents consent.
B- Ask the camp counselor or director for consent.
C- Ask the patient for consent
D- Perform the appendectomy.
83. ICU pt. elderly develop sacral ulcer with discharge and necrosis area & fat exposed
A- Primary-closer B- debridement with primary-closer
C- debridement with skin graft D- debridement with VAC
84. patient had a sigmoid polyp removed. they found on histopathology well
differentiated adenocarcinoma and the margins are free from cancer. what is the best next
step?
A- OBSERVATION B- SEGMOIDECTOMY
C- SEGMENTAL COLECTOMY D- FULGURATION OF THE POLYP SITE
History of gastric bypass with hx of leak, severe pain and mild hypotension, next step?
A. endoscopy B. us C. laparotomy D. laparoscopic drainage

PLEAZE, ANY NOTE OR CORRECTION OR ADDITIN SEND: [email protected]


‫نسألكم الدعاء بظهر الغيب‬
P a g e | 16
85. A patient underwent a sleeve gastrectomy last week, and he presents to the ER today
complaining of recurrent vomiting. ABGs show no abnormalities, and his vitals are stable.
Whatis the best way to manage this case? Reassure the patient .......
86. Pregnant with fetal distress and refused to do CS?
A- allow vaginal delivery B- her husband decision C- ethical committee D- CS
87. 36 male c/o difficulty on breathing and swallowing since 4 months x-ray: mid line
masscompressing trachea
A- thymoma B- lymphoma C- lung cancer D- thyroid goiter
88. Elderly male patient underwent colectomy for colon cancer in which micro metastasis
wasdetected in the lymph nodes, what is the best explanation
A- Good prognosis. B- Liver metastasis.
C- It is sensitive to chemotherapy. (Dukes class C cancer best for chemotherapy)
D- It is locally advanced
89. 29 Young aged males presented to ER after blunt trauma to Abdomen, CT scan
showsduodenal intramural hematoma your management is
A- Laparotomy with evacuation of the hematoma B- Dissection of duodenum C- Observation
90. Which of the following medication can be used as prophylaxis in appendectomy?
A- Cephalexin B- Ceftriaxone C- Metronidazole D- Vancomycin E- Ampicillin
91. man with oblong swelling on top of scrotum increase in size with Valsalva maneuver Dx
A- DIH B- indirect inguinal hernia C- varicocele D- femoral hernia
92. RTA victim, multiple trauma free fluid in abdomen and spleen lacerations + thoracic
aorticrupture
A- thoracotomy B- laparotomy C- CT scan
93. 30 y lady with UC came with has perianal lesions. When to do colonoscopy?
A- Immediately B- After 10 years
94. Stroke with hemorrhagic transformation. Developed papilledema. What would you
do?give mannitol

95. Pt 65 y/o immersive rectal bleeding what is the most likely diagnosis
A- Angiodysplasia B- Diverticulosis C- Colon cancer D- Rectal cancer
96. what would make you go for thoracocentesis?
A- loculated pleural fluid B- Ph of pleural fluid C- LDH of pleural fluid
97. Young pt. with altered mental status CT brain showed parieto-temporal hyper Dense lesion
A- Herpes encephalitis B- Meningoencephalitis C- Abscess
Pt after appendectomy, found to have 0.5cm carcinoid tumor in the meso appendix what is your
action:
A- Rt. Hemicolectomy B- no further ttt C- chemotherapy D- Radiotherapy

PLEAZE, ANY NOTE OR CORRECTION OR ADDITIN SEND: [email protected]


‫نسألكم الدعاء بظهر الغيب‬
P a g e | 17
98. Doctor want to examine female pt. and he call for nurse to attend with hem, which is
mostrelated to ethical more?
A- Non maleficence B- Privacy
99. child with 2cm spleen laceration and peri spleen fluid, how to manage?
A- NOM B- laparotomy with splenectomy C- laparotomy with splenoraphy
26 years old present with symptoms and signs of intestinal obstruction Imaging shows
intussusception Management
A- Surgical resection B- barium enema
100. How to differential btw crhon's and US
A- Aphthous nodule B- Extraintestinal manifestation C- Non-caseating granuloma
101. Patient post op complaining for 1-week intermittent fever on examination abdomen
andchest normal and the wound is clean What's the most appropriate step?
A- reassure B- CT abdominal C- x ray D- exploration
102. Male patient with renal colic. what is the best diagnostic test for renal colic?
A- KUB B- Renal US C- Abdomen CT D- Abdomen MRI
103. RUQ pain, for 12 hours, no fever, no jaundice. U.S findings "non thickened G.B wall
withmultiple gall stones, CBD IS Obscured What's your diagnosis
A- Pancreatitis B- Obstructive jaundice C- A-Cholecystitis D-mucocele
104. 46 yrs. old female smoker for 18 yrs. concern about cancer breast, appropriate screening for
her A- Mammogram
105. RTA victim, multiple trauma; free fluid in abdomen and splenic rupture
A- thoracotomy B. laparotomy C. CT scan
106. Infant with ventral hernia bulging out when he cries, and reduced when he is
sleeping.what is most the appropriate management?
A- observation B- herniotomy C- mesh repair D- laparoscopic
107. Pt male 50 y, alcoholic, abd pain, palpable gallbladder, weight loss, scenario
suggestive ofmalignancy. Liver enzymes acceptable except for significantly high ALP.
Diagnosis
A- Periampullary Tumor B- Hepatocellular CA
108. 45 A new screening test for Diabetes has a sensitivity of 90% and specificity of 80%
whichis the best interpretation?
A- 80% of non-diabetic has positive results B-80% of diabetes had positive results
C- The test was positive for 90% of diabetic patients D-For each 100 positive test results 90%
were diabetic
109. Warfarin did not stop by patient regardless of clear instructions the nurse in OR
noticedthat the patients didn't stop the drug as instructed and informs the surgeon, So
the surgeon
PLEAZE, ANY NOTE OR CORRECTION OR ADDITIN SEND: [email protected]
‫نسألكم الدعاء بظهر الغيب‬
P a g e | 18
postponed the operation
A- Near miss B- Adverse event C- Medical negligence
110. Stab injury in zone 1, examination shows diffuse emphysema & patient is stable what's
thenext appropriate step?
A- CT neck & chest B- Observation C- Surgical exploration
111. Small bowel obstruction & abdominal distention, tympanic on percussion. CT
multiplesmall bowel dilatation with transition point at distal small bowel. Mesenteric fat
something
A- observation B- colonoscopy C- dx laparoscopy D- exploratory laparotomy
112. Old lady follows up incidentally found grade 3 pelvic organ prolapse and she has
nocomplaint what to do?
A- Surgical repair B- Ring (not sure) C- No need for intervention
113. Patient with weight loss and dysphasia diagnosed by EGD and biopsy as
adenocarcinoma. how to know the T stage of cancer?
A- Abdominal US B- Thoracic CT-scan
C- CT chest abdomen pelvis D- Endoscopic US
114. Case of chronic limp ischemia what the gold standard in diagnosis?
A- Conventional Angio B- CT agio confirmatory C- Doppler initial
115. Pt with irreducible painful tender mass vitals is stable, otherwise examination is
normalexcept tender peri umbilical lump What is your management?
A- observation B- urgent surgery C- us abdomen
116. Patient post left thyroid lobectomy? complaining of severe shortness of breath
aftersurgery and pointing to her neck What to do
A- Take her back to surgical theater B- Bedside evacuation of hematoma C- Nasal
cannulation
117. Post RTA with decrease air entry, cervical emphysema and tympanic chest. Dx
A- Tracheobronchoncial injury B- Open pneumothorax C-
Tension pneumothorax
118. Fracture in the leg with deformity what to do
A- Control pain B- Control blood loss C- Save joint
119. 64-year-old patient with progressive jaundice, weight loss and vague abdominal
pain.Upon examination gall bladder was palpable What's the most likely diagnosis?
A- Periampullary cancer B- Klatskin tumor C- Colon cancer
120. A 10 years old boy came to the ER with right scrotal pain and swelling, on
examinationtender right testis, with decreased flow on Doppler study. Your diagnosis is?
A- Hernia B- Hematocele C- Testicular torsion D- Orchitis
121. 50. Elderly woman has epigastric pain, collapsed at home. In the ER she has mild low

PLEAZE, ANY NOTE OR CORRECTION OR ADDITIN SEND: [email protected]


‫نسألكم الدعاء بظهر الغيب‬
P a g e | 19
backpain and her BP 90/60. What's the most likely diagnosis?

PLEAZE, ANY NOTE OR CORRECTION OR ADDITIN SEND: [email protected]


‫نسألكم الدعاء بظهر الغيب‬
P a g e | 20
A- Mesenteric ischemia B- Leakage/ruptured aortic aneurysm
C- Perforated duodenal ulcer D- Gastric ulcer
122. Patient has HTN come with pulsatile abdomen swelling?
A- Aortic aneurysm B- renal cause
123. All can complicate excision of abdominal aortic aneurysm, EXCEPT?
A- Paraplegia B- Renal failure C- Hepatic failure D- Leg ischemia
124. The most common sign for the aortic aneurysm is the Erythema nodosum
125. Patient presented with severe epigastric pain radiating to the back. He has past
history ofrepeated epigastric pain. In Social history drinking alcohol. What's the most likely
diagnosis?
A- MI B- Perforated chronic peptic ulcer
126. The most accurate tool tor diagnosis of appendicitis
A- US B- Diagnostic laparoscopy C- CT scan
127. Patient with peptic ulcer using antacid, presented with forceful vomiting that
contains foodparticle A- Gastric Outlet obstruction
128. Picture of hyperthyroidism with us showing right solid nodule 2 cm and thyroid
scanshowing picture of toxic multinodular goiter. What is the most appropriate
management?
A- Subtotal thyroidectomy B- iodine Radio-ablation
C- Right thyroidectomy D- Near complete thyroidectomy
129. Female child with epigastric pain and bruise due to fall with history of neglected fall
fewdays ago. what to do now?
A- CT B- US C- Exploration
130. Most associated with nephrotic syndrome?
A- Gastritis B- Peritonitis.
131. Breast mass BIRAD V on us and enlarged pathological LNs. next step in management?
A- Core needle bx B- Excisional bx C- MRM D- Ct scan of bone and chest
132. Trauma with thigh hematoma and weak pulses distal to it. Next step in ER until
specializedphysician comes?
A- CTA B- Observe and analgesia C- Explore the wound
133. CT scan picture of left sided mca ischemic stroke and presented within
2hrs. What isManagement now?
A- Thrombolysis B- Heparin C- Nothing
134. He is diabetic hypertensive Weight 125, Hight 173 best to do?
A- Medication lower weight B- Bariatric surgeries> BMI more than 35+
135. Post appendectomy female came with LR abdomen mild tenderness Ex Normal by

PLEAZE, ANY NOTE OR CORRECTION OR ADDITIN SEND: [email protected]


‫نسألكم الدعاء بظهر الغيب‬
P a g e | 21
CTthere is 2*2 collection in Retrocecal

PLEAZE, ANY NOTE OR CORRECTION OR ADDITIN SEND: [email protected]


‫نسألكم الدعاء بظهر الغيب‬
P a g e | 22
A- Exploring laparotomy B- percutaneous drainage
C- laparoscopic D- conservative with Antibiotics
136. Radial nerve injury: wrist drop, common with humerus injury (humeral groove)
137. Ulnar nerve injury: claw hand, common with elbow injury
138. Median nerve injury: unable to do thumb opposition, common with wrist injury
30 years old female breastfeeding her baby present with painful lump on RT breast O/E:
redness hotness tenderness, no lymph node enlargement, fever 37.1, she starts on antibiotics
since period of time, US report show thick cyst with fluid collection, dx
A- abscess B- cyst
for clinical correlation What is the next appropriate management?
A- observe B- us guide aspiration C- Incision and drainage D- WLE
139. Pt. with repeated vomiting with a neck emphysema and opacity or fluid in pleural space?
A- Spontaneous esophageal rupture B- Lung bullae rupture
140. Patient come to ER with 4 days repeated vomiting what's the most common
electrolytedisturbance?
A- Hyperkalemia B- Hypercalcemia C- Hypokalemia D- Hypocalcemia
141. Patient with RTA coming to ER scalp injury BP 70/50 P130 RR35, initial will be give?
A- Colloid B- Crystalloid C- Blood of his group D- O -ve blood without cross
142. PATIENT MEDICALLY FREE NEED INGUINAL HERNIOPLASTY NEXT WEEK
A- ASA 1 B- ASA 2 C- ASA 3 D- ASA 4
143. Most common cause of pyogenic liver abscess is
A- Ascending cholangitis B- Intra abdominal sepsis
144. 65 years old male patient underwent hemicolectomy of cancer cecum one year ago
presented to ER by abd pain and vomiting 2 days ago EX abdomen distended but soft what is
the appreciate investigation
A- Plain x ray B- Barium enema C- Colonoscopy D- laparotomy
145. Patient post gastrectomy 3 day left hypo chondrium pain fever auscultation lt basal lung
what is cause
A- pneumonia B- atelectasis C- left sub phrenic collection
146. patient underwent gastrectomy with Rouen Y gastrojejunostomy 3-day NGT normal
drain 150 ml e abdomen lax, LABS normal except WBCs elevated what is the cause?
A- Duodenal stump blow out B- Jejunal leakage C- Gastrojejunal leak
During repair of obstructed umbilical hernia toxic fluid come on during dissection what type of
repair
A- Mayo repair B- Anatomical repair C- Repair with mesh D- Tension suture

PLEAZE, ANY NOTE OR CORRECTION OR ADDITIN SEND: [email protected]


‫نسألكم الدعاء بظهر الغيب‬
P a g e | 23
147. Helicobacter pylori
A- Common with gastritis B- Likely associated w gastritis C- Gram –VE flag
148. 9year old girl come to ER complaining of fatigue and lethargy physical examinations
she pale she has hepatomegaly and splenomegaly 6 cm below coastal area hemoglobin
low 50 sickle cell positive she had seam attack 2 times before what's your best
management
A- Regular blood transfusions B- Corticosteroids
C- Hydroxyurea D- Splenectomy
149. Patient bleeding from angiodysplasia left side
A- Left hemicolectomy B- Angio-embolization C-
Endoscopic laser ablation (Stable: c Unstable: B)
150. pancreatitis case managed conservatively at the day 5, laboratory: Everything normal
except Amylase 250, with dilated extra hepatic ducts appropriate management?
A- lap chole before discharge B- elective lap chole C- ERCP
151. Women Urine leakage while coughing and sneezing, no other symptoms
A- Enterocele B- Rectocele C- Paravaginal defect D- Hypermobile urethra
152. How to reduce colon cancer risk?
A- Life style modification B- Ferrous supplement
153. After resection of a pedunculated polyp the results was benign adenoma and patient
hasno family history of colon cancer what to advice for reduction of colon cancer?
A- Prophylactic sigmoidectomy B- Prophylactic colectomy C- Annual colonoscopy
154. Patient having air under diaphragm and normal bowel sound what to do?
A- Emergent exploratory laparotomy B- Gastrografin swallow
155. Pt presented with stabbed wound after wound exploration you found anterior
abdominal fascia penetration, (his vitals were stable) what's your next step'?
A- CT abd. B- MRI abd. C- Laparotomy D- Diagnostic laparoscopy
156. Which one of the following is correct regarding Level 2 axillary lymph nodes Please
selectone of the following?
A- Lie anterior to the pectoralis minor muscle
B- Lie inferior the lower edge of pectoralis minor
C- Lie posterior to pectoralis minor
D- Lie superomedial to pectoralis minor
157. A 25-year-old man, medically free. Presented to the ER with severe pain during
defecation and passage of amount of fresh blood after defecation. Physical examination
confirmed the presence of posterior anal fissure. What the most appropriate
A- Examination under anesthesia.

PLEAZE, ANY NOTE OR CORRECTION OR ADDITIN SEND: [email protected]


‫نسألكم الدعاء بظهر الغيب‬
P a g e | 24
B- Lateral internal sphincterotomy

PLEAZE, ANY NOTE OR CORRECTION OR ADDITIN SEND: [email protected]


‫نسألكم الدعاء بظهر الغيب‬
P a g e | 25
C- Chemical sphincterotomy with diltiazem
D- Botulinum injection
158. UTI in patient with allergy to penicillin, sulpha what is antibiotic?
A- nitro. B- ampicillin
159. Woman in delivery bleeding not stop she want to conceive in the future which
structure you should ligate?
A- Uterosacral ligament B- External iliac artery
C- Internal iliac artery D- Something vein
160. Case scenario surgeon did something wrong his assistant told him surgeon change
the action and no harm happened to patient good medical and surgical practice include
the following
A- The event must be record in operative notes
B- The assistant tells the patient only body
C- Nothing to be done
161. Surgeon after nephrectomy injured intestine peritonitis but back up repair intestine
patientleave hospital without complication after one week patient know he can
A- Sue's surgeon B- Sue hospital
C- Demand money D- Post on social media
162. What are stages of burn 3rd degree
A- Stasis B- Hyperemia C- congestion
163. Case of BCC the best for ttt
A- conventional excision B- moh surgery C- radiotherapy
164. Case of football player severe sudden rt iliac fossa pain
A- sports hernia B- inguinal hernia C- hematoma
165. ER call your senior went and leave you alone in OR in appendectomy you find tumor
overappendix what will you do?
A- Make only appendectomy and close abdomen.
B- Call senior and wait
C- Do hemicolectomy then call senior
166. Woody neck best investigation
A- TSH B- Biopsy
167. Woody neck scenario patient presented by hoarseness of voice the next step
A- decompressive thyroid isthmectomy B- total thyroidectomy C- core biopsy
168. Polytrauma patient admitted in ICU no brain issues but after 14 days develop
subarachnoid Hge the cause is
A- hypernatremia B- hyponatremia C- hyperkalemia D-hypercalcemia

PLEAZE, ANY NOTE OR CORRECTION OR ADDITIN SEND: [email protected]


‫نسألكم الدعاء بظهر الغيب‬
P a g e | 26
169. A case of gastric obstruction what is this the best ttt in previous case
A- Resection B- Removal w primary closure C- Bypass
170. Factor affecting recurrence of cancer colon
A- stage B- lymph nodes
171. Scenario Pt did upper endoscopy reveal grade 2 varices best ttt
A- sclerotherapy B- devascularization C- total shunt
172. Alcoholic old male p with 2 days melena diagnosis
A- bleeding duodenal ulcer B- bleeding gastric ulcer C- esophageal varices
173. You are consultant cardiothoracic surgeon and u will do operation success rate 10%
and afraid to tell patient because he may not sign consent what will you do and operation
very useful for patient
A- do operation without consent
B- do operation with consent from relatives
C- discuss with pt. his situation and for importance of operation
174. Child came to ER without his parents wound in face bleeding but he is stable
A- suture without parent consent
B- suture with relative come with him
C- apply gauze till his parents come
175. Electrolyte changes in gastric outlet obstruction
A- hypokalemia B- hyperkalemia C- hyponatremia
176. Early ECG change in that case
A- T wave saging B- P wave prominent C- ST depression
177. Scenario case of post intestinal resection anastomosis leak fever tachycardia IV fluid
A- RL B- NS C- D5 1/2NS
178. The most common cause of anal stenosis
A- Sclerotherapy B- Hemorrhoidectomy
C- Hemorrhoidectomy with rectal mucosa resection D- PIP
179. Female pt. 38 years old w epigastric pain during night recurrent attack relived
by IVhyoscine cause is
A- Duodenitis B- Pancreatitis C- Cholangitis
180. BCC therapy complication
A- Infection B- Recurrent C- Cosmetic discharge
181. Incisional hernia in multigravida female pt. most common complication post trans
peritoneal repair
A- Infection B- Adhesion C- Recurrence D- Perforation

PLEAZE, ANY NOTE OR CORRECTION OR ADDITIN SEND: [email protected]


‫نسألكم الدعاء بظهر الغيب‬
P a g e | 27
182. Old patient in ICU with acalculous cholecystitis best management?
A- cholecystostomy tube B- lap chole C- open chole
183. Burn patient present with Right upper quadrant pain dilated gallbladder thick wall and
pericystic fluid what is management?
A- cholecystostomy tube B- cholecystectomy C- MRCP0
184. GREEN. DISCHARGE BILATERAL MAMO AND US DUCT DILATATION WHATS NEXT TEST?
A- MRI B- BIOPSY C- FOLLOW UP
185. Patient with drop foot & lost the sensation in the webspace between 1st & 2nd
metatarsaljoints What is the damaged nerve?
A- Common peroneal nerve B- Deep Peroneal nerve
C- Tibial nerve D- Femoral nerve
186. Correction of fluid in patient with hypernatremia is overcorrection over 48 hours
187. Pt with CBD stone and ERCP bleeding and abdominal tenderness at site of
sphincterotomyCause of pain is?
A- Stone impaction B- Perforation C- Bleeding
188. Diabetic foot patient with ulcer 3*3cm reaching to SC and pain increase with walk
and decrease with rest
A- Ischemic B- Neuropathic
189. Child came alone with crushed limb and sever bleeding and should amputate
A- Amputate B- Wait parents
190. Hypertensive and diabetic patient on ACE inhibitors and B-blockers and oral
hypoglycemic then insulin (ASA 3)
191. Duodenal switch most affect?
A- VIT B12 B- VIT D
192. Pt with diarrhea, most affected?
A- K B- Mg C- Ca
193. When renal blood flow decrease which hormone will increase
A- ADH B- ALDOSTERONE C- STEROIDS
Patient with bleeding duodenal ulcer, injection of adrenaline by endoscope after 12 hours
rebleeding
A- Surgery B- Adrenaline C- Cautery
194. Young pt. has recurrent UTI, stones and electrolytes imbalance on imaging, bilateral
kidney enlargement with multiple variable size thin wall cysts throughout parenchyma.
What isthe diagnosis?
A- Medullary sponge disease C- Polycystic kidney disease

PLEAZE, ANY NOTE OR CORRECTION OR ADDITIN SEND: [email protected]


‫نسألكم الدعاء بظهر الغيب‬
P a g e | 28
195. Female complain of non-compressible mass, bilateral green nipple discharge,
mammogramshow BIRAD 2, what is your management?
A- Duct excision B- Reassure and follow up
196. Old diabetic, HTN and atrial fibrillation with signs of mesenteric ischemia, management is
A- Exploratory laparotomy
197. TIA and atrial fibrillation, what to give
A- Aspirin B- Heparin C- Warfarin INR 3-4 D- Warfarin INR 2-3
198. Obese Pt, DM, SCD, what type of GB stones
A- Cholesterol B- Pigmented C- Mix
199. Patient catch electrical wire, what decrease zone of stasis
A- Peripheral VD B- Control temp
200. 18 Month girl with asymmetrical breast enlargement other examination normal, what
youshould do?
A- Brain MRI B- Abd CT C- Pelvis CT D- Abd US
201. Farmer came to ER with severe abdominal pain and green discolouration after
hitting by wooden stick What is most appropriate management?
A- Abd CT B- Abd US C- Open surgery D- C&S test
202. In abdominal injuries, the most informative initial investigation is
A- CT B- Diagnostic peritoneal lavage C- US D- X-RAY
203. Which of the following is correct regarding sterilization?
A- Chemical sterilization is the commonest type
B- Scrub up team must keep hands above waist
C- Disinfection is enough for surgical instruments
D- Suspicious of organisms spread during operation not harmful
204. Standard screening tests on donor blood include all except
A- Hep. C B- Hep. B C- Rubella D- Syphilis
205. 25years old driver sustained a car accident came to ER w flaccid paralysis,
bradycardiaand hypotension, most likely diagnosis is
A- Neurogenic shock B- Cardiogenic C- Hypovolemic
206. In wound healing
A- Remodeling face may last up to 2years
B- Fetal wounds heal with primary intension
C- Type I collagen is present mainly in healing wounds
D- Prolonged continuous pressure decrease healing time
207. Paralytic ileus is associated with
A- Hypernatremia B- Hyperkalemia C- Hyponatremia D- Hypokalemia

PLEAZE, ANY NOTE OR CORRECTION OR ADDITIN SEND: [email protected]


‫نسألكم الدعاء بظهر الغيب‬
P a g e | 29
208. All are true except
A- Burn of low temperature but long contact produces mild injury
B- Epithelium is intact with erythema
C- Every burned pt. must receive tetanus vaccination
D- Any burn affecting face should be referred to a burn unit
209. Which of the following is indication of nutritional support?
A- Anorexia nervosa B- Intestinal fistula C- Malignancy D- All of the above
210. Which of the following procedure has high chance of developing pneumonia?
A- Anterior esophagectomy B- Lower segment CS C- Reduction of femoral fracture
211. late postoperative complications include
A- Reactionary Hge B- Short gut syndrome C- Delirium D- Nausea and vomiting
212. Weight loss is earlier with
A- Colon cancer B- Prostatic cancer C- Esophageal cancer D- Hepatoma
213. To detect hematogenous spread of tumor, all should be done except
A- Chest x ray B- Cystoscopy C- Abdominal CT D- Bone scan
214. Early complication of burn is
A- Dyspigmentation B- Sepsis C- Hypertrophic scar D- Contractures
215. Deep dermal burn
A- Heal with granulation tissue and fibrosis B- Painless
B- Have intact hair follicle D- Blanch on pressure
216. The commonest symptom of diverticular disease is
A- Bleeding per rectum B- Changes in bowel habits
C- Tenesmus D- Dull aching pain
217. 27years old pregnant female came with sever right upper quadrant pain started 5
hours ago with anorexia, vomiting and abdominal guarding on examination, most probable
diagnosis
A- Perforated peptic ulcer B- Hepatic hemangioma
C- Acute appendicitis D- Intestinal obstruction
218. 41year old female with known femoral hernia scheduled for operation came to ER
with severe pain over hernia and fever, on examination, hernia was tense, tender and
negative impulse on cough, the diagnosis is
A- Inflamed hernia B- Strangulated C- Obstructed D- incarcerated
219. Which tumor has least potential of malignant transformation?
A- Renal angiomyolipoma B- Ovarian embryonic carcinoma
C- Osteosarcoma D- Mesothelioma
220. The finger like projection of connective tissue core that lined with epithelium is called

PLEAZE, ANY NOTE OR CORRECTION OR ADDITIN SEND: [email protected]


‫نسألكم الدعاء بظهر الغيب‬
P a g e | 30
A- Fibroma B- Papilloma C- Desmoid tumor D- Ganglion

PLEAZE, ANY NOTE OR CORRECTION OR ADDITIN SEND: [email protected]


‫نسألكم الدعاء بظهر الغيب‬
P a g e | 31
221. 22year male came to clinic with abdominal pain, diarrhea and weight loss for 1month,
he gave history of occasional occult bleeding in stool, the most likely diagnosis is
A- Crohn disease Peptic ulcer C- Incarcerated hernia D- Intestinal obstruction
222. Features of previous diagnosis include all of the following except
A- Mucosal ulcerations separated by normal mucosa
B- All cases should be treated surgically
C- Most common site is ileum
D- Fistula is known complication
223. 29year old man came to clinic with itching and protrusion of small mass through anus
associated with minimal bleeding, increase when go to GYM and he used to reduce it
manually.The grade of this hemorrhoid is
A- I B- II C- III D- Iv
224. Which of the following is true?
A- Anal fistula is an abnormal connection between anal canal and perianal skin
B- Antibiotics alone sufficient in ttt of anorectal abscess
C- Traumatic fistulae are usually intersphincteric
D- Setons have increased risk of incontinence
225. The ttt of choice of ascites is
A- Portosystemic shunt B- Peritoneal shunt C- TIPS D- conservative
226. Patent processus vaginalis has an association with
A- Direct inguinal hernia B- Indirect inguinal hernia
C- Incisional hernia D- Lumbar hernia
227. Boundaries of Hasselback triangle include all the following except
A- Lateral border of rectus muscle B- Inferior epigastric artery
C- External iliac artery D- Inguinal ligament
228. in choledocholithiasis which of the following parameters decrease
A- Alkaline phosphatase B- Clotting factor x
C- Prothrombin time D- INR
229. Which of the following included in Ranson criteria?
A- WBCS B- ALT C- Amylase D- Lipase
230. The most common cause of chronic pancreatitis is
A- Hyperlipidemia B- Gallstones C- Idiopathic D- Alcoholism
231. 61year old smoker presented in clinic with jaundice, abdominal pain and weight loss.
on examination, fluid was heard with shaking and palpable GB and non-tender, the gold
standardinvestigation is
A- US B- CT SCAN C- ERCP D- CA19-9 marker

PLEAZE, ANY NOTE OR CORRECTION OR ADDITIN SEND: [email protected]


‫نسألكم الدعاء بظهر الغيب‬
P a g e | 32
232. All the following are superficial neck swellings except
A- Branchial cyst B- Sebaceous cyst C- Lipoma D- Neurofibromatosis
233. All are true in management of variceal bleeding except
A- Terlipressin may be used before confirmed diagnosis
B- Recurrent bleeding is prevented by surgery
C- Child C patient should undergo transplant evaluation
D- Sedatives should be given to reduce anxiety
234. In hepatic encephalopathy which is incorrect
A- Ammonia production can be reduced by neomycin
B- Lactulose is useful for ammonia reduction
C- Protein diet should increase in management
D- Hypokalemia is associated imbalance
235. True about Fibroadenoma
A- Rare before age of 40
B- Most common cause of breast mass in 4th and 5th decades
C- Recurrence after excision of large lesions is possible
D- Its size decrease with pregnancy
236. Which is cyanotic heart disease
A- Coarctation of aorta B- Tetralogy of Fallot
C- Ventricular septal defect D- Pulmonary stenosis
237. 53-year-old woman came to ER with chest pain and dyspnea, MI was excluded ECG
showed MR, the patient most commonly has history of
A- MI B- Infective endocarditis
C- Congenital heart disease D- Rheumatic fever
238. The most common midline single neck swelling is
A- Pharyngeal pouch B- Dermoid cyst
C- Laryngocele D- Thyroglossal cyst
239. All the following are true except
A- The most common cause of 1ry hyperparathyroidism is adenoma
B- Thyroid malignant lymphoma should be treated surgically
C- Steenson duct open opposite 2nd upper molar tooth
D- Recurrent laryngeal nerve injury during thyroidectomy can be permeant
240. Which of the following is associated with malignant breast lesions
A- Hyper echogenicity B- Widely scattered
C- Micro calcifications D- Smooth margins

PLEAZE, ANY NOTE OR CORRECTION OR ADDITIN SEND: [email protected]


‫نسألكم الدعاء بظهر الغيب‬
P a g e | 33
241. Non small lung cancer
A- Large cell carcinoma is the commonest type
B- Adenocarcinoma is diagnosis of exclusion
C- Patient with risk factor should be screened because it can be asymptomatic
D- 20% of patients are smokers
242. The gold standard investigation in GERD IS
A- Ambulatory PH monitoring B- Barium swallow
C- Endoscopy D- Clinical picture
243. Hiatus hernia
A- Reflux not seen in paraoesophageal type
B- Dysphagia commonest symptom in sliding type
C- Paraoesophageal type is treated medically
D- The gastroesophageal junction is intraabdominal in sliding type
244. Mother brought her 3months baby to clinic concerning about his large head and poor
feeding followed by vomiting. on examination scalp veins were dilated and eyes were
forcible deviated downward, the cause of this condition is
A- CSF overproduction B- Obstruction of CSF flow
C- Under absorption of CSF D- all of the above
245. The commonest complication following CABG
A- Atelectasis B- Infection C- Arrhythmia D- Bleeding
246. The most frequent presentation of pulmonary embolism is
A- Hemoptysis B- Dyspnea C- Syncope D- Chest pain
247. Metastatic osteosclerotic bones in plan X-ray primary most likely to be in
A- Prostate B- UB C- Lung D- Liver
248. Correct statement regarding the three phases of wound healing
A- Inflammation, epithelization, contracture
B- Inflammation, fibroplasias, contracture
C- Fibroplasia, epithelization, contracture
249. In wound healing maximal amount of collagen occurs at
A- 10 days B- 21days C- 6months D- Less than 1 week
250. In wound healing which is important to harden the healing
A- Tensile B- Elastin C- Collagen
251. In regards to scaphoid fractures, there is necrosis due to
A- There is dislocation fracture
B- The blood supply is distal to proximal
C- Tendon's cause tension on fractured fragments

PLEAZE, ANY NOTE OR CORRECTION OR ADDITIN SEND: [email protected]


‫نسألكم الدعاء بظهر الغيب‬
P a g e | 34
252. Young female presented to ER she has been involved in MVA, sustained tibial and
femur fracture Pt stabilized and admitted then developed sudden respiratory distress, she
is sufferingfrom (Fat embolism)
253. Male patient with duodenal ulcer has been vomiting for 2 weeks,what metabolic
disturbance will be
A- Hypochloremic, hypokalemic metabolic acidosis
B- Hypochloremic, hyperkalemic metabolic acidosis
C- Hypochloremic, hypokalemic metabolic alkalosis
254. Duodenal ulcer surgery with least risk of recurrence is
A- High selective vagotomy B- Vagotomy and pyloroplasty C- Vagotomy and antrectomy
255. Best long-term management of achalasia is
A- Nifedipine B- Nitrates C- Heller myotomy D- Fundoplication
256. Symptoms of thyrotoxicosis
A- Obesity B- Atrial fibrillation
C- Low energy level D- Loss of hair at lateral part of eye brows
257. Nipple discharge
A- Likely to be benign if spontaneous
B- Likely to be malignant if bloody and unilateral
C- If ductogram is normal no need for further evaluation
258. Most common thyroid cancer is
A- Papillary B- Follicular C- Anaplastic D- Medullary
259. Thyroid cancer that secretes calcitonin
A- Medullary B- Follicular C- Anaplastic D- Papillary
260. Most common finding in contamination of hyperalimentation subclavian catheter in
animmunocompromised patient on broad spectrum antibiotic
A- Pneumocystis carinii pneumonia
B- Pseudomonas aeruginosa
C- Candida albicans
261. KUB was done and incidentally gallstones were found. Next step is
A- Lap cholecystectomy B- Arrange for ERCP
C- US D- Laparotomy cholecystectomy
262. Patient threatened you for complaining if you don’t do the surgery
A- Consult physician B- Call social workers
C- Do it D- Ethical committee
Another answer is referring pt.

PLEAZE, ANY NOTE OR CORRECTION OR ADDITIN SEND: [email protected]


‫نسألكم الدعاء بظهر الغيب‬
P a g e | 35
263. Lung abscess
A- Always excised B- Mostly due to
aspiration c- Rarely requires antibiotics if well drained
264. Tension pneumothorax, immediate ttt is
A- IVF B- Needle aspiration
265. 2-year-old child refuse walking since hitting toy and rt leg twisted while falling
A- Spiral fracture of rt tibia B- Spiral fracture of rt femur
C- Soft tissue swelling of rt ankle D- Chip fracture of proximal rt tibia
266-Long scenario of middle-aged male with rectal bleeding, biopsy showed adenoma, at
whatinterval you do colonoscopy

A- 3-6months B- 3 years C- 10year D- No need

266. 30 years old female presented hypotensive with severe abdominal pain and last
period 2 months ago
A- Methotrexate B- Immediate surgery C- Refer to gynecologist
267. 3-month boy came with fever and history of circumcision, you take urine culture and it
was80000E coli and sensitive to Sulfa you gave him antibiotics for 10days and discharged
after 2 days mother said his fever improved
A- Complete course for 10days and no need for further action
B- Do US C- Repeat urine culture
268. 30 years old with hx of gallstones presented with epigastric pain for 6 days radiating
to back, vomiting, on examination diffuse distension and epigastric tenderness, labs
lipase and amylase were high, CXR showed left sided pleural effusion, what is the
investigation?
A- CT abdomen B- US C- Analysis of the fluid
269. Lung biopsy from
A- Midclavicular 6th intercostal space B- Midaxillary 6th C- 9th intercostal
270. 47 years old female single with positive family history of breast cancer, routine
mammogram showed bilateral increased density and glandular pattern. core needle
biopsy showed atypical ductal hyperplasia What is appropriate management?
A- Wire surgical excision B- Simple mastectomy
271. Diagnostic for sarcoma
A- Incisional B- Excisional C- Core needle D- Aspiration
272. Osteoporosis is one of the systemic involvement manifests of chrons disease. what
determine the severity of osteoporosis?
A- Age 20-40 B- Crohn involvement
273. What improve mortality in liver cirrhosis with esophageal varices and bleeding
PLEAZE, ANY NOTE OR CORRECTION OR ADDITIN SEND: [email protected]
‫نسألكم الدعاء بظهر الغيب‬
P a g e | 36
A- Octreotide's B- IV ceftriaxone

PLEAZE, ANY NOTE OR CORRECTION OR ADDITIN SEND: [email protected]


‫نسألكم الدعاء بظهر الغيب‬
P a g e | 37
274. When you order the test for screening cancer you will choose
A- Highly specific B- Highly sensitive C- Quick to perform D- Cheap
275. Which test is appropriate to study 5 different types of exercise given to obese ladies
overa period of time and follow there BMI?
A- Cohort B- Cross sectional C- Case control D- Clinical trial
276. Highest level of evidence to determine exercise effect on quality-of-life
A- Cohort B- Case control C- Clinical trial D- Observational study
277. Female patient breast feeding has unilateral tender breast mass that is involving
areola and attached to areola with small many cysts or something
A- Duct ectasia B- Intraductal papilloma
C- Fibroadenoma D- Paget disease
278. 47 years old patient with neck enlargement, US revealed mass on RT side 2*1cm
and another 3*2 on left side, where FNA should be taken
A- Left nodule B- Largest nodule C- Both nodules D- Right nodule
279. Signs of hypothyroidism with history of Hodgkin treated by radiation, investigations
A- TSH after 4 weeks B- T4 C- Thyroid scan D- US
280. Trauma patient with spleen laceration 2cm with fluid collection, what is your management
A- No surgical intervention B- Splenectomy C- Preserved surgery
281. Patient on metformin you admit him, what type of insulin
A- Sliding insulin SC B- IV insulin +long-acting insulin
C- Insulin BID D- Fixed rate IV insulin
282. Patient with neck pain and hyperthyroidism with upper respiratory infection, what to give
A- Propanol B- PTU C- Thyroid scan D- Methimazole
283. Old patient present with painless hematuria, medically free and no other symptoms,
vitallystable, what is the best way to diagnose
A- Cystoscopy B- Pelvis US C- CT abdomen D- IV pyelogram
284. Patient on C/S she developed massive bleeding where is site of bleeding
A- IMA B- Aorta C- Splenic artery D- Perforated peptic ulcer
285. Woman did CS came after 5 days complain of abdominal pain and tenderness and
distension, on imaging she has small bowel fistula, what are you going to do
A- Colostomy B- Resection and anastomosis C- Soft diet only
20 years old patient complaining of lower abdominal pain and tenderness like cystitistypical
case, how will you manage A- No need for ttt
B- Empirical ttt C- Wait for culture
D- Encourage hydration and Carnaby fruit

PLEAZE, ANY NOTE OR CORRECTION OR ADDITIN SEND: [email protected]


‫نسألكم الدعاء بظهر الغيب‬
P a g e | 38
286. RTA X-ray show widening mediastinum, what is the diagnosis
A- Aortic injury B- Cardiac contusion
287. RTA was stable in ambulance sudden deterioration with increased jvp and weak
thread pulse (Cardiac tamponade) cardiac shock
288. Truck driver presented with raised JVP and decreased BP and distant heart sounds,
what kind is the shock
A- Anaphylactic B- Septic C- Cardiogenic B- Hemorrhagic
289. Newborn with antenatal history of polyhydramnios cannot feed, drooling and
bubbling of his nose when feeds, imaging show nasogastric tube curled at esophageal
pouch. What is the complication of this procedure?
A- Phrenic nerve injury B- GERD C- Chylothorax
290. New born baby with right inguinal reducible hernia, high raising right testis and non-
swelling left inguinal hernia, what is the management
A- Observation B- Repair with mesh now C- xxx Wait till 6 years
291. Patient present with signs and symptoms of stroke, what next step in management
A- CT B- MRI C- EEG
292. Bedridden patient present with confusion and agitation, low blood pressure and high
HR,ECG show sinus tachycardia, what is the diagnosis
A- Stroke B- MI C- PE
293. Patient present with bleeding after 5 days of tonsillectomy What is the cause
A- Sepsis B- Incomplete removal C- Coagulopathy D- FB
294. 34-year female with breast fibroadenoma with atypia and hyperplasia risk for cancer
A- Her age B- Atypia C- Size D- Hyperplasia
295. Tumor lysis syndrome ( Hyperkalemia and hypocalcemia)
Old patient came to ER with confusion after falling from stairs, CT revealed extradural
hemorrhage, which artery is affected (Middle meningeal artery)
296. Most common cause of AMO in children (Bacterial)
297. Camping in community to educate people about health determents is considered as
A- Health education B- Active surveillance
298. Camping in community to decrease risk of hypertension is considered as
A- Primary B- Secondary C- Tertiary
299. Woman complain of pain before menses and resolves in 3rd day, how do you diagnose
By clinical symptoms
300. 45 years old female for colonoscopy screening (Low risk 50- high risk 10 years younger)
52 years old female known case of DM and knee osteoarthritis, has been using NSAID to relieve
her pain, she developed HTN, Na normal, K low borderline What do you think the cause of HTN

PLEAZE, ANY NOTE OR CORRECTION OR ADDITIN SEND: [email protected]


‫نسألكم الدعاء بظهر الغيب‬
P a g e | 39
A- Essential HTN B- Pheochromocytoma
C- NSAID induced HTN D- Primary hyper aldosteronism
301. 17 years old medically free female, in her class developed breasts later and never
menstruated, on examination she is tanner stage 5 but no menstruation What is the
diagnosis
A- Hypothalamic hypogonadism B- Imperforate hymen
C- Gonadal agenesis D- Testicular feminization
302. 18 years old unmarried girl ask about for screening for PAP smear
A- At 21-year B- When sexually active C- Come in one year
303. Intraductal papillary mucinous neoplasm ERCP picture
A- Fish eye sign B- fish mouth
304. In the past it was commensal, now it is known as pathogenic organism Staph edidermidis
305. A case of head trauma, in coma for 5 days, what is the initial way of feeding at this time?
A- NGT B- Gastrostomy C- Peripheral line D- Central line
306. Women want to take OCP and husband not want to take OCP
A- Empathy B- Follow wife C- Follow both
‫كل ما يتعلق بعمليات منع الحمل او ادوية مانعة للحمل يجب موافقة الزوجين‬.‫أما ما يتعلق بالجراحة الخاصة بطبيعة الوالدة سواء طبيعي أو‬
‫قيصرية فهذا فقط يتعلق بالمرأة ألنه ج سمها وهي حرة في االختيار وده معنى ال‬autonomy
307. Your colleague in the hospital has HBV positive, what will you do??
A- Ask him to tell his patients B- Write confidential report C- Don’t refer patient to him
308. Stage of change for smoking cessation
Precontemplation: current smokers who are not planning on quitting smoking in the next 6
months
Contemplation: current smoker who are planning quit smoking in the next 6 months but not have
made quit attempt in the past year
Preparation: current smokers who are definitely planning to quit within next 30 days and have
made a quit attempt in the past year
Action: individuals who are not currently smoking and have stopped smoking within the past 6
months
Maintenance: individuals who are not currently smoking and have stopped smoking for longer
than 6 months but less than 5 years
309. A child will go for circumcision, what is the best investigation??
A- BT B- Factor VII C- Factor 5
310. Scenario, a case of massive blood transfusion and the patient had cardiac arrest.
What isthe cause
A- Hyper K B- Hyper Ca?

PLEAZE, ANY NOTE OR CORRECTION OR ADDITIN SEND: [email protected]


‫نسألكم الدعاء بظهر الغيب‬
P a g e | 40
311. Patient on TPN, patient has anemia and will be given blood, the nurse gave the him
bloodin the same line of TPN, patient developed coma, what is the cause??
A- Hypoglycemia B- Hypocalcemia C- Hypokalemia D- Hyponatremia
312. The best initial treatment for a case of DKA?? IV fluids
313. The best fluid for a case of gastric outlet obstruction is??
A- Normal saline B- Ringer C- Glucose
314. A child presented with crushed limb with severe bleeding and need amputation, what
youwill do??
A- Amputate B- Wait the parents
315. Hypertensive patient on ACE inhibitor and b-blocker, also he is diabetic on oral
hypoglycemic drugs, he changed it to insulin, ASA??
A- I B- II C- III D- IV
316. The most accurate sign of chronic hypovolemia??
A- Ms. wasting B- Sunken eye C- Dry skin
317. Gene responsible for cancer colon
A- APC (tumor suppressor gene) B- KRAS C- CEA
318. Patient with liver cirrhosis and has ascites, presented with 3 cm liver mass,
management?? All labs within normal
A- Liver transplantation B- mass resection
319. Newly married female went to check up, what to check?
A- General appearance B- Pelvic examination C- Abdominal examination
320. Patient with history of lifting heavy weight, has tenderness, abdominal protrusion,
negative cough test, what is the diagnosis?
A- Ventral hernia B- Inguinal hernia C- Rectus sheath hematoma
321. Case of patient with increased JVP and decrease of heart sounds, what is the
type ofchock??
A- Cardiogenic B- Hypovolemic C- Obstructive shock
322. Patient fall down on out stretched hand include snuff box, what is the fracture??
A- Collis B- Scaphoid
323. Parents refused to vaccinate their child, what will you do?
A- Social service B- Ethics department
C- Vaccinate him D- Tell them advantage of the vaccine
324. 22 weeks newborn with severe congenital anomalies you want to intubate him and
hismother is your colleague and she refused to intubate, what will you do??
A- Respect her wishes B- Intubate C- Refer the patient to another doctor
325. Treatment of anaplastic thyroid cancer?

PLEAZE, ANY NOTE OR CORRECTION OR ADDITIN SEND: [email protected]


‫نسألكم الدعاء بظهر الغيب‬
P a g e | 41
palliative thyroidectomy for compressive symptoms
palliative chemo XRT
total thyroidectomy for resectable lesion
326. The most common thyroid cancer associating to hashimoto thyroiditis?
A- Papillary B- Follicular C- Medullary D- Lymphoma
327. Treatment of toxic goiter in pediatric patients?
A- radioactive iodine B-surgery C- antithyroid drugs
328. Grave’s disease is caused by? TSH receptor Ig G antibodies
329. Treatment of Riedle thyroiditis isthmectomy and tracheostomy
330. Female patient, during laparoscopic surgery, duodenal perforation happened, how to
manage? patient is stable. ‫سؤال جديد‬
A- Exploration B- Laparoscopy C- Conservative
331. A case of gall stone ileus, TTT>>> If patient stable and fit: enterolithotomy,
cholycystectomyand fistula closure
332. The most common site of obstruction in gall stone ileus is terminal ilium
333. BRCA 1 & 2 incidence of breast cancer? 75% - 85 %
334. Injury of RLN only ………. Vocal cord become median or paramedian due to action of
cricothyroid MS
 Injury of both external and RLN …. VC are away from Medline as they are now paralyzed.
 Unilateral RLN …… dyspnea on exertion
 Unilateral EXT laryngeal n & RLN …. HOARSNESS of voice
 Bilateral RLN …. STRIDOR
 BILATERAL EXT &RLN …… APHONIA
 INJURY OF EXTERNAL LARYNGEAL NERVE …… LOSS OF HIGH PITCHED SOUND.
335. Boerhaeve syndrome
 Management: repair and drainage
 Scenario, recurrent attacks of vomiting, chest pain.
Perforated esophagus
336. Mallory Weiss syndrome
 Presentation: episode of hematemesis after violent retching
or vomiting in alcoholic patient
 Site: tear in stomach (lesser curvature)
 Inv: endoscopy
 TTT: SPONTANEOUS RESOLVE AND BLEEDING STOP
(ANTIEMETIC)

PLEAZE, ANY NOTE OR CORRECTION OR ADDITIN SEND: [email protected]


‫نسألكم الدعاء بظهر الغيب‬
P a g e | 42
337. Inv of suspected
esophageal perforation during endoscopy Water soluble swallow
(Gastrografin)
338. Most common risk factor of SCC IN ESOPHAGUS ACHALESIA or SMOOKING
339. Myeloproliferative disease associated with huge spleen, what is the best invetigation?
CT with contrast
340. Origin of BCC Pluripotential cells of basal cell layer
341. The most common congenital anomaly associated to gastroschisis is INTESTINAL ATRESIA
342. Target cell in blood smear … in which disease? Thalassemia
st
343. Most common presentation of anal canal carcinoma 1 bleeding
A- Discharge B- Mass C- Pain D- Pruritis
344. Pt homosexual presented with ulcerative mass on the anus, painful, rectal
examinationcannot be done due to pain and stenosis, what is the diagnosis?
A- Chronic Anal fissure B- Anal canal carcinoma C- Rectal carcinoma
345. Patient with Cancer rectum with invasion to mesorectum, how to manage?
A- Neoadjuvant chemotherapy B- Adjuvant chemo radiation C- Trans anal excision
346. Mass in rectum 5 cm from the levator ani, moderately differentiated,
adenocarcinoma invading the mesorectum, what is the indication of neoadjuvant
chemoradiation?
A- Mesorectum invasion B- 5cm from levator ani
C- Adenocarcinoma D- Moderate diff.
347. Tear 6-7 cm in rectum near to half circumference of the rectum, by PR examination
youcan feel tip of wooden foreign body, how to remove this foreign body??
A- squeeze over the abdomen B- Open the rectum
C- Laparotomy and squeeze the intestine D- Let the patient strain
348. Tear 6-7 cm in rectum near to half circumference of the rectum, by PR examination
youcan't feel the foreign body, how to remove this foreign body??
A- Bimanual judge B- Anoscope
C- Laparotomy and squeeze the intestine D- Laparotomy and open the rectum
349. Patient with cancer rectum, patient started to complain from SOB, what will you order
forinvestigation??
A- CXR B- Pulmonary function test C- Ventilation perfusion scan
350. TRAUMA Patient with extra peritoneal bladder injury, management??
A- Suprapubic catheter B- Urgent exploration and repair
C- Catheter repair and assess after 2 weeks D- Catheter then US after 2
weeks

PLEAZE, ANY NOTE OR CORRECTION OR ADDITIN SEND: [email protected]


‫نسألكم الدعاء بظهر الغيب‬
P a g e | 43
351. Lymph node swelling, and was clear lymphatic fluid in this swelling, what to do?? Cystic
hygroma
A- Surgery B- Sclerotherapy C- Radiotherapy D- Chemotherapy

PLEAZE, ANY NOTE OR CORRECTION OR ADDITIN SEND: [email protected]


‫نسألكم الدعاء بظهر الغيب‬
P a g e | 44
352. Morbidly obese male patient, how to decide the best reduction surgery he will have
A- Barium enema B- CT abdomen C- US D- GI endoscopy
353. Patient presented with intestinal obstruction, underwent laparotomy, you find
obturatorhernia, constriction ring is dark purple, what is the pathophysiology?? Venous
congestion
354. Patient presented with incarcerated hernia, what is the indication of laparotomy??
A- Air under diaphragm B- Dilated bowel
C- Air fluid level D- Valvulae connivence
355. Inguinal hernia reaching the neck of scrotum, negative internal ring test, what is the
typeof hernia??
A- Direct inguinal hernia B- Femoral hernia C- Indirect inguinal hernia
356. Female patient with irreducible, non-tender umbilical hernia, what is the treatment??
A- Expectant treatment B- Elective repair
357. Epigastric pain or periumbilical pain referred to right iliac fossa + history of
inguinalhernia, by examination there is irreducible mass in the right iliac fossa,
Diagnosis??
A- Maydle hernia = W hernia B- Amyand’s hernia C- Richter hernia
N.B a case of acute appendicitis, so ttt is appendectomy through a herniotomy hernia repair without mesh

358. Patient underwent repair of incisional hernia, after 14 days she presented with mass
10 x10 cm not reducible, not tender, all parameters are normal. (mostly seroma)
A- Ab B- Hot fomentation C- Aspiration guided by us
359. A case of obstructed umbilical hernia, what is the Best investigation??
A- US B- CT abdomen C- Abdominal x ray
360. Seat belt injury, most common injured organ is
A- Duodenum B- Spleen C- Liver D- Mesentery
361. RTA speed 130 km/hr., wearing seat belt, presented to the ER, what is the initial
investigation?
A- US B- CT abdomen C- Abdominal x ray
362. Diagnosis of ectopic thyroid tissue in thyroglossal cyst
A- CT neck B- Neck US C- FNAC D- Labeled iodine (radionucleotide)
N.B the best is PET SCAN
363. Young female patient suddenly developed jaundice and fatigue, she had high ALP,
and high bilirubin, splenomegaly and hepatomegaly: US no finding, MRCP multiple foci of
strictureand dilatation Which is the best initial next step??
A- Liver biopsy B- Antinuclear antibody (not specific)
C- Repeat US D- Colonoscopy (to detect UC)
N.B 5-10 % of patients with PSC are associated with UC
PLEAZE, ANY NOTE OR CORRECTION OR ADDITIN SEND: [email protected]
‫نسألكم الدعاء بظهر الغيب‬
P a g e | 45
364. What is the treatment of cold abscess??
A- Incision B- Aspiration C- Ant TB drugs
365. The most common extra intestinal manifestation of UC??
Arthritis (21%)
Ankylosing spondylitis (2%)
Sclerosing cholangitis (4%)
366. Female patient underwent modified radical mastectomy, developed edema in the
upperlimb, management?
A- Limb elevation and analgesia B- Elastic stocking C- Incision and drainage
367. Patient with foot injury by a piece of wood, last dose of tetanus vaccine 12 years
back,what will you give him??
A- Tetanus toxoid B- Tetanus immunoglobulin
C- Tetanus antiserum D- Tetanus toxoid + immunoglobulin
368. A case of foot injury with wood piece, causing lymphangitis, what is the causative
organism??
A- Clostridium tetani B- Strept. Pyogenes C- Clostridium perfringes
369. Necrotizing fasciitis TTT?
A- Debridement B- Ab C- Debridement and antibiotic
370. 42 yrs. old female presented with severe abdominal pain not relieved except by
morphia,vomited 3 times, and one episode of bloody diarrhea, what is the best
investigation to diagnose
A- Abdominal radiology B- Bun C- Hct
371. 42 yr. old female presented with severe abdominal pain relieved only by morphia,
vomited3 times, and one episode of bloody diarrhea, how to treat this patient??
A- Celiac mesenteric anastomosis
B- Percutaneous transluminal angioplasty PTA if no peritonitis or perforation
C- Resection and anastomosis
D- Ileal bypass
372. Patient presented with critical limb ischemia, failed embolectomy, obliterated
poplitealartery, only patent superficial femoral artery, management??
A- AKA B- BKA C- DYSARTICULATION
373. Patient diabetic, with embolus occluding segment from common iliac up to common
femoral, with dry gangrene in the leg above knee, how to improve the prognosis and
decreaselevel of amputation??
A- Endovascular stent B- Embolectomy C- Axillofemoral bypass
374. Long scenario of middle age male 42 yr. with rectal bleeding, biopsy from sigmoid

PLEAZE, ANY NOTE OR CORRECTION OR ADDITIN SEND: [email protected]


‫نسألكم الدعاء بظهر الغيب‬
P a g e | 46
showedadenoma at what interval shall you do colonoscopy?

PLEAZE, ANY NOTE OR CORRECTION OR ADDITIN SEND: [email protected]


‫نسألكم الدعاء بظهر الغيب‬
P a g e | 47
A- 3- and 6-years B- 3 years C- 10 years D- No need
N.B SINGLE adenoma 5-10 years, Villous or dysplasia 3 years
375. Safety margin SLN Biopsy should be offered to all patients with clinically negative
nodalbasin and primary melanoma > 1 mm in depth.
Most common distant metastasis to the LUNG
SUPERFICIAL SPREADING Is the most common type
Small intestine is the most common site of gastrointestinal tract metastasis from melanoma

Insitu 0.5 cm

Less or 1 mm 1 cm

1.1 -2 mm 1-2 cm

More than 2 mm 2 cm

376. HYPOKALEMIA …. Flat t wave, prominent u wave


377. Pt in ICU, SIRS, elevated bun and create …. severe sepsis
378. Dark urine, hypotensive …. Toxic shock shock
379. Low PH, Low bicarbonate, Normal pco2 Diagnosis?
A- Compensated respiratory acidosis B- Uncompensated respiratory acidosis
C- Compensated metabolic acidosis D- Uncompensated metabolic
acidosis
380. Bilateral unequal limb in child, short femur diagnosis ??Short femur
381. Scenario pancreatectomy, post op febrile, hypotensive, what is the cause of hypotension?
A- Increase capillary permeability B- Decrease GFR
382. COMPARTMENTAL SYNDROME How to measure? intra vesical pressure
383. Gra1ding of intrabdominal pressure, Case CVP 16 (NORMAL 0-5)
A- G1 B- G2 C- G3 D- G4
384. Old age male patient, cachectic, with mass in the right side, diagnosis?? Cancer cecum
385. Old age female, nurse, fall down, developed distention, diagnosis?? Volvulus
386. Female patient with history of arthritis, developed epigastric pain, then become
generalized. Diagnosis?? Perforated peptic ulcer
387. a case of Perforated appendix Pt with acute abdomen, laparotomy, normal
appendix,terminal ileum shows thick with greenish white discharge with adhesions,
diagnosis??
A- Crohn’s B- TB C- Actinomycosis D- Typhoid
388. During operation for presumed appendicitis, appendix appear normal, however the

PLEAZE, ANY NOTE OR CORRECTION OR ADDITIN SEND: [email protected]


‫نسألكم الدعاء بظهر الغيب‬
P a g e | 48
terminal ilium is evidently thickened and feel rubbery, its serosa with grayish white
exudate and several loops of apparently normal small intestine are adherent to it, what is
the most likely

PLEAZE, ANY NOTE OR CORRECTION OR ADDITIN SEND: [email protected]


‫نسألكم الدعاء بظهر الغيب‬
P a g e | 49
diagnosis??
A- Crohn’s disease of terminal ilium B- Ulcerative colitis
C- Perforated Mickell's diverticulum D- Ileo cecal TB
E- Acute ileitis F- Actinomycosis ileitis (yellowish )
389. The most common SITE OF BILLIARY TRACT CANCER IS
A- Intrahepatic biliary tree B- Confluence of biliary tree
C- Extra hepatic bile duct C- gall bladder
N.B confluence of biliary tract = perihilar =klatskin tumor
390. The most appropriate diagnostic investigation for diagnosis of Klatskin tumor is MRCP
391. The best investigation which can differentiate between inguinal hernia, femoral hernia
andSafina varix is ….
A- MRI B- CT C- DUPLEX
392. The best treatment of litter's hernia (hernial sac contain tubular structure 6*1 cm,
Mickell'sdiverticulum?? Diverticulectomy and herniorrhaphy
393. Intrahepatic biliary radicle dilatation with normal CBD, next step??
A- MRCP B- ERCP C- PTC D- CT
394. While doing appendectomy in 12 yrs. old female you discovered simple ovarian cyst
A- Leave the cyst you don't have consent
B- Go and get a written consent from the father
C- Do the proper interference
D- Ask another colleague
395. 25 y/o women taking steroid for IBD c/o from abdominal pain and bilious vomiting , O/E
her abdomen is distended and tenderness in the right iliac fossa , she had done
colonoscopy 2 weeks ago which was normal , contrast barium showed single stricture at
terminal ilium 1 cm from ileocecal valve , most appropriate management ??
A- Stricturoplasty B- Right hemicolectomy
C- Conservative management D- Segmental resection with ileostomy
396. 4o year old female with CCC, preoperative US showing peri cystic fluid, thickening of
gall bladder wall, during lap. Cholecystectomy the GB anatomy lost. What is the most
appropriate next step??
A- Continue laparoscopic cholecystectomy B- Partial lap chole
C- Open complete chole D- Open partial chole
397. A case of hydatid cyst in the lung, patient has history of SOB, progressive with chest
pain.what is the management??
A- Albendazole B- pneumonectomy C- Aspiration D- PAIR

PLEAZE, ANY NOTE OR CORRECTION OR ADDITIN SEND: [email protected]


‫نسألكم الدعاء بظهر الغيب‬
P a g e | 50
398. Child coming in car accident with isolated head injury with subdural hematoma,
needs urgent exploration and all his family with him his father is coming on the way of
hospital. Whatis the proper action?
A- Wait his father arrival and take consent from him
B- Take the consent from the most senior family member
C- Inform the father by phone and take verbal consent
D- Go without consent as it is lifesaving procedure
399. The highest level of K is in which body fluid??
A- Saliva B- Urine C- Sweet
400. A case of permanent ileostomy, what is the most common complication??
A- Necrosis B- Parastomal hernia
C- non functioning D- Dehydration
401. Cause of anemia post partial gastrectomy??
A- Vit B12 deficiency B- Bleeding
C- Hypovolemia D- Iron deficiency anemia
402. Old age patient with rectal prolapse, what is the procedure?? Altemiere procedure
Most common presentation?? Mass protruding
Most common complication post-surgery? Infection then incontinence
403. Patient with pancreatic tumor best pain killer Morphia
404. A case of SIRS WBCS < 4000
405. 40 yr. old with profuse bleeding per rectum, by inv, hyper vascular area in transverse
colon by colonoscopy
A- Aortoenteric fistula B- Diverticulitis
406. Cancer prostate with Met's colon, liver mass CEA
407. CANCER prostate metastasizes to liver after 6-month Metachronous
408. Condyloma accuminata in anus & Patient known leukemia on chemotherapy Treatment??
A- Radio B- Resection with safety margin
409. liver is divided into Liver segments by Hepatic veins
410. scenario of seroma post-operative, how to manage??Percutaneous drainage
411. ECG changes, hypokalemia Flat t wave, prominent u wave
412. Bloody diarrhea, chronic abdominal pain, patient travelled to many countries, colonoscope
(circumferential inflammation from the rectum to the descending colon), negative bacterial
serology
A- Crohn's colitis B- Yersenia colitis C- UC D- diverticulitis
413. Severe epigastric pain, loss of liver dullness, stable patient
A- chronic DU B- rupture liver abscess.

PLEAZE, ANY NOTE OR CORRECTION OR ADDITIN SEND: [email protected]


‫نسألكم الدعاء بظهر الغيب‬
P a g e | 51
414. Child, inguinal swelling and pain after he played, scrotum normal?
A- complicated inguinal hernia B- sports hernia C- hematoma.
415. Same scenario, investigation? MRI.
416. Same scenario, cause?
A- tear in the posterior wall B- tear in the inferior epigastric artery.
417. Thigh puncture, blood loss, compression, bleeding doesn't stop, shocked, cold pulseless
limb?
A- duplex then surgery B- CTA then surgery C- surgery
418. Hernia not seen while lying but appears on standing:
A- colles sign B- grey turner sign C- carnet sign
419. Ileostomy through the rectus:
A- less hernia B- less prolapse C- continence.
420. Trauma, bilateral severe lung contusion, femur fracture, trauma to descending aorta,
mediastinal hematoma, stable:
A- aortic repair and clamping
B- aortic repair on femoral bypass
C- monitor vital signs in ICU
421. Adult female, chronic abdominal pain and diarrhea, positive family history, diagnosis:
A- Crohn's disease B- ulcerative colitis C- diverticular disease.
422. Ovarian mass found during appendectomy:
A- remove mass and preserve ovary
B- consent from first relative and oophorectomy !!
C- biopsy if D NOT PRESENT IN CHOSES D- leave it (the true answer )
423. Prognosis of acute pancreatitis: age (45 years), hypoxia (low), amylase (> 2000), amylase (>
400). Severe
424. Chronic pancreatitis, with image showing dilated pancreatic duct, alcoholic:
A- distal pancreatectomy B- total pancreatectomy
C- pancreatojejunostomy D- distal pancreatectomy and jejunostomy.
425. Image of swelling of the umbilicus then operative image showing quarry intestinal loop,
sure sign of intestinal necrosis:
A- absent arterial pulse B- increased serosal luster
B- increased peristalsis C- venous congestion.
426. Bleeding per rectum, sense of incomplete evacuation, PR free, lower abdominal pain:
A- cancer descending colon B- cancer rectum C- diverticulitis
427. Scenario, polydypsia, hypertension, hypernatremia, diagnosis? hyperaldosteronism

PLEAZE, ANY NOTE OR CORRECTION OR ADDITIN SEND: [email protected]


‫نسألكم الدعاء بظهر الغيب‬
P a g e | 52
428. Complicated UC (peritonitis), Hb 8, tachycardia, management?
A- proctocolectomy B- proctocolectomy with ileoanal pouch
C- colectomy D- proctocolectomy with Ileostomy.
429. Trauma, congested neck veins, muffled heart sounds, average blood pressure,
tachycardia, type of shock:
A- neurogenic, B- hypovolemic B- cardiogenic D- anaphylactic.
430. Trauma, tracheal shift, congested neck veins, massive hemothorax, first line treatment:
chest tube.
431. Abdominal compartment syndrome, pressure through urinary bladder is 28:
A- laparotomy and laparostomy B- laparotomy and drainage.
432. Hypotension after pancreatic surgery, cause: low cardiac output.
433. Strangulated hernia, query loop:
A- hot fomentation and 100% oxygen
B- resection and ileostomy
C- resection anastomosis.
434. Old age, chronic constipation, abdominal pain, colonoscope (image seems hyperemic),
physically free:
A- diverticulitis B- UC C- Crohn's disease D- laxative abuse
435. Problem in an instrument in multiple hospitals, report to whom:
A- ministry of health B- company C- health administrator
436. 70 years, neck mass 2 months, image, what happens if left: infection.
437. What used to prevent errors in hospitals?
A- efficiency B- equivalent C- patient safety
438. Old patient needs amputation, family refusing?
A- respect their decision B- report to the authorities
439. Diagnosed colonic adenocarcinoma, CT done but cannot stage?
A- colonoscope, B- MRI
440. Problems of long-standing omeprazole?
A- diarrhea is the most common complication
B- 10 times risk of osteoporosis
C- tell the patient nothing.
441. Soiling, sigmoidoscope normal, PR mucosal bulge on the left side at the tip of the finger
and indentation in the midline posterior, two images of pelvic MRI (mass on the left side at the
levator ani), no external opening, treatment?
A- Seton B- lay open from inside C- sealing by glue.

PLEAZE, ANY NOTE OR CORRECTION OR ADDITIN SEND: [email protected]


‫نسألكم الدعاء بظهر الغيب‬
P a g e | 53
442. Chronic epigastric pain not improved on PPI, management?
A- barium B- endoscope
443. Gastric cancer infiltrating musculosa mucosa? T1
444. Diverticulitis, pneumaturia? CT.
445. Lower abdominal pain, PR free, fever, best investigation? CT.
446. Prognostic factor for GIST (large mass in the stomach)?
A- mitotic figures B- size
447. Retro areolar mass, 3 x 4 cm, hard, irregular, serous discharge?
A- duct ectasia B- cancer breast
448. Image (breast mass 5 cm above nipple, hematoma, dimple, peau d'orange)?
A- hematoma B- fat necrosis.
449. Spurious diarrhea, weight loss, barium image (stop of dye at ascending colon)?
A- UC B- TB C- cancer colon
450. Greyish white exudate, thickened rubbery ileum, adherent normal bowel?
A- TB B- Crohn's
451. colitis, patient on cipro for 14 days, first management?
A- IV clindamycin B- stop cephalosporin
452. Severe agonizing pain relieved only by morphia (acute mesenteric ischemia), investigation
A- CT B- ERCP C- MRCP D- abdominal radiograph
453. Soiling 6 months, internal opening at the dentate line, no swelling, history of perianal
abscess drainage 6 months ago: anal fistula
454. Excessive sweeting in the hands and soles, first line of treatment
A- botulinum toxin injection B- sympathectomy
455. Trauma, massive hemothorax, shocked, pale, cold periphery: chest tube
456. Resident want to do a procedure in the clinic but he never did it before:
A- till the patient he cannot do it B- do it C- wait the consultant
457. Patient on TPN, then stopped and given blood, confusion: hypoglycemia
458. Most common complication after massive blood transfusion:
A- coagulopathy hypocalcemia hyperkalemia hyperthermia
B- coagulopathy hypocalcemia hyperkalemia hypothermia
C- coagulopathy hypocalcemia hyperkalemia hyperthermia.
459. Large nucleus and nucleoli in breast biopsy: simple mastectomy
460. Cancer rectum 5 cm from the anal verge, no LNs:
A- APR B- neoadjuvant CTH C- LAR D- adjuvant CRX.
461. Amino acid important for nutrition of mucosal cells:
A- arginine B- glutamine

PLEAZE, ANY NOTE OR CORRECTION OR ADDITIN SEND: [email protected]


‫نسألكم الدعاء بظهر الغيب‬
P a g e | 54
462. Origin of basal cell carcinoma: pluripotent stem cells
463. Gastroschisis associated with: intestinal atresia
464. Child, splenomegaly 4 cm below costal margin, both father and mother have blood
disease, jaundice, reticular cells, diagnosis:
A- sickle cell anemia B- ITP C- thalassemia D- spherocytosis
465. Chronic epigastric pain relieved by bletching, esophagitis, fundus of the stomach near
esophagus, management:
A- Nissen fundoplication B- lap fundoplication C- anatomical repair
466. Hirschsprung disease, most association: Trypanosoma cruzi
467. 70 years, smoking 2 packs per day for 50 years, nocturia, difficult micturition, inguinal
hernia, strongest risk factor for hernia:
A- old age B- smoking C- PBH
468. Alcoholic, severe epigastric pain, jaundice, 3 attacks of black stool, fluid thrill:
A- chronic DU!! B- cancer stomach C- chronic pancreatitis.
469. Characteristics of mucinous cystic neoplasm of the pancreas: high CEA in cyst fluid
470. Pancreatic cyst, low amylase, high glycogen in cyst fluid: serous cystadenoma
471. Cyst in main pancreatic duct, small cysts or papilloma in distal pancreatic duct branches,
management:
A- ERCP and stent B- distal pancreatectomy
C- total pancreatectomy D- pancreatojejunostomy.
472. Blood pressure 80/60, HR 110, RR 18, grade of shock: second
473. Best resuscitation fluid for trauma
A- blood B- isotonic saline ‫المفروض‬RINGER
474. TEF case, distressed, NGT arrested in chest: continuous suction
475. Colo vesical fistula due to diverticular disease, urine culture showed E. coli: CT
476. Ulcer 12 cm from anal verge, polyp in the ascending colon, colonoscope done, staging:
MRI CT
477. Ranson criteria, the score means:
A- severe pancreatitis B- mortality rate < 5%.
478. Rectal foreign body, tear involving half rectal circumference, 6-7 cm from anal verge,
laparotomy:
A-proctectomy and coloanal anastomosis B- direct repair and colostomy
479. Chronic peptic ulcer, vomiting, succussion splash, first management: NGT decompression.
480. Child with subdural Hge, father not present:
A- consent from nearest relative B- doctor sign consent C- wait his father.
481. Undescended testis, diagnosis: MRI (no laparoscope).

PLEAZE, ANY NOTE OR CORRECTION OR ADDITIN SEND: [email protected]


‫نسألكم الدعاء بظهر الغيب‬
P a g e | 55
482. IO, laparotomy, resection anastomosis, no peritonitis or spillage, type of wound:
A- clean B- clean contaminated C- contaminated.
483. Infant, severe crying, firm hot tender bluish inguinal mass, normal groin:
A- torsion testis B- irreducible inguinal hernia.
484. Hypothyroidism (high TSH, normal T3 and T4) and proptosis, initial cause of proptosis:
A- stimulatory anti TSH B- inhibitory anti TSH
C- T lymphocytes C- B lymphocytes.
485. Thyroidectomy and lost high pitched sound:
A- recurrent laryngeal nerve injury B- superior laryngeal nerve injury
486. Ovarian cyst surgery, mild wound pain and discharge, fever 38, WBCs 12, what analgesic
A- epidural B- NSAID C- IV analgesic D- oral paracetamol.
487. Cholecystectomy 10 months ago, 2 months swelling 5 cm below xiphisternum, defect 1 cm
in linea alba
A- fatty hernia B- port hernia
488. Same scenario, port hernia, management:
A- laparotomy B- laparoscopy C- CT
489. History of spine trauma at T6, paraplegia, cause of urine retention:
A- detrusor instability B- detrusor paralysis C- detrusor dyssynergism
490. Flail chest, sucking chest wound, fracture 8, 9, 10, 11, initial management:
A- intubation B- three side dressing (Helmich valve manauver for 3 sided dressing)
C- chest tube through the wound, analgesics.
491. Organism that was considered commensal but now become pathogenic: staph epidermidis
492. Old age, fall 2 weeks ago, disturbed level of consciousness, CT concave frontal lesion,
diagnosis:
A- axonal brain injury B- cope counter cope injury C- neglected subdural hematoma
493. Old age, submental swelling that moves with protrusion of the tongue but not with
deglutition (? ranula), fate
A- infection B- malignancy.
494. Pulsatile abdominal mass, hypotension (80/60):
A- sonar then OR B- CTA then OR
C- OR D- fluid resuscitation then reassesses.
495. Appendectomy, after 3 days right upper abdominal pain, shoulder pain, fever,leukocytosis:
A- laparotomy B- CT C-X RAY
496. Ileostomy 3 months for a sigmoid anastomosis, normal electrolytes, fluid maintenance?
A- saline B- DNS C- ringer

PLEAZE, ANY NOTE OR CORRECTION OR ADDITIN SEND: [email protected]


‫نسألكم الدعاء بظهر الغيب‬
P a g e | 56
497. Diabetic patient, 45 years, generalized abdominal pain, vomiting, high glucose, negative
glucose and ketone in urine, management:
A- insulin B- IV fluids C- antibiotics
498. Female, abdominal pain, radiate to medial aspect of the thigh till the knee, hypotension,
WBCs 18, management:
A- MRI B- CT C- laparotomy
499. Stab chest, skin and SC tissue 2 cm, right 8th intercostal space, stable, conscious,
management:
A- conservative B- thoracotomy C- thoracostomy.
500. Image of sigmoid volvulus diagnosis:
A- cecal volvulus B- sigmoid volvulus
501. 3 days post appendectomy, discharge, fever, leukocytosis, initial management: culture
andsensitivity.
502. Consultant transect CBD for the third time, what to do: report in the file.
503. Hormone that is secreted in response to renal hypoperfusion: aldosterone.
504. Scenario hyperparathyroidism, preoperative investigation: sestamebi scan.
505. Watery diarrhea, hypokalemia, mass in the pancreas: VIPoma.
506. Piles, first degree, proper management: high fiber diet…
507. Third degree piles (reduced manually), management: Surgery
508. On screening, 100 colonic polyps with cancer colon, responsible gene:
A- tumor suppressor gene B- mismatch repair gene
509. Patient will undergo a major cardiothoracic surgery with 10% success rate, doctor think
that patient will refuse surgery, not doing the operation will harm the patient:
A- ask a colleague to do the surgery
B- do surgery with no consent
C- inform the patient and take consent
510. Chronic gastritis due to H pylori? type B
511. ER doctor will end his duty, there is a patient in the ER, his colleague has not arrived
A- stabilize the patient till his colleague comes
B- leave the patient until his colleague comes
C- call his colleague to come.
512. Chronic liver disease, ascites, low albumin, high INR, high bilirubin, normal enzymes, mass
3 cm, how to determine severity of liver disease
A- MELD score B- Child-Paugh score.
513. Abdominal compartment syndrome, low blood pressure, low Hb, low PO2, high creatinine,
K 6.5, most concerning: cardiac arrest.

PLEAZE, ANY NOTE OR CORRECTION OR ADDITIN SEND: [email protected]


‫نسألكم الدعاء بظهر الغيب‬
P a g e | 57
514. Female need bariatric surgery, hesitated or incompetent:
A- do it with a colleague B- husband sign the consent
C- consult psychiatrics D- do surgery.
515. CT axial, aortic aneurysm, pain in left lower back left lower abdomen and left testis, blood
pressure average:
A- diverticulitis B- appendicitis C- leak inf aortic aneurysm
516. Trauma in middle chest, left sided emphysema, diagnosed as gastric perforation, left sided
chest tube and no improvement:
A- esophagoscope B- bronchoscope
517. Smoker, hemoptysis, cavity in the left upper lobe of lung:
A- CT B- rigid bronchoscope under anesthesia
C- noradrenaline injection C- conservative
518. Surgeon marked the limb to be amputated before surgery, wrong side draped, surgeon
discovered the problem and corrected it how to prevent:
A- marked with permanent pen
B- take patient information before surgery
C- time out during surgery.
519. Antibiotic is prescribed in cases of acute appendicitis for how long??

A- 3-5 days B- 5-7 days C- 7-10days D- less2 days


Nb acute app less than 2 -- If perforated 3 to 5 d
520. patient with congestive heart failure, lower abdominal pain and vomiting, with moderate
ascites, ascitic fluid examination show low albumin, normal WBC, how to manage??
A- 2nd generation cephalosporin B- paracentesis
C- exploratory laparotomy D- DPL
521. A case of acute appendicitis, underwent open appendectomy, developed discharge from
the wound, what is the organism?? Staph arus
522. How to minimize surgical site infection, what do you recommend?
A- give Iv antibiotic 1 hr. before surgery B- preoperative shaving
523. Patient with cancer tongue, modified neck dissection type 1 on right side, modified neck
dissection type 2 on left side, patient developed vomiting, drowsy and confused, headache,
duplex show thrombosis of left IJV what will you do?
A- neurosurgery consultation B- CT
C- reexploration and thrombectomy D- anticoagulant
524. Mechanism of tissue expansion??
A- mitosis B- connective tissue disruption

PLEAZE, ANY NOTE OR CORRECTION OR ADDITIN SEND: [email protected]


‫نسألكم الدعاء بظهر الغيب‬
P a g e | 58
525. Female patient with sacral sinus with discharge, bimanual examination discharge caseus
material, what is the diagnosis??
A- pilonidal sinus B- anal dermoid C- post anal dermoid D- coxgeal teratoma
526. old age male patient falls on the bathroom 1 week back, he presented with neurological
manifestation, CT brain was done showing concave lesion, what is the diagnosis??
A- concussion B- subdural hematoma
527. child was hit by a ball to his head, loss of consciousness, then regain his consciousness,
he deteriorated again, Diagnosis?? Extradural Hge
528. scenario, of mesenteric ischemia, thumb print sign, Diagnosis?? ischemic colitis
529. pulsatile abdominal mass, Bl. pr 80/40, management??
A- Exploration B- Us C- CT D- resuscitation and reassessment
530. patient admitted to hospital, mostly for fracture few days later, he developed retrosternal
chest pain, diaphoresis, confusion, Diagnosis??
A- MI B- PE C- PUD
531. During lap cholecystectomy, PIC of GB, hemorrhagic, with adhesions, and, loss of normal
anatomy, what will you do??
A- partial cholecystectomy B- open cholecystectomy
532. you are working in tertiary hospital, you received 75 cases, one of these case 75 yrs. old
ale pt., abdomen tense and rigid, fracture pelvis, absent distal pulse, wound in thigh, BP 70/40
RR 40 triage this patient?
A- urgent B- immediate C- delayed D- dead
533. scenario of breast cyst, aspiration done, but not completely collapsed, what will you do??
biopsy
534. case of 35 yr. old female breast mass 4*5 cm, in the lateral upper quadrant, with palpable
LN, what will you do??
A- mammogram B- MRI C- True cut biopsy
535. case of 35 yr. old female breast mass 4*5 cm, in the lateral upper quadrant, with palpable
LN, what is most confirmatory investigation??
A- mammogram B- MRI C- True cut biopsy
536. Male patient has history of leukemia presented with features of anal cancer underwent
surgical excision of the tumor, margin was not free what will you do??
A- Excision with free margin B- Radiotherapy C- Wide local excision and flap
537. 85 yr. old female with symptoms of acute cholecystitis, without stones, high WBCS, how
to manage??
A- Lap chole B- Open chole
C- Cholecystostomy D- Medical management

PLEAZE, ANY NOTE OR CORRECTION OR ADDITIN SEND: [email protected]


‫نسألكم الدعاء بظهر الغيب‬
P a g e | 59
538. A case of rupture AAA, blood pressure 80/40 what will you do??
A- Us B- Ct C- Urgent exploration D- Resuscitation and reassess
539. Diabetic patient presented with foul smelling from dorsal foot wound associated with
crepitus extended to mid-thigh, management?
A- Amputation B- Debridement C- Antibiotic
540. Female pt. diabetic with past history of DVT 25 yrs. back, and underwent excision of
malignant melanoma of 2 yrs. back, presented with painful swelling of the leg showing ugly
limb with crustations, what is the diagnosis?? ‫صورة‬
A- Hemosiderin B- Necrobiosis lipodica diabeticorum
541. The best investigation for pancreatic divisum??
A- Ct B- Us C- ERCP D- MRCP
542. During laparoscopic cholecystectomy, complete transection of CBD just
above the duodenum, how to manage this complication??
A- Choledochojejonostomy
B- Choledochodudenostomy
C- Repair over T-tube
543. Safety margin of SCC??
A- 3 mm B- 4mm C- 5mm D- 10mm
544. Origin of SCC Keratinizing cell layer of the epidermis
545. Patient with inflammatory swelling with fluctuation, over the dorsum of foot, Doppler US
showing all vessels triphasic except dorsalis pedis is monophasic, what will you do??
A- Debridement B- Amputation
546. 8 yr. old boy with intussusception ,‫ صورة‬what is the most common cause of this?
A- Payers patches enlargement B- Benign polyp
C- Cancer D- Congenital band
‫السؤال كان كدا واالختيارات ملهاش عالقة ببعض‬management?? appropriate the is What 549.
A- Systemic anticoagulant
B- I & D of perineal collection
C- Analgesic and antipyretic
550. 8 years old boy presented with stap neck in the middle and lateral side of neck, patient is
stable, but has expanding hematoma, management??
A- Exploration B- Embolization C- Angiography D- Conservative
551. Capacitate coupling, cause of thermal injury??
A- Electric current transfer to organ touched by the metal trocher
B- Sandwiching

PLEAZE, ANY NOTE OR CORRECTION OR ADDITIN SEND: [email protected]


‫نسألكم الدعاء بظهر الغيب‬
P a g e | 60
552. Scenario of a case doctor prescribed clindamycin for patient, later culture shows
resistant to the antibiotic, doctor was not informed about the result he continued same a drug
and patient died from sepsis, how to minimize this??
A- Drug code B- Reco C- Poor system communication
‫صورة‬ ‫مكرر السؤال مرتين مع اختالف اإلجابات‬rectum MRI 553.
554. 23 yrs. old female presented with symptoms of Gerd with worsening dysphagia, next
step??
A- endoscopy B- manometry C- ph monitoring
555. patient underwent total gastrectomy, at 4th day post op. patient has leakage from
anastomosis, no sepsis, what you will manage??
A- conservative B- omental patch
C- endoscopic stenting D- redo of anastomosis
556. female 56 yrs. old, loss of weight, hx of constipation, x ray abdomen show air under
diaphragm, what the surgeon will do for diagnosis CT
557. a case of male patient with signs of achalasia, small tapering, and proximal esophageal
dilatation, how to manage??
Dx. manometry
initial treatment?? Balloon dilatation
best treatment?? Heller myotomy
((best is lap myotomy with dor partial anterior fundoplication)) in another question
558. Female patient 4 times CS presented with incisional hernia, what is the most appropriate
factor for management plan??
A- Size of the defect B- Location C- Content of the hernia
559. Same scenario, what is the most appropriate investigation for incisional hernia??
A- US B- Laparoscopy C- CT
560. Female Patient underwent lap chole 10 months back, presented with irreducible
epigastric mass 2*3 cm with discomfort no impulse on cough, what is the most appropriate
investigation?
A- Ct B- Us C- Laparoscopy
561. Scenario of list of two cases for cochlear implant, one will do left side cochlear implant
and the other will do bilateral, the turn now for the case of bilateral, but the patient was
cancelled for some reasons and no body informed the surgeon and they pushed the one for
unilateral left side and the surgeon opened on the right side, then they discovered this mistake
intraoperatively, where is the defect??
A- Booking defect B- Marking defect

PLEAZE, ANY NOTE OR CORRECTION OR ADDITIN SEND: [email protected]


‫نسألكم الدعاء بظهر الغيب‬
P a g e | 61
562. Scenario of surgeon did site mark on left foot will go for amputation, but the nurse
sterilized and prepared the right, the surgeon remembered the correct site from previous
marking. Near miss
563. Which of the following prevent confuse to the site of surgery??
A- Time out
B- Operative technician does side marking
C- Senior resident does side marking
564. How to train the junior resident?
A- Prepare curriculum B- Round meetings
565. 85 yrs. old female patient diabetic, with acute cholecystitis, how to manage ?? without any
parameters
A- Open cholecystectomy B- Medical ttt
C- Laparoscopic cholecystectomy D- Cholecystostomy
566. Patient underwent lap cholecystectomy, 2nd day post op, drain show 100 cc bile,
abdomen is soft and lax, what is next??
A- Us B- MRCP C- ERCP D- HIDA scan
567. Male patient, COPD underwent anterior resection, epidural catheter was inserted for pain
management, 8 hrs. post-operative developed fever, what do you think, the cause of fever?
A- Atelectasis B- Body reaction to surgery
C- Pneumonia D- Epidural abscess
568. Male patient with cancer rectosigmoid started to develop SOB of 2 weeks, what is the
cause??
A- Anxiety B- Metastasis C- Aspiration
569. Patient COPD, has fracture neck femur, pulmonary consultation was done and the doctor
prescribed aggressive treatment for 4 days, patient developed pulmonary embolism, what is
the cause??
A- Immobilization B- Steroids
570. Patient with flank pain, bone pain, with hard neck mass, with hypercalcemia, what is the
most likely diagnosis??
A- Parathyroid carcinoma
B- Metastasis from anaplastic carcinoma of thyroid C-primary hyperparathyrodism
571. Patient with midline firm solid neck nodule, move with deglution and swallowing, which of
the following investigation confirm the diagnosis??
A- US B- TSH C- CT D- Radioactive iodine
572. 57 yrs. old female with pic of thyroglossal cyst ‫ صورة‬appeared 2 month ago, move with
deglution only, patient refused surgery, what is the fate of this swelling?

PLEAZE, ANY NOTE OR CORRECTION OR ADDITIN SEND: [email protected]


‫نسألكم الدعاء بظهر الغيب‬
P a g e | 62
A- Infection B- Malignancy C- Laryngeal compression
D- No complication D- Spontaneous resolution
573. Neck mass, hard, fixed suffering from progressive hoarseness of voice, appeared 10
weeks ago, what is the best diagnostic investigation??
A- FNAC B- Core biopsy C- Isthmectomy
574. Patient admitted for treatment of pulmonary embolism, received multiple injections
patient suffered from difficulty of sleeping and become agitated, and the staff loss the control,
what is the cause??
A- Hospital delirium B- Intracranial he
575. Newborn with congenital hernia, parents thought medical advice for their infant in the
clinic, what is the management?
A- Expeditious elective surgery
B- Elective surgery at age of 3 m
C- Wait until body weight be 10 kg
D- After 2 yrs.
576. What is the most reliable investigation in obstructive jaundice?
A- Bilirubin B- Alkaline phosphatase
C- Case of VIPoma D- Case of glucagonoma
577. Scenario of features of insulinoma. patient underwent CT but not confirmatory for the
diagnosis, what is the other option investigation for diagnosis? EUS
578. Patient with MEN2, has medullary thyroid carcinoma, what is the most common
associated diseases??
A- Pheochromocytoma B- hyperparathyroidism
579. CT of rupture spleen, and asked for diagnosis ‫صورة‬
580. CT of GIST, asked for diagnosis. ‫صورة‬
581. Patient with abdominal trauma, presented with generalized abdominal pain, on exam,
patient had guarding, ecchymosis in the flanks, high amylase, CT done showing tear but there
is no duct injury, what will you do??
A- Observation B- External drainage
582. Scenario of duodenal hematoma, asked for treatment?? Conserve
583. A case of rupture diaphragm, underwent thoracoscopic repair, after 4 hrs. he developed
tachycardia dyspnea, cyanosis, what happened??
A- Pneumothorax B- Air embolism
584. Scenario for patient of stab chest and developed cardiac tamponade

PLEAZE, ANY NOTE OR CORRECTION OR ADDITIN SEND: [email protected]


‫نسألكم الدعاء بظهر الغيب‬
P a g e | 63
585. Female patient during ERCP for stone extraction, failed due to perforation, and she
developed hematemesis and melena and mild tenderness BP 80/50, how to manage??
A- Endoscopy repeat B- Laparoscopy C- Celiac angiography
586. Same last scenario and asked about the cause of epigastric tenderness??
A- Duodenal perforation b- Pancreatic injury c- Stone impaction
587. Picture of DIULAFOY lesion ‫صورة‬diagnosis??
588. Female patient with duodenal ulcer, shocked, received blood, what will you do after
resuscitation?
A- Endoscopy and cauterize bleeder
B- Surgery and sewing of duodenum
C- Laparoscopic
D- Celiac angiography
589. A case of major case of burn admitted to burn ICU, patient developed massive bleeding
per rectum, what is the initial step after resuscitation??
A- NGT B- Laparoscopy C- Endoscopy D- Colonoscopy
590. A case of huge diverticular abscess you did percutaneous drainage, when will you do the
definitive surgery??
A- After 6 wks. B- After 8 wks. C- After 12 wks.
591. A case of metabolic acidosis, low HCO3, with intestinal fistula, what is the cause of this
condition??
A- Intestinal fistula B- Hypovolemic shock
592. Sever e hypernatremia, resuscitation or treatment??
A- Hypotonic saline B- Isotonic saline C- Starch
593 Scenario of MRI of anal fistula without external opening?? ‫سؤالين مكررين باختيارات مختلفة‬
A- Fistula B- Submucous abscess C- Pelvi-rectal abscess
594. A case of Colo vesical fistula, what is the best investigation??
A- CT abdomen and pelvis B- Double contrast barium
595. What is the indication of neoadjuvant chemotherapy in case of cancer rectum 5 cm from
anal verge, no lymph node, high grade adenocarcinoma??
A- Mesorectum invasion B- Near sphincter
C- 5cm from anal verge D- High grade tumor
596. What is the indication of neoadjuvant chemotherapy in case of cancer colon??
Pericolic fat infiltration
597. Anal carcinoma, main treatment??
A- Radiotherapy B- Chemoradiotherapy C- Surgery

PLEAZE, ANY NOTE OR CORRECTION OR ADDITIN SEND: [email protected]


‫نسألكم الدعاء بظهر الغيب‬
P a g e | 64
598. Patient with leukemia, underwent excision of anal mass, excision was not complete with
positive margins, what will you do?
A- Reexcision till free margin
B- Wide local excision and graft
C- Radiotherapy
599. In case of hypovolemic shock, in which organ autoregulation of its own blood supply?
A- Kidney B- Brain C- Intestine D- Muscle
600. What is true regard acute cholecystitis??
A- 75% associated with stone
B- Positive with organism
C- HIDA scan normal even with acute cholecystitis
601. A case of female patient with myeloproliferative disease, presented with severe left
hypochondrial pain referred to the shoulder, blood film normal, what will you do??
A- Lap splenectomy B- Open splenectomy C- Endovascular stent D- Conserve
602. A case of necrotizing fasciitis or gas gangrene, culture show gram positive rods, what is
the best antibiotic you will give?
A- Penicillin B- Tetracycline C- Clindamycin
603. Female patient developed pulmonary embolism what is the best investigation??
A- ECG B- X ray C- ABG
604. What is the finding with fine needle aspiration from mucinous cystadenoma??
A- High CEA B- High amylase C- High glucagon
605. Patient with GERD not responding to medical treatment, which surgery will do??
A- Nissen total fundoplication B- Partial fundoplication
C- Dorr's fundoplication D- Trouped fundoplication
D- Belsy fundoplication
606. Obese female patient, uncontrolled diabetic with GERD, what is the best bariatric surgery
for her?
A- Reux en y bypass B- Gastric sleeve
C- Adjustable Gastric band D- Gastric balloon
607. Obturator hernia, after reduction of the intestinal loop, small segment was necrotic from
the site of constricting ring, what is type of necrosis??
A- Hemorrhagic B- Liquefactive C- Coagulative
608. Patient with symptoms GERD, on diet and lose 6 kg from his body weight, developed
worsening of symptoms and develop chest infection which symptom should take care of it?
A- Dysphagia B- Recurrent chest infection C- Weight loss

PLEAZE, ANY NOTE OR CORRECTION OR ADDITIN SEND: [email protected]


‫نسألكم الدعاء بظهر الغيب‬
P a g e | 65
609. 50 yrs. old female patient with history of constipation and weight loss, with x ray showing
air under diaphragm, what next investigation is more helpful for diagnosis??
A- CT B- Gastrographin C- US D- MRI
610. SCENARIO of a child with subdural Hge, need to enter the OR, what will you do?
A- Take consent from father by phone
B- Take consent from the nearest relative
C- Take another advise from your colleague
611. What is contraindication for anti-reflux measures??
A- Motility disorder B- Barret esophagus
612. Scenario of surgeon ligate CBD 3 times during lap chole, what will you do?
A- Write on operative notes what happened
613. 35 yr. old male presented with bilateral crush injury both thighs with fracture femur, what
should be in mind regard this patient?
A- Hge B- Myoglobinuria C- Hypotension D- Fracture
614. Patient with hyperkalemia k 6.5, what is the risk for this patient? Cardiac arrest
615. The most serious problem to give parenteral nutrition in central line?
A- Infection B- Electrolyte imbalance C- Intestinal atrophy
616. Terminal ilium with grayish exudate, with adhesion, firm and rubbery, diagnosis??
A- Crhon's disease!! B- TB C- UC
617. A case of rectal cancer invading serosa with +ve 3 pelvic lymph node, what is the
staging??
A- Stage A B- Stage B C- Stage C1 D- Stage C2
618. A case of during inguinal hernia repair, you find Mickell's diverticulum, how to manage??
A- Hernia repair only B- Hernia repair with mesh
C- Hernia repair without mesh D- Resection anastomosis with hernia repair
diverticulectomy and hernia repair
619. what is the best indicator of resuscitation in case of hypovolemic shock??
A- CVP B- UOP C- Peripheral oxygen saturation
620. A case presented with intestinal obstruction, during laparotomy, you found amass in the
in the rectosigmoid area what will you do??
A- Radical left hemicolectomy B- DEFUNCTIONING Colostomy
C- Biopsy and exteriorization D- cecostomy
621. female patient with dysphagia and chest pain of 6 months, stomach is up and parallel to
the esophagus, LES is in place what is the diagnosis?
rolling hiatus hernia complicated with gastric volvulus

PLEAZE, ANY NOTE OR CORRECTION OR ADDITIN SEND: [email protected]


‫نسألكم الدعاء بظهر الغيب‬
P a g e | 66
622. same scenario gastric volvulus, management?
A- anatomical repair B- fundoplication
623. a case of malignant tumor in the bone associated with onion peel sign, diagnosis??
A- Ewing sarcoma B- osteo sarcoma
624. A patient aged 40 years is diagnosed e a unilateral inguinal reducible hernia. On further
examination, nothing else is detected. What should be done? ELLICTIVE HERNIAL REPAIR.
625. Elderly came e subdural hematoma e signs of lateralization imaging revealed 3 mm
shifting. His GCS – 7/15 then he intubated and resuscitated. What to do next?
A-IV mannitol. B - CRANIOTOMY
626. Percentage of having GERD after sleeve? 10 %
627. A 59 y women undergoes a total thyroidectomy. The patient was extubated and
immediately developed shortness of breath and respiratory distress. Examination revealed
that both vocal cords are in semi closed position. How will you secure the airways?
Tracheostomy.
(The patient has a bilateral injury that compromises the airway. It usually manifesting immediately
post op. and potentially requiring tracheostomy. Cricothyroidotomy is an ER procedure indicated in
angioedema, foreign body in upper airway, sever oropharyngeal / nasal bleeding.)
628. Confirmed COVID 19 pt. developed ARDS and required mechanical ventilation. Which of
the following is the most appropriate setting?
Low tidal volume ventilation (VT 4-8 ml/kg body weight)
629. A confirmed COVID 19 pt. required endotracheal intubation for ventilator support. Who is
the most appropriate person to perform intubation? Most experienced person.
630. Ventilator associated pneumonia (VAP) was encountered in several patients in the
medical intensive care unit. Which of the following is the most effective and the most
straightforward method to prevent transmission of microbes?
Wash hands between the patients
631. In light of the COVID 19 pandemic, what is the WHO recommendation for washing hands
when only using water and soap? 40 -60 seconds.
632. When screening is in low prevalence community, what kind of result do you expect?
False positive.
633. A 40-year man with a 2-year history of abdominal hernia presented with symptoms of
bowel obstruction. The skin over hernia is red in color. What complication has the patient
developed? Strangulation.
634. Which of the following is the most common site of tubule-villous adenoma colored pink?
Rectum.

PLEAZE, ANY NOTE OR CORRECTION OR ADDITIN SEND: [email protected]


‫نسألكم الدعاء بظهر الغيب‬
P a g e | 67
635. A 60 y Pt, came e the history of hashimoto disease who developed a huge goiter. Biopsy
of the thyroid gland was taken which showed malignant thyroid cancer. Which type of
carcinoma could this be? Lymphoma.
636. A couple comes to asking about surrogate pregnancy. You don’t know much about this
topic. What should you do? Give a follow up appointment and study the case further
637. When can cholecystectomy be done in a pregnant? 2nd trimester
638. A primigravida pt., was trying to conceive for 10 years. She finally conceived and reach
10 weeks but later on developed a complete abortion. This trauma emotionally affected her
and her husband a lot. The doctor tried to bring the couple together by addressing their issue
and emotions. What communication skills did the doctor practice here? Reflection.
639. Campaigning in the community to decrease risk of hypertension is considered as?
Primary.
640. A 30-year-old lady came to the A/E department wearing inappropriate clothes and
speaking in a language not known in a seductive way. What should you do?
Call a nurse who speaks the same language.
641. A pregnant woman went into labor. On examination, fetal distress was seen. She was
counseled for surgery but she refused bluntly. What appropriate action should be taken?
Allow vaginal delivery based on her wishes.
642. A 40-yrs female presented with lower limb edema. On examination, the distal pulses were
intact. Only dilation of greater saphenous vein’s tributaries was seen. Which of the following is
the investigation of choice? Venous duplex ultrasound.
643. A 55-yrs pt., e Cushing's syndrome is admitted for adrenalectomy. Which drugs should be
given? Pre-op corticosteroids.
644. During a forceps delivery of a baby, the LT stylomastoid foramen was injured by the
forceps blade. Which of the following is going to occur due to this?
Loss of taste sensation of the anterior 2/3 of the tongue.
645. Membranous urethra injury management? Retro pubic catheter.
646. Pt. e neck injury in zone 1, vitally stable, what is the next? Neck exploration.!!
647. A year back Hx of trauma and patient is paraplegic came now e SOB and cough
diagnosed as PE e DVT and pt. is HD unstable? Thrombolysis.
648. Pt. came e symptoms of TIA. What is the next step? CAROTID DUPLEX ,,,,BEST MRA
649. Pt. came e his shoulder flat, arm is adducted and internally rotated, what is the diagnosis?
Post. Dislocation.
650. A 22-yrs old male was brought to the emergency duo to sever femoral pain after trauma
to the leg. X ray was done and shows a displaced fracture of the femoral shaft with 30 %

PLEAZE, ANY NOTE OR CORRECTION OR ADDITIN SEND: [email protected]


‫نسألكم الدعاء بظهر الغيب‬
P a g e | 68
angulation. What is your next step in management?
Open reduction e nail (interlocking intramedullary nailing)
651. When should post splenectomy pneumococcal vaccination give?
A-2 weeks after splenectomy. B-2- 3 weeks prior elective op.
652. A 43-yrs man is brought to the ER after a stab wound to the abdomen. On arrival he is
alert, pulse is 110, RR 16/m, BP 112/80. Examination shows a stab wound about 2 cm long near
the umbilicus to the left e omentum eviscerated from the wound. What is your next step?
A-Exploratory laparotomy. B-CT C –OBSERVATION
653. A 35-yrs female pt. presented to the ER e a 2 days history of epigastric pain. 2 weeks ago,
she had acute biliary pancreatitis. CT was done and shows 4 cm peripancreatic fluid. There is
no necrotic tissue. What is your next step? Conservative management.
NB
 Internal drainage  if not resolved spontaneously and in uncomplicated pseudo cysts
requiring intervention.
 External drainage  if infected and without a mature wall.
 Pseudo cysts < 6 cm and present for < 6 weeksusually resolve spontaneously.
 Excision  bleeding, systemic sepsis, concern for malignancy.
654. A 26-yrs old man e epigastric pain and nausea was admitted after a CT of the abdomen
revealed finding suggestive of entero-enteric intussusception. He treated conservatively. On
day 4 he looks ill and has diffuse abdominal pain. What is the next step? Surgical reduction.
655. (see photo of the Q) he has HTN, DM & LT hemiplegia. HR is 90, BP is 125/84. On
examination, distended abdomen e mild tenderness. Bowel sounds are decreased. A standing
X ray abdomen is shown. Which of the following is the most likely cause of this finding?

Cecal volvulus.
656. An 80-yrs old male pt. brought to the ER. He was eating when suddenly started to
complain of severe abdominal pain and he lost his consciousness. The temp was 38.5, RR is
23b/m, pulse is 120/m, Bp 100/70, and examination shows a rigid and tender abdomen. What is
the next step? Exploratory laparotomy. (Acute mesenteric ischemia).

PLEAZE, ANY NOTE OR CORRECTION OR ADDITIN SEND: [email protected]


‫نسألكم الدعاء بظهر الغيب‬
P a g e | 69
657. A 62-yrs old man comes to the physician to follow up after colectomy. He was diagnosed
e colorectal cancer and undergo a total colectomy. When he will follow up CEA level?
Every 3 months for 2 years. Then every 6 months for additional 3 years.
NB
 Endorectal ultrasound  every 4 months for 3 years.
 Chest abdomen CT every 6 - 12 months for 5 years.
 Colonoscopy  every 1 year.
658. An 80-yrs old male presented e RLQ abdominal pain, laparoscopic appendectomy was
done. On gross examination, a mass of 8 mm detected at the mid appendix. When should you
follow up with this Pt? Colonoscopy after 6 weeks. (not 6 months)
NB >>> CT is the imaging of choice for staging and determine the extension.
659. A 59-yrs old male presented e hematochezia. A colonoscopy was done and a polyp was
seen at the sigmoid. Polypectomy was done and sent for histopathology. The result of
histopathology come with differentiated adenocarcinoma with free margins from cancer. What
is the next step? Observation.
660. Two patients come to you, one of them has a mutation carrier for familial adenomatous
polyposis and the other a mutation carrier for Peutz-Jughers syndrome. What is the best
screening time for cancer?
Colonoscopy annually continued lifelong for FAP.
Colonoscopy at age 8 every 3 years for PJS.
661. A 70-yrs old man presented e abdominal pain radiating to his RT flank. He has
hypertension and coronary heart disease. He has smoked a pack of cigarettes daily for 30
years. Pulse 96b/m. BP 125/80. O2 saturation 95%. Examination shows a painful pulsatile
abdominal mass. What is the appropriate next step in management?
CT angiography (suspected symptomatic AAA)
NB >>> Suspected asymptomatic AAA U/S abdomen.
662. A 36-y old man is brought to ER e a stab wound to the abdomen. Pulse 110/m. RR 16/m. BP
112/70. The wound was explored and there is anterior abdominal fascia penetration. What is
the next step? CT abdomen. (Stable pt.)
663. A 60 yrs old man comes to the clinic because he is vomiting blood. He was diagnosed
recently e pancreatitis. The pt. stabilized and endoscopy was done and revealed gastric
fundus bleeding with dilated veins, sclerotherapy was done. Duplex abdominal ultrasound
showed splenic vein thrombosis. What is your management? Splenectomy
664. A 19-yrs old male presented e swelling in his arm. Imaging shows a tumor in the triceps.
How can be biopsied? Core needle biopsy

PLEAZE, ANY NOTE OR CORRECTION OR ADDITIN SEND: [email protected]


‫نسألكم الدعاء بظهر الغيب‬
P a g e | 70
665. A 70 yrs. old women comes for evaluation of unusual bleeding for the past 2 weeks.
Endometrial biopsy from the body of the uterus shows endometrial adenocarcinoma. Which of
the following the most likely primary site of metastasis? Body of the uterusExternal iliac
NB >> Middle third of the vagina & cervix  internal iliac.
Ovaries  para-aortic.
666. A 59 yrs. old women undergoes a total thyroidectomy. The pt. was extubated and
immediately developed shortness of breath and respiratory distress. Examination reveals that
both vocal cords are in a semi closed position. How will you secure the airway?
Tracheostomy tube
667. An 87-y old man is brought to the ER because of headache, nausea, and vomiting for 24 h.
two weeks ago, he falls into the bathroom and hit his head. His GCS 7/14. On examination,
decreased muscle strength in the RT leg and arm. CT of the head without contrast shows a
high density, 13 mm concentric collection. The pt. was intubated and resuscitated. What is
your next step in management? Craniotomy and surgical evacuation
668. A 40-yrs old pt. presented to the physician. The pt. diagnosed with lumbosacral
radiculopathy at S1 (sciatica). What do you think the complaint of the patient was?
Pain in the buttock, back of the calf, and side of the foot
669. Sulpha side effect? Kernicterus
670. Female with abd mass history of cesarian section planed for laparoscopic repair hernia,
most common complication:
A- Pain B- Infection C- Adhesion formation D- Cosmetic
671. Pt with gastric band for obesity 2y ago present with vomiting and distention, ttt?
A- endoscopic B-Laparoscopic C- Laparotomy
672. Old pt. with complete rectal prolapse, ttt? altimeter
673. Longitudinal tear in CBD during laparoscopic Cholecystectomy management??
A- primary closure with suture B- ERCP and stent via the rent
C- clip of the rent and drain D- closure over T- tube
674. Myeloproliferative huge spleen with severe Lt hypochondrial pain for diagnosis??
A- Laparoscopy B- MRI C- CT with Contrast
675. Hip metastasis after ttt of breast cancer management of pain??
A- radiotherapy B- opioid C-Bisphosphonate's infusion
676. Bleeding per rectum, colonoscopy and barium are –ve except for hyper vascular area in
transverse colon most common??
A- colon cancer B- dieulafoy lesion C- Aortoenteric fistula
677. 73 old male with an alcoholic liver disease previous appendectomy, cachectic, vomiting
+ve succession splash

PLEAZE, ANY NOTE OR CORRECTION OR ADDITIN SEND: [email protected]


‫نسألكم الدعاء بظهر الغيب‬
P a g e | 71
A- intestinal tumor B- adhesion
C- ascites D- gastric outlet obstruction
678. Severe diarrhea, pt. disturbed conscious, Na 162, K 4.9 ... best fluid replacement??
A- Saline B- Ringer C- D5 D- Dextrose- saline
679. Female 20 yrs., history of -ve abd exploration for acute abd. 2 months ago, came with
vomiting distention, +ve thrill fluid aspiration +ve amylase. Management??
A- ERCP B- Laparoscopy C- CT
680. Rectal cancer 4cm from anal margin for nodal staging??
A- CT abd & pelvis B- endoanal U/S
C- MRI D- PET
681. Trauma >> exploration >> infected >> Septic shock the cause of poor peripheral tissue
perfusion??
A- Low cardiac output B- Increase resistance
C- ineffective perfusion in peripheral tissue D- decrease glomerular filtration rate
682. Risk for breast cancer??
A- menarche at 14 B- contraceptive pill
C- leave born after 30 D- history mother with cancer breast
683. Most effective surgery for wt. reduction?? gastric bypass
684. Hard thyroid nodule woody with compression manifestation with history of episodes of
hypothyroidism and jaundice, best diagnosis??
A- FNA (Cytology) B- Radionuclide
C- Thyroid function D- isthmectomy or open bx
685. pt. with hypertension, hypernatremia, hypokalemia and mass above Kidney??
A- Cushing B- Adrenal tumor
686. Best method for localization of parathyroid nodule before surgery??
A- Subtraction scan B- Sestamebi scan
687. Spherocytosis planned for Splenectomy investigation??
A- fragility test B- US to rule out gall bladder stone
688. Melanoma 2 mm thickness safety margin??
A- 0.5cm B- 1cm C- 1.5cm D- 2 cm
689. 24 yrs. explosive injury??
A- Burn B- Shrapnel
690. Explosion with neck injury, exploration esophageal & proximal SCA injury ttt??
A- repair of both B- repair artery & esophagostomy
C- Ligation artery & repair esophagus D- Ligation artery & esophagostomy
691. Buttock infection with crepitus with gram +ve spore forming bacilli ttt?? Penicillin G

PLEAZE, ANY NOTE OR CORRECTION OR ADDITIN SEND: [email protected]


‫نسألكم الدعاء بظهر الغيب‬
P a g e | 72
692. best mesh for hernia repair
A- non absorbable B- delayed absorbable C- inelastic
693. Case dark urine?? toxic shock
694. Hand infection most associated with osteomyelitis?? Felon
695. Pt with history lumpectomy for phyllode tumor after recurrence??
A- Lumpectomy + radio B- Lumpectomy + Chemo C- Simple mastectomy
696. Anatomic Liver division depend on??
A- hepatic vein B- portal vein C- hepatic artery
697. In achalasia there is absence of??
A- complete relaxation of Low esophagus
B- In complete relaxation of Low esophagus
C- esophageal peristalsis
698. Most common anomaly with gastroschisis??
A- Malrotation B- Intestinal atresia
699. exploration for appendectomy, injury cecal diverticula, soiling, cecum is massively
involved ttt??
A- Ileocecectomy B- Rt hemicolectomy
700. diverticulitis with parasigmoid abscess 6x6, ttt??
A- antibiotics B- precut. Aspiration
701. pain in LLQ, CT showed abscess contagious confluent with sigmoid cause??
A- rupture diverticulum B- bleeding in abscess
702. 32-year-old female with 24 weeks pregnant Present to ER with an acute onset of
abdominal pain, fever and vomiting. She said the pain wake her up in middle of night with
sudden onset of epigastric pain that is now diffuse, she has no vaginal bleeding and fetal
monitoring demonstrates normal vital for the fetus, upon physical examination the pt. has
diffuse tenderness with guarding throughout the abdomen, were in epigastric region , pelvic
examination is normal , she has a leukocytosis of 15,000 abdominal Xray serious showed
dilated bowel loops but no other findings, what is your next step in management of this
patient??
A- abd U/S
B- CT of abdomen and pelvis with contrast
C- Admit and observation with serial abd exam
D- Exploratory laparoscopy
703. 30-year-old pt. presented with stool mixed with blood for 2 weeks, upper GI and lower
colonoscopy -ve, by examination palpable spleen, diagnosis is??

PLEAZE, ANY NOTE OR CORRECTION OR ADDITIN SEND: [email protected]


‫نسألكم الدعاء بظهر الغيب‬
P a g e | 73
A- typhoid ulcer B- Diverticulosis
C- duodenal ulcer D-Meckel diverticulosis
704. the risk of regional node metastasis in 0.70 mm thick melanoma is
A- Less than 5% B- 10% C- 20% D- 50%
705. Most significant study to check for esophageal tumor invasion to nearly bronchus??
A- bronchoscopy B- MRI C- esophagoscopy D- Ct chest
706. Post neck femur surgery, what medication will be given as VTE prophylaxis??
A- Aspirin B- Low molecular weight heparin
C-Full dose of heparin D- Clopidogrel
707. Pt to do elective inguinal hernia repair recurrent, best operation??
A- TAPP B- TEP C- Open
708. Pt with inguino-scrotal hernia presented with abd distention, vomiting and severe pain,
most likely about the hernia (not tender)??
A- Obstructed B- Strangulated C- Incarcerated
709. Pt came from Mexico with RUQ pain?? Amebic liver abscess
710. 30-year-old pt. with total extraperitoneal repair of hernia, the pressure must be??
A- More than 10mmHg B- 11-15mmHg C- 16-20mmHg
711. 56-yr pt. has history of acute pancreatitis show splenic aneurysm with calcification, ttt??
A- Observation and follow up
B- Ligation of aneurysm
C- Splenectomy with removal of affected artery
712. Hypokalemic changes in ECG are??
A- prolonged QT segment B- Prominent U wave C- widened QRS complex
713. Scenario of TIA investigation, best by??
A- CT Angio B- Doppler US C- CT Brain
714. 63y admitted for an elective colon resection for recurrent attacks of sigmoid diverticulitis,
you want to administer prophylactic antibiotics in choosing a regimen you should be aware
that the most common organism found in the colon of normal individuals is??
A- Escherichia coli B- Clostridium difficile
C- Pseudomonas Species D- Bacteroides species
715. On the 2nd day after AAA repair, pt. passes grossly bloody stool, the next step??
A- Immediate exploratory laparotomy B- Sigmoidoscopy
C- Ct abdomen with contrast D- Barium enema
716. Pt with burns to the entire back, scalp 50% pf head and neck and posterior thighs has
what percentage of his total body surface area burned??
A- 40% B- 28% C- 20% D- 32%

PLEAZE, ANY NOTE OR CORRECTION OR ADDITIN SEND: [email protected]


‫نسألكم الدعاء بظهر الغيب‬
P a g e | 74
717. Most common strong anticoagulant secreted by medicinal leeches is??
A- Heparin B- Warfarin C- Hirludin
718. Burn degree of electrical high voltage on boy 15year old??
A- 1st B- 2nd C- 3rd D- 4th
719. CBD stricture and management post CBD injury??
A- Choledochodudenostomy with stent B- Choledochodudenostomy without stent
C- ERCP with stent D- end to end anastomosis(hepatico jejunostomy)
720. Pt with anal pain with no soaking cloth. Colonoscopy free, pain with DRE??
A- intersphincteric abscess B- pile C- Fissure
721. Which can compromise respiratory during delivery to baby??
A- Cystic hygroma B- Torticollis
722. Most common complication after hemorrhoidectomy?? Urine retention
723. Output from ileostomy must be kept under colostomy??
A- 500-700 B- 1500
724. Pt with septic shock, which Hb value considered for blood transfusion??
A- 6-7 B- 7-8 C- 8-9 D- 10
725. Unconjugated bilirubin, indirect, water insoluble & conjugated bilirubin direct, water
soluble.
726. Type of gastritis caused by H pylori?? type B
727. TTT of annular pancreas?? Duodenojejunostomy
728. Pancreatic tumor 2cm with hypokalemia?? VIPoma
729. Pt underwent lap chole with post-operative day one bile leak, best ttt?? ERCP with stent
730. Best investigation to diagnosis CHPS?? endoscopy for adults & U/S for pediatrics
731. Pt complaining of pruritis at night around anus, no soiling of clothes ,no history of
bleeding or pain or fever,diagnosis is??
A- Anal fissure B- Perianal abscess C- Pile D- Perianal fistula
732. Pt complaining of mid sigmoid colon cancer, the best management is??
A- Anterior resection
B- Resection and anastomosis colorectal anastomosis.
C- Abdominal perineal
D- Hartman procedure
733. Pt suffering from perforated diverticulitis drainage was done with colostomy. pt. received
3rd generation cephalosporine before operation and post-operative for 1 week, pt. developed
fever, the most usually due to??
A- pt. should receive anti anaerobic

PLEAZE, ANY NOTE OR CORRECTION OR ADDITIN SEND: [email protected]


‫نسألكم الدعاء بظهر الغيب‬
P a g e | 75
B- should receive 1st generation cephalosporine
C- multiple pre op doses of antibiotics
734. Underweight infant presented with dripping of salvia with abdominal distention and
cyanosis, dyspnea (Ryle tube fill with air) Xray shows Ryle in chest. what is the initial
management??
A- Continues cleaning and suction of esophagus
B- Repair of esophageal fistula
C- Gastrostomy to decompress distended stomach
D- Intubation
735. Endorectal U/S how many layers detected?
A- 2 B- 3 C- 4 D- 5
736. Percentage of malignant potential in ulcerative colitis after 20years??
2% (after 10 years)
8% (after 20 years)
18% (after 30 years)
737. Pt with inflammatory colitis, which suggest crohn disease diagnosis??
A- Ileitis B- Rectal sparing C- Crypt abscess D- Proctitis
738. About azathioprine in treatment of IBD??
A- Decrease leukocyte B- Used instead of corticosteroids
C- It is onset of action 6-12 weeks D- Can give iv
739. ttt of desmoid tumor in mesentery of patient with FAP??
A- Tamoxifen B- Cortisone C- Radiotherapy D- Methotrexate
740. ttt of infected bullous dermatitis with crepitus under skin??
A- Clindamycin B- Penicillin G
741. Pt with BP 120/80, creatinine and urea were high, diagnosis??
A- SIRS B- Sepsis C- Severe sepsis D- Septic shock
742. Pt had Rt hemicolectomy for FPC 10 yrs. ago, now presented with a 14 cm parietal abd
mass at the operation site, what is the proper management??
A- Triple chemotherapy B- Conservative management
C- Wide local excision D- Combined chemo-radiotherapy
743. Pt with gastric ulcer by endoscopy due to H-pylori, the priority to??
A- Eradicate H pylori B- Reduce acid reflux C- Endoscopic biopsy of ulcer
744. Pt with peptic ulcer, not respond ttt. Best management is??
A- Vagotomy B- High selective vagotomy C- Billroth operation
745. Boy 10-year-old c/o pain at hip, relieved by rest, diagnosis?? Perthes disease

PLEAZE, ANY NOTE OR CORRECTION OR ADDITIN SEND: [email protected]


‫نسألكم الدعاء بظهر الغيب‬
P a g e | 76
746. Pt complaining of bone pain, depression, loss of memory, fatigability, most likely
causes??
A- High serum Ca and intact PTH
B- Decrease Ca urine level
C- Decrease serum phosphate level
747. 30-year-old female with longitudinal; tender tear on the 9 o'clock lithotomy position with
bright bleeding per rectum, what finding need to do biopsy??
A- Site B- Size C- Bleeding D- Recurrence
748. 56 yrs. male with constipation, bleeding per rectum, cachexy, has circumference anal
verge adenocarcinoma, management??
A- Defunctioning colostomy
B- Abdominoperineal resection with permanent colostomy
C- Chemo radiotherapy therapy
D- Radiotherapy
NB>>>neoadjuvant CTH + RT if present in choices
749. Most common cause of urine tract infection with E coli Colo vesical fistula??
A- Diverticulosis B- Crohn disease C- Ulcerative colitis
750. Pt with history of diverticulitis, came with high grade fever, lower abd pain, U/S showed
6x6 homogenesis collection on pelvis, what will you do?
A- Percutaneous drainage B- Antibiotics C- Fluids D- Follow up
751. Pt on antibiotics for treatment of cellulitis for more than 13 weeks, developed colicky abd
pain, diarrhea, fever, most important test to detect the cause??
A- Stool cytotoxin test B- ELISA C- Blood culture
752. Pt with diffuse painless thyroid gland enlargement with T3, T4 were normal, TSH is high
and thyroid antibodies +ve??
A- Thyroidectomy B- Radioiodine
C- Thyroxine D- Antithyroid drug
753. Pt complaining of abd mass, on the anal canal biopsy showed human papilloma virus??
A- Paget's disease B- Bowen disease C- Adenocarcinoma
754. Type of shock need vasopressin??
A- Hypovolemic B- Cardiogenic C- Distributive
755. What type of shock has the following characteristic, COP low, SVR high, BP low, O2 low??
A- Cardiogenic B- Hypovolemic C- Distributive
756. In ICU pt. developed hypotension, hyponatremia, hypoglycemia and hyperkalemia. What
is the most important drug given to save the pt.??
A- Hydrocortisone B- Fludrocortisone C- Ca D- NaCl

PLEAZE, ANY NOTE OR CORRECTION OR ADDITIN SEND: [email protected]


‫نسألكم الدعاء بظهر الغيب‬
P a g e | 77
757. Pt developed hypernatremia, hypokalemia. hypertension, CT showed mass on kidney.
most likely diagnosis is??
A- Adrenocortical carcinoma B- Renal tumor C- Pheochromocytoma
758. Pt complaining of rt side chest pain, chest Xray showed Rt side lung opacity cyst
breathing diminished on Rt side, diagnosis is hydrated cyst, most appropriate ttt??
A- Albendazole B- Pneumonectomy C- PAIR D- Excision
nd
759. Pregnant woman 2 trimester 13 weeks has tremor, palpitation, rt neck swelling, most
likely ttt??
A- Thyroidectomy B- Antithyroid drug C- Radioiodine D- Radiation
760. Paget disease of the bone management?? Bisphosphonate
761. Extended jugular vein in hypotension pt. with normal respiratory air entry??
A- Hemothorax B- Pneumothorax C- Cardiac tamponed
762. Pyloric adenocarcinoma??
A- D 1 B- D 2 C- D 3 D- D 4
763. Middle age 1-month epigastric pain + diarrhea, gastrin 120 Next
A- Somatostatin receptor scan B- Endo lumen US
C- PPI D- Secretin stimulation test
764. Post liver abscess rupture exploration, abd. Distention, oliguria, fever, tachycardia, mild
hypotension & Intravesical pressure 28 cr. 8??
A- Colostomy B- Exploration & evacuation C- Colectomy D- CT abd
765. 4 wks. old child with clear vomiting, what’s the best investigation?? US
766. Pt. with vomiting 3 times daily, inv. K: 1.8, what’s the suspected ECG??
Prominent U wave with flat T wave
767. Male pt. diarrhea 1 month, colic, anal fistula opening above dental line What’s the best
investigation??
A- CT colon B- MRI C- Colonoscopy D- Br. Enema
768. Female presented with RUQ pain, jaundice, Labs: high direct bilirubin, high indirect
bilirubin, high Alkaline phosphatase Temp, 38.6 ...what’s the next step??
A- MRCP B- ERCP C- CT Abd.
769. What’s associated to coin indicate malignancy??
A- Calcification B- Stellate like figure
770. 65 yrs. old female presented with constipation, distention & abd. Cramps History:
constipation 2 months ago, but she believes the cause is changing diet She did hysterectomy
10 years ago Exam; increased intestinal sounds with tenderness over Rt. Iliac fossa What’s the
diagnosis?
A- Acute appendicitis B- Adhesive IO C- Sigmoid volvulus

PLEAZE, ANY NOTE OR CORRECTION OR ADDITIN SEND: [email protected]


‫نسألكم الدعاء بظهر الغيب‬
P a g e | 78
771. 37 yrs. female with midline swelling, doesn’t move with deglution, moves with tongue
protrusion Patient refuses any surgery, what’s the fate of this swelling?
A- Spontaneous disappearance B- Malignancy transformation
C- Laryngeal compression D- infection
772. Pt. with AF did embolectomy of femoral artery then started pain, numbness, swelling of
the foot Exam; tender calf mass, popliteal artery is felt, Dx?
A- DVT B- Compartmental syndrome C- Re-embolization
773. pt. with sudden epigastric pain became generalized abd. Pain & tenderness Xray; air
under diaphragm ... Dx?
A- Gastric perforation B- Appendicular rupture C- Ileal perforation
774. Pt. with Acute Rt. Iliac fossa pain for 2 days with vomiting, pain started around umbilicus,
PR tenderness, Dx? Acute appendicitis
775. Female with no history of jaundice, during lap cholecystectomy, you found wide cystic
duct & wide CBD, what will you do?
A- cholecystectomy with US
B- per (intra)-operative cholangiography
C- open cholecystectomy & palpate CBD
D- intraoperative US
776. Factors affecting recurrence of colon cancer
A- staging B- lymph nodes
777. Upper endoscopy shows grade 2 varices, what’s the best management?
A- sclerotherapy B- devascularizati C- total shunt D -Band ligation
778. Old age male alcoholic presented with melena for 2 days, what’s the cause?
A- bleeding Duodenal ulcer B- bleeding gastric ulcer C- esophageal varices
779. Jaundiced newborn, US: no evidence of extra hepatic biliary ducts What’s the age beyond
it the operation will have poor results
A- 8 wks. B- 12 wks. C- 14 wks. D- 18 wks.
780. Old age male suffering from upper abd. Pain, CT: dilated distal 2/3 of pancreas, MRCP
shows stone in junction between proximal 2/3 & distal1/3 of Pancreatic duct. Best
management?
A- Distal pancreatectomy B- Duodenopancreatectomy C- ERCP & stent
781. 2 yrs old baby with bilateral undescended testis, Most diagnostic test?
A- laparoscopy B- karyotyping
782. Malnourished patient, what is element should be normal if patient is for surgery?

PLEAZE, ANY NOTE OR CORRECTION OR ADDITIN SEND: [email protected]


‫نسألكم الدعاء بظهر الغيب‬
P a g e | 79
783. pt. underwent papillary thyroidectomy yesterday, on normal saline& diet, presented with
generalized seizures. what’s the cause?
A- Cerebral metastasis B- Hypocalcemia hyponatremias
784. Female with swelling 3 cm with peau de orange with supra clavicular L.N, No evidence of
distant metastasis
A- T3 N0 M0 B- T3 N1 M0 C- T4 Nb1 M0 D- T4 Na1 M1
785. Trauma female patient with Rt. Hypochondrial pain, yesterday she missed herself until
next day presented with severe Rt. Hypochondrial pain, U.S. normal, possible organ to be
injured?
A- spleen B- Liver C- stomach intestine
786. Patient with sudden lumbar back pain radiating to flanks, He gave history of the pain not
radiating 2 days ago. Vitals: pulse: 80, BP 110/70, temp. 37 After 1 hour: pulse 106, BP 80/40,
temp. 37 What’s the management?
A- CT abd. B- Laparotomy C- Endoscopic repair for the aneurysm
787. Trauma patient with hypochondrial pain radiating to shoulders, Vitals: pulse 108, BP
100/70. what’s the initial management?
A- US B- CT C- LAPAROTOMY
788. Patient with hypovolemic shock, mild anxious, BP 120/80, p: 106 What’s the stage of shock?
A- 1 B- 2 C- 3 D- 4
789. Scenario of pt. has anal swelling and bleeding; swelling is protruded not reduced except
manually what is the management?
A- hemorrhoidectomy B- band C- ligation D- laxative
790. Scenario of pt. suffering from bleeding per rectum PR show ulcerative mass 4cm from
anal verge, management??
A- colonoscopy e biopsy B- CT C- MRI
791. Scenario of pt. mass 2,5cm at base of appendix, management??
A- Rt hemicolectomy B- Appendectomy
792. Scenario of pt. show ulcer in greater curvature management??
A- upper endoscopy e biopsy B- gastrectomy
793. Scenario of pt. suffer from smooth swelling at mid-1/3 of the esophagus what is the best
management?
A- endoscopy e biopsy B- esophagectomy C-enucleation
794. The commonest histological malignancy in Git
A- adenocarcinoma B- squamous cell carcinoma
795. The best prognosis of breast cancer
A- mucinous B- Paget C- tubular
796. Commensal that discovered cause pathogenesis?? staph epidemic
PLEAZE, ANY NOTE OR CORRECTION OR ADDITIN SEND: [email protected]
‫نسألكم الدعاء بظهر الغيب‬
P a g e | 80
797. Scenario of recurrent ing. scrotal hernia, irreducible only, not obstructed, not
strangulated e history of prostatectomy management??
A- open and repair e mesh B- laparoscopic repair
798. Boy e piece of wood in feet, red streaking what is the causative organism??
A-streptococcus pyogenes
799. Inguinal scrotal hernia what is sign?? Cannot palpate the vas
800. Gun shot in leg with paraesthesia and weak peripheral pulsation, management??
A- exploration, repair B- expectant ttt C- CT angiography
801. Which tumour not producing gynecomastia
A- lung B- Bone C- Prostate D- Testis
802. Prognosis of cancer bladder based???
A- age B- hematuria C- invasion of wall D- size of mass
803. Neonate 16 day old with Abd. Distension WBCS=8900 diagnosis??
A- enterocolitis B- volvulus C- intussception
804. Case with non-bilious vomiting, hypokalemia, ECG changes??
A- depressed ST segment B- elevated T wave
805. Daily needed protein for adults?? 0,8 mg/kg/day
806. IV fluid resuscitation 70kg person ??
A- 20ml/kg B- 50ml/kg C- 100ml/kg
807. Post trauma x-ray show air in mediastinum best next investigation??
A- CT chest B- esophagoscopy C- angiography D-water soluble contrast
808. Pt. with upper Git bleeding all investigation and coagulation is normal diagnosis??
A- dieulafoy lesion B- peptic ulcer
809. Gall bladder polyp 4mm management??
A- Expectant ttt B- Lap. Cholecystectomy C- Extended cholecystectomy
810. 6months after cholecystectomy MRCP image show defect in CBD, diagnosis??
A- CBD stone B- Hepatic duct structure C- CBD injury D- Cholangiocarcinoma
811. Bile found in peritoneum cystic duct leakage occurs post cholecystectomy management??
A- Tube B- ERCP C- Primary repair
812. Aspiration of pancreatic sac reveals fluid rich in carbohydrates diagnosis??
A- Pseudo cyst B- Mucinous cystadenocarcinoma C- Serous cystadenocarcinoma
813. Hepatic mass with increased platelets consumption??
A- Hemangioma B- HCC C- FNH
814. Swimmer female underwent modified mastectomy and breast reconstruction which type
of flap will interfere with swimming?? Lat dorsi flap

PLEAZE, ANY NOTE OR CORRECTION OR ADDITIN SEND: [email protected]


‫نسألكم الدعاء بظهر الغيب‬
P a g e | 81
815. Breast mass in upper inner quadrant aspiration reveals malignant cells management?
A- radical mastectomy B- MRM
C- Radical mastectomy, CTH D- Radical mastectomy, CTH, RTH
816. Gall bladder with adenocarcinoma in mucosa, management??
A- Cholecystectomy alone B- Wedge resection of liver C-extended cholecystectomy
817. Gall bladder with squamous cell carcinoma not reach serosa management??
A- Celiac L.N biopsy B- Extended cholecystectomy C- Liver resection
818. Stab neck in midline in the level of cricoid with hematoma management??
A- CT B- Angiography C- Exploration
819. Stab neck below angle of mandible with hematoma, weak carotid pulse drowsiness,
management??
A- exploration B- CT C- angiography
820. Most risky preoperative risk factor?
A- Aortic stenosis B- Bronchitis
821. Most common cause of recurrent epigastric hernia?
A- Using absorbable sutures in the repair B- Sport
C- Missed defect D- Increased intrabdominal pressure
822. 45 years old male patient with late vomiting and abdominal distension, diagnosis?
A- Crohn’s disease B- Sigmoid cancer C- Diverticulitis
823. After chest trauma weak femoral pulse and chest x-ray revealed air and wide
mediastinum,diagnosis?
A- Aortic rupture B- Flail chest C- Tension pneumothorax
824. What’s true about fibrocystic change in breast?
A- Disappears after menopause B- Premalignant C- Need surgery
825. Case of crohn’s disease with perforated distal ileum, management?
A- Primary repair B- Resection of perforated segment and ileostomy
C -Ileocecectomy
826. Long case of alcoholic old male patient with liver cirrhosis and ascites, management?
A- TIPS B- TIPS and transplantation
827. Pain in flank with urine output 500cc/24h next investigations?
A- CT B- Cystoscopy C- IVP
828. Pt. weights 105 kg & height 170 cm went to endocrinologist, barium swallow shows
elevated gastroesophageal junction diaphragm, management?
A- Nutritional and hormonal ttt B- Lap gastric bypass
C- Intragastric balloon D- Lap banding

PLEAZE, ANY NOTE OR CORRECTION OR ADDITIN SEND: [email protected]


‫نسألكم الدعاء بظهر الغيب‬
P a g e | 82
829. Image shows rt breast erythematous mass in inner lower quadrant 3cmX3cm in size, no
LN detected, management? Antibiotic
830. Patient with history of gastric ulcer came to ER with bleeding duodenal ulcer
management?
A- Truncal vagotomy
B- Truncal vagotomy + pyloroplasty
C- Truncal vagotomy + distal gastrectomy
831. Post-operative hypotension + empty drained and no urine output, management?
A- Diuretics B- Catheter flush C- Dopamine D- IV fluids
832. RTA post laparotomy dark brownish urine, management
A- Urine analysis B- Na bicarbonate C-iv fluids

833. RTA and pt. thrown away; he can’t move his lower limbs which are warm, what
type of shock?
A- Hypovolemic B- Cardiogenic C- Neurogenic
834. Interloop infection after abdominal exploration, management? Antibiotic and drainage
835. Most radiosensitive tumor?
A- Neuroblastoma B- Nephroblastoma C- Lymphoma
836. Most common malignancy in age 30 years
A- Osteosarcoma B- Wilm’s tumor C- Seminoma
837. Trigeminal nerve injury is caused by
A- Facture temporal bone B- Fracture maxilla C- Fracture mandible
838. Ulcer in cheek, no LN palpable
A- Melanoma B- BCC C- SCC
839. Newborn with persistent vomiting x-ray shows no air in GIT? esophageal atresia
840. Patient with massive truncal burn the way for emergent resuscitation
A- unburned peripheral limb B- venous cut down C- central venous line
841. What is the least tumor cause gynecomastia?
A- colon B- lung C- prostate D- gall
bladderE- testis F- renal G- bone
842. Patient came to ER with stab wound in the neck complaining from S.O.B and bubbling
from the wound, x-ray chest shows air in mediastinum what is the diagnosis?
A- Rupture trachea B- Rupture esophagus
C- Traumatic pneumothorax D- Entrapped external air
843. 3-year-old patient presented with stool mixed with blood for two weeks upper and lower
GI endoscopy are –ve, by examination palpable spleen, diagnosis?
A- Typhoid ulcer B- Duodenal ulcer C- Diverticulitis D- Mickell diverticulum

PLEAZE, ANY NOTE OR CORRECTION OR ADDITIN SEND: [email protected]


‫نسألكم الدعاء بظهر الغيب‬
P a g e | 83
844. 56 years old patient have history of acute pancreatitis show splenic aneurysm with
calcification, TTT?
A- Observation and follow up B- Ligation of aneurysm
C- Splenectomy with removal of the affected artery
845. The hernia that is never completely reduced
A- Epigastric B- Littre C- Sliding D- Pantaloon
846. Indirect inguinal hernia reaches the scrotum characterized by
A- Can’t palpate vas
B- +ve transillumination test
C- irreducible
847. PoP at calf muscle appears for 1 day in pt. 25 years old, spread via s.c. struck to groin, the
most causative is?
A- E coli B- S pyogenes C- S difficil D- C.W. (clostridium welchii)
848. In patient with incisional hernia, during adhesiolysis resulted perforation of transverse
colon with gross fecal contamination, primary closure was done what is after?
A- Repair and mesh
B- Primary, later repair
C- Closure in mass with tension sutures
849. Patient suffering from perforated diverticulitis, drainage was done with colostomy,
patient received 3rd generation cephalosporines before and postoperative for one week, pt.
developed fever, by investigations intraperitoneal abscess at day 6 postoperative, this is most
commonly due to
A- Patient must receive anti anaerobic
B- Should receive 1st generation cephalosporine
C- Multiple preoperative doses of antibiotics
850. Underweight infant presented with dripping of saliva with abdominal distension, cyanosis
and dyspnea, Ryle tube fill with air, x-ray show Ryle in chest, what is the initial management?
A- Continuous cleaning and suction of esophagus
B- Repair of esophagotracheal fistula
C- Gastrostomy to decompress the distended stomach
D- intubation
851. What is the most common cause of hemobilia?
Iatrogenic trauma to the liver and biliary tree
852. What is the first test to rule out hemobilia? Esophagogastroduodenoscopy
853. What is the first-line therapy for hemobilia? Angiography (embolization)

PLEAZE, ANY NOTE OR CORRECTION OR ADDITIN SEND: [email protected]


‫نسألكم الدعاء بظهر الغيب‬
P a g e | 84
854. Female patient with long time of bilateral leg swelling and skin change, best:
A- Doppler U/S B- Venography C- ABPI
855. Female patient with big toe bluish discoloration and severe pain with gangrene, ABPI is:
A- >1 B- 0.8 – 0.9 C- 0.5 – 0.6 D- < 0.3
856. Most common site of biliary carcinoma:
A- Distal part of CBD B- Hilum C- Cystic D- Intrahepatic
Confluence or perihelium or bifurcation of CHD
857. Patient with right upper quadrant pain, fever, yellow sclera, bilirubin high
A- Pancreatitis B-cholecystitis C- Cholangitis
858. A diagram of surgical resident and non- surgical resident followed for 4 years outcome of
depression stud, what is the type of study:
A- Cohort B- Series case C- Control study D- Navy tape test
859. Study on effect of dairy products on group with colon cancer and group with no colon
cancer, what is the type of study:
A- Cohort study B- Control study
C- Section study D- Correlation
860. During lap. Chole., you injured the CBD and told the patient post-operative, what does
this mean?
A- Disclosing on apparent complication B- Disclosing complication
C- Disclosing error D- Disclosing an apparent error
861. Study conducts to see the effect of ear phone Bluetooth and brain tumor, what is the
best study?
A- Cross sectional B- Cohort C- Case control D- Correlation
862. 3 weeks old infant with multiple non-bilious vomiting, diagnosis:
A- Duodenal atresia B- Pylorus hypertrophy
C- Annular pancreas D- Malrotation
863. Post operation of thyroidectomy, fever, hypertension, tachycardia: Thyroid storm
864. Organ resistant to infection
A- Intraabdominal B- Blood C- Pulmonary
865. Patient with controlled diabetes
A- ASA I B- ASA II C- ASA III D- ASA IV
866. Patient with RTA, Bl. Pr. 90/60, HR 130, RR 26, best fluid?
A- Blood B- Colloid C- Crystalloid (if best initial)
867. Ileostomy, best fluid? Normal saline
868. Myeloproliferative with splenomegaly, best investigation? CT with contrast

PLEAZE, ANY NOTE OR CORRECTION OR ADDITIN SEND: [email protected]


‫نسألكم الدعاء بظهر الغيب‬
P a g e | 85
869. Child with splenomegaly, jaundice, yellow sclera, dark urine and +ve H/O family both
parents have blood dyscrasia, Hb 9.2, best treatment?
A- Splenectomy B- Corticosteroids
C- Packed RBCs transfusion D- Whole blood transfusion
870. Landmark to differentiate between indirect and direct hernia:
A- Scrotum B- Vas deferens C- Pubic tubercle D- Internal ring
871. Para umbilical hernia, vomiting, constipation, sac irreducible, not tender, type of hernia?
A- Obstructed B- Irreducible C- Incarcerated D- Strangulated
872. Hernia with crescentic shaped umbilicus is?
A- Umbilical B- Epigastric C- Spigelian hernia D- Para- umbilical
873. What is the cause of rectal prolapse in amale pt patient 40 years old?
A- Pelvic floor weakness
B- Long term history of diarrhea or constipation
C- Previous injury to the anal or hip area
874. Patient, 40 years old with rectal prolapse,with chronic constipation best management?
A- Altemeier B- Delorme C-resection and rectopexy D-Wells rectopexy
875. Sigmoid volvulus, next appropriate management;
A- Flatus tube B- Nasogastric tube
876. Symptoms of GERD + barium swallow shows that gastroesophageal junction above
diaphragm but the stomach is in place:
A- Heller myotomy
B- Partial fundoplication
C- Nissen fundoplication (complete)
877. Female patient with abdominal distension and vomiting O/E stricture 6 cm from ileocecal
valve no passable (only one structure):
A- Stricturoplasty B- Right hemicolectomy
C- Resection and anastomosis D- Resection and ileostomy
878. Old age male with lesion lateral to eye, lesion show pigmentation and lateral ulceration
A- S.C.C B- B.C.C C- Melanoma D- Marjolijn ulcer
879. Best investigation to detect GERD: PH monitoring
880. Girl comes to ER with her mother, the father out of the town, diagnosis was appendicitis
A- Prepare for surgery
B- Take verbal consent from the mother
C- Wait till the father come back
D- Explain the procedure to the girl and take the mother signature

PLEAZE, ANY NOTE OR CORRECTION OR ADDITIN SEND: [email protected]


‫نسألكم الدعاء بظهر الغيب‬
P a g e | 86
881. Female 65 years old with face skin lesion (pic) beside the nose (check) for excision (basal
cell carcinoma), safety margin?
A- 0.5 mm B- 1 mm C- 2 mm D- 10 mm
N.B. In general safety margin is 5 mm (0.5)
882. 30/500 babies who were exposed to Di-ethyl-stilbestrol in utero were followed up for 20
years and the occurrence of cancer in this group was compared with that of 1500 babies not
exposed to DES, over the same period of time, what is the study design?
A- Cohort B- Case control C- Cross sectional D- Correlation study
883. Female patient old age diabetic, uncontrolled came to ER with blackish discoloration at lt.
foot 2 weeks duration, no palpable anterior and posterior tibial artery, leukocytosis, CTA
stenosis at distal superficial femoral artery, complete occlusion popliteal and intra popliteal
artery, no collaterals, no run off, management?
A- Bypass graft B- Below knee amputation C- Above knee amputation
884. Patient RTA, urine output 25 ml/ h, cold clammy sweating, mild anxious, RR= 25, BP=
normal, pulse= 110, what is the volume of blood loss?
A- < 15% B- 15-30% C- 30-40% D- > 40%
885. Diabetes mellitus among health care workers is 30 out of 100 doctors were diabetic as
compared to 50 out of 200 nurses, remember (doctors-nurses) (diabetic-non diabetic), which is
the most appropriate statistical test?
A- T-test B- ANOVA C- Chi-square D- Correlation
886. Best study to assess the relationship between Bluetooth headsets and brain cancer?
A- Retrospective study B- Case control C- Ecological D- Correlation
887. Likely associated with vascular injury most least:
A- Mid ulnar bone fracture B- Supracondylar fracture humerus
C- Supracondylar fracture femur D- Post dislocation of knee joint
888. The diagnosis of esophageal achalasia is confirmed by?
A- Bird’s peak appearance on barium swallow
B- Sub atmospheric intraluminal esophageal pressure on manometry
C- Endoscopic evidence of distal esophagitis
D- Failure of lower esophageal sphincter relaxation on manometry
889. 41 years old undergoes complex repair of a deep laceration in her hand, when removing
the dressing on postoperative day 2, a large clot with mild surrounding erythema is
encountered, which of the following statements regarding the inflammatory phase of wound
healing:
A- The complement component C5a and platelet factor attract neutrophils to the wound

PLEAZE, ANY NOTE OR CORRECTION OR ADDITIN SEND: [email protected]


‫نسألكم الدعاء بظهر الغيب‬
P a g e | 87
B- The presence of neutrophils in the wound is essential for normal wound healing
C- It lasts up to 24 hrs. after the injury occur
890. Medullary thyroid cancer:
A- More common than papillary thyroid cancer
B- Tend to metastasize by lymphatics
C- Genetic survey may lead to prophylactic total thyroidectomy
891. Male with grave’s disease after thyroidectomy, fever, tachycardia, tachypnea,
hypotension
A- Thyroid storm B- Septicemia C- Infection
892. Type of shock need vasopressin
A- Hypovolemic B- Cardiogenic C- Distributive
893. Gallbladder stone suggested by U/S + dilated CBD without stone, patient is 60 years old,
best investigation?
A- ERCP B- MRCP C- CT D- PTC
894. Gallbladder + fever + pain in rt. Upper quadrant, tenderness, she started antibiotics after
3 days with no improvement, best investigation:
A- ERCP B- MRCP C- CT
895. Patient, 22 years old presented with pain at rt. Iliac fossa with rebound tenderness, WBCs
12.6, temp 38.2, diagnosis is:
A- Salpingitis B- Appendicitis C- UTI
896. Diverticulitis with peri sigmoidal abscess 6x6, treatment:
A- Antibiotics B- Percutaneous aspiration
897. What type of shock has the following characteristics, COP low, SVR high, low BP and low
O2?
A- Cardiogenic B- Hypovolemic C- Distributive
898. 45 years old male patient with late vomiting and abdominal distension, diagnosis?
A- Crohn’s disease B- Sigmoid carcinoma
899. Hand infection most associated with osteomyelitis: Felon
900. Patient with history of diverticulosis come with high grade fever, lower abdominal pain,
U/S shows 6x6 homogenous collection in the pelvis, what will you do?
A- Percutaneous drainage B- Antibiotics C- Fluids
901. Patient complaining of pruritis at night around the anus, no history of bleeding or pain or
fever, diagnosis is?
A- Anal fissure B- Perianal abscess C- Piles D- Perianal fistula
902. Pregnant patient, 24 weeks came with acute onset of abdominal pain, fever, and vomiting,
she states that pain woke her up at night, now pain is diffuse, no vaginal bleeding, fetus exam

PLEAZE, ANY NOTE OR CORRECTION OR ADDITIN SEND: [email protected]


‫نسألكم الدعاء بظهر الغيب‬
P a g e | 88
is ok, guarding whole abd., pelvic exam is normal, WBCs 15000, x-ray shows abd. Dilated
loops, what is next step?
A- U/S B- CT abd. and pelvic with contrast
C- Exp. Laparotomy D- Diagnostic laparoscopy
903. Male retrosternal pain, relieved, no other positive data:
A- Boerhaave syndrome B- MI C- Stable angina
904. Patient underwent lap. Chole. With post-operative day one bile leak, best treatment?
ERCP & stent
905. Female patient, 65 years old, presented with constipation, distension, and abdominal
cramps, give history that she is constipated 2 months ago, but she believed that the cause was
changing diet, she also does hysterectomy 10 years ago, on examination, there is increase
intestinal sound with tenderness in right iliac fossa, what is the diagnosis?
A- Acute appendicitis B- Adhesion intestinal obstruction
906. Case of old male, 70 years old with mid esophagus cancer with no metastasis,
management:
A- Stent B- Resection
907. Patient in blast injury, have pneumothorax burn 40%, fracture femoral and shrapnel,
injury orientation?
A- Fracture femur B- Burn C- Shrapnel D- Pneumothorax
908. Patient in a blast injury, best IVF is:
A- Normal saline B- Ringer lactate C- 3% saline
909. Most effective treatment of flail chest is:
A- Adhesive tape
B- Fixation with wire and screw
C- Epidural catheter and analgesic
910. In TNM and staging of breast cancer according to fibroadenoma what is incorrect:
A- Mostly benign B- Mostly mobile
C- Diagnosis by mammogram D- May be huge
911. Child has wound in right foot with wood broke, red, hot and tender, most likely organism:
A- Clostridium tetani B- Clostridium perfringens
912. Fluid resuscitation for infant: 20ml/kg bolus
A- 50-60 B- 60-65 C- 65-70 D- 70-80
913. Gallbladder stone asymptomatic and patient has history of enteric fever, best treatment:
A- Cholecystectomy B- Conservative

PLEAZE, ANY NOTE OR CORRECTION OR ADDITIN SEND: [email protected]


‫نسألكم الدعاء بظهر الغيب‬
P a g e | 89
914. Patient RTA with a fracture femur only, C/O dyspnea, petechia in upper chest and neck,
diagnosis?
A- Fat embolism B- Tension pneumothorax
915. Patient with hand injury with wire, took tetanus completed 12 years ago, best to be given?
A- Tetanus toxoid
B- Immunoglobulin
C- Tetanus + toxoid+ immunoglobulin
916. Case of iatrogenic esophageal perforation, diagnosis? Water soluble swallow
917. A case of achalasia, initial investigation: Barium swallow
918. A case of parameter of septic shock:
A- Sepsis + organ dysfunction B- SIRS + infection
C- Severe sepsis + hypotension D- Pulse > 90 + WBCs > 12000
919. Patient presented with central obesity, striae, and buffalo hump; best investigation is:
A- CT brain B- Urine cortisone
920. Occurred medical mistake and you want to learn the students, what is the appropriate
method for learning? Make a meeting about mortality and morbidity
921. Bilirubin:
A- Most of direct bilirubin is a result of hemolysis
B- Indirect bilirubin is a result of biliary tree obstruction
C- Most of bilirubin in serum is indirect and carried by albumin
922. Patient with uneventful lap. Chole. For 6 months presented with jaundice for 1 month with
MRCP photo, diagnosis:
A- Cholangiocarcinoma B- Stricture C- Missed stone
923. Long case…. Liver hydatid cyst, what is the layer of excision?
A- Laminated layer and adventitia B- Germinal layer and adventitia
C- Adventitia and capsule D- Germinal layer and laminated layer
924. Stab wound in the neck 2 cm below the angle of the mandible with progressive
hematoma, decrease blood pressure and tachycardia:
A- Exploration B- Angiography C- Conservative and follow up
925. A case of pancreatitis, with age 60, WBCs 21000, increase blood glucose level, increase
LDH 450, AST 244:
A- Severe case of pancreatitis
B- Mortality < 5%
C- Amylase is indicator of severe disease
926. Female patient with breast mass 1 cm by mammogram stellate mass with irregular
border, microclacifications and distortion of surrounding structures, by true cut biopsy there

PLEAZE, ANY NOTE OR CORRECTION OR ADDITIN SEND: [email protected]


‫نسألكم الدعاء بظهر الغيب‬
P a g e | 90
is hyperplastic tissue, management:
A- Excisional biopsy by wire detection preoperative
B- Modified radical mastectomy
C- Simple mastectomy
927. Case of male patient, alcoholic with post vomiting esophageal tear, management:
A- Conservative and follow up
B- Thoracotomy and esophageal repair
928. A case of periampullary tumors, the best staging:
A- Endoluminal U/S B- CT C- ERCP D- MRCP
929. Female patient with gastric carcinoma in pylorus with celiac lymph node (advanced
tumor) with gastric outlet obstruction:
A- Total gastrectomy B- Subtotal gastrectomy C- Gastrojejunostomy
930. Female patient on treatment for arthritis complaining of epigastric pain and rt. iliac pain
with signs of peritonitis Perforated peptic ulcer
931. Most associated with chronic pancreatitis:
A- Weight loss B- Pain C- Fever
932. Patient with jaundice and epigastric pain, increase alkaline phosphatase, increase
bilirubin, increase SGOT & SGPT, increase GGT, the condition more than 7 weeks, most
common cause:
A- Pancreatic tumor (cancer head) B- Choledocholithiasis C-
Cholangiocarcinoma
933. Patient with right hypochondrial pain with dyspepsia with pain referred to the
shoulderwith U/S shows thickness of gallbladder wall
A- Acute cholecystitis B- Acute pancreatitis C- Peptic ulcer
934. Patient with palpable mass in right hypochondrium with U/S dilated gallbladder, thin wall:
A- Mucocele B- Acute cholecystitis
935. Female patient Felty syndrome:
A- Peritonitis + splenomegaly + rheumatoid
B- splenomegaly + rheumatoid + pancreatitis
C- splenomegaly + neutropenia + Peritonitis
D- neutropenia + splenomegaly + rheumatoid
936. Trauma to chest examination revealed centralized trachea, paradoxical movement of 6, 7,
8 left rips, x-ray show 6, 7, 8 left ribs and blurred costophrenic angle, PO2 92%, parameter is
normal, what is the definitive management?
A- Intubation and mechanical ventilation B- Tube thoracotomy C- Adhesive strap
937. 65-year patient came to clinic with 3-year history of bleeding per rectum with and without

PLEAZE, ANY NOTE OR CORRECTION OR ADDITIN SEND: [email protected]


‫نسألكم الدعاء بظهر الغيب‬
P a g e | 91
stool, urgency, 3 years ago he had coronary heart surgery and on antiplatelet, 6 years ago has

PLEAZE, ANY NOTE OR CORRECTION OR ADDITIN SEND: [email protected]


‫نسألكم الدعاء بظهر الغيب‬
P a g e | 92
been treated by radiotherapy for cancer of prostate, what is the cause of his condition?
A- Radiation proctitis B- Colon cancer C- Bleeding due to antiplatelet drug
938. Patient post-surgical exploration for pyogenic liver abscess presented with abdominal
distension, fever, tachycardia, tachypnea, oliguria, urinary catheter connected to manometry
pressure is 28:
A- Evacuation of collection
B- Laparotomy and colostomy
C- Laparotomy and laparostomy
939. 22-year-old medical student is seen by the student health service prior to beginning
school, routine labs are drawn, the medical student immunized against hepatitis B in childhood
will have which hepatitis profile?
A- Hbs Ab +ve, Hbs Ag +ve, Hbc Ab +ve
B- Hbs Ab +ve, Hbs Ag +ve, Hbc Ab -ve
C- Hbs Ab -ve, Hbs Ag -ve, Hbc Ab -ve
D- Hbs Ab +ve, Hbs Ag -ve, Hbc Ab -ve
940. Upper right quadrant pain male patient 44 years old presented in ER with fever, H/O lap.
Chole. For 4 days, U/S reveals abdominal collection in gallbladder fossa and hepatorenal
pouch, MRCP shows leakage from cystic duct stump, proper management:
A- ERCP + stent B- Open surgery and ligation of tear C- Surgical drainage
941. Patient with tachycardia, increase temp., decrease BP, what is the cause of hypotension?
A- Low cardiac output
B- Increased systemic vascular resistance
C- Decreased arteriolar resistance
942. A newly diagnosed HIV patient worry to tell his wife and ask not to tell her, what will you
do?
A- Tell his wife’s mother and ask her to tell her daughter
B- Tell his wife
C- Protect the patient confidentiality
D- Reassure him and offer your assistance to hell his wife
943. Seizure patient taking drug decrease attention, and he is a bus driver, your action?
A- Respect confidentiality and don’t tell anyone
B- Tell him not to drive
C- Tell his supervisor
944. A study conducted to evaluate medication among depressed patients, they were not
taking their medications regularly:
A- Unacceptable study because it affects drug efficacy

PLEAZE, ANY NOTE OR CORRECTION OR ADDITIN SEND: [email protected]


‫نسألكم الدعاء بظهر الغيب‬
P a g e | 93
B- Acceptable because patients signed the consent voluntary
C- Unacceptable because we can’t take consent from depressed patients
945. Enlarged thyroid gland with normal T3 & T4 and increase in TSH + thyroid antibodies,
what is the treatment?
A- Thyroxin B- Radioactive C- Antithyroid
946. Patient with dyspepsia, polyuria, hypernatremia, hypokalemia: Aldosterone
excess
947. Patient with hypernatremia, hypokalemia + mass in upper pole of the kidney:
Adrenal carcinoma
948. Patient undergo surgery for an abdominal neoplasm, he discusses it with his surgeon,
surgeon describes the procedure to him in detail, what is still necessary for the patient to be
able to give informed consent to this procedure?
A- A medical screen including ECG and coagulation panel
B- A psychiatric assessment
C- An explanation of alternatives to the surgery
D- Consultation with a family member of the patient coagulation panel
949. A child presented to the emergency with wound in his head, with him was his brother and
nursemaid but father and mother were in their way starting to come to hospital:
A- Start to stitch the wound
B- Cover the wound and wait for the mother or father
950. Cardiologist his patient need operation in heart but the patient doesn’t know the
procedure with many complications, what should be done?
A- Start to work without patient knowledge
B- You should inform the patient of all the details of the process
C- Abandoning the operation
951. Patient 84 yrs. old, his wife 29 yrs. old, he presented to ER with vomiting, nausea,
abdominal pain, distended and tympanic abdomen, rebound tenderness, temp. 38.5, RR 120,
HR 101, H/O 2 MI & CVA 5 years ago, after CVA he has trouble in speaking, he doesn’t have
power of autonomy, patient needs operation, his wife refused, his son demands that
everything possible must be done to save his father’s life, what level of care is appropriate?
A- Attempt intervention by local district court judge
B- Initiation of hospice care
C- Medical and supportive care only
D- Urgent surgical exploration
952. Mean of cholesterol in Arab area, American and European people:
A- Chi-square B- Student t-test C- ANOVA D- Correlation

PLEAZE, ANY NOTE OR CORRECTION OR ADDITIN SEND: [email protected]


‫نسألكم الدعاء بظهر الغيب‬
P a g e | 94
953. One of the following is a medical error:
A- Mistaken diagnosis or prescription B- Negligence
C- Management plan not completed as intended D- All of the above
954. Parameters to compare by Chi-square:
A- Age and BP B- Smoking and BP
C- Gender and smoking D- Heart disease and smoking
955. Gastric cancer (adenocarcinoma) removed completely during surgery but after operation
and with histologic examination there was +ve margin because it was seen by microscopy:
A- R0 B- R1 C- R2 D- R3
956. Primary energy in acute injury:
A- Fat B- Muscle C- Glycogen D- Ketone bodies
957. Patient gave you a gift and you refused, next day patient attempted suicide because she
felt rejected, what you should do in this situation?
A- Accept the gift and give it to patient charity
B- Explain for the patient that it’s unethical to accept gift then refuse
C- Write paper on OPD door that gifts are unacceptable
D- Accept the gift for yourself
958. Patient RTA, he had fracture leg, needs foot amputation, after you explained the risk and
benefits of procedure, he agreed because he hears voices telling him to do it, what do you do?
A- Find another decision maker because patient is incompetent
B- Proceed to the lifesaving surgery (since it’s an emergency intervention)
C- Involve another surgeon to confirm
959. Which one is not correct in four principles of Beauchamp & Childress:
A- Beneficence B- Non-maleficence C- Respect for persons D- Justice
960. Patient with known case of hypothyroidism presented by thyroid lump and hoarseness of
voice and dysphagia, O/E: woody firm fixed thyroid swelling, best investigation?
A- FNA B- Open biopsy C- TSH D- Iodine isotope
961. Patient with thyroid mass, FNA shows amyloid, treatment?
A- Lobectomy with neck dissection
B- Lobectomy with radiation
C- Total thyroidectomy with modified neck dissection
D- Total thyroidectomy with radiation
962. Patient underwent hemorrhoidectomy after 2 days he came at OPD for follow up, he was
with his son, doctor asked the nurse to nurse the patient, which one of the following exceeds
the privacy of the patient?
A- Examine in presence of nurse

PLEAZE, ANY NOTE OR CORRECTION OR ADDITIN SEND: [email protected]


‫نسألكم الدعاء بظهر الغيب‬
P a g e | 95
B- Examine without his son
C- Examine without white sheet
963. Male patient, CT abdomen shows hepatocellular carcinoma, what next investigation for
work up?
A- CT brain B- MRI abdomen and pelvis C- Pet scan D- Bone scan
964. Most common in MEN 1 is:
A- Parathyroid B- Thyroid C- Adrenal D- Pancreas
965. Patient with H/O 15 years duration of duodenal ulcer presented with multiple vomiting,
best I.V. fluid to be given is:
A- Ringer lactate B- Normal saline C- 3% saline D- Hetastarch
966. Gastrinoma location, the investigation of choice is:
A- MRI B- CT C- SRS D- MIBG
967. Patient very malnourished, what is the element be normal is the patient is for surgery:
A- Vit C B- Copper C- Folic acid D- Ca
968. Post UGI endoscopy, physician was worried about esophageal perforation, what is the
best investigation to confirm it? Diluted Gastrographin with thin film barium
969. Minimal safety margin for SCC:
A- 2 mm B- 4 mm C- 10 mm D- 15 mm
970. Patient with myeloproliferative disease and splenomegaly, next step?
A- CT e contrast B- Bone marrow biopsy C- Splenectomy D- Observation
971. Abnormal bleeding with normal prothrombin time occurs with all except:
A- Heparin overdose B- Cirrhosis C- Hemophilia D- Von Willebrand’s disease
972. Female 30-year-old, she has history of diarrhea, blood and mucous for 5 years back, her
mother has the same history from long time, most appropriate diagnosis?
A- Diverticulitis B- Crohn’s disease
C- Ulcerative colitis D- Colon carcinoma
973. Gastric cancer with right gastric 3 LNs need resection:
A- D1 B- D2 C- D3
974. Gastric cancer reaches muscularis propria:
A- T1 B- T2 C- T3 D- T4
975. Alcoholic old man, suddenly vomited and developed chest pain and dysphagia,
diagnosis? Perforated esophagus
976. Which system less affected by ischemia?
A- cardio B- GIT C-Liver D- Brain

PLEAZE, ANY NOTE OR CORRECTION OR ADDITIN SEND: [email protected]


‫نسألكم الدعاء بظهر الغيب‬
P a g e | 96
977. Female patient 34 years old, G4 P2, C/O pf abd. Pain, nausea, vomiting, during her 1st
trimester of pregnancy, HCG > 100, pelvic U/S shows snowstorm, which of the following is
true: risk of malignancy 20 %
978. Treatment of proctalgia fugax??
A- Amitriptyline B- Salbutamol inhaler C- Puborectalis release D- Biofeedback

979. Picture of dark colored nail resulted after trauma by heavy blunt object, what is
thediagnosis??
A- Subungual hematoma B- Malignant melanoma subungual type
980. What is true from the following choices? (all choices describe acute pancreatitis)
S.amylase level decrease to normal after 3-4 days
981. Old age female with midline neck swelling appears 3 days back, what is the fate of this
swelling?
A- Infection B- Resolve spontaneous C- Laryngeal compression
982. Female patient with history of right breast trauma, she has firm to hard mass, diagnosis?
A- fat necrosis B- breast cancer C- breast hematoma
983. ‫شيال‬exposed to fall of building over him، patient entrapped and get out after 3 hours، both
lower limbs had multiple bruises and he has oliguria، what is the cause of this condition ?
A- Reperfusion injury B- Acute limb ischemia
984. Which facial bone fracture is associated with diplopia on upward gaze?
A- Inferior orbital wall B- Medial orbital wall C- Lateral orbital wall
985. Picture of intestinal loop after constriction showing strangulation, he asked about the
pathology?
A- Venous congestion B- Arterial injury C- Shiny serosa
986. A case of hypothyroidism, patient developed hypotension, hypercapnia, hypoxia, and the
patient collapsed, what is the most urgent management??
A- O2 therapy B- IV thyroxine C- Cortisone D- Mechanical ventilation
987. Which of the following has the most prognostic value in acute pancreatitis??
A- Amylase B- Lipase C- AST D- Po2
988. Diabetic patient with gangrene in the forefoot, will undergo for amputation, which type??
A- Syme amputation B- Mid metatarsal amputation C- Below knee amputation
the of curvature lesser the from coming lesion Exophytic ‫صورة اشعه لمقطعية بالمعدة توضح االتي‬989.
stomach of patient suffering from dyspepsia and fullness, diagnosis??
A- Leiomyosarcoma B- GIST
990. ‫صورة اشعه عادية لل‬Sigmoid volvulus (diagnosis, treatment,)
991. Brain tumors
A- Golden investigation is CT

PLEAZE, ANY NOTE OR CORRECTION OR ADDITIN SEND: [email protected]


‫نسألكم الدعاء بظهر الغيب‬
P a g e | 97
B- Size affect symptoms rather than the site
C- Prognosis is better in females more than males
D- Prednisone is important in management brain tumors
992. Ankle brachial index is ……. For patient with gangrene in the big toe, suffering from
claudication pain and rest pain.
A- 1 B- 0.6 C- <0.3
993. Picture of gastric ulcer, biopsy was taken showing the ulcer is limited to the mucosa of
stomach, what is the appropriate management?
A- Wedge resection B- Gastrectomy C- Endoscopic mucosal resection
N.B Early-stage gastric cancer can be resected by endoscopy
994. Stap wound in the chest, patient has abdominal pain, what is the best investigation for
detection of diaphragmatic injury?
A- Thoracoscopy B- Laparoscopy C- CT D- US
995. A case of blunt abdominal trauma, patient has right upper quadrant pain and lower chest,
with pain in the tip of right shoulder, which organ of the following is most likely to be injured?
A- Liver B- Stomach C- Spleen D- Intestine
996. A case of sport hernia, pathology?? Tear of the posterior wall of inguinal canal
997. Scenario of 100 colorectal polyp, with positive family history of FAP syndrome,
management??
A- Colectomy B- Miles operation(APR)
998. Baroreceptor affect …?
A- Affect cop B- Increase vascular resistance
999. Cancer colon patient developed SOB, Investigation?? CXR
1000. Female patient with acute mesenteric ischemia, best investigation??
Abdominal radiography
‫مريض كان هيعمل بتر لقدم سكرى اليمنى والممرضة عقمت اليسر ى والطبيب افتكر كويس انه هيبتر اليمنى فتفادى‬miss Near 1001. ‫بتر‬
‫اليسرى‬
1002. The most appropriate investigation foe assessment of a 2 yrs. child with bilateral
undescended testis? Laparoscopy
1003. A nurse with needle stick injury, she refused to report the infection control, what will you
do?
A- Report the hospital B- Leave here go home C- Write incident report
1004. Basics of patient medical services??
A- Safety B- Patient centered care
1005. Staging of cancer rectum reaching the perirectal fat, with lymph node metastasis and
liver Mets? T3N2M1

PLEAZE, ANY NOTE OR CORRECTION OR ADDITIN SEND: [email protected]


‫نسألكم الدعاء بظهر الغيب‬
P a g e | 98
1006. The most important investigation in case of chronic pancreatitis is??
A- MRCP B- CT C- ERCP D- MRI
1007. Male patient with injury of foot, took antibiotic, then developed sepsis, what is the
pathology? Increased capillary permeability
1008. Best investigation of cancer esophagus? EUS
1009. A case of insulinoma, can't be detected by CT, what is next? EUS
1010. A Case Of pancreatitis, admitted to the ICU, developed hypoxia, diagnosis?? ARDS
1011. Patient underwent pancreatectomy, on 3rd day post op. patient developed frothy
discharge, what is the cause?
A- Decrease COP B- Decrease vascular resistant
1012. Child ingested aquastic material, and give picture of barium swallow showing
esophageal stricture in the distal segment, management?
A- Esophagectomy
B- Endoscopic esophageal dilatation
C- Stent
1013. Child with respiratory distress, critical case and need urgent procedure, parents refuse
the procedure, what is your action?
A- Respect their wishes
B- You will do the procedure as it is lifesaving
C- Try to convince the parents
1014. Patient underwent total gastrectomy and esophagojejunostomy, after drinking, clear or
frothy discharge, what is the diagnosis?
A- Duodenal blow out B- Leak from the anastomosis
1015. Patient went to Mexico, then developed right hypochondrial pain, diagnosis??
Amoebic liver abscess
1016. What is the investigation of choice for amoebic liver abscess? Serology
1017. A case of chronic pancreatitis with multiple strictures and dilatation, how to manage?
A- Distal pancreatectomy
B- Lateral pancreatic jejunostomy
C- Pancreaticoduodenectomy
1018. The most important investigation of mucinous cystadenoma?
A- CEA B- Amylase
1019. A case of patient with painless right UQ swelling, Courvoisier sign, diagnosis?
Cancer head of pancrease
1020. A case of diabetes insipidus, polydipsia, polyuria post head trauma, what is the cause?
Loss of ADH

PLEAZE, ANY NOTE OR CORRECTION OR ADDITIN SEND: [email protected]


‫نسألكم الدعاء بظهر الغيب‬
P a g e | 99
1021. The most common cause of chronic pancreatitis is??
A- Alcoholism B- Acute pancreatitis
No choledocholithiasis in the choices
1022. ‫ صورة اشعه‬of aortic aneurysm mainly to the left side e right lower abdominal pain, radiated
to the right testis, diagnosis??
A- Appendicitis B- Diverticulitis C- Aneurysm dissection
1023. Married female 28 yrs. old presented with acute abdominal pain and hypotension last
menstrual cycle was 1.5-month, diagnosis?? Rupture ectopic pregnancy
1024. Female patient presented with continuous vomiting of 3 days and then developed
abdominal pain, diagnosis?? Torsion ovary
1025. ‫صورة‬adhesions and gangrenous patch of intestine during laparoscopic exploration، how
to manage??
A- Laparotomy B- Laparoscopic repair C- Ileostomy
1026. Female patient presented with abdominal swelling 10*10 cm post umbilical hernia repair,
afebrile, not expansile on cough, diagnosis? Post op seroma
1027. Irreducible inguinal hernia, pain, no fever, type of repair?
A- Lockwood repair B- Herniotomy C- Liechtenstein repair
1028. Patient with previous bilateral hernia repair, the best procedure for management
recurrent hernia? TAPP
1029. Thin-walled gall bladder, diagnosis? GB mucocele
1030. Patient presented on the 7th day post op. with chest pain with decrease air entry on
lower part of left chest wall, temp 39, diagnosis?
A- Atelectasis B- Pneumonia C- DVT, PE D- UTI
1031. Patient with colon surgery with anastomosis, presented with leak from site of
anastomosis, fever, high TLC management?? Reexploration
1032. The best investigation to differentiate between perforated peptic ulcer and acute
pancreatitis??
A- Ct with contrast B- Abdominal x ray C- MRI
1033. The best operation for management of perforated duodenum?
A- Repair with omental patch B- Repair with truncal vagotomy
C- Repair with selective vagotomy D- Repair with antrectomy
1034. Patient with leukemia, presented with swelling coming from anus, itching, bleeding,
diagnosis??
A- Sentinel pile B- Anal carcinoma
1035. Best fluid management for ileostomy diarrhea?? Ringer lactate

PLEAZE, ANY NOTE OR CORRECTION OR ADDITIN SEND: [email protected]


‫نسألكم الدعاء بظهر الغيب‬
P a g e | 100
1036. Another scenario with hypernatremia diarrhea, best resuscitated with??
Hypotonic solution
1037. Patient with severe facial trauma associated with facial fracture, anesthesia can't
intubate the patient, next?
A- Needle cricothyroidotomy B- Surgical cricothyroidotomy C- Tracheostomy
1038. A case of chronic liver disease with hepatic mass, how to assess his condition??
A- CHILD PUGH CLASSIFICATION B- MELD CLASSIFICATION
1039. Patient with generalized lymphadenopathy, axilla, neck, suffer from intermittent attacks
of low-grade fever, and has cystic lesion in the spleen, diagnosis??
A- Kala azar B- Malaria C- TB
1040. Patient presented with pain in the left upper quadrant pain, with, irritation of the
diaphragm. bl pr 100/70, tachycardia, how to investigate? US
1041. What is the earliest sign of hypovolemic shock?
A- Tachycardia B- Wide pulse pressure
C- Hypotension D- Disturbed conscious level
1042. Patient with high blood glucose and glucosuria, management? IV fluids
1043. For Whom you will report the device which is not working well, even in other hospitals??
Hospital administration
1044. How to avoid false amputation of foot? Marking
1045. A case of post op. with pulmonary edema? Antidiuretic hormone
1046. During laparotomy, there is perforated ilium 20 cm proximal to the ileocecal valve, with
adhesions between the distal part of ilium and the colon, wall of the ileum and the mesentery
are thickened, diagnosis??
A- Crohn's disease B- TB(rare perforation) C- Actinomycosis D- Ilial perforation
1047. Sign of chromic dehydration
A- Muscle wasting (true answer) B- Sunken eye
1048. Scenario of patient smoker for 50 years, with prostatism presented with hernia, the most
important risk factor?
A- Smoking B- Prostatism C- Constipation
1049. Same scenario, for previous patient as a preoperative investigation which is the best??
A- Endorectal US B- CXR and ECG
1050. Dusky ileostomy, management?
A- Hot sponging B- Expectant management
C- Refashioning D- closure of ileostomy
1051. Patient underwent colostomy, he developed mass beside the colostomy show impulse
on cough, diagnosis?? Parastomal hernia

PLEAZE, ANY NOTE OR CORRECTION OR ADDITIN SEND: [email protected]


‫نسألكم الدعاء بظهر الغيب‬
P a g e | 101
1052. Patient with 25 % 2nd and 3rd degree burn, patient condition improved and burn wounds
healed, and patient is for discharge home, what is your advice to him?
A- Tell him burned area may be erythematous and itchy
B- Apply pressure garment all the day and take it off at night
C- If skin fissuring occurs, do sterile dressing and report your doctor
D- Apply skin emollient
1053. Patient has deep wound in the hand, doctor prescribed antibiotic for him, patient took
AB only 2-3 days then stopped, then he presented by fever, hypotension, diagnosis?
A- Tetanus infection B- Septic shock
1054. Patient with history of previous abdominal operation while examination, there is
indentation on the anterior wall of rectum (pelvic abscess), how to manage?
A- CT guided aspiration B- US guided aspiration C-Transrectal drainage
1055. Stap wound in the chest, patient has congested neck veins, muffled heart sounds,
diagnosis?? Cardiac tamponade
1056. Scenario of chest trauma, patient presented with trachea shifted to right side,
hyperresonance by auscultation, diagnosis?
A- Left tension pneumothorax B- Right tension pneumothorax
1057. RTA patient with dullness over the chest, non-audible breath sounds diagnosis??
Hemothorax
1058. Submucosa appear in endoluminal US as ……??
A- Second hypoechoic layer B- third hyperechoic layer
1059. The best investigation for T staging of colon cancer is??
A- EUS B- CT C-MRI (for nodal staging)
1060. The best investigation for detection of obturator lymph node in cancer rectum is …??
MRI
1061. Case of crushed both lower limbs with hyperkalemia, what is most concern?
Cardiac arrest
1062. Patient known hypothyroidism, post operation, she developed confusion, hypoxia,
hypercapnia, (myxedema coma), treatment??
A- Cortisone B- IV levo thyroxine C- Mechanical ventilation
1063. Patient with huge colloid goiter with pressure manifestations like dyspnea and
dysphagia, patient is euthyroid, management?
A- Anti-thyroid drugs B- Radioactive iodine C- Subtotal thyroidectomy
1064. Which of the following improve exophthalmos??
A- Antithyroid drugs B- Steroids C- Thyroidectomy

PLEAZE, ANY NOTE OR CORRECTION OR ADDITIN SEND: [email protected]


‫نسألكم الدعاء بظهر الغيب‬
P a g e | 102
1065. Multiple PNS opening of abscess are seen laterally in a picture, what is the pathology??
A- Abnormal hair growth B- Infected congenital sinuses C- Bad hygiene(with abscess)
1066. Post Thyroid surgery, patient developed loss of high-pitched sound, which nerve is
affected??
A- SLN B- RLN C- BOTH
1067. Duodenal ulcer, superficial ulcer ‫ صورة‬what is the priority in treatment?
A- Antacid B- Eradication of H pylori
1068. Patient presented with pus and discharge from umbilicus, what is the best investigation?
Sinogram
1069. Female patient with bone pain, flank pain, asthenia, high PTH, high level of ca, what is
the diagnosis?
A- Primary hyperparathyroidism B- Parathyroid adenoma
1070. Old age female with non-tender parotid swelling, well defined smooth, diagnosis??
A- Pleomorphic adenoma B- Lymph node C- Mucoepidermoid tumor
1071. Patient presented with thyroid nodule, normal TSH and normal T3 T4, the best
confirmatory inv??
A- MRI B- US C- CT D- FNAC
1072. PIC of endoscopy, showing round gastric swelling, diagnosis, treatment??
GIST, WEDGE RESECTION
1073. PIC of GIST, the most prognostic factor??
A- Size B- Mitotic figure
1074. Distance 15 cm from incisor teeth
A- Inferior constrictor muscle
B- Diaphragm
C- Aortic and bronchial constriction
1075. Commonest sign of bone metastasis in breast cancer?
A- Shaft femur B- Spine
1076. Which of the following is more important in wound healing??
A- Antibiotic B- Wound debridement
1077. During laparotomy for sever peritonitis, neglected patient, incidental perforated ilium,
management??
A- Repair B- Ileostomy C- Resection and anastomosis
1078. Most common malignancy of the omentum: Metastases
1079. What is the most reliable method to help determine the cause of ascites?
The serum-ascites albumin gradient (SAAG)

PLEAZE, ANY NOTE OR CORRECTION OR ADDITIN SEND: [email protected]


‫نسألكم الدعاء بظهر الغيب‬
P a g e | 103
1080. How do you calculate the SAAG?
Serum albumin concentration - ascites albumin concentration
1081. How much albumin should be given for every liter of ascites removed after large-volume
paracentesis (>5 L)?
6 to 8 g of albumin/L of ascites removed
1082. Most common malignancy associated with chylous ascites: Lymphoma
1083. Most common early complication following hernia repair: Urinary retention

1084. What is the time period that hernias usually recur?


Usually within the first 2 years after repair
1085. What is a sports hernia?
Characterized by a dilated superficial ring of the inguinal canal and chronic groin pain in
athletes

1086. A 49-year-old male has a right-sided perihilar mass incidentally found on CT


scan performed after a motor vehicle trauma 1 month ago. He has a 30 pack-year smoking
history.He reports his clothes fit more loosely. On examination, he has purple striae on his
abdomen and prominent fat on his posterior neck. CT scan confirms a 4-cm irregular mass
without enlarged lymph nodes. Treatment is most likely to involve which of the following?
A- Radiation therapy alone B- Combination chemotherapy and radiation
C- Surgical resection D- Mediastinoscopy and if node negative, then surgical resection.
1087. A 4-year-old boy presents to the ED after an accidental ingestion of lye. The child is
exhibiting stridor and hoarseness. The child is emergently intubated and resuscitated with IV
fluids. An upright chest radiograph shows no evidence of perforation. The patient is admitted
to the intensive care unit. The next step in the management is:
A. Esophagoscopy B. Esophagography
C. Chest computed tomography scan D. Close observation
1088. A patient with multiple small bowel resections for Crohn’s disease presents with an
acute flare. Due to his preexisting short length of bowel and current profuse diarrhea, he is
suspected of having functionally a short gut syndrome and is started on TPN. Refeeding
syndrome is characterized by which of the following electrolyte abnormalities?
A. Hyponatremia, hypokalemia, and hypercalcemia
B. Hyperphosphatemia, hypokalemia, and hypocalcemia
C. Hypokalemia, hypomagnesemia, and hypophosphatemia
D. Hypocalcemia, hyponatremia, and hypomagnesemia
1089. The most common virus transmitted by transfusion is (CM VIRUS)
A. Hepatitis A B. Hepatitis B C. Hepatitis C D. (HIV)

PLEAZE, ANY NOTE OR CORRECTION OR ADDITIN SEND: [email protected]


‫نسألكم الدعاء بظهر الغيب‬
P a g e | 104
1090. 30-year-old man with no past medical history undergoes emergency hernia repair. He
has a 10-pack-per-year smoking history. What is his American Society of Anesthesiologists

PLEAZE, ANY NOTE OR CORRECTION OR ADDITIN SEND: [email protected]


‫نسألكم الدعاء بظهر الغيب‬
P a g e | 105
(ASA) physical status classification?
A. ASA class I B. ASA class II C. ASA class III D. ASA class V
1091. Most common symptoms of esophageal perforation
A- Sever chest pain B- Dyspnea C- Fever D- Emphysema
1092. Hiatal hernia e GERD. Tx
A- Partial fundoplication B- Nissen fundoplication C- Heller seromyotomy repair
1093. Hiatal hernia e GERD. Mechanism
A- short esophagus B- LES incompetent
1094. Mid esophageal mass, celiac LN. Patient well for surgical procedure
A- Chemo-radiotherapy B- Stent C- Esophagectomy D- Chemotherapy
1095. Tx of Zencker diverticular
A- Pharyngo-diverticulectomy
B- Diverticulo-myotomy
C- Diverticulectomy + Cricopharyngotomy
1096. Stab wound in chest, S/S of cardiac tamponade
A- Cardiogenic shock B- Neurogenic shock
C- Hypovolemic shock D- Obstructive shock
1097. Most prognostic factor for Esophageal cancer
A- LN B- Depth C- Vascular invasion
1098. Most common cause of lower GI bleeding
A- Diverticulosis B- Dieulafoy C- Intussusception
(no hemorrhoid in choice)
1099. History of anaplastic thyroid, S/S of Dysphagia, no Dyspnea. Next Dx test
A- FNA B- Open biopsy C- U/S D- Neck CT
1100. First fluid for burn? RL
1101. First fluid for pelvic fracture, V/S stable? RL
1102. Picture of Electrical burn. Ask for stasis zone where?
A- Upper chest B- Arm C- Fore arm
1103. Same Role of 9 How mush percentage left upper LL + partial of chest?
A- 4,5 B- 9 C- 13.5 D- 18
1104. Hydatic cyst in lung (Rt lobe). Tx
A- Pneumonectomy B- Cyst excision C-PAIR D- Albendazole
1105. Liver abscess. Tx? Percutaneous Drainage
1106. Brain CT photo/ patient Hx of melanoma treatment, hydatid cyst
A- Brain metastases by melanoma B- Hydatic cyst brain

PLEAZE, ANY NOTE OR CORRECTION OR ADDITIN SEND: [email protected]


‫نسألكم الدعاء بظهر الغيب‬
P a g e | 106
1107. Two months head trauma, vomiting, tiered, affect conscious level. CT Subdural
hematoma. Management?
A- Serial CT B- Burr hole C- Craniotomy
1108. Trauma, Diabetic keto-acidosis, O/E peritonitis. lab keto-acidosis. Best initial
management?
A- Fluid replacement B- Laparotomy C- Ab D- Oral hypoglycemic
1109. Trauma, rigid abdomen, V/S stable. Best initial Tx
A- Exploration laparotomy B- Fluid resuscitation C- US/CT
1110. Best prognosis breast cancer? Tubular
1111. Anesthesia put two different medication same shape and color. What call this error
A- Medication error B- Latent error C- Anesthesia error
1112. Patient in CPR. you want to examine genitalia. What to do consent
A- Shout you want to do exam B- Do it C- Wait for family
1113. Patient with gangrenous LL, unconscious, family refuse operation because they said he
will die
A- Follow family consent B- Push for amputation
C- Ask another surgeon for amputation then proceed
D- Meet ethical committee
1114. Patient trauma, OR done, post op pulmonary edema. Hormone cause? ADH
1115. Necrotic pancreatitis, peritoneum fluid……, +ve E coli. Next?
A- Exploration, debridement B- Antibiotic C- Percutaneous drainage
1116. Mucin cyst of pancreas? ↑CEA and low amylase
1117. Melanoma 4mm, excision? 20mm-30mm
1118. Local advanced breast cancer é HER +ve, vertebral metastasis. Tx?
A- Mastectomy + vertebral radiotherapy
B- Mastectomy + vertebral chemotherapy
C- Mastectomy + chemotherapy
1119. Femoral hernia. TX?
A- Lockwood procedure B- Lichtenstein procedure
C- Laparoscopic surgery D- Herniotomy
1120. Crohn disease, +ve peritonitis x4 days ileum perforation 20cm from ileo-cecal valve. TX?
A- Resection –anastomosis B- Primary repair
C- Loop ileostomy D- Rt Hemicolectomy
1121. Patient é liver cyst é peripheral calcification. Dx?
A- Hydatic cyst B- Pyogenic abscess

PLEAZE, ANY NOTE OR CORRECTION OR ADDITIN SEND: [email protected]


‫نسألكم الدعاء بظهر الغيب‬
P a g e | 107
1122. Patient with compartment syndrome of abdomen. CVP=16mmgh. what is degree of
abdomen compartment?
A- I B- II C- III D- IV
1123. Case Fever post Rt hemicolectomy 24h post-operative
A- Atelectasis B- Leak C- UTI
1124. S/S of IO, Xray pneumobilia.TX? Enterotomy (ileotomy é stone
removal)

1125. MCC of chronic pancreatitis? Alcohol


1126. Hypokalemia + diarrhea? VIPoma
1127. Most common site of Gastrinoma in pancreas?
A- Head B- Body C- Neck D- Tail
1128. Case of mucosa é lip pigmentation é intussusception. Dx? Peutz Jeger syndrome
1129. Septic shock? ↑ Capillary permeability.
1130. Post open cholecystectomy, with abdominal wall pulge? incisional hernia
1131. Which lab ↑ initially ............for Dx acute pancreatitis
A- amylase B- lipase C- BUA D- Hematocrit
1132. Photo intussusception for adult. What to do before surgery resection?
A- Sigmoidoscopy B- Hydrostatic reduction
1133. Abdominal pain, fever, GI bleeding, Diarrhea?
A-Typhoid B- ulcerative colitis
1134. Photo rectal prolapsed. MCC? Chronic constipation
1135. Anterior anal fissure. Cause? Delivery
1136. Anal fistula e diarrhea? Colonoscopy
1137. Triangle of DOOM, medial boundary? Vas deferens
1138. Triangle of pain boucle medially boundary? Spermatic vessels
1139. Most common complicated hernia?
A- Femoral B- umbilical C- inguinal
D- paraumbilical E- incisional
1140. Central neck cyst, fluctuated, red. Best management?
A- Sistrunk B- Incision and drainage C- Aspiration D- Antibiotic
1141. Case patient complain while lifting have object develop chest pain, X-ray
pneumomediastina? Boerhaave syndrome
1142. Old age C/O pain while eating? Chronic mesenteric ischemia
1143. History of MI, LGI Bleeding? Sigmoidoscopy
1144. Sigmoid ulcer + ascending colon polyps. Which Best investigation to do initially?
A- MRI B- CT C- EUS D- Barium enema

PLEAZE, ANY NOTE OR CORRECTION OR ADDITIN SEND: [email protected]


‫نسألكم الدعاء بظهر الغيب‬
P a g e | 108
1145. Lateral Neck mass. +ve Transillumination, ↑ size with crying?
A- Branchial cyst B- Cystic Hygroma
1146. Most Common abdominal Tumor in children? Neuroblastoma (<2years)
1147. First hormone secreted from kidney as a response for renal
hypoperfusion(=hypovolemia)
A- Renin B- Aldosterone
1148. Post EGD scope, perforation. What Next? Gastrographin
1149. Dx of achalasia? Manometry
1150. Graves' disease? TSH stimulation Anti body
1151. T3, T4 normal, TSH ↑, on thyroxine
A- Hypothyroidism B- Hyperthyroidism
1152. Spherocytosis? Do US before OR for gall bladder stones
1153. Biliary Tree injury, T tube done, sub hepatic drain removes, ↑↑bilirubin 800. Cause
A- distal stone B- biliary injury
1154. Distended GB, intra / extra hepatic dilated, weight loss? Head of pancreas cancer
1155. Liver amebiases? Serology
1156. Hydatic liver? ELISA
1157. Sign of chronic cholecystitis. photo GB
A- Thick wall B- dilated cystic duct
1158. Dx of Acute pancreatitis
A- CT B- US C- ERCP D- HIDA
1159. To prevent infection of Necrotizing pancreatitis?
A- Enteral feeding B- Antibiotic C- Punction aspiration D- Necroscopy
1160. Decreased risk of local surgical site infection?
A- Clippen of hair B- Prevent perioperative hypothermia
1161. Basketball player C/O sudden sever pain in Rt lower abdomen, scrotum free?
A- Sport hernia B- Hematoma
1162. Ovary Tumor primary origin in
A- Stomach B- Colon
1163. Gastric bleeding after epinephrine injected, Rebleeding 24h? Endoscopic
reinjection

1164. Injury of forearm, inability of flexion index and middle finger? Median nerve injury
1165. Tumor resected with Microscope +ve? R1
1166. Dx of acute cholecystitis. Initially? US
1167. Rectal fistula without perianal opening. CT pelvic collection on posterior aspect?
A- Pelvi-rectal fistula B- intersphincteric abscess

PLEAZE, ANY NOTE OR CORRECTION OR ADDITIN SEND: [email protected]


‫نسألكم الدعاء بظهر الغيب‬
P a g e | 109
1168. In OPD, female 30 years old patient C/O abdominal pain e WBC 35000
A- Acute pancreatitis B- Diverticular abscess
C- Acute Appendicitis D- Pelvic inflammatory disease
1169. New married, Hypotension, ↑ HR, abd pain? Rupture Ectopic
pregnancy

1170. Hiatal hernia with GEJ above diaphragm short esophagus


management? Collis gastroplasty

1171. While doing appendectomy in 12 yrs. female you discovered simple ovarian cyst and you
can do it efficiently without increase in operation time what to do?
A- Leave the cyst as you didn't have consent
B- Go and get a written consent from her father
C- Do the proper interference without consent to prevent another laparotomy
D- Ask another colleagues opinion
E- Do the best for the patient (by your paternity)
1172. Scenario of perforated appendix course of antibiotic
A- 1-2 days B- 3-5 days C- 5-7 days D- 7-10 days
1173. Old age alcoholic patient CT pancreatic calcification dilation come to ER by sever
epigastric pain referred to back Amylase very high diagnosis?
A- chronic pancreatitis B- Acute on top of chronic pancreatitis
1174. Child hitted by hoarse in his abdomen mild tenderness guarding labs normal
investigation
A- U/S B- DPL C- X-RAY D- CBC
1175. Ulcer in lesser curvature medical ttt failed, need surgery type of surgery
A- Billroth 1 B- Billroth 1 & gastroplasty C-antrectomy+truncal vagotomy
1176. Open cholecystectomy 2 month ago presented by reducible swelling palpable edge
between operation scar and swelling
A- Dehiscence B- seroma C- Incisional hernia
1177. Surgeon do complete transection just above duodenum what will you do
A- Choledochojejonostomy B- Hepaticojejunostomy
1178. Scenario of shock stab wound chest admitted in hospital after few days' saturation↓. HR↑
A- hypovolemic shock B- cardiogenic shock C- septic shock
1179. Symptom of GERD manometry no achalasia Barium swallow dilated esophagus
&stricture ring
A- endoscopic dilation B- heller myotomy
C- laparoscopic myotomy D- nifedipine
1180. Stab wound UOQ abdomen mild tenderness in UOQ management vitally stable
PLEAZE, ANY NOTE OR CORRECTION OR ADDITIN SEND: [email protected]
‫نسألكم الدعاء بظهر الغيب‬
P a g e | 110
A- admitted &expectant management B- CT C- DPL

PLEAZE, ANY NOTE OR CORRECTION OR ADDITIN SEND: [email protected]


‫نسألكم الدعاء بظهر الغيب‬
P a g e | 111
1181. TNM stage in cancer rectum to tumor invade mesorectum pararectal LN secondaries in liver
Only one choice has M1.
1182. Melanoma in lateral side of leg clerk ‖ biopsy done management
A- wide local resection with safety margin B- resection with inguinal LN dissection
1183. Blunt abdominal trauma in RTA UOQ tenderness x ray normal there is CT image (image
of CT not clear for me
A- liver tear B- gastric perforation C- gall bladder rapture!!
1184. Junior resident first time in appendectomy senior consultant assists him but consultant
was angry because he was late so he did operation by himself after closure of fascia senior
resident came to or and said in loud voice this is wrong patient about this situation
A priority for patient safety
1185. Child one year his mother feel mass in rt lumbar region diagnosis
A- neuroblastoma B- nephroblastoma C- lymphoma
1186. Female pt. with 2 years ago cholecystectomy came with jaundice cause
A- retained stone B- CBD stricture
1187. Gene in cancer colon non hereditary
A- APC gene B- P 53 C- cross gene D- MLH gene
1188. Stomach staging mass reach musclaris propria
A- T1 B- T2 C- T3 D- T4
1189. FB in rectum inverted mug appear in image you can palpated by PR dilated anus about 6
cm from anal verge and tear about 50% of circumference Tx
A- bimanual judgment
B- laparotomy and open rectum
C- laparotomy and squeeze rectum
1190. Anaethiast try many sizes of mask to one patient? standardization
1191. Amputation in pt. unconscious ,relatives refuse the operation
A- do operation because its life saving B- obey his relative C-ethical comitte
1192. Submucosa of esophagus appear in EUS as? Third hyperechoic layer
1193. Old age with cut wound in finger he stops antibiotic after 2 days one week after he came
to ER by picture of? Septic shock
1194. Immunized pt. 12 years ago injured wit barbie wire Tx
A- Tetanus Toxoid B- Immunoglobulin
C- antiserum toxin D- Tetanus Toxoid & immunoglobulin
1195. Addict patient with multiple pricks on forearm subcutaneous gas (gas gangrene) very
bad condition metabolic acidosis 7.12 hypoxia

PLEAZE, ANY NOTE OR CORRECTION OR ADDITIN SEND: [email protected]


‫نسألكم الدعاء بظهر الغيب‬
P a g e | 112
A- ICU admission B- fluid replacement
C- massive debridement D- Antibiotic
1196. Upper GI sever bleeding endoscopy done after resuscitation failed patient receive 2 unit
of blood next step
A- capsule endoscopy B- Another endoscopy
C- laparotomy& ligation D- tagged RBC
1197. Middle age female patient with mass in UOQ in breast not fixed to skin or muscle, mass
1.5 cm biopsy was taken and reveal infiltrative duct carcinoma, no Mets, negative LN Tx?
A- Wide local excision with sentinel LN B- MRM!! C- Lumpectomy
1198. Scenario of hypovolemic shock of RTA clear sign of grade 3 hypovolemia shock best
fluid for resuscitation
A- RL B- Blood
1199. Parameter of hypovolemic shock HR 110 mild hypotension? Grade 2 hypovolemia shock
1200. Adenocarcinoma of stomach undergo operation pathology report positive margin
A- R1 B- R2 C- R3 D- R4
1201. RTA spine # spinal cord injury? neurogenic shock
1202. Neurogenic shock presentation? Bradycardia hypotension warm
hand

1203.Septic shock scenario needs pathophysiology? Decrease arteriole


resistance ↓

1204. Mass in esophagus, best investigation? EUS (endoscopic U/S)


1205. Paraoesophageal hernia, management? Surgical repair
1206. Scenario of a patient, filling defect in radiography, no loss of wait, and there’s gastric
upset,
A- Lymphoma B- Gastric carcinoma (leiomyosarcoma)
C- GIST D- leiomyoma (in esophagus)
1207. Perforated peptic ulcer, what’s the definitive ttt? Repair + graham patch
1208. Complications of gastric operation, Billroth I, II
A- Dumping syndrome (palpitation, sweating, and hypotension ½ hr. after meal)
B- Pain after meal relieved after drinking (bile reflux gastritis)
1209. A patient with myeloproliferative disease, presented with pain in left hypochondrium
and referred to the back, what’s the pathophysiology of the pain?
A- Hemorrhage B- Infection C- Aneurysm D- Infarction
1210. Gastric lymphoma, what’s the best Tx?
A- Radiotherapy B- Chemotherapy C- Surgery
1211. Pathophysiology of exophthalmos in grave’s disease?
PLEAZE, ANY NOTE OR CORRECTION OR ADDITIN SEND: [email protected]
‫نسألكم الدعاء بظهر الغيب‬
P a g e | 113
A- Activation of B-lymphocyte B- Activation of T-lymphocyte
C- Thyroid stimulating antibody D- Thyroid inhibitory antibody

PLEAZE, ANY NOTE OR CORRECTION OR ADDITIN SEND: [email protected]


‫نسألكم الدعاء بظهر الغيب‬
P a g e | 114
1212. A patient in 10 weeks he shows rapidly growing thyroid, fine needle showed it’s
malignant, what’s the type of malignancy?
A- Follicular B- Papillary C- Medullary D- Anaplastic
1213. A patient with hard thyroid (Riedel’s thyroiditis), management?
Isthmectomy
1214. Thyroid mass >1.3 cm, management? Fine needle (biopsy)
1215. Most common disease in lactating breast?
A- Bacterial mastitis B- Abscess C- Fibrocystic disease
1216. Cyst in breast aspirated and fluid was clear but not completely resolved, next step?
A- Mammography B- Fine needle C- Core biopsy
1217. Female patient with trauma to the breast (picture), mass in upper outer quadrant, not
tender, no axillary lymph nodes? Fat necrosis
1218. Case of Crohn’s, Pathophysiology? Transmural fibrosis
1219. A case of intussusception (picture), scenario of a patient came with intestinal
obstruction and opened up for surgery and the surgeon find the (picture) intra operative, what
should be done pre-operative for this case?
A- Air enema (pneumatic reduction) B- Barium enema C- colonoscopy
1220. A picture showing mass in right hypochondrium reaching umbilicus, there is
multipledaughter cells (hydatid cyst in liver), investigation? ELISA
1221. Treatment of previous case? Surgical
1222. Scenario of a patient travelled to Mexico and returned after a short while he suffered
from a stroke and became well then, he developed CNS manifestations and done CT and this
(picture) was found, what’s the diagnosis?
A- Cerebral stroke B- Hydatid cyst in brain
1223. Best method to measure the severity of liver mass?
A- CHILD classification B- MILD classification
1224. A case of a patient, complaining of discharge from anal canal, by inspection there’s no
opening, MRI was done (picture shows a track from 2 o’clock to 6 o’clock), diagnosis?
A- Ischiorectal abscess B- Levator ani abscess C- Cryptoglandular abscess
1225. Anal fistula (picture shows an opening), what’s the type of fistula? Low anal fistula
1226. Mass in anal verge, 5 cm away, most important thing in treatment?
To detect whether there’s meso-rectum invasion or not
1227. Isolated fracture femur, management?
A- Thomas B- Cast
C- Intra-medullary nail (open technique) D- Inter-medullary nail (closed technique)

PLEAZE, ANY NOTE OR CORRECTION OR ADDITIN SEND: [email protected]


‫نسألكم الدعاء بظهر الغيب‬
P a g e | 115
1228. Senior resident ortho noticed that rate of infection is high , on reviewing the OPD
f i l e s he noticed that the consultant doesn’t take the proper anti septic measures, what’s the
type of

PLEAZE, ANY NOTE OR CORRECTION OR ADDITIN SEND: [email protected]


‫نسألكم الدعاء بظهر الغيب‬
P a g e | 116
this study?
A- Prospective B- Retrospective
1229. wrong side of surgery, patient throw himself from his room (suicide attempt), what’s the
cause?
A- Close call B- Never event C- Absence of risk management protocol
1230. Female patient wants to have a weight reduction surgery but she is diagnosed with a
mental illness, what should be the management?
A- Psychiatric consultation B- Husband consent
C- Do surgery D- Take another surgeon’s opinion
1231. Female patient wants to have a cosmetic procedure in her face but she is diagnosed with
a psychiatric illness, what should you do?
A- Husband consent B- Psychiatric consultation
1232. Young female patient involved in an accident, came to ER with a compromised
vascularity to the lower limb, you should take her to surgery which among its complications is
amputation, her father is not present in the hospital but her grandfather is there, what should
you do?
A- Take her immediately to surgery (limb saving)
B- Taking her father’s consent on the phone
C- Taking her grandfather’s consent
1233. Patient with melanoma, depth > 2 mm, free margin of this lesion?
A- < 0.5 cm B- 1-2 cm C- 2-4 cm D- > 4 cm
1234. Picture of a lesion in the forearm and there’s axillary lymph nodes, free margin is
detected by?
A- Depth of the lesion (melanoma) B- Positive lymph nodes C- Size of the lesion
1235. Patient came to ER unconscious, open his eyes on verbal orders, confused, localize pain,
what’s GCS? 12
1236. Patient came to ER with fracture to facial bone, was playing football and a ball hit his
lower jaw causing fracture mandible, he was stable but suddenly he fainted, what’s the cause?
A- Hypoglycemia B- Hypoxia C- Hypotension
1237. A patient with intra cranial hemorrhage with increasing CNS manifestations,
management? Craniotomy
1238. Patient came to ER with solid hemothorax and fracture ribs, chest tube was inserted but
with no improvement, what should you do?
A- Re-insert another chest tube B- Mechanical ventilation
C- Ventilation under pressure D- Support airway + CT

PLEAZE, ANY NOTE OR CORRECTION OR ADDITIN SEND: [email protected]


‫نسألكم الدعاء بظهر الغيب‬
P a g e | 117
1239. Patient underwent surgery (gastrectomy), 3rd day postoperative he shows decrease air
entry on lower part of lt. lung, he was feverish 39.5?
A- Subphrenic collection B- Pneumonia
C- Catheter related infection D- Atelectasis
1240. What’s the most common complication of peptic ulcer?
A- Bleeding B- Perforation C- Malignancy
1241. Scenario of a patient underwent gastrectomy, 4th day postoperative there was a 100-cc
collection, patient is stable, what’s the source of leakage?
A- Duodenal stump blow out
B- Esophago-jejunostomy leak
C- Pancreatic leak
1242. Patient with impacted CBD stone, he did ERCP, sphincterotomy with stone extraction,
bleeding happened and hematemesis, 2 days after patient presented with repeated attacks of
hematemesis and melena, what’s the cause of pain?
A- Pancreatic injury(pancreatitis) B- Duodenal leak C- Active bleeder
1243. A picture of stomach with a lesion about 1.5 cm, diagnosed as a leiomyoma, what’s the
management?
A- Eneculation (if in esophagus)
B- Wedge resection (in stomach)
C- Total gastrectomy
1244. Leiomyoma, what’s the most prognostic factor?
A- Tumor size B- Mitotic count
1245. Scenario of a patient with multiple stones in distal part of pancreas, alcoholic patient and
there’s chain of lakes in main pancreatic duct?
A- Distal pancreatectomy
B- Longitudinal pancreatico-jejunostomy
C- External drainage
1246. Female patient, 65 years old, diabetic and on oral hypoglycemic, presented with severe
upper abdominal pain, increased amylase and lipase (acute pancreatitis), what is the cause?
A- Drug induced B- Fever C- Alcohol D- Gall stones
1247. Most common in MEN I? Parathyroid
1248. A patient presented with jaundice, with history of pancreatic cyst which was aspirated
and the fluid examined and showed low amylase and high glycogen level, what’s the type of
cyst?
A- Serous cyst B- Mucinous cyst
C- Pancreatic pseudo cyst D- Interpapillary mucinous neoplasm

PLEAZE, ANY NOTE OR CORRECTION OR ADDITIN SEND: [email protected]


‫نسألكم الدعاء بظهر الغيب‬
P a g e | 118
1249. Scenario of a patient with acute abdomen, pain relieved only by morphia, what’s the
most proper investigation to diagnose this case?
A- Kinase B- Urate C- Cytokine D- Abdominal radiography
1250. Scenario of patient admitted to ICU due to acute pancreatitis, later shows anuria,
distension, CVP 16, what’s diagnosis?
A- Intestinal obstruction
B- Acute abdominal compartment syndrome
1251. A trauma patient, bleeding, tachycardia 130 (grade III), best fluid for resuscitation?
A- Blood B- Ringer lactate C- Saline
1252. Scenario of a patient with hypovolemic shock, pulse 110, mild hypotension? (grade II)
1253. A patient with acute pancreatitis, admitted to ICU, increased creat., Bl. Pr. 80/30?
(Septic shock)
1254. Crohn’s case diagnosed from a long time, presented with exacerbation and diarrhea,
best drug to give this patient?
A- Steroids B- 5ASA C- Azathioprine
1255. Old age patient, with cirrhosis, ascites, U/S shows mass in liver about 3 cm, to do staging
for this case what investigation is the best?
A- CT brain B- Endoscopy C- Laparoscopy
1256. Alcoholic, abdominal pain, weight loss, diarrhea, multiple strictures and dilatations of
the pancreatic duct, multiple stones in the distal pancreatic duct?
Lateral pancreatojejunostomy
1257. Alcoholic, severe epigastric pain radiating to back, very high amylase and lipase, next
investigation?
A- ERCP B- CT C- MRCP D- EUS
1258. Clinical case of acute pancreatitis, best test for diagnosis?
A- amylase B- lipase
1259. Clinical case of acute pancreatitis, clinical and laboratory data of Ranson score are
given, age 45, low PO2, WBCs 21.000, LDH 400, AST 244, glucose 11 mmol/L (normal up to 5.5),
very high amylase?
A- a case of sever pancreatitis B- mortality rate < 5%
C- high amylase indicates worse prognosis
1260. Similar case, most important prognostic factor?
A- low PO2 B- age > 40 C- high amylase
1261. 90 kg patient with acute hemorrhagic pancreatitis in the ICU, urine output 30 mL/h,
shocked, resuscitation fluid? ringer lactate

PLEAZE, ANY NOTE OR CORRECTION OR ADDITIN SEND: [email protected]


‫نسألكم الدعاء بظهر الغيب‬
P a g e | 119
1262. Pancreaticoduodenectomy after traumatic injury of the pancrease and duodenum, drain
output is frothy secretions, capillary refill is 4 sec, cause of shock?
dilatation of peripheral blood vessels
1263. Another trauma patient (not remember details) with shock, capillary refil 2 sec,
cause of shock? hypovolemic
1265. Alcoholic, acute pancreatitis 3 months ago, there is still epigastric pain, on follow up
there is epigastric cyst 11 x 15 cm, management?
A- internal drainage B- conservative treatment C- external drainage
1266. Progressive jaundice, deep yellow scleral, dark urine, pale stool, distended gallbladder
with stones, dilated CBD, investigation?
A- CT B- ERCP
1267. Same case, likely diagnosis?
A- distal cholangiocarcinoma B- klatskin tumor
N.B: (there is no cancer head pancreas in the choices)
1268. Patient underwent lap cholecystectomy, image of postoperative ERCP showing dilated
CBD and two filling defects, diagnosis? missed stone
1269. Difficult laparoscopic cholecystectomy, converted to open cholecystectomy, drain
output increased from 100 mL in the first postoperative day to 600 mL after few days,
management? ERCP and stent
1270. Intraoperative picture during laparoscopic cholecystectomy, inability to define the
anatomy of Calot's triangle, what to do?
A- convert to open B- fundus first dissection
C- intraoperative cholangiogram D- do cholecystectomy
1271. Same picture, patient with acute cholecystitis that was initially managed conservatively,
medical treatment failed and urgent lap chole was done, dense omental adhesions to the gall
bladder were lysed, what indicates chronicity?
A- adhesions B- thick wall C- sub serosal hemorrhage
1272. 18-Female middle aged, right upper abdominal pain, vomiting, investigation:
US, HIDA scan, US showed thin-walled distended gall bladder, no fever or leukocytosis,
diagnosis?
A- mucocele B- empyema C- acute cholecystitis D- cancer gallbladder
1273. 20-Patient with dyspepsia or upper abdominal pain, US showed thick-walled gallbladder
with no stones, management?
A- lap chole B- conservative treatment
1274. 21-Patient underwent CBD exploration, strange image of post-operative T tube
cholangiogram (with dilated CBD, dye in second part of the duodenum, pancreatic duct and

PLEAZE, ANY NOTE OR CORRECTION OR ADDITIN SEND: [email protected]


‫نسألكم الدعاء بظهر الغيب‬
P a g e | 120
common channel also apparent), diagnosis?
A- choledochal cyst B- CBD stricture C- long common channel
1275. Lap chole 10 years ago, epigastric pain, fever, jaundice, investigation to help in the
diagnosis?
A- ERCP B- US C-MRCP D-CT
1276. 23-A straight forward case of cholangitis (long case with right hypochondrial pain, fever,
chills, jaundice, gall stones), after iv fluids what is your management?
A- iv antibiotics and ERCP B- observation
1277. 24-Patient with sudden severe epigastric pain, epigastric and right lower quadrant
tenderness, what initial investigation that will help you to reach the diagnosis?
A- X ray erect showing the diaphragm B- CT abdomen
1278. 25-Right upper quadrant pain, fever, jaundice, image of CT showing large cyst in the
liver with enhancing wall, diagnosis? liver abscess
1279. 26-Most common cause of liver abscess? ascending cholangitis
1280. 27-Right upper quadrant pain, fever, by imaging there is liver cyst, 40 mL chocolate pus
aspirated from it, responsible organism? entamoeba histolytica
1281. 28-Patient with right upper abdominal pain, palpable mass, she has pets in the house, no
fever or leukocytosis, liver cyst on imaging, I don't remember all details (but it was easy case
of hydatid cyst), how to reach the diagnosis?
A- ELISA B- aspiration of the cyst
1282. 29-Neonate (2 weeks) with jaundice, contracted gallbladder, invisible extrahepatic bile
ducts, time limit for surgery to prevent irreversible damage?
A- 8 weeks B- 12 weeks
1283. 30-T image of gastric mass, endoscopy and biopsy showed leiomyoma, what is the most
important prognostic factor?
A- size B- number of mitosis
1284. 31-Same pictures and scenario asking about most probable diagnosis?
A- GIST B- Leiomyosarcoma C- lymphoma (no leiomyoma)
1285. Direct question about the most diagnostic test for gastric outlet obstruction in infants?
A- US B- barium meal
1286. A patient with hypertrophied Rugel folds of the stomach, I don't remember the scenario
but some medical treatment was given and failed, asking about the management?
I choose total gastrectomy and I don't remember the other choices
1287. A patient was referred from the burn unit to the ER for massive lower GI bleeding, there
was no blood in the nasogastric tube?
A- upper GIT endoscopy B- angiography

PLEAZE, ANY NOTE OR CORRECTION OR ADDITIN SEND: [email protected]


‫نسألكم الدعاء بظهر الغيب‬
P a g e | 121
1288. Patient with stress gastritis, endoscope failed to stop bleeding due to profuse
haemorrhage from all over the gastric mucosa, patient was shocked, management?
A- angiographic embolization B- surgery
1289. Best investigation for bleeding Meckel' diverticulum? Technetium scan
1290. A patient with upper GI bleeding, there was no evidence of liver disease, peptic ulcer,
the cause? I choose Dieulafoy lesion
1291. Patient underwent closure of ileostomy, no oral intake, which fluid used for
maintenance?
A- dextrose saline B- saline C- ringer
1292. A trauma patient, short case with intact airway, adequate breathing and hemodynamics
(but blood pressure 100/60), what fluid used for initial resuscitation during evaluation?
A- ringer lactate B- normal saline C- blood
1293. Patient with liver cell failure and ascites, underwent paracentesis and become shocked,
resuscitation with?
A- saline B- ringer C- plasma D- salt free albumin
1294. Patient known GERD, progressive dysphagia over years, barium swallow showed
narrowing of the lower esophagus with intact mucosal pattern, manometry excluded
achalasia, management?
A- endoscopic dilatation B- esophagocardiomyotomy C- nifedipine
1295. Female patient with dysphagia to fluids, barium swallow showed smooth stricture of the
distal end of the esophagus, I don't remember all details but it was a typical case of achalasia,
best treatment?
A- balloon dilatation B- esophagocardiomyotomy C- nifedipine
1296. Patient with hurt burn and regurgitation (very short case), what is the best to diagnose
GERD?
A- endoscopy B- 24 h PH monitoring C- manometry
1297. A similar very short case of GERD, what is the most sensitive test to diagnose GERD?
A- endoscopy B- 24 h PH monitoring C- manometry
1298. Old female with dysphagia relieved by a large bletch (I think a case of paraoesophageal
hernia), investigation?
A- CT abdomen and chest with contrast B- barium swallow C- endoscope
1299. Old patient with dysphagia, some details are mentioned, patient diagnosed as metastatic
cancer esophagus, management?
A- radiotherapy B- stent C- esophagectomy
1300. After difficult endoscopy there is neck pain, crepitus, erythema?
A- perforation of the cervical esophagus B- Ludwig angina

PLEAZE, ANY NOTE OR CORRECTION OR ADDITIN SEND: [email protected]


‫نسألكم الدعاء بظهر الغيب‬
P a g e | 122
1301. After upper GI endoscopy, doctor concerned about perforation, how to investigate?
A- barium swallow B- diluted contrast swallow
1302. Old patient, with symptomatic inguinal hernia 10 years, smoke 2 packs per day 50 years,
dysuria and difficult micturition, residual urine 120 mL, prostate size 55 g, management?
A- hernioplasty B- herniotomy
C- TURP then hernioplasty D- alpha blocker then hernioplasty
1303. Exact same case, what is the most important cause of this hernia?
A- male gender B- old age C- BPH D- smoking
1304. 2 images, one of PUH with discolored skin, the other is intraoperative image of
strangulated loop, the loop appears to have a black spot in the image but the text describes
the loop as purple not black, a patient with 10-year history of reducible PUH, become
irreducible 8 hours, during surgery the sac contained serosanguinous fluid, the loop is purple,
management?
A- warm packs B- resection anastomosis
C- wedge resection D- reduce the loop into the abdomen
1305. The exact same pictures and case, the initial pathophysiology?
A- the adhesions B- the edge of the defect
C- venous obstruction D- arterial obstruction
1306. A history of long-standing reducible hernia, the hernia become irreducible 1 week ago,
no pain, the hernia is soft and not tender?
A- irreducible hernia B- strangulated hernia C- incarcerated hernia
1307. 2 images of a patient with a large lower abdominal incisional hernia that appears on
standing, the patient had 4 times CS, hernia was repaired using the laparoscope, the most
common late complication?
A- adhesions B- fistula C- mesh migration D- wound failure
1308. 24 years old male, during playing a groin swelling appeared, it was tender and not
reducible, management?
A- analgesics and physiotherapy
B- surgical evacuation of the hematoma
C- surgical hernia repair
1309. Exact same case, cause?
A- rupture inferior epigastric artery B- congenital inguinal hernia sac
1310. Exact same scenario, investigation?
A- CT B-MRI C- aspiration
1311. Patient was crushed by a car with bilateral fracture femur, what is your concern?
A- maintain urine output

PLEAZE, ANY NOTE OR CORRECTION OR ADDITIN SEND: [email protected]


‫نسألكم الدعاء بظهر الغيب‬
P a g e | 123
B- external fixation of the fracture
C- internal fixation of the fracture
D-hypotension
1312. Patient extracted from a collapsed home after hours, his leg is painful, swollen, no
fracture, what is the pathophysiology?
A- ischemia B-reperfusion injury
1313. Almost the same case and the patient is hypotensive, what is the cause of hypotension?
A- decreased COP B- hemorrhage C- peripheral vasodilatation
1314. Almost same case, dark brown urine, cause? myoglobinuria
1315. Trauma patient, unconscious, oxygen saturation 95%, shocked, distended abdomen,
fracture femur, initial management?
A- maintain airy way B- wide bore cannula and iv fluids
1316. Trauma patient with dyspnea, shock, decreased air entry and resonance on the left side,
tracheal deviation to the right side, diagnosis? left sided tension pneumothorax
1317. Trauma patient with left lower chest bruises, dullness of the left chest, decreased
movement of the left chest, crepitus (? I think due to fracture ribs), diagnosis?
A- flail chest B- left hemothorax
1318. Exact same case, patient not shocked and stable, management? chest tube
1319. Trauma patient, mediastinal emphysema, enlarging subcutaneous emphysema, dyspnea
and respiratory distress despite chest tube and large amount of air leakage, investigation?
A- bronchoscope B- esophagogram C- CT chest
1320. Trauma patient with hemorrhage (not remember the scenario), blood pressure 90/60,
hear rate 110, anxious, degree of shock?
A- I B- II C- III D- IV
1321. Another trauma patient with hemorrhage (not remember the scenario), blood pressure
110/60, heart rate 110, amount of blood loss?
A- 10-15% B- 15-30% C- 30-40% D- > 40%
1322. Tumors occurs most commonly in which salivary gland? parotid
1323. Child with midline neck swelling that moves with protrusion of the tongue, most
common site?
A- below hyoid bone B- overlying thyroid cartilage
1324. 70 years old female, midline cystic swelling in the neck (with image) 2 weeks ago,
swelling moves with protrusion of the tongue but not with deglutition, patient refused surgery,
what will happen?
A- infection B- laryngeal compression
C- resolution D- nothing (no malignancy)

PLEAZE, ANY NOTE OR CORRECTION OR ADDITIN SEND: [email protected]


‫نسألكم الدعاء بظهر الغيب‬
P a g e | 124
1325. Infant with lateral neck mass, increases in size during crying, compressible, translucent,
ultrasound showed it multicystic, diagnosis? cystic hygroma
1326. Subtotal thyroidectomy 10 years ago, recurrent, lab shows high TSH with normal T3 and
T4, cause of recurrence?
A- hypothyroidism B- thyroxin replacement therapy
1327. Patient with diffuse goiter, exophthalmos, lab shows elevated TSH and low T3 and T4,
initial pathophysiology of the condition (not goiter)?
A- inhibitory anti TSH Ab B- stimulatory anti TSH Ab
C- T lymphocytes D- B lymphocytes
1328. 19 years old female, rounded, firm, mobile breast mass, US and FNA showed
fibroadenoma 3 x 2 cm, management? excision (no follow up, other options are clearly
wrong)

1329. 32 years female, with proved fibroadenoma of the breast (clinically, mammography,
FNA) as previous case, 1 x 1.5 cm, she was afraid as her mother died of cancer breast at age
43, management? reassurance and follow up (no excision, other options are irrelevant)
1330. Male with biopsy proved breast cancer, no family history of breast cancer, asking about
the mutation?
A- P53 B- BRCA1 C- BRCA2 D- new mutation
1331. During laparotomy for hepatic resection (doesn't mention for what) something was
found in the mesentery, there was a picture showing multiple diverticula at the mesenteric
border of a jejunal loop, management? Jejunal diverticulum
A- biopsy (if LNS) B- resection of the loop(if bleeding or perforation ) C- postoperative
antibiotics (if diverticulitis)
1332. A patient presented with abdominal pain, mass, black pigmentation of the mouth & hand
and feet, father removed a polyp from the colon many years ago, diagnosis?
Peutz-Jeghers syndrome
1333. During appendectomy, appendix was normal, a loop of ileum was thickened & rubbery &
covered by whitish grey exudate, many loops were adherent to that loop, diagnosis?
A- TB B- Crohn's C- actinomycosis
1334. Image during laparoscopic exploration for acute abdomen showing a perforation in a
small intestinal loop with pyogenic membrane in the peritoneum, management?
A- laparotomy B- laparoscopic repair of the perforation
1335. Appendectomy 10 years, patient presents with abdominal pain, distention, vomiting for 2
days and absolute constipation for 1day, hyper audible intestinal sounds, conservative
treatment for 72 hours but with no improvement, x ray image (showing dilated ileal and jejunal
loops) management?

PLEAZE, ANY NOTE OR CORRECTION OR ADDITIN SEND: [email protected]


‫نسألكم الدعاء بظهر الغيب‬
P a g e | 125
A- exploration B- Gastrographin follow through

PLEAZE, ANY NOTE OR CORRECTION OR ADDITIN SEND: [email protected]


‫نسألكم الدعاء بظهر الغيب‬
P a g e | 126
1336. A patient with myeloproliferative disorder, splenomegaly, severe left hypochondrial pain
radiating to the left shoulder, investigation? CT (for splenic infarction)
1337. 10 days after appendectomy, right hypochondrial pain radiating to the right shoulder,
swinging fever, tenderness over the eight-lower chest, abdomen soft and lax except mild right
hypochondrial tenderness, investigation? CT abdomen (for subphrenic abscess)
1338. 10 days after appendectomy, patient came to remove the stitches, wound was swollen &
warm, large amount of pus on removing the stitches, initial management?
A- culture B- irrigate the with saline
C- pack the wound with betadine-soaked gauze D- secondary closure of the wound
1339. Patient injured his finger with knife, self-dressing, after 4 days the GP give him
antibiotics and he stop it after 2 days, now erythema of the finger, crepitus in the arm, drowsy,
management? debridement
1340. Same scenario, asking about pathophysiology? increased capillary permeability
1341. Patient with right gluteal bulla with surrounding erythema and crepitus extending into
the right thigh, aspirate from the bulla showed gram positive rods, best antibiotic?
A- penicillin G B- clindamycin C-vancomycin
1342. Diabetic patient with black foul-smelling patch over the left foot with crepitus extending
to the thigh (almost), management? debridement
1343. Organism that was thought to be skin commensal but found to colonize catheters and
cause bacteremia and endocarditis?
A- staph epidermidis B- pseudomonas aeruginosa
1344. Stimulation of baroreceptors in the aortic arch leads to?
A- decrease heart rate
B- central sympathetic stimulation
C- vc of skeletal blood vessels
1345. Adolescent with right hip pain during activity relieved by rest, 0.5 cm shortening of the
right lower limb?
A- Perth's disease B- slipped capital femoral epiphysis
1346. Dyspnea, x ray showed mediastinal shadow, CT showed left upper lung mass and
mediastinal LNs, investigation?
A- open lung biopsy B- mediastinoscopy
1347. Lesion of the big toe, contain HPV by some sort of biopsy?
verruca vulgaris (common wart)
1348. Patient with > 1000 polyps in the colon and rectum, family history of similar condition,
treatment?
A- total colectomy B- polypectomy (no total proctocolectomy)

PLEAZE, ANY NOTE OR CORRECTION OR ADDITIN SEND: [email protected]


‫نسألكم الدعاء بظهر الغيب‬
P a g e | 127
1349. Patient with hard ulcerative mass 11 cm from the anal verge and multiple polyps in the
ascending colon, first investigation for staging?
A- CT abdomen B- MRI C- EUS D-colonoscopy
1350. Patient with cancer rectum 5 cm from the anal verge infiltrating the mesorectum?
neoadjuvant chemoradiotherapy
1351. Direct question about what allows for sphincter preservation in case of cancer lower
rectum? neoadjuvant chemoradiotherapy
1352. Cancer rectum infiltrating the mesorectum, with metastases to inferior mesenteric LNs
and to the liver, stage? T4 N2 M1 (this was the only choice with M1)
1353. Patient with altered bowel habit, spurious diarrhea, bleeding per rectum (don't
remember the details but typical history of cancer colon), image of barium enema with typical
apple core appearance, diagnosis? cancer colon
1354. Similar but not same scenario (typical for cancer colon), another image of barium enema
with typical apple core appearance, next investigation? colonoscopy
1355. Old patient with sudden abdominal pain & constipation & distention, image of abdominal
x ray with typical sigmoid volvulus, diagnosis? sigmoid volvulus
1356. Another scenario of acute abdominal pain & vomiting & constipation, image of
abdominal x ray typical of cecal volvulus, what you are afraid from?
A- gangrene B- perforation
1357. Another case of acute abdomen (don't remember the details) but with two CT images
and he said that there is cecal volvulus shown in these images, then patient was relieved
(spontaneous detorsion occurred), management?
A- exploration and resection B-exploration and fixation C- observation
1358. A case of known ulcerative colitis, developed abdominal pain & distention, image of
abdominal x ray showing hugely dilated transverse colon, diagnosis? toxic megacolon
1359. A case of lower GI bleeding, endoscope cannot find the source, angiography found
vascular abnormality in the transverse colon, diagnosis?
angiodysplasia (there was no Dieulafoy lesion)
1360. Image of complete rectal prolapse, what is the most common associated complain?
A- bleeding B- mucous discharge C- fecal incontinence
1361. Patient complains of perianal itching that becomes more severe at night, no history of
soiling of the cloths, diagnosis?
A- anal fissure B- anal fistula C- partial rectal prolapse
1362. Patient presented with severe perianal pain & perianal swelling, examination showed
prolapsed mucosa with ulceration and necrosis, PR was impossible?
A- prolapsed piles B- anal canal carcinoma C- perianal abscess

PLEAZE, ANY NOTE OR CORRECTION OR ADDITIN SEND: [email protected]


‫نسألكم الدعاء بظهر الغيب‬
P a g e | 128
1363. After internal sphincterotomy for anal fissure the patient developed incontinence to gas
and liquid stool, PR showed injured external anal sphincter, how to diagnose the structural
abnormality of the anal canal?
A- EUS B- manometry C- pudendal nerve terminal motor latency
1364. Patient with perianal fistula with the internal opening above the dentate line, he
complains of abdominal pain & diarrhea, investigation?
A- MRI fistulogram B- colonoscopy C- CT enterocolitis
1365. Patient found shocked in the floor of the bathroom, profuse bleeding per rectum, full
thickness rectal tear 50% of the circumference, higher in the rectum there was a foreign body,
what could make the patient condition worse?
A- mucosal rectal tear B- full thickness rectal tear C- impacted foreign body
1366. Exact same case, management?
A- compression of the abdomen
B- perianal extraction
C- laparotomy and squeeze the foreign body
D- laparotomy and open the rectum to remove the foreign body
1367. Patient underwent hip replacement, postoperative medications were iv fluids &
analgesics & proton pump inhibitors, 7 days later she developed dyspnea, what could prevent
this complication?
A- prophylactic heparin B- prophylactic aspirin
1368. In the third postoperative day (don't remember the operation but almost abdominal
operation), fever, decreased air entry on the left lower long base, diagnosis?
A- pneumonia B- atelectasis? C- subphrenic abscess
1369. Crohn's disease patient, managed by resection anastomosis, postoperatively
developedintestinal fistula (drain output 1 liter intestinal) but no peritonitis or sepsis (don't
remember thedetails but this was evident in the scenarios), lab show metabolic acidosis
with low HCO3,cause? intestinal fistula (loss of intestinal juice containing HCO3)
1370. Image of a large naevus of the forearm as was described in the text (its surface was
irregular in the image), it has recently enlarged and changed in shape, what determines the
excision margin? depth of dermal invasion
1371. Image of a large swelling in the back, it is cystic, with low grade fever and weight loss,
history of pain in the spine (and I think imaging modality describing a problem in L1, not sure),
cold abscess management?
A- incision and drainage B- aspiration C- Antituberculous drugs

PLEAZE, ANY NOTE OR CORRECTION OR ADDITIN SEND: [email protected]


‫نسألكم الدعاء بظهر الغيب‬
P a g e | 129
1372. A patient with diabetic foot, orthopedic surgeons and general surgeons are arguing
where to admit the patient, how to prevent this problem?
instructions from the medical director
***follow hospital policy
1373. Who is responsible to solve ethical problems in the hospital (direct question)?
ethical committee
1374. 13 years old child, his parents brought him the emergency department, he needs urgent
intervention but his parents refused to sign the consent?
A- take consent from other family member
B- do not do the intervention
C- do the intervention if lifesaving
NB: if there is authority more accurate
1375. Doctor wants to do a chest surgery to the patient but success rate is 10% but if not done
the patient will deteriorate, doctor is afraid that the patient will refuse to sign the consent?
A- tell the patient and let him take the decision
B- take consent from a relative
C- don't tell the patient so that he may sign the consent
D- do the operation without consent because you know what is good for the patient
1376. A similar scenario but the doctor wants to do surgery to the patient that can be
complicated by amputation and if surgery not the disease will progress and amputation will be
also be a must?
A- tell the patient and let him take the decision
B- take consent from a relative
C- don't tell the patient so that he may sign the consent
D- do the operation without consent because you know what is good for the patient
1377. An old female with dementia, diagnosed with metastatic cancer breast, her sone asked
you not to tell her?
A- tell her B- don't tell her C- ask him why he wants you not to tell her
1378. Short question just like that, surgical intervention is indicated in which patient?
A- no responders B- transient responders
C- responders D- unconscious
1379. You found that certain drug lot with specific patch number causes a specific side effect
in your patient, who to tell? the hospital administrator
1380. You discovered a problem in an energy device, same problem was discovered in other
hospitals also? report to the hospital administrator

PLEAZE, ANY NOTE OR CORRECTION OR ADDITIN SEND: [email protected]


‫نسألكم الدعاء بظهر الغيب‬
P a g e | 130
1381. A patient for right cochlear implant entered the OR instead of another with bilateral cochlear
implant and bilateral cochlear implants were inserted, how to prevent this error?
WHO safety chick list
1382. A patient for left lower limb amputation was admitted as for right lower limb amputation
due to a fault from the admission nurse, during surgery after the right lower limb was almost
amputated the error was discovered and bilateral amputation was done, how to prevent this
error? WHO safety chick list
1383. Trauma patient, open his eye to painful stimulation, withdrawal to pain, respond to
questions with wrong answers, GCS?
A- 6 B- 7 C- 9 D- 10
1384. Old patient falls on the stairs 1 week ago, now presenting with decreased conscious
level and seizures, CT report says concave dense lesion in the right temporal area?
ruptured subdural hematoma
1385. Image of pilonidal sinus with multiple opening (the text mentioned it was over the sacral
area), patient complains of pain and discharge, diagnosis?
A- pilonidal sinus B- anal fistula
1386. Exact same image and scenario, treatment?
A- warm paths
B- incision and drainage of the abscess
C- excision of the abscess and all opening or tracks
1387. inguinal hernia repair ....... 10 years later hernia again in same position, type of hernia?
A- recurrent B- incisional C- inguinal
1388. anterior resection, post op day 7, abd pain, tenderness generalized, distension. fever 39
A- sub phrenic collection B- pul. embolism C- suspected leak
1389. wound infection in un controlled DM what to do to decrease wound infection?
A- control high blood sugar post op B- shaving C- antibiotics within one hour before OT
1390. barro receptor stimulation cause ?? decrease Heart rate
1391. anterior resection .... post op, fever, radiology: pelvic collection about 6 cm?
A- per cut. drainage B- antibiotics C- conservative
1392. App. abscess by radiology, most surgical concern?
A- rupture (perforation) B- high fever
1393. Rt inguinal hernia, contain Meckel's diverticulum, 5 cm * base 1.5 cm?
A- diverticulectomy plus hernia repair
B- diverticulectomy
C- resection plus hernial repair

PLEAZE, ANY NOTE OR CORRECTION OR ADDITIN SEND: [email protected]


‫نسألكم الدعاء بظهر الغيب‬
P a g e | 131
1394. Image of small segment stricture of terminal ileum about 2 to 3 cm from ileocecal valve
and pt. with manifestations of intestinal obstruction type of operation?
A- resection and anastomosis B- Stricturoplasty C- Rt hemi colectomy
1395. Pathogenesis of BCC?
A- ultraviolet B- genetics
1396. Case of wt. loss after bariatric surgery 10 yr. ago with significant wt. loss complain of
abd. pain sever + distension and constipation with pain radiate to rt thigh and knee decision?
A- laparotomy B- laparoscopy C- conservative
1397. How to Differentiate between perforated PU and acute pancreatitis
A- x-ray barium B- x-ray abdomen C- CT with contrast
1398. 77 yr. old male present with severe abdominal pain for 4 days ass. with diarrhea with
blood inv. done >> thumbprint sign
A- pseudo membranous colitis B- ischemic colitis
1399. Male pt. on ciprofloxacin for 10 days and develop diarrhea and diagnosis confirmed
membranous colitis ttt? oral Flagyl (others)
1400. Intra op room, nurse give wrong something?
A- commission B- omission
1401. pt. with impacted stone in CBD and ERCP fail to remove it, but pt. still in pain in upper
abd and mild tenderness what is the cause of pain?
A- bleeding (haemobilia) B- stone in CBD C- perforated duo
1402. MC cause of bile duct cancer is
A- PSC B- choledochal cyst C- stone of bile duct
1403. Male with history of prostatectomy for CA prostate 10 yr. ago, now complain of abd. pain
and bleeding per rectum, colonoscopy show ca colon, other work up show liver Mets about
liver Mets?
A- synchronous with colon cancer
B- metachronous with prostate cancer
C- CEA is elevated with Mets with prostate
1404. Known case of leukemia has generalized lymphadenopathy and hepatosplenomegaly
what you will do for diagnosis of lymphadenopathy?
A- FNAC B- LN excision biopsy
1405. Infant with Rt inguinal hernia till scrotum name of this swelling is
A- patent process vaginalis B- direct hernia C- indirect hernia
1406. Umbilical sinus? sinogram
1407. Long case of umbilical hernia, firm and not reducible no distension pt. not in int obst.
Diagnosis?

PLEAZE, ANY NOTE OR CORRECTION OR ADDITIN SEND: [email protected]


‫نسألكم الدعاء بظهر الغيب‬
P a g e | 132
A- strangulated umb. hernia content viscus B- irreducible umb. hernia contain omentum
C- obstructed umb. Hernia D- strangulated umb. hernia with omentum
1408. Case of diverticulitis on flagyl and dalacin, abd tenderness of Lt side and repeated
vomiting, surgery done, there is and there is sever inf. of LT side of colon pus was taken for
culture, case not improved on medication, what you will do?
A- stop flagyl and dalacin B- stop flagyl and dalacin and give tineam
1409. The most sensitive in diagnosis of pancreatic divisum
A- ERCP B- MRCP
1410. Post lap cholecystectomy, leak, 1st step to do?
A- MRCP C- ERCP C- US D- HIDA scan
1411. Patient was found shocked in bathroom, has rectal injury with foreign body with more
than half of the circumference, full thickness, 6-8 cm from anal verge (not sure about the
distance) exploration was done and the rectum was opened transversally, what’s the optimal
procedure?
A- Colostomy and primary repair B- Resection anastomosis and colostomy
1412. old patient was planned for procedure (not urgent), he refused the operation but the
family want to do it, what to do? Respond to the pt. wishes
1413. Male underwent sleeve more than 10 yrs. ago, presented with weight loss and features
of intestinal obstruction, there is pain radiated to right thigh and knee, what to do? (Obturator
hernia)??
A- exploration B- laparoscopy
1414. Patient underwent exploration, was found to have diverticulitis, culture sensitivity done
(no result was shown in the scenario), prescribed on metronidazole and dalacin, patient
wasn’t improved?
A- stops both antibiotics B- gives 3rd generation C- give tinam
1415. Mesenteric ischemia presentation, (with picture)? abdominal radiography
1416. Most common cause of cholangiocarcinoma?
A- gall stones B- primary sclerosing cholangitis C- choledochal cyst
1417. Shocked patient with congested neck veins, low cop, high SVR ?
A- cardiogenic B- distributive C- obstructive D- hypovolemic
1418. Colo vesical fistula, how to diagnose?
A- barium enema B- CT C- us (cystoscopy not mentioned)
1419. GIST AND LIOMYOMA, prognostic factors?
A- Mitotic features B- size
1420. Umbilical sinus? sinogram

PLEAZE, ANY NOTE OR CORRECTION OR ADDITIN SEND: [email protected]


‫نسألكم الدعاء بظهر الغيب‬
P a g e | 133
1421. Ca esophagus while introducing stent, perforation occurred, patient developed
mediastinitis, air under diaphragm?
A- tracheostomy / chest tube? B- conservative
C primary repair D- esophagostomy
NB: esophagectomy is right but not present in the choices so I will go for diversion
1422. Chest trauma with tracheobronchial tear? bronchoscopy
1423. PE, diagnosis?
A- CT Angio B- ABGs C- ECG
1424. Trauma with disturb conscious level. US showed mild free intra-abdominal fluid pt
unstable?
A- CT B- DPL
NB: if pt. is HD stable =CT, unstable = DPL
1425. Collapsed building? reperfusion injury
1426. Most common cause of acute and chronic pancreatitis?
acute= gall stones, chronic =alcohol consumption
1427. Factor that decreased the incidence of infection in acute pancreatitis? Enteral feeding
1428. Leukemia with hepatosplenomegaly and lymphadenopathy, diagnosis of LN?
A- FNAC B- LN excision biopsy
1429. Blunt chest trauma, with hematoma, rupture descending thoracic aorta, and mediastinal
hematoma?
A- immediate repair B- delayed repair C- repair within 6 hours
1430. perianal skin lesion in patient with leukemia, cause?
A- HPV B- adenocarcinoma
1431. Anterior anal fissure tear (almost 5 cm) in female, commonest cause?
A- normal delivery B- constipation
1432. Rectal carcinoma 11cm from anal verge, infiltrating mesorectal fascia, indication for
neoadjuvant ttt? mesorectal invasion
1433. Prepyloric ulcer, 3cm, indurated and firm?
A- Billroth ii B- truncal vagotomy with antrectomy
1434. Breast lump, aspirated, green fluid, the mass is completely disappeared, next step?
A- true cut B- surgery C- cytology D- observation and follow up
1435. Patient was on antibiotic, developed diarrhea, diagnosis done pseudomembranous
colitis, treatment?
A- I.V. vancomycin B- Oral metronidazole C- Ciprofloxacin
1436. Patient was on antibiotic and rectum is hyperemic, yellow fluke, best treatment?
A- Ciprocin B- Ceftazidime C- Metronidazole D- Triclabendazole

PLEAZE, ANY NOTE OR CORRECTION OR ADDITIN SEND: [email protected]


‫نسألكم الدعاء بظهر الغيب‬
P a g e | 134
1437. ITP patient underwent splenectomy but still platelet 50000, treatment?
A- Steroid B- Azathioprine C- Immunoglobulin D- conservative ttt
1438. Trauma to right hypochondrium, most liable organ to injury?
A- Spleen B- Stomach C- Liver D- Lung
1439. Patient with severe epigastric pain, generalized, chest x-ray shows air under diaphragm,
on exploration 1 cm perforation is found in stomach 3 cm proximal to pylorus with 2 cm
induration around perforation, treatment?
A- Billroth II gastrectomy B- Gastro-jejunostomy
C- Pyloromyotomy with vagotomy D- Close perforation and toilet
1440. Patient with upper GI 2 episodes of bleeding, O/E abdominal distension (ascites),
diagnosis?
A- Peptic ulcer B- Esophageal varices
C- Dieulafoy lesion D- Duodenal ulcer
1441. The most common cause of Colo-vesical fistula is?
A- Cancer B- Crohn’s(ileo colic –ileo ileal) C- Diverticulitis
1442. The indication for surgery in ulcerative colitis? Toxic mega colon
1443. History RIF pain and appendectomy done, 1 cm of carcinoid is found post-surgery, best?
A- Nothing B- Right hemicolectomy
1444. H/O hemorrhoidectomy for 2 years, the patient presented with difficulty defecation,
diagnosis?
A- Stenosis B- Missed cancer
1445. Patient admitted in hospital because partial intestinal obstruction, what is the indication
for surgery during observation?
A- Hypotension B- Persistent abdominal pain
1446. Patient with ITP (immune thrombocytopenic purpura), underwent splenectomy, after 1
year platelet decreased again, cause?
A- Uncomplete spleen resection B- Accessory spleen
1447. Patient with spherocytosis send to surgeon before doing surgery we need which test?
A- C.T. B- Fragility test C- U.S. abdomen
1448. Succinylcholine is contraindicated for?
A- Patient with hepatitis B- Parotidectomy
C- Celiotomy 2 weeks after spinal cord injury D- Thyroidectomy
1449. Splenic artery aneurysm
A- Can present with double rupture phenomenon
a ruptured aneurysm can present with the double-rupture phenomenon, in which the
aneurysm ruptures into the lesser omental sac primarily and blood overflows into the
peritoneal cavity after 6 to 96 hours

PLEAZE, ANY NOTE OR CORRECTION OR ADDITIN SEND: [email protected]


‫نسألكم الدعاء بظهر الغيب‬
P a g e | 135
B- Is more common in males
C- Is seldom multiple

PLEAZE, ANY NOTE OR CORRECTION OR ADDITIN SEND: [email protected]


‫نسألكم الدعاء بظهر الغيب‬
P a g e | 136
1450. One week after coumadin therapy, a patient develops severe pain in right leg with areasof skin
necrosis, appropriate action is?
A- Transfusion of fresh frozen plasma
B- Intravenous vit. K
C- Stopping coumadin and starting heparin
1451. Mixed venous saturation is increased in?
A- Hypovolemic shock B- Septic shock
C- Cardiogenic shock D- Neurogenic shock
1452. Paraneoplastic syndrome is most commonly associated with?
A- Small cell lung cancer(SIADH) B- Bronchial carcinoid C- Bronchoalveolar carcinoma
1453. Seat belt in chest and abdomen, mostly cause injury to?
A- Retroperitoneum B- Pancreas C- Hollow viscus D- Rim of pelvis
1454. Patient presented in ER with lower GI bleeding, O/E pigmentation in perioral, hands and
feet, with family history of colon cancer in his father’s family, diagnosis? Peutz-jeghers
1455. Helicobacter pylori
A- Most common in gastric ulcer than duodenal ulcer
B- Cause gastric malignancy
C- Positive in 70-80% in esophagus and duodenum in normal people
1456. Trauma cesarean section is indicated if the fetus is viable and?
A- The mother is unstable and cardiac arrest is anticipated
B- 5 minutes of cardiopulmonary resuscitation were successful
C- 5 minutes of cardiopulmonary resuscitation were unsuccessful
D- cardiopulmonary resuscitation has just been initiated
1457. After vigorous exercise, an athlete develops pain on dorsiflexion of the foot and
decreased sensation in the first web space, appropriate action should be?
A- Color duplex scan and immediate heparinization
B- Leg elevation, ice packs, and non-steroidal anti-inflammatory medication
C- Immediate fasciotomy (leg elevation contraindicated in compartmental syndrome)
D- Immediate femoral arteriogram
1458. Female patient with history of breast feeding stopped for 6 months presented with
breast mass, FNA shows whitish fluid, diagnosis?
A- Cancer B- Galactocele
1459. Most common breast disease in lactating female is?
A- Cancer B- Mondor’s disease C- Mastitis D- Abscess

PLEAZE, ANY NOTE OR CORRECTION OR ADDITIN SEND: [email protected]


‫نسألكم الدعاء بظهر الغيب‬
P a g e | 137
1460. Young female with 3 cm breast mass, mobile and firm, FNA shows fibroadenoma, best
management?
A- Follow up B- Excision biopsy C- Danazol
1461. Patient with history of right hemicolectomy for T1a, N0, M0 colon cancer, at present
complain of right lower quadrant pain with constipation, best investigation?
A- CT (avoid perforation) B- Colonoscopy C- MRI
1462. Patient present with colitis and temperature 39, pulse 95/min, RR 22/min, BP 90/60,
diagnosis?
A- SIRS B- Sepsis C- Severe sepsis D- Septic shock
1463. Male patient with diverticulosis, symptoms and signs of diverticulitis, best investigation?
CT abdomen
1464. Male patient, old age, with pneumaturia, what is the most common cause of this
condition?
A- Diverticulosis B- Cancer C- Crohn’s D- Radiation
1465. Male patient presented to clinic with abdominal pain and weight loss for 3 months ago,
he underwent right hemicolectomy 3 years ago for colon cancer, what is the best
investigation?
A- Colonoscopy B- CT abdomen C- PET D- US
1466. Barium swallow show corkscrew esophagus with normal manometry
A- Diffuse esophageal spasm B- Achalasia
1467. Female patient with long time of bilateral leg swelling and skin change, best
investigation?
A- Duplex US B- Venography C- ABPI
1468. Female patient with big toe bluish discoloration and severe pain with photo (gangrene),
ABPI is?
A- > 1 B- 0.8 – 0.9 C- 0.5 – 0.6 D- < 0.3
1469. Female patient with leg edema, need to be treated by gradual pneumatic compression
and garment compression, which are contraindications if ABI?
A- > 1 B- 0.8 – 0.9 C- < 0.5
1470. Clotting factor correlating with vit. K is?(2,7,9,10)
A- VII B- XI C- X
1471. 2 years old boy brought by his mother, O/E testicles not found in scrotum and inguinal
region, best investigation to confirm diagnosis?
A- CT B- MRI B- Laparoscopy
N.B. if bilateral …. do karyotyping to define the gender

PLEAZE, ANY NOTE OR CORRECTION OR ADDITIN SEND: [email protected]


‫نسألكم الدعاء بظهر الغيب‬
P a g e | 138
1472. Patient with chronic chest empyema, which is non-tuberculosis, best treatment
associated with antibiotics is?
A- Thoracocentesis B- Decortication C- Pleurodesis
1473. Patient with chest trauma, chest tube is inserted, but still air leak associated with
increasing pneumomediastinum and subcutaneous emphysema, best investigation?
A- Esophagoscopy B- Bronchoscopy C- Bronchography
1474. Photo of old age female patient with chest skin lesion (like BCC), skin biopsy margins
are?
A- 5 mm B- 1 mm C- 2 mm D- 10 mm
N.B 4-6 mm
1475. Female patient, 72 years old, post prandial pain 15 minutes, anorexia, most appropriate
diagnosis?
A- Colon cancer B- Chronic mesenteric disease
1476. Most common site of biliary carcinoma cholangiocarcinoma
A- Distal part of CBD B- Hilum C- Cystic D- Intrahepatic duct
1477. Patient with upper right quadrant pain, fever yellow sclera, high bilirubin
A- Pancreatitis B- Cholangitis
1478. Upper abdominal pain recurrent steatorrhea, diabetic alcoholic, with picture of CBD,
multiple dilatation pancreatic duct?
A- CBD stone B- Pancreatic carcinoma
C- Chronic pancreatitis D- Pancreatic cyst
1479. Tissue loss almost always results from ligation of the?
A- Popliteal artery B- Common femoral artery
C- Superficial femoral artery D- Portal vein
1480. Post operation of thyroidectomy, fever, hypertension, tachycardia? Thyroid storm
1481. Patient with inguinal swelling, hernioplasty done without opening hernial sac, type of
this hernia is?
A- Direct B- Indirect C- Femoral D- Ventral
1482. Patient with history of inguinal swelling, presented with painful inguinal swelling +
abdominal distension and constipation for 1 day, no fever, diagnosis?
A- Irreducible hernia B- Incarcerated hernia
C- Obstructed hernia D- Strangulated hernia
1483. 3 weeks old infant with multiple non- bilious vomiting, diagnosis?
A- Duodenal atresia B- Pylorus hypertrophy
C- Annular pancreas D- Malrotation

PLEAZE, ANY NOTE OR CORRECTION OR ADDITIN SEND: [email protected]


‫نسألكم الدعاء بظهر الغيب‬
P a g e | 139
1484. During lap. Chole. You injured the CBD and told the patient post-operative, what does
this mean?
A- Disclosing an apparent complication B- Disclosing complication
C- Disclosing error D- Disclosing an apparent error
1485. Study conducts to see the effect of earphone Bluetooth and brain tumor, what is the best
study?
A- Cross sectional B- Cohort C- Case control D- Correlation
1486. diagram of surgical resident and non-surgical resident followed for 4 years outcome of
depression study, what’s the type of study?
A- Cohort B- Series case C- Control study D- Navy tape test
1487. Study on effect of dairy products on group with colon cancer and group with no colon
cancer, what is the type of study?
A- Cohort study B- Control study C- Section study D- Correlation
1488. Hyponatremia is a complication of topical application of?
A- Silver nitrate B- Povidone iodine C- Sulfamylon D- Bacitracin
1489. Infant continuous crying, O/E swelling scrotal, firm, bluish in color, not fluctuant, no
groin or abdominal swelling
A- Congenital hydrocele B- Testicular teratoma
C- Testicular torsion D- Strangulated inguinal hernia
1490. Splenic injury with 3 cm laceration, which grade ?
A- I B- II C- III D- IV
1491. Organ resistant to infection?
A- Intra-abdominal (GIT due to immunoglobulin A)
B- Pulmonary
C- Blood
1492. Patient with RTA, BP 90/60, HR 130, RR 26, best fluid?
A- Blood B- Colloid C- Crystalloid
1493. Patient 35 years old, presented with breast mass in upper outer quadrant with non-
cyclic pain, next investigation?
A- FNA(bestinvestigation) B- Mammogram C- MRI breast
1494. Patient with BP 170/90, T3, T4, TSH are normal, Hb is normal, platelet is 100000, WBC is
normal, to control BP should send the patient to?
A- Cardiology B- Pulmonology C- Endocrinology D- Hematology
1495. Landmark to differentiate between indirect and direct hernia?
A- Scrotum B- Vas deferens C- Pubic tubercle D- Internal ring

PLEAZE, ANY NOTE OR CORRECTION OR ADDITIN SEND: [email protected]


‫نسألكم الدعاء بظهر الغيب‬
P a g e | 140
1496. Para umbilical hernia, vomiting, constipation, sac irreducible, not tender, type of hernia?
A- Obstructed B- Irreducible C- Incarcerated D- Strangulated
1497. Hernia with crescentic shaped umbilicus is?
A- Umbilical B- Epigastric C- Para- umbilical D- Spigelian hernia
1498. The endothelial cells are the source of
A- Factor II B- Factor V C- Factor VII D- Factor VIII
1499. Which of the following is likely to be multiple?
A- Gastrinoma B- Insulinoma C- Somatostatinoma D- Glucagonoma
1500. The mechanism of action of heparin is?
A- Potentiation of antithrombin III factor
B- Inhibition of cyclooxygenase
C- Direct inhibition of thrombin
1501. Which of the following amino ester?
A- Lidocaine B- Tetracaine C- Bupivacaine D- Mepivacaine
1502. Vit k
A- Is water soluble B- Is absorbed in proximal small bowel
C- Requires bile salts for absorption D- Is required for factor VIII synthesis
1503. Female patient, anemic, pallor, jaundice, splenomegaly, target cells at blood smear
A- Thalassemia B- Spherocytosis C- Sickle cell
1504. Myeloproliferative disease with spleen enlargement?
A- U/S B- CT with contrast C- CT
1505. Post repair after AAA repair 2 days, there is bloody discharge from the rectum, you
suspect colonic ischemia, what do you do?
A- Mesenteric angiography B- Sigmoidoscopy
C- Upper GIT endoscopy D- CT
1506. Patient RTA, urine output 25 ml/hr., cool clammy sweating, mild anxious, RR 25, BP
normal, pulse 110, what is the volume of blood was lost?
A- < 15% B- 15-30% C- 30-40% D- > 40%
1507. Female patient with 4 CS, she has big intra-abdominal mass, post laparoscopic most
common complication? (2 pictures abdominal mass bulged, and flat abdomen)
A- Infection B- Wound failure(recurrence) C- Adhesion D- Fistulation
1508. Ileostomy best fluid? Normal saline or D5 0.5 NS
1509. Diabetes mellitus among health care workers is 30 out of 100 doctors were diabetic as
compared to 50 out of 200 nurses, which is the most appropriate statistical test? (Remember
doctors-nurses, diabetic-non diabetic)
A- T-test B- ANOVA C- Chi-square D- Correlation

PLEAZE, ANY NOTE OR CORRECTION OR ADDITIN SEND: [email protected]


‫نسألكم الدعاء بظهر الغيب‬
P a g e | 141
1510. Best study to assess the relationship between Bluetooth headsets and brain cancer?
A- Retrospective study B- Case control C- Ecological D- Correlation
1511. 30/500 babies who were exposed to Di-ethyl-stillbestrol in utero were followed up for 20
years and the occurrence of cancer in this group was compared with that of 1500 babies not
exposed to DES over the same period of time, what’s the study design?
A- Cohort B- Case control C- Cross sectional D- Correlation study
1512. History of lap. Chole. Since few days then bile collection in GB bed, definitive treatment?
A- Reoperation B- ERCP + stent C-US guided aspiration
1513. Patient with right side upper abdominal pain with fever, not responding to medication,
U/S abdomen shows GB distended with thin wall, diagnosis?
A- Acute cholecystitis B- pyocele
C- Cholangitis D- Mucocele
1514. Bilirubin
A- Most of direct bilirubin is a result of hemolysis
B- Indirect bilirubin is a result of biliary tree obstruction
C- Most of bilirubin in serum is indirect and carried by albumin
1515. Patient with uneventful lap. chole. for 6 months, presented with jaundice for 1 month
(with MRCP photo), diagnosis?
A- Cholangiocarcinoma B- Stricture C- Missed stone
1516. Picture of ulcer at posterior lateral aspect of the knee, female patient 35 years old, past
history of burn at same site 20 years back with scar no LNS, what’s the proper management?
A- Wide local excision
B- Wide local excision with sentinel LN biopsy
C- Wide local excision with LN dissection
D- Above knee amputation
1517. 75 years old female patient with recurrent attacks of loss of conscious with blurring
vision, no history of chronic illness next investigation ?
A- CT B- CTA C- Duplex
1518. Female old age, diabetic, uncontrolled came to ER with blackish discoloration at left foot
of 2 weeks duration no palpable anterior and posterior tibial artery, leukocytosis, CTA shows
stenosis at distal superficial femoral artery, complete occlusion of popliteal and infra-popliteal
artery, no collaterals, no run off, management?
A- Bypass graft B- Below knee amputation C- Above knee amputation
1519. Old age male with lesion lateral to eye, lesion show pigmentation and lateral ulceration
A- SCC B- BCC C- Melanoma D- Marjolin ulcer

PLEAZE, ANY NOTE OR CORRECTION OR ADDITIN SEND: [email protected]


‫نسألكم الدعاء بظهر الغيب‬
P a g e | 142
1520. Symptoms of GERD + barium swallow shows that gastroesophageal junction above
diaphragm but the stomach in place
A- Heller myotomy B- Partial fundoplication C- Nissen fundoplication (complete)
1521. Female patient with abdominal distension and vomiting, O/E stricture 6 cm from
ileocecal valve not passable (only one stricture)
A- Stricturoplasty B- Right hemicolectomy
C- Resection and anastomosis D- Resection and ileostomy
1522. Which of the following provides the main energy source during critical illness/ injury?
A- Skeletal muscle B- Liver C- Adipose tissue D- Kidney
1523. Cervical sympathectomy is least likely to improve?
A- Hyperhidrosis B- Scleroderma C- Causalgia D- Frostbite
1524. Girl came to ER with her mother, the father is out of town, diagnosis was appendicitis
A- Prepare for surgery
B- Take verbal consent from the mother
C- Wait till the father come back
D- Explain the procedure to the girl and take the mother signature
1525. Sigmoid volvulus (picture), next appropriate management?
A- Flatus tube B- Nasogastric tube
1526. 30 years old female patient, she has history of diarrhea and blood and mucous for 5
years, and her mother has the same history from long time, most appropriate diagnosis?
A- Diverticulitis B- Crohn’s disease C- Ulcerative colitis D- Colon carcinoma
1527. Alcoholic old man suddenly vomited and developed chest pain and dysphagia,
diagnosis? Esophageal perforation
1528. Post UGI endoscopy physician was worried about esophageal perforation, what is the
best investigation to confirm it? Diluted Gastrographin with thin film barium
1529. Gastrinoma location, the investigation of choice?
A- MRI B- CT C- SRS D- MIBG
1530. Patient very malnourished, what’s the element be normal if the patient is for surgery?
A- Vit c B- Copper C- Folic acid D- Ca ++
1531. Patient with myeloproliferative disease and splenomegaly, what’s the next step?
A- CT B- Bone marrow biopsy C- Splenectomy D- Observation
1532. Abnormal bleeding with normal prothrombin time occurs with?
A- Heparin overdose B- Cirrhosis
C- Hemophilia D- Von Willebrand’s disease
1533. 24 hours after admission to ICU, a postoperative patient has bright blood through the
nasogastric tube, all of following shown efficacy in preventing stress gastritis except?

PLEAZE, ANY NOTE OR CORRECTION OR ADDITIN SEND: [email protected]


‫نسألكم الدعاء بظهر الغيب‬
P a g e | 143
A- Sucralfate B- Proton pump inhibitors
C- Enteral diet D- Histamin-2 (H2) receptor antagonist
1534. 41 years old female, undergoes complex repair of a deep laceration in her hand, when
removing the dressing on postoperative day 2, a large clot with mild surrounding erythema is
encountered, which of the following statement regarding the inflammatory phase of wound
healing?
A- The complement component C5a and platelet factor attract neutrophils to the wound
B- The presence of neutrophils in the wound is essential for normal wound healing
C- It lasts up to 24 hours after the injury is incurred
1535. Child has wound in right foot with wood broke, red, hot, tender, most likely organism?
A- Clostridium tetani B- Clostridium perfringens
1536. Fluid resuscitation for infant? (20 ml/ kg)
1537. Gall bladder stone, asymptomatic and patient has history of enteric fever, best
treatment?
A- Cholecystectomy B- Conservative
1538. Gall bladder stone suggested by U/S + dilated CBD without stone, patient 60 years old,
best investigation?
A- ERCP B- MRCP C- CT D- PTC
1539. Gallbladder + fever + pain in right upper quadrant, tenderness, she started on antibiotics
after 3 days with no improvement, best investigation?(cholangitis)
A- ERCP B- MRCP C- CT
1540. Patient with 2 trials of unsuccessful esophagoscopy presented in ER with chest pain, the
surgeon was worried about perforation, the best for confirmation?
A- Barium swallow B- Esophagoscopy
C- CT D- Water soluble contrast swallow (Gastrografin)

1541. Female patient presented in ER with history of years of dysphagia, the dysphagia is
more for liquids and associated with history of coughing and sore throat, diagnosis?
A- DES (diffuse esophageal spasm) B- Dysphagia lusoria
C- Dysphagia of cardia D- Esophageal cancer
1542. Patient with history of 15 years duration of duodenal ulcer presented with multiple
vomiting, best I.V. fluid to be given is?
A- Ringer lactate B- Normal saline C- 3% saline D- Hetastarch
1543. Patient with known case of hypothyroidism presented with thyroid lump, hoarseness of
voice and dysphagia, O/E woody firm fixed thyroid swelling, best investigation?
A- FNA B- Open biopsy C- TSH D- Iodine isotope

PLEAZE, ANY NOTE OR CORRECTION OR ADDITIN SEND: [email protected]


‫نسألكم الدعاء بظهر الغيب‬
P a g e | 144
1544. Patient with thyroid mass, FNA shows amyloid, treatment?
A- Lobectomy with neck dissection B- Lobectomy with radiation
C- Total thyroidectomy with modified neck dissection D- Total thyroidectomy with radiation
1545. Patient present with central obesity, striae and buffalo hump, best investigation is?
A- Ct brain B- Urine cortisone
1546. Occurred medical mistake and you want to teach the students, what’s the appropriate
method for learning?
A- Make a meeting about mortality and morbidity
B- Root cause analysis
C- Must tell anyone from hospital about what happened
D- Make interrogation about what happened
1547. Patient underwent hemorrhoidectomy, after 2 days he came at OPD for follow up, he was
with his son, doctor asked the nurse to nurse the patient, which one of the following exceeds
the privacy of the patient?
A- Examine in the presence of the nurse
B- Examine without his son
C- Examine without white sheet
1548. Male patient, CT abdomen shows hepatocellular carcinoma, what next investigation for
work up?
A- CT brain B- MRI abdomen and pelvis
C- PET scan D- Bone scan
1549. RTA, ecchymosis, mid abdomen at site of seat belt, possible injury?
A- Spleen B- Liver C- Intestine D- Mesenteric
1550. Esophageal cancer with dysphagia LN at celiac artery, treatment?
A- Stent B- Esophagectomy C- Chemotherapy
1551. Primary energy source in acute energy
A- Fat B- Muscle C- Glycogen D- Ketone bodies
1552. Which one is not correct in four principles of Beauchamp, Childress??
A- Beneficence B- Non maleficence
C- Respect for persons D- Justice
1553. Patient gave you a gift and you refused, next day pt. attempted suicide because she felt
rejected, what you should do in this situation??
A- Accept the gift and give it to pt. charity
B- Explain to pt. that is unethical to accept gift and then refuse
C- Write paper on OPD door that gifts are unacceptable
D- Accept the gift for yourself

PLEAZE, ANY NOTE OR CORRECTION OR ADDITIN SEND: [email protected]


‫نسألكم الدعاء بظهر الغيب‬
P a g e | 145
1554. pt. RTA, he had fracture leg, and need foot amputation, after you explain the risk,
benefits of procedure, he agreed because he hears voices, telling him to do it, what to do??
A- Find another decision maker because pt. is incompetent
B- Proceed to live saving surgery since it is emergency intervention
C- Involve another surgeon to confirm the importance of surgery benefits
1555. Police man come with a guy to ER while you are the physician asking you for alcohol
screening but the guy refused, what do you do??
A- Respect confidence of the pt. B- Respond to police man request
C- Call a supervision D- None of the above
1556. Parameters to compare by chi-square
A- Age, bp B- Smoking, bp
C- Gender, smoking D- heart disease, smoking
1557. Mean of cholesterol in Arab area, American, European people??
A- Chi- square B- Student T test C- ANOVA(compare3) D- Correlation
1558. 70 years old pt. with sepsis has ph:7,18 which of the following statements is true
regarding to his metabolic acidosis??
A- The most common cause of excess acid is prolonged nasogastric suction
B- Metabolic acidosis results from the loss of bio carbonate on the gain of fixed acid
1559. Arrest of cell cycle in the metaphase in the action of?
A- Cyclophosphamide B- Methotrexate
C- Doxorubicin D- Vincristine
1560. pt. 84 y his wife 29y he presents to ER e’ nausea, vomiting, Abd. Pain, distended,
tympanic, rebound, T•/38,5 PR/20 HR/101 h/o two mi, CVA 5 years ago, after CVA he has
trouble in speaking, he doesn’t have power of attorney, pt. need operation, his wife refused,
his son demands that everything possible be done to save his father’s life What level of care is
appropriate??
A- Attempt intervention with local district court judge
B- Initiation of hospital care
C- Medical and supportive care only
D- Urgent surgical exploration
1561. pt. undergo for surgery in Abd. Neoplasm he discusses it with his surgeon, surgeon
describes the procedure to him in details, what is still necessary for the pt to be able to give
informed consent to this procedure??
A- A medical screen including ECG and coagulation panel
B- Psychiatric assessment

PLEAZE, ANY NOTE OR CORRECTION OR ADDITIN SEND: [email protected]


‫نسألكم الدعاء بظهر الغيب‬
P a g e | 146
C- An explanation of alternative to surgery
D- Consultations with one of family members of the pt. coagulation panel
1562. enlarged thyroid gland with normal T3, T4, high Tsh⬛
' +thyroid antibodies what is the
treatment??
A- Thyroxine B- Radioactive C- Anti thyroid
1563. pt. with dyspepsia, polyuria, hyper natremia, hypo kalmia? Aldosterone excess
1564. pt. with hyper natremia, hypo kalmia +mass at upper pole of kidney? adrenal carcinoma
1565. Child presents at emergency with wound in his head, with him was his brother,
nursemaid but his mother and father were in way starting to come at hospital
A- Start the stitches of the wound
B- Cover the wound, wait father, mother
1566. Cardiologist his pt. need operation in heart but pt. doesn’t know the procedure with
many complications what should you do??
A- Start to work without knowing the pt.
B- You should inform the Pt of all details of the process
C- Abandoning the operation
1567. Seizure pt. taking drug decreases attention and he a bus driver, what is your action??
A- respect confidentiality and don’t tell anyone
B- say to him not to drive
C-tell his supervisor
1568. A study conducted to evaluate medication among depressed pts they weren’t taking
their medications regularly?
A- unacceptable study because it affects drug efficiency
B- acceptable because the pt. signed the consent voluntary
C- unacceptable because we can’t take consent from depressed pts
1569. HIV newly diagnosed worry to tell his wife and ask not to tell his wife, what will you do?
A- Tell his wife’s mother and tell her to tell her daughter
B- Tell his wife
C- Protect the patient confidentiality
D- Reassure him and offer your assistance to tell his wife
1570. pt. with RTA a fracture femur only c/o dyspnoea +petechiae in upper chest + neck,
diagnosis??
A- Fat embolism B- Tension pneumothorax
1571. pt. with hard injury with wire took tetanus completed for 12 years best to given?
A- Tetanus toxoid B- Immunoglobulin C- Tetanus toxoid + immunoglobulin

PLEAZE, ANY NOTE OR CORRECTION OR ADDITIN SEND: [email protected]


‫نسألكم الدعاء بظهر الغيب‬
P a g e | 147
1572. Which type of shock need vasopressor??
A- Hypovolemic shock B- Distributive shock C- Endocrine shock
1573. pt. in blast injury best IVF is??
A- Normal saline B- Ringer lactate C- 3% saline
1574. 22 years old medical student is seen by the student health services prior to beginning
school routine laps are drawn, the medical student is immunised against hepatitis b in
childhood will have which hepatitis profile??
A- Hbs Ab+, Hbs Ag+, Hbc Ab+
B- Hbs Ab+, Hbs Ag+, Hbc Ab-
C- Hbs Ab-, Hbs Ag-, Hbc Ab-
D- Hbs Ab+, Hbs Ag-, Hbc Ab-
1575. upper RQ pain male pt. 44 years old present in ER with fever H/O lap chole for 4 days us
reveals Abd. Collection in gallbladder fossa and hepatorenal pouch MRCP shows leakage
from cystic duct stump, proper management??
A- ERCP + stent B- Open surgery and legation of tear C- Surgical drainage D-
percutaneous drainage
1576. Medullary thyroid cancer??
A- Most common than papillary thyroid cancer
B- Tend to metastasis by lymphatics
C- Genetic survey may lead to prophylactic total thyroidectomy
1577. Male with grave disease after thyroidectomy, fever, tachycardia, tachypnoea,
hypotension
A- Thyroid storm B- Septicaemia C- Infection
1578. Most effective ttt of flail chest is??
A- adhesive tapes
B- fixation with wire and a screw
C- epidural catheter and analgesic
1579. Likely associated with vascular injury most least??
A- Mid ulnar bone fracture
B- Supra condylar fracture humerus
C- Supra condylar fracture femur
D- Post dislocation of knee joint
1580. The diagnosis of oesophageal achalasia is confirmed by??
A- Bind’s beak appearance on barium swallow
B- Sub atmospheric intraluminal oesophageal pressure on manometry
C- Endoscopic evidence of distal oesophagitis
D- Failure of lower oesophageal sphincter relaxation on manometry
PLEAZE, ANY NOTE OR CORRECTION OR ADDITIN SEND: [email protected]
‫نسألكم الدعاء بظهر الغيب‬
P a g e | 148
1581. Anatomic liver division depends on??
A- Hepatic vein B- Portal vein C- Hepatic artery
1582. In achalasia there is absence of??
A- Complete relaxation of low oesophagus
B- Incomplete relaxation of low oesophagus
C- Oesophageal peristalsis
1583. Splenic aneurysm?
A- Observation and follow up
B- Ligation of the aneurysm
C- Splenectomy with removal of the affected artery
1584. During incisional hernia, injury to colon, colon then repaired primarily, what to do with
the hernia?
A- primary closure B- mesh repair
C- tension suture D- delay 4 days then proceed to mesh repair
1585. Most common hernia to strangulate?
A- Femoral B- Inguinal C- Epigastric D- Spigelian
1586. Hernia above umbilicus you can feel linea alba between the swelling and the umbilicus?
A- Umbilical B- Para umbilical C- Epigastric
1587. 25 years old football player, came with sudden swelling in the right inguinal region
which was tender, scrotum is normal containing NO bruises?
A- Torsion of spermatic cord B- Injury to posterior wall of inguinal canal
C- Congenital sac D- Rupture of inferior hypogastric vessels
1588. Lesion, causing osteomyelitis (picture of injured finger)?
A- Felon B- Paronychia C- Tenosynovitis
1589. Patient underwent surgical exploration for pyogenic liver abscess, presented with
abdominal distension, fever, tachycardia, tachypnea, oliguria, urinary catheter connected to
manometer is 28?
A- Evacuation of the collection
B- Laparotomy and colostomy
C- Laparotomy and laparostomy
1590. Patient presented with tachycardia PR 140bpm, tachypnea, elevated urea and
creatinine, BP 80/30 temp 37?
A- Sepsis B- Severe sepsis C- Septic shock
1591. Patient has neurogenic shock post spinal cord trauma presented with paraplegia, which
parameter indicates neurogenic shock?

PLEAZE, ANY NOTE OR CORRECTION OR ADDITIN SEND: [email protected]


‫نسألكم الدعاء بظهر الغيب‬
P a g e | 149
A- Bradycardia B- Hypotension
C- Tachycardia D- Trauma to spinal cord
1592. Patient with high temp, high pulse, low BP, what cause hypotension?
A- Low COP
B- Increased SVR
C- Decreased arteriolar resistance
1593. Trauma, resulted in tension pneumothorax, what’s the initial management?
A- Needle thoracostomy B- Tube thoracostomy (chest tube)
1594. Old man with pneumaturia, and recurrent UTI, found to have Colo vesical fistula, what is
the commonest cause for this condition?
A- Cancer colon B- Crohn’s disease C- UC D- Diverticulitis
1595. Trauma to the chest, examination revealed central trachea, paradoxical movement of
6,7,8th left ribs, x-rays show 6,7,8 left ribs fracture, and blurred costophrenic angle, PO2 92%,
other parameters are normal, what’s the definitive management?
A- Tube thoracostomy
B- Intubation and mechanical ventilation
C- Adhesive chest strap
1596. 65 years old, came to clinic with 3 years’ history of bleeding per rectum with and without
stool, associated with urgency. 3 yrs. ago he had coronary surgery and was prescribed on
antiplatelet. he was treated by radiotherapy for cancer prostate 6 years ago, what’s the cause
of this condition?
A- Radiation proctitis B- Ca colon C- Bleeding due to antiplatelet drugs
1597. 25 years old lady has breast lump 3 cm, confirmed by US to be fibroadenoma, what’s the
next step?
A- Excision B- Mammogram C- Biopsy D- Self-examination monthly
1598. Most associated with chronic pancreatitis?
A- Weight loss B- Pain C- Fever
1599. Patient presented with jaundice and abdominal pain, has increased alkaline
phosphatase, increased bilirubin, GGT, SPGT and SGOT, this condition is more than 2 weeks,
what’s the most common cause?
A- Choledocholithiasis
B- Cholangiocarcinoma
C- Pancreatic tumor (ca head of pancreas)
1600. Patient is known diabetic and ischemic heart disease presented with mucocele
gallbladder?un fit for surgery

PLEAZE, ANY NOTE OR CORRECTION OR ADDITIN SEND: [email protected]


‫نسألكم الدعاء بظهر الغيب‬
P a g e | 150
A- Open cholecystectomy B- p/c cholecystostomy
C- lap. Cholecystectomy D- interval cholecystectomy
1601. Patient presented with right hypochondrium pain referred to the right shoulder with
dyspepsia, US shows thickness wall?
A- acute cholecystitis B- acute pancreatitis C- PUD
1602. Patient presented with palpable mass at RUQ, ultrasound shows dilated GB with thin
wall?
A- Mucocele B- Acute cholecystitis D-chronic cholecystitis
1603. Day 10 post appendectomy, upon removal of stitches profuse pus discharged from the
wound, patient is a febrile, normal TLC?
A- Irrigate with saline B- Povidone iodine socked gauze at wound dressing
C- Secondary suture D- nothing
NB: if there is culture, do culture
1604. 6 years old child with URTI, 5 days later developed distress and hypercapnia and
generalized abdominal pain?
A- Appendicitis B- Pneumococcal bacterial infection C- Peritonitis
1605. 20 years old, with constipation, distended abdomen, generalized abdominal pain, fresh
bleeding per rectum?
A- Perforation B- Intussception C- Dileafoy
1606. Male patient, presented with diarrhea for 1 month, with abdominal colicky pain. anal
fistula opening above the dentate line, what’s the best investigation?
A- CT B- MRI C- Colonoscopy D- BA enema
1607. Male patient, alcoholic, presented with esophageal tear post vomiting, management?
A- Thoracotomy and esophageal repair B- Conservative and follow up (Mallory Weiss synd)
1608. A case of periampullary tumor, what’s the best modality for staging?
A- EUS B- CT C- ERCP D- MRCP
1609. Female patient has pyloric carcinoma with celiac LN involvement (advanced tumor) with
gastric out let obstruction?
A- Total gastrectomy B- Gastrojejunostomy C- Subtotal gastrectomy
1610. Female patient on treatment for arthritis, presented c/o epigastric pain and right iliac
pain with sigh of peritonitis? Perforated PU
1611. Female patient with felty's syndrome?
A- Peritonitis –splenomegaly –rheumatoid B- Rheumatoid-splenomegaly –pancreatitis
C- Splenomegaly-neutropenia-peritonitis D- Neutropenia-splenomegaly-
rheumatoid
1612. Rectal tumor, 5 cm from the anal verge, advanced. best treatment?

PLEAZE, ANY NOTE OR CORRECTION OR ADDITIN SEND: [email protected]


‫نسألكم الدعاء بظهر الغيب‬
P a g e | 151
A- Neo-adjuvant therapy B- APR C-anterior resection

PLEAZE, ANY NOTE OR CORRECTION OR ADDITIN SEND: [email protected]


‫نسألكم الدعاء بظهر الغيب‬
P a g e | 152
1613. Male patient with esophageal carcinoma, the best method for palliative treatment?
A- Laser ablation B- Stent
1614. Female patient with breast mass 1 cm by mammogram, stellate mass, with irregular
border and micro-calcification, true cut biopsy revealed hyperplastic tissues, what’s the
management?
A- Excisional biopsy by wire detection preoperative
B- Modified radical mastectomy
C- Simple mastectomy
1615. Female patient with past history of subtotal thyroidectomy for MNG, presented with
recurrence of goiter with normal T3, T4 and elevated TSH, what’s the cause? Hypothyroidism
1616. Long case of hydatid cyst of the liver, what’s the layer of excision?
A- Germinal layer and laminated layer
B- Laminated layer and adventitia
C- Germinal layer and adventitia
D- Adventitia and capsule
1617. Stab wound to the neck 2 cm below the angle of the mandible with progressive
hematoma, low BP, and tachycardia
A- Exploration B- Angiography C- Conservative and follow-up
1618. A case of pancreatitis, age 60, WBCs 21,000 glucose 11mmol, LDH high 450 AST 244?
A- Severe case of pancreatitis
B- MR less than 5%
C- Amylase is indicator for severe disease
1619. Case of iatrogenic esophageal perforation, diagnosis? Water soluble contrast swallow
1620. A case of parameters of septic shock
A- Sepsis + organ dysfunction B- SIRS + infection
C- Severe sepsis + hypotension D- Pulse more than 90, WBC more than 12,000
1621. Internal opening 2cm from anal verge, no ext. opening, MRI was done (‫)صووره‬, PR examine
palpable collection at post. Lat. Aspect 2cm from the opening what`s the type
A- low anal B- Pelvi-rectal C- subcutaneous(ischiorectal) D- sub mucous
1622. Young girl presented with abd. Pain, family history of FAB what the best investigation
A- CT B- Colonoscope C-abdominal US D-MRI abdomen
1623. 20 yrs. old girl, presented with diarrhea, mucous discharge, loss of wt. +ve family history
of same case diagnosis
A- ulcerative colitis B- crhon's C- cancer colon

PLEAZE, ANY NOTE OR CORRECTION OR ADDITIN SEND: [email protected]


‫نسألكم الدعاء بظهر الغيب‬
P a g e | 153
1624. Breast mass, hard, irregular 3cm retro areolar with nipple retraction, serous discharge
diagnosis
A- intraductal papilloma B- ductectesia C- cancer breast
1625. pt. presented with diarrhea, mucous, lower abd. Pain colonoscopy done show flat
adenomatous polyp 2,2cm what`s the me sure concern
A- bleeding B- intussception C- malignancy
1626. After lat. Sphincterotomy to ttt fissure pt. Presented with incontinences to lose stool and
gas how to asses this case?
A- endoanal u\s B- pudendal nerve terminal motor latency
1627. Lower abd pain mainly to rt., referred to scrotum, groin BP 139\79 Ct picture show 2
white point in front vertebra what the diagnosis
A- ureteric stone B- leaking aortic aneurysm
C- acute appendix D- acute diverticula
1628. Infection, crepitus at buttock what`s the proper antibiotic? penicillin G
1629. I.V addict, severe infection at arm. normal vessels, sensation ttt.
A- amputation B- antibiotic C- aggressive debridement
1630. After lap choli there is leak from cystic duct ? ERCP with stent if high output
1631. Swelling at back 2*2 cm., caseous material, punctum on top, diagnosis
A- lipoma B- sebaceous cyst C- abscess
1632. Most common complication after parotid surgery
A- frey`s syndrome B- fascial N palsy
1633. Which of the following elevated with activation of baroreceptors? catecholamine
1634. pt. with polyuria, polydipsia, increase K, hypertension, Na increase, adrenal mass which
of the following elevate? aldosterone (hyperaldosteronism)
1635. Female old age after abd. Surgery there in parotid enlargement what is the cause
A- parotitis B- parotid duct stone
1636. After abd. Surgery and there are SSI how to prevent the SSI?
A- antiseptic before operation B- clipping at the surgery time
C- antibiotic within 12 hrs. D- avoid hypothermia perioperative

1637. Businessman come to hospital and u consult him then he appreciates with u and need
tomake friend relationship with u what will u do ethical point for u
A- do relationship with him B- refuse any relationship
1638. pt. in hospital and on ICU then relative can`t pay for hospital what will u do
A- Discharge pt.
B- Referred to other hospital
C- keep pt. on vent. Without any compromise

PLEAZE, ANY NOTE OR CORRECTION OR ADDITIN SEND: [email protected]


‫نسألكم الدعاء بظهر الغيب‬
P a g e | 154
1639. There is gangrene on lt. foot and pt. in OR and nurse prepare rt one, surgeon come and
amputate rt, instead of lt how to avoid these mistakes? marking
1640. Most common organ affected from burn? Skin
1641. Flame burn ai closed space (suspected inhalation injury) manipulation?
endotracheal intubation
1642. child 12 yrs. There is bilat. Neck swelling how to reach diagnosis(LNS)
A- Ct neck B- biopsy and histopathology C- FNA
1643. Near total thyroidectomy for toxic multinodular goiter and then recurrent Tsh low
,elevation of T3, T4 normal what the cause
A- Hypothyroidism B- hyperthyroidism
C- suppression T cell D- autonomous nodule
1644. Pediatrician with RTA, crushed L.L, X ray show Bilat. fracture femur pt. hypotensive what
is the major concern?
A- hypotension B- hemorrhage C- myoglobinuria
1645. Child with large swelling, transillumination clear fluid what`s the diagnosis?
cystic hygroma
1646. Chest trauma, distended neck vein, trachea shift to lt. BP 70\0, no air entry at rt. Side,hyper
resonant chest.
A- cardiac tamponade B- tension pneumothorax C- massive hemothorax
1647. 76 yrs.at last 6 months. There is neck mass, hard, fixed, hoarseness of voice
A- FNA B- isthmectomy C-CT neck
1648. Which of the following compromise the respiration during delivery? cystic hygroma
1649. RTA, scalp laceration, fascial, fracture femur and at 3rd day become unconscious what
the cause
A- 2nd hemorrhage(subdural hematoma) B- fat embolism C-electrolyte disturbance
1650. 3rd day post-operative day from gastrectomy decrease air entry at lower part of lt. lung,
fever
A- sub phrenic collection B- pneumonia
C- catheter related infection D- atelectasis
1651. Picture of burn at posterolateral aspect of knee 3rd degree, contraction, ulcer way if
coverage
A- 1ry intension B- 2ry C- skin graft D- flap reconstruction
1652. 1.5 yrs. old boy with unilat. Undescended testis just at the deep inguinal ring what the
rational of surgical intervention?
A- to the risk of malignancy B- fear of trauma C- to decrease risk of if infertility
1653. Case of limb trauma and asked about the method of follow up for fear of compartment
syndrome
PLEAZE, ANY NOTE OR CORRECTION OR ADDITIN SEND: [email protected]
‫نسألكم الدعاء بظهر الغيب‬
P a g e | 155
A- Doppler u/s B- bl. Pressure measure
C- CVP D- follow up measure with swan g.
Catheter(tonometr)
1654. Crohn's disease with exacerbation what is best drug?
A- steroid B- azathioprine C-5ASA D- biological ttt
1655. Old age cirrhosis, ascites, mass 3 cm. at liver staging for case which investigation
A- CT(for the liver it self)and MRI B- laparoscopic(abdominal staging) D-PET SCANwhole body
1656. Pt. Cancer colon 50% of wall circumference colonoscopy show multiple linear regarding
plain for resection pt.
A- 1ry resection B- 2ry only C- Staging D- Resection to 1ry, 2ry
1657. Suspected cancer colon what is the best investigation?
colonoscopy

1658. Symp., sign of esoph. Mass what is the best investigation? EUS
1659. Good assessment for cancer esoph? EUS
1660. Cancer esoph. Advanced stage, stent, then become lower chest pain what to do
A- Conservative (if stent migration) B- 1ry repair C-esophagostomy
N.B if its perforation esophagectomy
1661. Sympt. Of GERD, gastroesoph. Junction above level of diaphragm what is the cause
A- wide curora B- decrease gastro esoph. Pressure
C-increase intra abdominal pressure D- -ve intrathoracic pressure
1662. Pt. with lower Abd. Pain, pulsatile abd mass BP 80 /40? Exploration
laparotomy (AAA)

1663. Circumferential mass at anus, path. Sq. Cell carcinoma management


A- radiation B- chemo radiation C- staging re ass
1664. Recurrent inguinal hernia best management? Laparoscopic TAAP
1665. Female pt., abd. pain referred to thigh, sign, symp. For obstruction? strangulated hernia
1666. Most prognostic factor of leiomyoma
A- Mitotic count B- tumor size
1667. leiomyoma at stomach 2cm lesion management
A- enucleation ( in esophagus) B- wedge resection
1668. Multiple stone at distal part of pancrease, pt. alcoholic management?
A- distal pancreatectomy B- longitudinal pancreatojejunostomy
1669. pt., diabetic, oral hypoglycemic, presented with upper abd, pain, lipase high., amylase
high cause
A- drug B- gall stone
1670. Cancer rectum can`t be assed invasion of mesorectum what u do?

PLEAZE, ANY NOTE OR CORRECTION OR ADDITIN SEND: [email protected]


‫نسألكم الدعاء بظهر الغيب‬
P a g e | 156
A- endorectal U\S B- MRI
1671. pt. jaundice with history of pancreatic cyst, aspirate fluids show low amylase, increase
glycogen
A- serous B- mucinous C- pseudocyst

PLEAZE, ANY NOTE OR CORRECTION OR ADDITIN SEND: [email protected]


‫نسألكم الدعاء بظهر الغيب‬
P a g e | 157
1672. Pt. with upper acute abd. realvead by morphia diagnosis
A- urea B- creatinine kinase C- Abd. Radiography
1673. Pt, acute pancreatitis in ICU he presented with anuria, distension, CVP 16
A- acute abd, compartmental syndrome B- intestinal obstruction
1674. Increase arterial systemic vascular resistance, increase CVP, saturation low, cardiac
output low
A- hypovolemic B- cardiogenic
1675. ER doctor finished his duty and waiting for his colleague who will come after 15 min,
there is patient in ER, what to do?
A- Leave the patient and left ER
B- assess and stabilize the patient till his colleague arrives
1676. Old lady, has end stage tumor, her son requested not to tell his mom about her
condition, what to do?
A- Tell her any way
B- Ask the son why not to tell her
C- Don’t tell her
1677. Child post RTA, has huge hematoma on his thigh and profuse bleeding, needs urgent
intervention, his father on the way to the hospital, what to do?
A- wait for his father to sign the consent
B- proceed with operation without consent
1678. Doctor is doing intra-articular injection for patients with osteoarthritis in his clinic
without optimum sterilization, as a registrar who wants to document the incidents what to use?
A- incident report B- prospective study C- retrospective study
1679. While doing CPR to the patient, the nurse noticed that the doctor is going to give wrong
medication, and she told him but he ignored her?
A- negligence B- Authority gradient
1680. Pt post operation, while in airport, the warning device used for body surveillance beeps,
what to do in OR to avoid that? sign out
1681. GB empyema in elderly not fit for surgery? percutaneous cholecystostomy
1682. Intraoperative cholangiography discovered missed stone in CBD, how to manage?
choledecoscope
1683. Pt had cholecystectomy 2 yrs. ago, came with RUQ pain, fever and jaundice, what’s is
the expected diagnosis?(charcots triad)
A- Missed stone B- ascending cholangitis
1684. 70 yrs. old man from nursing home admitted with obstructive sympt.to have Cecal
volvulus, responded to conservative ttt, what the most appropriate next management to do?

PLEAZE, ANY NOTE OR CORRECTION OR ADDITIN SEND: [email protected]


‫نسألكم الدعاء بظهر الغيب‬
P a g e | 158
A- rt. Colectomy with end ileostomy B- rt. Hemicolectomy with 1ry anastomosis

C- cecopexy (if unstable) D- tube cecostomy


1685. while doing ERCP for missed stone, bleeding at the ampulla of vater, 2nd day pt.
presented with acute epigastric Pain what is the cause of pain
A- duodenal bleeding B- pancreatic injury C-duodenal injury D- missed stone
1686. Acute pancreatitis, he asked about the severity and Mandatory rate? 5
***Patients with one or two criteria have a predicted mortality of less than 1%, with three
criteria (10%) or four criteria (15%); with more than seven criteria 50%
1687. Cholangiocarcinoma (deep jaundice), how to diagnose? MRCP
1688. How to differentiate between pancreatitis and perforated PU? CT x ray erect abd us
1689. Perianal discharge and cord like structure, there was a past history of perianal abscess
drained 2 yrs. ago, what’s the diagnosis? Perianal fistula
1690. Diverticulitis how to assess complicated diverticulitis fistula? CT
1691. why the previous pt. developed collection and abscess formation after antibiotic ttt?
no coverage foe anaerobic bacteria(metronidazole)
1692. How to differentiate between femoral hernia and saphina varix?
A- CT B- duplex C- MRI
1693. Lady presented with acute abdominal pain and sign of peritonitis Last Menstrual Period
was 6weeks ago? ruptured ectopic
1694. RTA face laceration, on the 2nd day, developed confusion and restless?
A- delayed concussion B- 2ndry cerebral Hge C- Hypovolemic shock
1695. RTA, pt is shocked and unconscious, how to open his airway?
A- ETT B- orotracheal tube C- nasotracheal tube D- face mask
1696. RTA, fracture ribs, paradoxical chest movement? flail chest
1697. Chest trauma, stap wound at epigastrium and cause cardiac tamponade, what is the site
injured at the heart? Right ventricle
1698. Chest trauma, surgical emphysema,rupture bronchus how to diagnose? Bronchoscopy
1699. Burn 40%, how to assess tissue perfusion? Urine out put
1700. Large burn ulcer on the lateral aspect of the knee (Marjolin), what indicating bad
prognosis? inguinal LNs involvement
1701. Nasal BCC in face behind nose, ttt? moh’s technique 2mm safety margin in face
1702. pt. was planned for TURP, came comatose, with full bladder, what’s next? on atime for
operation
A- folly’s catheter B- suprapubic catheter
1703. Child while riding bicycle, developed blunt trauma to his abdomen, what is the most
organ to be injured?

PLEAZE, ANY NOTE OR CORRECTION OR ADDITIN SEND: [email protected]


‫نسألكم الدعاء بظهر الغيب‬
P a g e | 159
A- spleen B- liver C- mesentery D- kidney
1704. Fournier's gangrene, necrotizing fasciitis, ttt? Debridement +antibiotics

PLEAZE, ANY NOTE OR CORRECTION OR ADDITIN SEND: [email protected]


‫نسألكم الدعاء بظهر الغيب‬
P a g e | 160
1705. Child had tetanus vaccine 6 yrs. ago, injured with screw? Tetanus toxoid
1706. 35 yrs. old man had splenectomy post trauma, received post splenectomy vaccination
after operation, what to do next?
A- given antibiotics for 2 yrs. B- no further ttt
1707. Middle aged female developed profuse bleeding after tooth extraction, she mentioned
past hx of bleeding tendency, all bleeding profiles were normal apart from platelets 40.000
A- ITP B- TTP C- thalassemia D- gran’s man’s
1708. T-tube drainage per day should be?less than 500 ml
A- 0-100 ml B- 100-250 ml
1709. pt. with right foot gangrene extending to above his knee, CT angiogram done and
showed left sided occlusion of ex. iliac vessels, ttt?
A- stent B- embolectomy C- bypass
1710. Wrestler, has punched over the lateral aspect of his face, suspected fracture infraorbital
bone, what you suspect to occur? Diplopia
1711. Middle age lady presented with 2 days hx of severe abdominal pain that only get
relieved with morphine, she described emesis twice hematemesis and Malena once, how to
reach the diagnosis?
A- hematocrit B- s. urea C- TLC D- abdominal angiography
1712. pt. known to have chronic liver disease and ascites, came in shock, fluids to give?
Salt free albumin
1713. Diffuse neck swelling, Tsh, T3, T4 normal, how to diagnose?
A- radio-scan B- us C- FNAC
1714. 55 yrs. old female, with1 cm breast mass seen by U\S, what to do next?
A- FNAC B- core biopsy C-mamogram D_MRI
1715. Open book pelvic fracture, pt. is shocked III, what fluid to give for resuscitation? blood
1716. pt. with hx of PUD, came to ER with hematemesis, UGI endoscopy showed an ulcer in the
posterior wall of the first part of the duodenum with bleeding vessel and hematoma in the
floor, bleeding stopped, but 2 days later hematemesis recurred, what to do?
A- repeat endoscopy B- suture(ligating) the vessel C- Octreotide
1717. RTA, fracture long bone, 2nd day the pt. developed respiratory distress and petechial
rash? Fat embolism
1718. Man wt. 125 kg, known to have COPD, with hx of snoring and sleep apnea, planned for
operation (not mentioned the name of operation), what not to give post-operatively?
A- NSAID B- sedatives
1719. H-pylori causing which type of gastropathy? Type B

PLEAZE, ANY NOTE OR CORRECTION OR ADDITIN SEND: [email protected]


‫نسألكم الدعاء بظهر الغيب‬
P a g e | 161
1720. Trachea-esophageal fistula, what you see in CXR?
A- coiled nasogastric tube B- stomach bubbles
1721. Femoral hernia ttt
A- lap. Repair B- Lockwood C- herniotomy
1722. Female presented with obturator hernia (obturator is mentioned in the Qs), the trapped
bowel was found darkly purple, what’s the cause?
A- arterial obstruction B- venous congestion
1723. Child in ICU, family couldn’t afford hospital stay because of the cost, what you will do
‫يفضل زي ما هو‬
1724. Collection after lap-chole?
A- ERCP B- US aspiration
1725. Wound, not properly treated with antibiotics, get inflamed (red, tender, hot and swollen),
with fever and BP 80\50
A- wound infection B- severe sepsis C- septic shock
1726. Picture of large leg swelling with hyperpigmentation, crusted in diabetic female?
A- lipodermatosclerosis B- hemosiderin deposition C- group A strept
1727. Repair of aortic aneurysm, what’s the most dangerous complication?
A- aorta–enteric fistula B- renal failure C- aneurysm
1728. RTA, pelvic fracture and thigh wound with profuse bleeding = there is hypovolemic
shock parameters, what last to occur?
A- increase venous return
B- increased cortisone level
C- increased arteriolar resistance
1729. Hirschsprung diagnosis?
A- full thickness rectal biopsy B- rectal mucosal biopsy initially
1730. Vasopressor use in? Distributive shock
1731. pt. transferred to word from burn unit, developed melena, fluid resuscitation done, what
next?
A- NGT B- gastroscopy
1732. pt. with chronic arthritis, came with an acute abdomen?
A- acute pancreatitis B- perforated PU
1733. Features of soft tissue necrosis (necrotizing fasciitis)?
A- hand with diffuse swelling, warm B- wound + fever more than 40
C- non pitting edema D- wound with pus discharge

PLEAZE, ANY NOTE OR CORRECTION OR ADDITIN SEND: [email protected]


‫نسألكم الدعاء بظهر الغيب‬
P a g e | 162
1734. Ventral hernia in female with +ve cough impulse, disappear after lying supine, it appears
near a slit above umbilicus (slit means incision)?
A- umbilical B- paraumbilical C- epigastric D- incisional
1735. Neoadjuvant ttt of breast cancer, what does it indicate?
A- surgery then chemotherapy B- chemotherapy then surgery
1736. Wrong sided nephrectomy, and accidental bowel injury, the pt. should?
A- sue the doctor B- sue the hospital C- compensation
1737. Cause of common bile duct stricture? Incident injury
1738. Diagnosis for umbilical sinus? Sinogram
1739. pt. after operation by 7 days become feverish? Leaking with abscess formation
1740. pt. become feverish after 8 hrs. postoperative?
A- Atelectasis B- body reaction

1741-medical management of melanoma:

PLEAZE, ANY NOTE OR CORRECTION OR ADDITIN SEND: [email protected]


‫نسألكم الدعاء بظهر الغيب‬
‫‪P a g e | 163‬‬

‫‪CAPSULES‬‬
‫بسم هلال الرحمن الرحيم‬
‫وسريعة من كل حتة في الجراحة هتفيدك جدا‬ ‫دي تجميعه مهمة‪-1‬‬
‫في مراجعة ليلة االمتحان‪ ،‬كانت محتاجة تنسيق أفضل بس سيبناها‬
‫‪.‬كده عشان الوقت كان ضيق ممكن نضبطها أكتر اإلصدار القادم‬
‫المعلومات متراجعة بس لو لقيت معلومة محتاجة تصحيح ابعت‪-2‬‬
‫المصدر‬ ‫ومعاه‬ ‫على الميل ده التصحيح‬
‫‪[email protected]‬‬
‫أي حد هيقرأ هتشيل ذنبه ايوه‘ بشيلهالك ●‬
‫‪v‬‬ ‫لو مبعتش التصحيح‪،‬‬
‫‪:‬‬
‫لو حابب تضيف معلومة ابعت برده على نفس الميل و احنا‪-3‬‬
‫هنضيفه في اإلصدار القادم ويبقى في ميزان حسناتك‬
‫أسألكم الدعاء بظهر الغيب‪-4‬‬
‫●‬
‫متنساش الدعاء‪v -5‬‬
‫‪:‬‬

‫‪PLEAZE, ANY NOTE OR CORRECTION OR ADDITIN SEND:‬‬ ‫‪[email protected]‬‬


‫نسألكم الدعاء بظهر الغيب‬
P a g e | 164

CAPSULES

PLEAZE, ANY NOTE OR CORRECTION OR ADDITIN SEND: [email protected]


‫نسألكم الدعاء بظهر الغيب‬
P a g e | 165

 

 

PLEAZE, ANY NOTE OR CORRECTION OR ADDITIN SEND: [email protected]


‫نسألكم الدعاء بظهر الغيب‬
P a g e | 166

 

PLEAZE, ANY NOTE OR CORRECTION OR ADDITIN SEND: [email protected]


‫نسألكم الدعاء بظهر الغيب‬
P a g e | 167

PLEAZE, ANY NOTE OR CORRECTION OR ADDITIN SEND: [email protected]


‫نسألكم الدعاء بظهر الغيب‬
P a g e | 168

PLEAZE, ANY NOTE OR CORRECTION OR ADDITIN SEND: [email protected]


‫نسألكم الدعاء بظهر الغيب‬
P a g e | 169

PLEAZE, ANY NOTE OR CORRECTION OR ADDITIN SEND: [email protected]


‫نسألكم الدعاء بظهر الغيب‬
P a g e | 170

PLEAZE, ANY NOTE OR CORRECTION OR ADDITIN SEND: [email protected]


‫نسألكم الدعاء بظهر الغيب‬
P a g e | 171

PLEAZE, ANY NOTE OR CORRECTION OR ADDITIN SEND: [email protected]


‫نسألكم الدعاء بظهر الغيب‬
P a g e | 172








PLEAZE, ANY NOTE OR CORRECTION OR ADDITIN SEND: [email protected]


‫نسألكم الدعاء بظهر الغيب‬
P a g e | 173
 Pelvic abscess post diverticulitis:
Management according to Henchey classification:

- Stage I: < 4cm with pericolic collection……. antibiotics


- Stage II: > 4cm and distant air > 5cm ……. Lab drainage
- Stage III: purulent peritonitis …. lab drainage
- Stage IV: feculent peritonitis ……. Surgery -hartman
-Sigmoidectomy
 Cancers that cause gynecomastia:
All endocrine and exocrine tumors except bone and GB (least tumor)

 Differences between 1ry, 2ry and 3ry hyperparathyroidism:


- 1ry …… pathology in the gland…. increase Ca and decrease phosphate
- 2ry …… pathology in the kidney …. decrease in Ca and increase in
phosphate
- 3ry……. Autonomas parathyroid nodule ……. no effect ……. hypocalcemia
with renal transplantation
N.B. With all types there is increase in PTH

 Hydatid cyst in the lung…. management (size)


- 2-3 cm ……. Albendazole
- 4-6 cm ……. Pneumonectomy
- If > 50% of lobe ……. Lobectomy
N.B. no PAIR in lung, it may cause anaphylaxis

 Hydatid cyst in liver management, according to GARPI classification:


- Stage I: simple cyst …. (if > 5cm) albendazole (if < 5cm) albendazole +
PAIR

PLEAZE, ANY NOTE OR CORRECTION OR ADDITIN SEND: [email protected]


‫نسألكم الدعاء بظهر الغيب‬
P a g e | 174

- Stage II: septated but ill defined ……. (if > 5cm) albendazole (if < 5cm)
albendazole + PAIR
- Stage III: septated but well defined + daughter ……. surgery
- Stage IV: heterogenous e daughter …… surgery
- Stage V: calcified wall ……. conservative
N.B. In all stages start with albendazole

 Stone in pancreatic duct: (size and site)


Size:

- < 1cm …. ERCP and extraction by balloon


- 1cm …. according to site:
 in head or body: ERCP and stent
 extensive and strictures: lat. Pancreatic jejunostomy (Pestu operation)
 Pancreatic injury management: (isolated)

MRCP OR ERCP

injured duct
intact duct
ERCP

multiple lacerations slight silent Conservative


resection ( pancreatic external drain I.V.F and AB
dudenectomy)

N.B we decide whether it’s isolated or not through abdominal CT then if suspect
pancreatic duct injury not detected by CT do ERCP (the most accurate tool)

PLEAZE, ANY NOTE OR CORRECTION OR ADDITIN SEND: [email protected]


‫نسألكم الدعاء بظهر الغيب‬
P a g e | 175

Common bile duct stone:

- Pre- op.: ERCP + lab explo + cholecystectomy


- Inta- op.: lab or open exploration of CBD + cholecystectomy then ERCP
- Post op.: ERCP …… if no leak: exploration CBD + cholecystectomy
……. if there is leak: drainage

 Hypokalemia in ECG:
- Q- wave: slightly peaked
- Q-T interval: prolonged
- S-T segment: depressed
- T- wave: inverted or flat
- U- wave: prominent (important)
- Arrhythmias
 Hyperkalemia in ECG:
- Q-T interval: short
- S-T segment: depressed
- T wave: tall peaked (important)
 Hypocalcemia in ECG:
- Q-T interval: prolonged
- S-T segment: prolonged
- T wave: typically, unchanged (important)
 Hypercalcemia in ECG:
- Q-T interval: short
- S-T segment: short
- T wave: flat
N.B. like ECG changes of MI

PLEAZE, ANY NOTE OR CORRECTION OR ADDITIN SEND: [email protected]


‫نسألكم الدعاء بظهر الغيب‬
P a g e | 176

 Hypomagnesemia in ECG:
- QRS complex: wide
 Hypothermia: - bradyarrhythmia - osporn J wave
 SRS:
- More sensitive to Gastrinoma and glucagonoma
 Somatostatin diagnosis:
- EUS, SRS
- Localized in head of pancreas
 Gastrinoma:
- Most common pancreatic endocrine tumor in MEN 1
- Multiple
- Site: gastrin triangle in duodenum
- Diagnosis: secretin stimulatory test or Ca stimulatory test or gastrin level
>500- 1000
- Localization by SRS
 Glucagonoma:
- Site: tail of pancreas
- Hyperglycemia
- Hypo aminoacidemia
- Necrolytic migratory erythema
- Diagnosis: glucagon level
- Localization by SRS
 Insulinoma:
- Site: head, body and uncinate process
- Most common pancreatic tumour
- Diagnosis: C peptide pre insulin

PLEAZE, ANY NOTE OR CORRECTION OR ADDITIN SEND: [email protected]


‫نسألكم الدعاء بظهر الغيب‬
P a g e | 177

- Glucose < 50, improve after giving glucose


- Localization: endoscopic U/S
 VIPoma:
- Symptoms: abd pain and diarrhea
- Hypokalemia
- Diagnosis: VIP level
- Localization: EUS or CT
- Site: head of pancreas
 Head of pancreas:
- VIPoma and insulinoma
 Tail of pancreas:
- Glucagonoma and somatostatinoma
 MEN I:
- Pituitary adenoma
- Parathyroid hyperplasia (most common)
- Pancreatic tumor (mainly gastrinoma)
 MEN II A:
- Parathyroid hyperplasia
- Medullary carcinoma
- Pheochromocytoma
 MEN II B:
- Neuroma tumors
- Marphenoid syndrome
- Medullary carcinoma
- Pheochromocytoma
 Esophageal varices management:

PLEAZE, ANY NOTE OR CORRECTION OR ADDITIN SEND: [email protected]


‫نسألكم الدعاء بظهر الغيب‬
P a g e | 178

- Stage I: re-endoscopy within 1 year…… if still bleeding ….. injection


sclerotherapy
- Stage II & III: propranolol inj. But if the patient is intolerant ….. rubber
band
- Stage IV: devascularization and shunt
 Management of esophageal varices:
Upper endoscopy

- No varices (stage 0): surveillance by EG scope after 2-3 years


- < 5ml with no red sign (stage I): surveillance
- < 5ml with red sign (stage I): B- blocker
- >5 ml (stage II or III): B- blocker or band ligation
 Organism was commensal and become pathogen:
- Staph epidermidis…. tunneled catheter hemodialysis injection
- Pseudomonas aeruginosa
 Investigations for GERD:
- Best: 24 PH monitoring
- Proper: endoscope
 Investigations for achalasia:
- Best: manometry
- Proper: barium swallow
 Pancreatic pseudocyst:
- Increase amylase
 Intra papillary mucinous neoplasm (IPMN):
- Increase amylase
- Increase CEA
 Mucinous cystadenoma:

PLEAZE, ANY NOTE OR CORRECTION OR ADDITIN SEND: [email protected]


‫نسألكم الدعاء بظهر الغيب‬
P a g e | 179

- Decrease amylase
- Increase CEA
 Serous cystadenoma:
- Decrease amylase
- Decrease CEA
 Gall bladder carcinoma management:
- TiA: limited to mucosa …. cholecystectomy
- TiB and deeper: muscularis and deeper…. wedge liver resection + extended
cholecystectomy or segmentectomy + lymphadenectomy
 Zone II neck injury (stab):
- Exploration
 Zone I or III neck injury (stab):
- Stable: investigate
- Unstable: exploration
 Obstruction of sigmoid:
- Constipation, distension and late vomiting
 Small bowel obstruction:
- Pain, early vomiting, late constipation and distension
 Indication of TIPs: - uncontrollable variceal bleeding.
- Refractory ascites
 Patient post RTA -> dark brownish urine acute tubular necrosis due to
crush injury.
- Management: urine alkalinization by Na. bicarb.
 If patient post RTA e limb swelling & pain  reperfusion injury.
 Management of biliary atresia: - better to be within 2 weeks, if more than 8
weeks bad results.

PLEAZE, ANY NOTE OR CORRECTION OR ADDITIN SEND: [email protected]


‫نسألكم الدعاء بظهر الغيب‬
P a g e | 180

 If pancreatitis e splenic v. thrombosis  splenectomy.


 If pancreatitis e splenic A. thrombosis  conservative.
 Splenic A. aneurism: - # resection if:
- symptomatic
- Pregnant pt.
- > 2cm size.
# Ligation if in middle third.
# Resection & splenectomy  if distal.
# Embolization V. painful.
 Hernia normally reducible  PUH.
 Hernia never reducible  epigastric hernia.
 Indication of cesarian section in pregnant e trauma: -
- Profound shock e impending cardiac arrest.
- Uterine injury or fetal distress.
 Staging of malignancy in stomach, esophagus, periampullary, pancreas
Endoscopic ultrasound.
 Fealty's S  neutropenia, splenomegaly, rheumatoid, anemia,
thrombocytopenia.
 ABI (Ankle Brachial Index) for assessment of ischemia: -
- Normal  1
- Mild  0.7 – 1
- Moderate  0.5 – 0.7
- Severe  <0.5

 Compartmental syn.
- Pressure > 25 mmHg.

PLEAZE, ANY NOTE OR CORRECTION OR ADDITIN SEND: [email protected]


‫نسألكم الدعاء بظهر الغيب‬
P a g e | 181

- 1st presentation sever pain.


- Management fasciotomy.
 Normal intra-compartmental pressure  15 -25 mmhg.
 Pressure intra-compartmental by Tonometry.
 Annular pancreas Associated e
- Polyhydramnios.
- Down syndrome.
- Esophageal atresia.
- Imperforated anus.
- Mickle’s diverticulum.
Diagnosed by u/s, ERCP.
Managed by
- Duodeno-duodenostomy.
- Duodeno- jejunostomy.
 Pancreatic division: - failure of fusion pancreatic parts.
So, head & body separated from tail
Diagnosed by: ERCP or MRCP.
Managed by: ERCP & sphincrotomy.
 Investigation before bariatric surgery  U/S.
 Investigation before sleeve  upper endoscopy.
 I.O. post gastric sleeve  Adhesion.
 I.O. post gastric bypass  internal hernia.
 Sliding hiatus hernia caused by  short esophagus.
- Stomach in place & GE junction above diaphragm.
- Symptoms of GERD.
Management

PLEAZE, ANY NOTE OR CORRECTION OR ADDITIN SEND: [email protected]


‫نسألكم الدعاء بظهر الغيب‬
P a g e | 182

- Colle’s gastroplasty.
- Conservative ttt for GERD.
 Paraoesophageal hernia
- GEJ  below diaphragm.
- Wide crura  stomach above diaphragm.
- Symptoms of pressure & may be strangulated.
- Surgical management
 Most diagnostic scan for pheochromocytoma  MIPG scan
 Diagnostic scan for osteoporosis  DEXA scan (bone density scan)
 Sigmoid volvulus  from Lt to RT iliac side & inverted U shape.
 Cecal volvulus  coffee bean appearance

from Rt to Lt iliac.
 Mesenteric mid gut volvulus  apple peel or twisted ribbon.

PLEAZE, ANY NOTE OR CORRECTION OR ADDITIN SEND: [email protected]


‫نسألكم الدعاء بظهر الغيب‬
P a g e | 183

 Cancer rectum: Diagnosis


- CT abdomen & pelvis
- Examination under anesthesia & biopsy
T1 within 10 cm from anal verge.
- Trans anal local excision  if e invaded margin 
radical rectal excision.
- T2  surgery.
- T3 or N1 preoperative chemo-radio therapy.
 Cancer colon stage 3  adjuvant chemotherapy (post op.)
 Surgery for cancer rectum  total meso-rectal excision.
- Low anterior resection: -
If tumor > 5 cm from the anal verge.
- Abdomen-perineal resection: -
If tumor invade the anal canal or < 5 cm anal verge.
 Safety margin in cancer rectum excision (mainly distal)
- 1 cm in wall & 4 cm in mesorectum
- If upper rectum 4 cm

PLEAZE, ANY NOTE OR CORRECTION OR ADDITIN SEND: [email protected]


‫نسألكم الدعاء بظهر الغيب‬
P a g e | 184

- If lower rectum (no meso)  so 1 cm.


 Cancer colon etiology: -
- Sporadic: 60 % of cases (no familial H.)
- Familial: e no gene
- Hereditary: known genetic factor
E.g.: FAP, hereditary non polyposis cancer rectum.
 Cancer rectum T3 or N1:
- If lower 2/3  neoadjuvant chemo-radiotherapy.
- If upper 1/3  as cancer colon.
 Best investigation of cancer rectum  MRI.
 Screening of cancer rectum & colon  colonoscopy
- If average risk people e no family history  start 50
y.old & every 10 y
- If +ve family H. start at 40 yr. or 10 year younger
than his relative discover & continue every 10 y.
- If FAP  start at 12 yr. & continue every year.
 If – ve at age of 25 e no polyp  ↓
frequency.
 If –ve at age of 35  ↓ frequency.
 If –ve at age of 50 do every 5 years.
- If hereditary non polyposis cancer rectum start at
20 yr. & continue every year.
 After cancer colon excision  follow up after 1 year
- If –ve: follow up after 3 years
- If –ve: follow up every 5 years.
 If Pt e adenoma  colonoscopy after 3 y if –ve every 5 y.

PLEAZE, ANY NOTE OR CORRECTION OR ADDITIN SEND: [email protected]


‫نسألكم الدعاء بظهر الغيب‬
P a g e | 185

 If high risk adenoma  1 y, then 3 y, then 5y. (like cancer colon)


 Screening of colonic adenoma(polyp)  by colonoscopy.
- If tubular adenoma  average (5-10 years)
- If multiple (1-2) & sessile - <10 ml  every 10
years
- - >10 ml  every 5 years.
- If multiple (3-10) or serrated or villus  every 3y.
- - If multiple (>10) < 3 years.
 Adenocarcinoma of colon or rectum after resection:
- If free margin  observation
- If not  surgical resection.
 T staging of cancer colon & management:
- T1  submucosa  trans anal endoscopic
resection.
- T2mucosal laparoscopic resection.
- T3invading the meso rectum  neoadjuvant
chemo then surgery.
- T4surrounding structures.
 To detect metastasis  CT abd & pelvis.
 For nodal staging  MRI.
 Crohn’s disease: common site  terminal ilium. Transmural lesion e skip
lesion e pyoderma gangerinosum.
 Ulcerative colitis  mucosal & sub mucosal  no skip lesion e erythema
nodosum.
Start ttt e steroid.
Classification of gastric LN dissection

PLEAZE, ANY NOTE OR CORRECTION OR ADDITIN SEND: [email protected]


‫نسألكم الدعاء بظهر الغيب‬
P a g e | 186

 D1: rt and lt cardia, lesser and greater curvature


 D2: hepatic and lt gastric and pylorus rt and lt
 D3: down to transverse colon and costopancreatic
Safety margin of

BCC: 3-5 mm

SCC: 5-10 mm

Malignant melanoma: >or= 1ml - 1 cm

1-2 ml - 1-2 cm

<2 ml - 2 cm

Indications of admission in burn cases

 Children < 10 % BSA


 Adult < 15 % BSA
 Chemical or electrical burn
 Specific areas (face – hand – perineum)
 Inhalation burn
 Educational or environmental barrier for home care
 Infected burn in child or diabetic patient
 Circumferential burn (limb or chest)
Major burn if:

 <15% BSA in children


 <20% BSA in adult
Inhalation burn

PLEAZE, ANY NOTE OR CORRECTION OR ADDITIN SEND: [email protected]


‫نسألكم الدعاء بظهر الغيب‬
P a g e | 187

 History of burn in closed space


 Facial edema
 Hoarseness of voice
 Carbon particles around nostrils and on the tongue
 Diagnosis by bronchoscope
Pseudoepitheliosis hyperkeratosis – repeat biopsy

Management of sacral pressure sores

 Grade I and II – conserve


 Grade III and IV – surgical debridement + coverage by skin graft or flap
Instructions:

 Change patient position every 2 hours


 Air mattress
 Improve general condition and nutrition
VAC therapy (-ve pressure) (clean wound)

If the patient is 3rd or 4th degree not fit for surgery

Marjolin ulcer (squamous cell carcinoma)

Management: role of surgery:

 excision and coverage by skin graft or flap with clean safety margin
 Amputation to avoid lymphatic spread caused by manipulation
Spherocytosis:

 Diagnosis with fragile test


 U/S pre-operative (to show gallbladder)

PLEAZE, ANY NOTE OR CORRECTION OR ADDITIN SEND: [email protected]


‫نسألكم الدعاء بظهر الغيب‬
P a g e | 188

 If the patient doesn’t improve after splenectomy …. Accessory spleen which


most common present in hilum, re- operation
ITP:

 Bleeding tendency due to antibodies to platelets


 PT and INR are normal
 Treated first by steroids…. if failed splenectomy
 Normal spleen size
Neurogenic shock:

 Hypotension, bradycardia and warm hand


Septic shock:

 Tachycardia and warm hand


Helicobacter pylori:

 Duodenal ulcer 90%, gastric ulcer 70%


 Type B gastritis
 Follow up of improvement by: urea breath test
 Most common cause of gastric ulcer
Gastrinoma:

 Multiple
 Most common site: Gastrinoma triangle
 May be present in pancreas or heterotopic gastric tissue in Meckel’s
diverticulum (more common)
 Diagnosed by: secretin stimulating test (secretin releasing secretion)
Glucagonoma:

 Associated with hyperglycemia


 Associated with fleeting arthritis (poly-migratory arthritis)

PLEAZE, ANY NOTE OR CORRECTION OR ADDITIN SEND: [email protected]


‫نسألكم الدعاء بظهر الغيب‬
P a g e | 189

 High fasting glucagon


 Hyperglycemia
 Associated with necrolytic migratory erythema
Insulinoma:

 Associated with hypoglycemia


 mostly benign
 In beta cells of Langerhans
 Increase insulin
 Treatment: surgery or injection by alcohol
 Improved by feeding
 C peptide test is done for diagnosis, glucose level < 50 mg/dl
Zollinger Ellison syndrome:

 Multiple peptic ulcers or in abnormal sites, or resistant to treatment


 Gastric acid output:
maximum basal output: maximum acid output…. if > 0.6 = Gastrinoma
VIPoma:

 3rd commonest malignancy in pancreatic islet cells


 Watery diarrhea, hypokalemia and achlorhydria (WDHA syndrome)
Meckel’s diverticulum:

 Most common tumor is carcinoid


 2nd most common tumor is Gastrinoma
 Most common ectopic cells are gastric cells
Most common tumors in endocrine pancreatic tumors:

 1st: insulinoma
 2nd: Gastrinoma

PLEAZE, ANY NOTE OR CORRECTION OR ADDITIN SEND: [email protected]


‫نسألكم الدعاء بظهر الغيب‬
P a g e | 190

 3rd: VIPoma
Somatostatinoma: (3S)

 Diabetes (sugar)
 Steatorrhea
 Stones (gallbladder)
Ulcerative colitis:

 Most associated with erythema nodosum (extra-intestinal manifestations)


Crohn’s:

Associated with pyoderma gangrenosum (picture of ulcerative nodule or pustule on


chin of tibia)

CHPS:

 Projectile non bilious vomiting


 Age: around 1st month of life
 Hypokalemia, hypochloremia, metabolic alkalosis, aciduria
 Treatment: normal saline
 Investigation: U/S
 Operation: pyloromyotomy (Ramstedt operation)
Intestinal atresia (jejunal atresia)

 In a child with gastroschisis


 Bilious vomiting
Post thyroidectomy:

 If hypertension and fever: thyroid storm


 If hypotension, hypoglycemia and no fever: adrenal insufficiency
Duodenal atresia:

PLEAZE, ANY NOTE OR CORRECTION OR ADDITIN SEND: [email protected]


‫نسألكم الدعاء بظهر الغيب‬
P a g e | 191

 Non-bilious vomiting
 Vomiting in 1st few days of life
Wound healing:

- Vit. C: very important for healing


- Vit. A: against action of steroid on healing
Factors that impair wound healing:

1. Diabetes
2. Local wound infection ‫أهم سبب‬
3. Steroid use
 L.N dissection with malignant melanoma:
Medial to pectoralis minor
 Non bilious vomiting: with CHIPS & pyloric stenosis
 Bilious vomiting: with duodenal & jejunal atresia
 Common anomaly with gastroschisis is: (intestinal atresia)
 Omphalocele accompanied by:
Cardiac problems and complicated with infection
 Pyloric stenosis presents within month
 Duodenal atresia presents within days
 ASA II: pt with comorbidities but controlled
 ASA III: PT with comorbidities but in incapacitating
‫يعنى ال تعوق حياته‬
Splenic trauma grading:

Grade I: hematoma < 10 ml or laceration < 1 cm → conservative


Grade II: hematoma 10-50 ml and tear 1-3cm → conservative
Grade III: hematoma> 50ml and tear > 3m →Conservative surgery if
unstable

PLEAZE, ANY NOTE OR CORRECTION OR ADDITIN SEND: [email protected]


‫نسألكم الدعاء بظهر الغيب‬
P a g e | 192

→conservative if stable
Grade IV: plush ie blood vessel injury by CT: embolization then
reembolization then splenectomy
Grade V: shuttered spleen→ splenectomy
Shock grading: (hypovolemic)

Grade I: <15% i.e., 750ml BP → normal


Pulse →100
Urine OP → 30ml
RR → 20
Pt is alert
ttt→ crystalloid

Grade II: 15-30% i.e., 750-1500 ml BP → slight ↓


Pulse → 100-120
Urine OP→ 20-30ml
RR→20-30
Pt is anxious
Ttt→ crystalloid

Grade III: 30%-60% i.e., 1500-2000 m BP → ↓ but > 90/60


Pulse →120-140
Urine OP→ 5-15 ml
RR →30-40
Pt is confused
ttt→ crystalloid+ blood

PLEAZE, ANY NOTE OR CORRECTION OR ADDITIN SEND: [email protected]


‫نسألكم الدعاء بظهر الغيب‬
P a g e | 193

Grade IV: >40% i.e., >2000 ml BP→ < 90-60


Pulse → 140
Urine OP→ 0-5 ml
RR→ > 40
Pt is comatose
ttt→crystalloid+ blood
 Hernia: above & medial to pubic tubercle …. Indirect inguinal
Below & lateral to pubic tubercle…. femoral hernia
Below & medial: complete indirect
DD of direct & indirect hernia by ring test:
If no bulge: +ve …indirect
If bulge: -ve …. direct
Cysts of pancreas

→ pseudocyst ↑amylase

→ Mucinous cyst ↑ CEA

→ IPMN (fish mouth in ERCP) ↑amylase & ↑CEA


 Pancreatic stone → ERCP & stent
 Pt with multiple dilatations & constriction → petsow →lateral
pancreatico-jejunostomy
Pancreatic trauma:

→distal to lt superior mesenteric →distal pancreatectomy

→ Proximal to lt superior mesenteric → drainage then central pancreatectomy

→ If injured CPD → Whipple

 Fluid resuscitation in pancreatitis → RL

PLEAZE, ANY NOTE OR CORRECTION OR ADDITIN SEND: [email protected]


‫نسألكم الدعاء بظهر الغيب‬
P a g e | 194

 Main feeding in pancreatitis → naso jejunal tube


 Best investigation for pancreas → CT
Pancreatic pseudocyst: - < 6cm → follow up

- > 6 cm → endoscopic drainage

- Infected → percutaneous drainage

- >10cm → surgical internal drainage


Hernia:

- change umbilical shape → PUH


- in between xiphisternum & umbilicus or can feel linea alba → epigastric
hernia
- never reducible → epigastric hernia
- most common groin mass → inguinal hernia
- most liable for obstruction → femoral hernia
- inguinal hernia: if pt. < 48 → follow up
If pt. > 48 & symptomatic → laparoscopic ttt

- Femoral hernia at any age: laparoscopic ttt


- Recurrent hernia: TAPP
- Hernia not tender: obstructed
- Hernia tender: strangulated
Myeloproliferative disease of the spleen:

- One of causes of huge splenomegaly


- Diagnosed by CT with contrast or liver-spleen scan
- To diagnose associated hypersplenism → do bone marrow biopsy
** collagen formed after 48 hr reach maximum ⬛
~after 21 day

PLEAZE, ANY NOTE OR CORRECTION OR ADDITIN SEND: [email protected]


‫نسألكم الدعاء بظهر الغيب‬
P a g e | 195

Maximum tensile strength after 8 weeks

**skin neoplasm: basal cell carcinoma in face origin pluripotent cell.

Excionn by safety margin 5 mm

Ttt: surgical excision

** squamous cell carcinoma

Origin: ceratinized cell

Ttt: excision safety margin 5-10mm e’ radiotherapy

** melanoma <or=1m ⬛
~1cm

~1-2cm
<1-2m⬛

⬛2cm
~
>2m⬛

~ lymph node dissection up to medial to pectolaris minor


Inaxilla⬛

**Burn:

first degree: blunshing skin

Second degree: epidermis, superficial derms

Third degree: full thickness of the skin

** Role of 9:

one upper limp ⬛


~. 9%
⬛18
one lower limp~
~9
Head⬛
Chest:

~18
anterior⬛
Posterior ⬛
~18

PLEAZE, ANY NOTE OR CORRECTION OR ADDITIN SEND: [email protected]


‫نسألكم الدعاء بظهر الغيب‬
P a g e | 196

Perineum. ⬛
~1
Neck ⬛
~1

** fluid therapy: 4ml+weight +BSA

** complications of burn ⬛
~ marjolin ulcer ⬛
~ excision e’ graft or amputation

⬛ repeated biopsy
** hyper pseudo epithiotnis ~

** wound healing: ⬛
J⬛by cortisin vit A stop action of cortison
Vit C: important for healing

Keloid ⬛~ regional cortison injection cause of delayed wound healing ⬛


~
infection

** distributive shock: anaphylactic

Neurogenic

Septic


Jsystemic vascular resistance

⬛S.V.R shock‫باقى أنواع ال‬


'

In men:

Most common ⬛
~ parathyroid
~pancreas
Most common tumor⬛
Gastric cancer R0 ⬛
~ ‫مفيهاش حاجه‬

~microscopic +ve e’ in margin


R1⬛

⬛macroscopic
R2~

Main ttt

PLEAZE, ANY NOTE OR CORRECTION OR ADDITIN SEND: [email protected]


‫نسألكم الدعاء بظهر الغيب‬
P a g e | 197

Disease modified relating drugs immune suppressive therapy: Adaleno-map is the


main ttt

Not steroid alone

Steroid+immune suppressive

**causes of hiatus hernia: weakness of muscles of diaphragm (cause of diaphragm)

Which fix the cardio oesophageal junction

Has two types: sliding

Rolling

In sliding ⬛
~gastro oesophageal junction alone diaphragm and stomach in place
Ttt: Nissen's fundoplication +or- collies gastroplasty

⬛ gastro oesophageal junction below diaphragm and stomach pulled up


~
In rolling⬛
above diaphragm beside oesophagus

Cause: wide crura

Ttt: surgical correction

** diagnosis of Gastrinoma ⬛ ⬛
~ S.R.S scan Diagnosis of pheochromocytoma ~
M.I.P.G scan

Thyroglossal cyst, you want to search for prescience of thyroid tissue inside
⬛ thyroid scan: Tc99 or Radioactive iodine
thyroid gland ~

Outside gland: Tc99 or F. D j pet scan

Renal scaring in children or degree of damage in parenchymal tissue in


~ DMSA scan
kidney and functioning or not⬛
 MEN 1:
Most common tumour: para thyroid hyperplasia

** tumour lysis: hyperkalaemia & Hypocalcemia

 Blast injury:

PLEAZE, ANY NOTE OR CORRECTION OR ADDITIN SEND: [email protected]


‫نسألكم الدعاء بظهر الغيب‬
P a g e | 198

1ry: pneumothorax

2ry: sharp nail

3ry: fracture

4th: burn

 Necrotising pancreatitis ~
⬛ drainage ⬛
~culture, sensitivity
 ⬛
~ necrosectomy
If bacteria⬛
If no bacteria ⬛
~ follow up

 Pregnant women:
V.Q ratio

CT angio

CT chest

 Congested neck veins, shifted trachea, hyper ros. ⬛


~tension pneumothorax
 Congested neck veins, wide mediastinum, muffled heart sound ⬛ ~ cardiac
tamponed
 Wide mediastinum, expanded lung, ~ ⬛rupture aorta
 Shifted trachea, dullness of lung on side~
⬛massive pleural effusion
 Most late sign of compartmental ~ ⬛pulselessness
 First sign⬛
~pain

** Nipple discharge:
Spontaneous
Non spontaneous
Non spontaneous:<40 years ⬛
~no invest.
>40 years ⬛
~imaging
Spontaneous: < 30 years ⬛
~ sonar +or- mamo
>30 years ⬛
~ mamo + us
~MRI or ductogram
BIRAD 1-3⬛

PLEAZE, ANY NOTE OR CORRECTION OR ADDITIN SEND: [email protected]


‫نسألكم الدعاء بظهر الغيب‬
P a g e | 199

⬛tissue biopsy
BIRAD>3 ~

⬛MELD creat Na, bilirubin,


‫لو جاب عيان بيتكلم عن‬pt.~

bilirubin, encephalopathy Ascites, albumin ⬛


~CHILD PUPH ‫لو جاب عيان بيتكلم عن‬

** sliding hernia: junction above diaphragm


Causes: short oesophagus
Symptoms: GERD symptoms

** para oesophageal hernia:


Wide cardia
Junction below diaphragm
Stomach herniated
True hernia
No reflux
Pressure symptoms
** fibroadenoma or breast mass e’ pain
Ttt: oral contraceptives pills

** radiation proctitis: acute


Chronic
‫ سنه‬٠٣ ‫ شهور بعد االشعاع حتى‬٦ ‫تحدث من‬
** fibroadenoma: triple assessment
Imaging
Clinical assessments
⬛ core biopsy or FNAS
Pt<40 years ~
~ excisional biopsy
Pt> 40 years ⬛
Main ttt of ulcerative colitis or crohn's ⬛
~ is immune suppressive or immune
modulation

Steroid is in severe cases e’ exacerbation


** Actionmycis ⬛ ~ greenish white discharge
** Crohn’s disease ⬛~ transmural inflammation and thickened wall
** TB ileitis ⬛
~ lymph node

In septic shock or sever sepsis:


If you find:

PLEAZE, ANY NOTE OR CORRECTION OR ADDITIN SEND: [email protected]


‫نسألكم الدعاء بظهر الغيب‬
P a g e | 200

Main arterial pressure


 Older than 65-70~
⬛ severe sepsis
 Less than 65 ~
⬛ septic shock

 Blood loss:
1st degree:
 < 15% (750 ml)
 No effect on blood pressure
 Urine output: 30 ml
 Respiratory rate: 20
 Pulse: <100
 Slightly anxious
 Crystalloid
nd
2 degree:
 15-30% (750-1500 ml)
 No effect on blood pressure
 Urine output: 20-30 ml
 Respiratory rate: 20-30
 Pulse: 100-120
 Mild anxious
 crystalloid
rd
3 degree:
 30-40% (1500-2000 ml)
 Blood pressure: Start to decrease blood pressure
 Urine output: 5-15 ml
 Respiratory rate: 30-40
 Pulse: 120-140
 Agitated or confused
 blood
th
4 degree:
 >40% (>2000 ml)
 Blood pressure: Decreased
 Urine output: 0-5 ml
 Respiratory rate: >40
 Pulse: >140

PLEAZE, ANY NOTE OR CORRECTION OR ADDITIN SEND: [email protected]


‫نسألكم الدعاء بظهر الغيب‬
P a g e | 201

 Comatose
 Blood

 Hydatid cyst:
 Causative organism: Echinococcus granulosus
 A CT picture with water Lily sign (large cyst inside cyst in liver)
 Treatment:
- if simple cyst <5 cm …… give albendazole
-If simple cyst > 5cm…… give albendazole + percutaneous drainage
-If > 10 cm or septated…… percutaneous drainage
-If daughter cyst is present (even if < 5cm) …… surgery (deroofing)
-If calcified……. Conservative albendazole
 Amebic liver abscess:
 Causative organism: Entamoeba histolytica
 Anchovy sauce appearance
 Serology: serology for entamoeba
 Treatment: -if < 5 cm give metronidazole or tinidazole only
- If > 5 cm or failed medical treatment 5-7 days or abscess in left
lobe or impending rupture or 2ry bacterial infection ……. surgery ( needle
aspiration or catheter drainage “better”)

Benign hepatic tumors:


Most common benign hepatic tumor is hemangioma
 Associated with consumption of platelets
 By U/S: well demarcated homogenous hyperechoic
 Conservative treatment and instruct the patient to avoid exercise because it’s
more liable to rupture
Follicular nodular hyperplasia:
 Etiology: not clear
 By U/S: homogenous hypoechoic or isoechoic + stellate lesions
Liver adenoma:
 Related to oral contraceptive pills

PLEAZE, ANY NOTE OR CORRECTION OR ADDITIN SEND: [email protected]


‫نسألكم الدعاء بظهر الغيب‬
P a g e | 202

 By U/S: heterogenous
 Treated by stoppage of OCP

 Malignant hepatic tumors:


Most common is HCC then cholangiocarcinoma
Cholangiocarcinoma:
 Most common site: hilum of liver (at confluence of hepatic duct)
 Dilated intra hepatic biliary radicals, normal extra hepatic biliary radicals,
normal gallbladder
 Investigation: MRCP

 Basal cell carcinoma (rodent ulcer):


 2nd most common skin and subcutaneous malignancy after SCC
 Most common site: in face (near the check)
 Safety margin: in any site in body 5mm but in face 2mm
 Most questions: a picture of a pigmented lesion in face, origin pluripotent
cells, locally malignant tumor (as adamantinoma of the mandible and
ameloblastoma) and doesn’t cause metastases

 Melanoma:
Safety margin:
 if lesion < or = 1 mm …… 1cm
 If lesion 1-2 mm …. 1-2 cm
 If lesion >4 mm …. 2 cm

 Thyroid:
Investigations for thyroid enlargement: TSH and U/S
Simple multinodular goiter:
 Normal TSH
 Treatment: total thyroidectomy if cause compression manifestations
 If there is dominant nodule (nodule > 1cm): fine needle
Toxic goiter:
Grave’s:

PLEAZE, ANY NOTE OR CORRECTION OR ADDITIN SEND: [email protected]


‫نسألكم الدعاء بظهر الغيب‬
P a g e | 203

 Diffuse enlargement
 1st line of ttt: anti thyroid drugs, 2nd line of ttt: surgery or radioactive iodine (
if drug ttt failed, if there is no remission after 1.5 years, if there’s
complications from drug ttt, or fail to control thyroxine level)
Toxic multinodular goiter:
 ttt: exclusive surgery and role of anti-thyroid drugs just for preparation
Toxic adenoma:
 ttt: radioactive iodine unless there is any contraindication

 A case of toxic goiter, U/S shows single thyroid nodule …. isotope scan must
be done (to determine whether it’s a hot nodule causing toxicity or cold nodule
and the rest of the gland is causing toxicity)
 If cold nodule: with fine needle and U/S decide whether it’s malignant or
benign
 If follicular neoplasm…. hemithyroidectomy
 If papillary carcinoma (according to size):
-< 1 cm and single nodule …… hemithyroidectomy
-if >4 cm …… total thyroidectomy
-if 1-4 cm and patient is young and there is no extra lympho-vascular
invasion and no nodal metastases ……. Hemithyroidectomy
-if 1-4 cm and patient is old age and there is lymph-vascular invasion to the
capsule and there are nodal metastases ……. total thyroidectomy
-if in the questions the lesion is 1-4 cm with no specifics…… total
thyroidectomy

 Thalassemia:
 microcytic hypochromic with target cell in peripheral blood smear.
 Myeloproliferative disease:
 Bone marrow malignancy
 Diagnosed by bone marrow aspiration
 Associated with huge spleen
 CT with IV contrast
 Treatment: splenectomy
 GIT loss or hypovolemic shock or pancreatitis or hemorrhagic shock:
 1st give Ringer lactate except

PLEAZE, ANY NOTE OR CORRECTION OR ADDITIN SEND: [email protected]


‫نسألكم الدعاء بظهر الغيب‬
P a g e | 204

 CHPS: normal saline then K


 Severe diarrhea with hypernatremia with CNS manifestations: D5
 Severe diarrhea with hypernatremia without CNS manifestations: D5 half
NS
 Severe vomiting and patient are confused: normal saline then K
 Hereditary spherocytosis or hemolytic anemia or sickle cell trait:
 If present in crisis (show signs of hemolysis): don’t do splenectomy, instead
give him packed RBCs
 Hyponatremia:
 Corrected by normal saline (as well as isonatremia), while hypernatremia
causing dehydration is corrected by hypotonic saline (D5 or D5 NS 0.45%)
 Abscess > 5cm or complicated (surgical drainage)
 U/S guided drainage
 CT guided drainage
 Percutaneous drainage
 Catheter drainage
 If a nurse is present during patient physical examination…. privacy

 If expose the patient without his permission or don’t cover him…. privacy
 If didn’t close the curtains…. Privacy
 Antiplatelet & anticoagulant don’t cause GI bleeding except in presence of a
source like PU
 Fibroadenoma Triple ass. (Clinical, Imaging, Biopsy), if it’s fibroadenoma
follow up with US or mammography till 6 month, if size increases make
excision, if size regresses end follow up.. If it isn’t fibroadenoma from the start
after triple ass. make excision also. Size doesn’t make you change the triple ass.
Of fibroadenoma. If the size is small you can make cryoablation or liposuction
 fracture long bone with ( betechi or rash in upper chest, confusion, dysnea ) it’s
fat embolism occurs from 24 to 72 hours after fracture.
 Any obstructive jaundice with stone >>> ERCP .. Cholangitis>>> ERCP.. Any
other thing (cr. Head of pancrease) >>> MRCP
 prophylactic surgery: FAP before 20 yrs old >> prophylactic colectomy

PLEAZE, ANY NOTE OR CORRECTION OR ADDITIN SEND: [email protected]


‫نسألكم الدعاء بظهر الغيب‬
P a g e | 205

 Medullary carcinoma with MEN 2 >>> prophylactic


thyroidectomy
 Lobular cr. In situ with family history or HER2 positive >>> prophylactic
mastectomy
 PLAST inj.:1ry >> pneumothorax, 2ry >>> sharpnail, 3ry >> Fracture, 4ry >>
Burn& closed wound
 HTN & fever post thyroidectomy >>> Thyroid storm (ttt BB with antipyretic &
fluids)
 Hypotension, Hypoglycemia post thyroidectomy >> Adrenal insufficiency ( ttt
steroids )
 Medullary Cr. >>> parafollicular cells, high calcitonin, Congo red, genetic
counseling
 Ttt >>> Total thyroidectomy with neck dissection or central lymph node
dissection
 Tachycardia, tachypnea, HTN or Hypotension with fever (Gravis after stress)
>>> Thyroid storm
 Hypotension & Hypoglycemia >> Adrenal insufficiency
 Atypical ductal hyperplasia >> wide (or wire) local excision
 Most common cholangiocarcinoma (klatskin tumor) at the Hilum of the liver
>>> MRI (normal gall bladder, normal extrahepatic biliary ducts, dilated
intrahepatic ducts)
 Sickle >> Auto splenic infarction (Hyposplenism)
 Most common inherited clotting disorder >> VWD: Autosomal dominant
(prolonged PTT & bleeding time) (ttt: Cryoprecipitate).
 SIRS >> TLC <4000 or >12000, temp. <36 or >38, tachycardia, tachypnea
 2 or more with source of infection >>> Sepsis, End organ failure &
Hypotension >>> Severe Sepsis

PLEAZE, ANY NOTE OR CORRECTION OR ADDITIN SEND: [email protected]


‫نسألكم الدعاء بظهر الغيب‬
P a g e | 206

 Refractory Hypotension or needs inotropes >>> Septic shock


 Diverticulosis (UTI with E.coli) >>> Most common cause for Colo vesical
fistula or pneumaturia
 Stricture erythematous patch on gall bladder after colonoscopy or
sigmoidoscopy >>> Diverticulosis
 Accidental discovered Diverticulosis >> Nothing to do
 Collection with Diverticulosis >>> Percutaneous drainage
 Pt. during appendicectomy with white patches on terminal ileum >>> TB
(Antituberculous drugs for 6-9 months)
 Greenish patches during appendicectomy >>> Actinomycosis
 Pseudomembranous colitis >>> Chronic use of antibiotics then diarrhea
(clostridium difficile)
 INV. >>> Toxins in stool or dirty yellowish patches in colonoscopy
 Ttt >>> stop causative AB, oral vancomycin or IV Metronidazole
 Neurogenic shock >> Hypotension, bradycardia
 Warm hand septic shock >> Tachycardia & warm hand
 Non-small cell lung cr. (or squamous) >>> SVC obstruction
 Small cell lung Cr. >>> Cushing or myasthenia gravis (premalignant $)
 Hashimoto malignant transformation >> Most common Lymphoma, 2nd most
common papillary carcinoma
 Hashimoto >> high TSH, Normal or decreased T3 & T4 >>> ttt: Thyroxin
except in lymphocytic transformation or pressure symp.>> Surgery
 Antithyroid drugs in Graves is the 1st line ttt & should be stopped & replaced
with the 2nd line ttt (Surgery “total or near total thyroidectomy” or Radioactive
Iodine in 3 circumstances:
 No remission after 18 months 2- failure to control toxicity 3- complications
from the drugs

PLEAZE, ANY NOTE OR CORRECTION OR ADDITIN SEND: [email protected]


‫نسألكم الدعاء بظهر الغيب‬
P a g e | 207

 Surgery is preferred in Graves especially with exophthalmos


 Hashimoto has 3 stages: 1st stage hyperthyroidism, 2nd stage Euthyroid, 3rd
stage hypothyroid
 Thyroid case; 1st US & thyroid profile if Hyperthyroidism>> thyroid scan
 if Euthyroid >> FNA
 ttt of papillary cr. >>> <1cm lobectomy,
 >4cm total thyroidectomy,
 between 1-4 cm with young age, no lymph nodes, no distant Mets, no
radioactive iodine >>> Lobectomy.
 If FNA shows follicular cells, you should do hemithyroidectomy with frozen
section to differentiate between follicular adenoma & carcinoma
 Psammoma bodies >> Papillary Cr.>> Lymph node Mets
 Hurthle cells>> Follicular Cr., can't be determined with FNA
 Medullary Cr. >> High calcitonin, Congo red or Amyloid >> total
thyroidectomy with lymph node dissection
 Anaplastic Cr. >> direct invasion>> frozen neck
 Antibodies in: Graves: TSI, Hashimoto: Antimicrosomal & Anti thyroglobulin
& Antipyroxidase
 If lateral cervical lymph nodes proved to have Mets >> Modified radical neck
dissection
 Lymph node Mets: Metastatic LNs & Thyroid pathology
 Lateral apparent thyroid: LNs (with thyroid tissue in Biopsy) & no thyroid
pathology
 Ectopic thyroid: Normal Thyroid tissue in thyroglossal tract with or without
normal thyroid gland
 Thyroid Lymphoma: 2-5 % from all thyroid malignancy, Non-Hodgkin's B-cell
lymphoma

PLEAZE, ANY NOTE OR CORRECTION OR ADDITIN SEND: [email protected]


‫نسألكم الدعاء بظهر الغيب‬
P a g e | 208

 Best inv. To detect the thyroid tissue outside the thyroid gland is PET-CT
 Most common Bilateral breast Cr. >> Lobular carcinoma
 Worst prognosis in Breast Cr. >>> Infiltrative duct cr. (Carcinoma en cuirasses)
 Best prognosis >> Tubular Cr. Then Mucinous
 Multicentric Cr. >> Lobular carcinoma in situ
 Pathological breast discharge: unilateral, continuous, spontaneous (without
compression), Serous bloody, from single duct, Mass, nipple or skin retraction
 Physiological: Bilat., interrupted, Nonspontaneous, clear or may be
serosanguinous or milky, from multiple ducts
 If physiological discharge >> Follow-up (clinical if <40 yrs. & Mammogram if
>40yrs)
 If pathological discharge>>> US if <40 yrs., Mammogram if >40 yrs.,
 if normal imaging: ductogram or ductoscopy & MRI (Best modality)
 single duct ectasia >> Microductomy
 Multiple ducts ectasia>> Local (Wide) duct excision
 If abnormal imaging (Microcalcification & No mass), Suspicious of
Malignancy >>> Biopsy (Core biopsy is the best)
 18. Rectal prolapse: * Loose connective tissue connecting between pelvic floor
muscles & Rectum
 * Most common Symp.: Mass protruding from the Anus or Pain
 * Most common Comp.: Incontinence
 Most common type of repair: In old Age >> Perineal rectosigmoidectomy or
Rectal mucosectomy
 Ttt in Children >> Circulage
 Most common comp. for surgery >>> Infection





PLEAZE, ANY NOTE OR CORRECTION OR ADDITIN SEND: [email protected]
‫نسألكم الدعاء بظهر الغيب‬
‫‪P a g e | 209‬‬
‫‪‬‬
‫‪‬‬

‫‪Arabic Ethics‬‬
‫عشان‬ ‫بالعربي‬ ‫ركز هنا في معلومات‪ -1‬مهمة ومعتمدة‪ ،‬كتبتلك كله‬
‫●‬
‫‪:‬‬ ‫ما ياخدش منك ساعة مذاكرة‪ ،‬ادعيلى ‪v‬‬
‫في معلومات حسيت ان ملهاش الزمة وكنت هشيلها بس قولت‪-2‬‬
‫●‬
‫‪:‬‬ ‫اسيبها ل تكون مهمة وال حاجة‪ ،‬اسيبها لحنكة القارئ بقي ●‬
‫‪v‬‬
‫في بعض الشباتر السيناريوهات ملهاش توضيح‪ ،‬محبيتش افتى‪-3‬‬
‫وسيبتها زى ما هي‪ ،‬لو لقيت ليها توضيح مؤكد‪ ،‬ابعت على الميل‬
‫‪[email protected]‬‬
‫وهنضيفها في اإلصدار الجاى‬
‫نسألك الدعاء بظهر الغيب‪-4‬‬
‫●‬
‫متنساش الدعاء‪v -5‬‬
‫‪:‬‬

‫‪PLEAZE, ANY NOTE OR CORRECTION OR ADDITIN SEND:‬‬ ‫‪[email protected]‬‬


‫نسألكم الدعاء بظهر الغيب‬
‫‪P a g e | 210‬‬

‫‪Arabic Ethics‬‬
‫الشابتر األول‬
‫‪1‬الحالة‬

‫أسبوعا‪ 13‬عا ًما تبلغ من العمر ‪28‬واجهت إحدى المقيمة في مناوبتها الخاصة بأمراض النساء والتوليد حالة امرأة حامل تبلغ من العمر‬
‫وقدم أطباء األورام ‪ .‬تم تشخيص المرأة بأنها مصابة بسرطان المبيض في المرحلة الثانية‪.‬من الحمل‪ ،‬وهي بالفعل أم لثالثة أطفال أصحاء‬
‫لم تكن المقيمة متأكدة مما إذا كان من القانوني‪ ،‬من منظور إسالمي‪ ،‬إنهاء‪.‬توصية لفريق التوليد إلنهاء الحمل لبدء العالج الكيميائي‬
‫لم تجد أي توجيه واضح من منظور إسالمي في الكتب المدرسية الطبية التي وجدتها في المكتبة‪ ،‬والتي كتبت جميعها ونشرت من‪.‬الحمل‬
‫سألت زمالئها في قسم الشؤون الدينية بالمستشفى عما إذا كانوا قد كتبوا سياسة أو بيانات تستند إلى الفتوى حول هذه ‪.‬منظور غربي‬
‫لم تجد إجابة واضحة‪ ،‬لذلك بدأت في البحث على اإلنترنت ووجدت أخي ًرا بعض الفتاوى المكتوبة باللغة العربية التي تسمح ‪.‬المسألة‬
‫ومع ذلك‪ ،‬لم تكن المقيمة راضية تما ًما‪ ،‬وكانت محبطة ج ًدا من الوقت والجهد ‪.‬بأفعال مماثلة في أنواع مماثلة من المرضى‬
‫اللذين‬
‫حا لها‪ ،‬حيث كانت الفتوى‬ ‫وعالوة على ذلك‪ ،‬فإن ما وجدته لم يكن واض‪.‬اضطرت إلى بذلهما للعثور على إجابة للحالة التي واجهتها‬
‫‪.‬مليئة بالمصطلحات الفقهية التي لم تكن على دراية بها‬

‫بالنسبة ألي جانب سريري متعلق برعاية المرضى‪ ،‬يجب على الفريق الطبي‪ ،‬بما في ذلك المقيمين‪ ،‬التأكد من أنهم يقدمون للمريض )ج‬
‫وفي كثير من األحيان قد يسأل المريض نفسه الطبيب عن الفتوى ‪.‬الرع اية التي تتوافق مع القيم األخالقية والمعتقدات الدينية للمريض‬
‫هذا مهم بشكل‪ .‬قد تكون هذه األسئلة بسيطة‪ ،‬مثل كيفية أداء التيمم‪ ،‬أو ما هي األدوية التي يمكن أن تعيق صيام رمضان‪.‬المتعلقة بحالتهم‬
‫خاص عندما يتعلق األمر بقضية حساسة للغاية مثل إنهاء الحمل‪ ،‬والتي يتم شرح قواعدها األخالقية والدينية أيضا في الوحدة الخاصة‬
‫على الرغم من أن دور المستشفى هو تقديم إرشادات لألطباء لتوحيد الممارسة‪ ،‬إال أن هناك مصادر أخرى يمكن‪.‬بقرارات نهاية الحياة‬
‫وتشمل هذه لجنة األخالقيات في المستشفى إن وجدت‪ ،‬وقسم الشؤون الدينية‪. ،‬لألطباء الرجوع إليها لحل القضايا األخالقية والدينية‬
‫وتجدر اإلشارة إلى أن المملكة العربية السعودية قد تبنت سياسة تجعل أعضاء الهيئة ‪.‬والفتاوى المجمعة في كتب تجيب على أسئلة مماثلة‬
‫تم ذكر بعض‪.‬العليا للعلماء وحدهم مؤهلين إلعطاء الفتاوى‪ ،‬لتجنب تعدد المصادر التي تؤدي إلى مزيد من اللبس أكثر من التوضيح‬
‫)انظر أدناه(الموارد في القراءات المقترحة‬
‫الشابتر الثاني‬
‫‪1‬الحالة‬

‫طلب منها استشاريها أن ترى ‪.‬الدكتورة هدى مقيمة سعودية مسلمة تقوم بالتناوب في واحدة من أكبر مستشفيات النساء في تورنتو‪ ،‬كندا‬
‫سألت المريضة الدكتورة هدى عن ‪ .‬عا ًما‪ ،‬وغير متزوجة‪ ،‬وتعيش مع صديقها ‪16‬مريضة في العيادة الخارجية تبلغ من العمر‬
‫ًا للترتيب ‪.‬أفضل وسيلة لمنع الحمل يمكن أن تستخدمها لتجنب الحمل‬ ‫ًا واألكثر أما ن‬
‫كما طلبت منها النصيحة بشأن الطريقة األكثر أما ن‬
‫شعرت الدكتورة هدى بعدم االرتياح واالرتباك الشديد حيال كيفية التعامل مع هذه المريضة‪ ،‬حيث ‪.‬إللجهاض في حالة فشل وسائل منع الحمل‬
‫عالوة على ذلك‪ ،‬فإن نوع اإلجهاض الذي تطلبه موكلتها محظور في ‪.‬أن معتقداتها الدينية ال تسمح بعالقات جنسية قبل الزواج أو خارجه‬
‫أخبرت المريض أنها ال تستطيع أن تزودها بالنصيحة التي تريدها وطلبت منها التفضل بتحديد موعد آخر حيث يمكنها مقابلة‪.‬اإلسالم‬
‫‪.‬االستشاري‬

‫هذا ال يعني أنها‪ .‬تقع على عاتق الدكتورة هدى واجب العناية بمريضها وواجب شرح وجهة نظرها لزمالئها والمستشارين‪ .‬قم بواجبك في الرعاية )ج‬
‫يجب أن تقدم تدخ ال طب ياا ضد معتقداتها‬

‫صا‪ ،‬إذا شعر ‪/‬بشكل عام‪ ،‬يحق للطبيب أحياناا إحالة مرضيه ‪ .‬كن صادقا ‪.‬الرجوع‬ ‫شعرت‪ /‬مرضيته إلى مزود آخر يمكنه تقديم خدمة أفضل وأكثر تخص ا‬
‫باإلضافة إلى ذلك‪ ،‬يجب أن ‪ .‬ال ينطبق هذا على حاالت الطوارئ‪ ،‬أو في حالة عدم توفر مقدم خدمة يمكن للمريض الوصول إليه‪.‬بعدم الكفاءة في إدارة الحالة‬
‫يكون صادقاا مع مريضه في حالة رغبة المريض في معرفة سبب عدم قدرته على تقديم مثل هذه الخدمة مباشرة‪ ،‬أي لماذا يحتاج إلى إحالة‬
‫المريض إلى مزود آخر‬

‫باستخدام الملخص الموجز المذكور في هذه الوحدة‪ ،‬باإلضافة إلى الموارد األخرى‪ ،‬حاول أن تشرح لمرضاك وزمالئك سبب ‪.‬شرح اإلسالم‪ ،‬ولكن ال داعي له‬
‫أخبرهم أن اإلسالم لديه مجموعة من األهداف المحددة التي تشمل الحفاظ على الحياة‪ ،‬وأن هناك ‪.‬عدم قدرتك كمسلم‪ ،‬على سبيل المثال‪ ،‬على المساعدة في اإلجهاض‬
‫بشكل عام‪ ،‬كن دقيقا ا وال تشرح بالتفصيل ما ينص عليه اإلسالم في‪ .‬استخدم المبادئ ذات الصلة ‪.‬بعض المبادئ التوجيهية التي تتبعها‬

‫‪PLEAZE, ANY NOTE OR CORRECTION OR ADDITIN SEND:‬‬ ‫‪[email protected]‬‬


‫نسألكم الدعاء بظهر الغيب‬
‫‪P a g e | 211‬‬
‫إذا‪ .‬من الجيد الدعوة إلى اإلسالم‪ ،‬لكن اإلعداد العرضي للطبيب المريض ليس أفضل مكان للقيام بذلك‪.‬قضايا أخرى‪ ،‬إال إذا طلب منك ذلك بشكل مباشر‬
‫‪ ،‬فيمكنك دعوتهم إلى بعض األنشطة في أقرب مسجد أو مركز إسالمي)بما في ذلك زمالئك(كنت مهت اما باالتص ال بالناس‬

‫في معظم األوقات‪ ،‬ستجدها ‪ .‬من األهمية بمكان أن تكون على دراية بكيفية عمل مؤسستك الصحية وما هي السياسات المعمول بها‪.‬تعرف على النظام واتبعه‬
‫إذا لم يكن األمر كذلك‪ ،‬فاستشر زمالئك المسلمين اآلخرين‪ ،‬أو اسأل مستشارك عن كيفية إدارة المواقف التي ‪.‬واضحة ومتسقة مع أدورك كطبيب مس لم‬
‫"أدوار وواجبات األطباء" يرجى الرجوع إلى وحدة ‪.‬تكون فيها خدمة الرعاية الصحية التي يطلبها المريض غير مسموح بها في اإلسالم‬
‫‪".‬االستنكاف الضميرية "للحصول على رؤية أوضح لمفهوم‬

‫الشابتر الثالث‬
‫‪1‬الحالة‬

‫في جولته‪ ،‬كان الجراح المعروف السيد بوتشر حري صا دائ ًما على حضور جميع المقيمين والمتدربين في وحدته‪ ،‬باإلضافة إلى‬
‫أ ع ض اء‬
‫يشار إليها ‪ .‬الذين كان دائ ًما لهم)اختصاصي التغذية‪ ،‬وأخصائي العالج الطبيعي‪ ،‬والممرضات‪ ،‬وغيرهم(فريق الرعاية الصحية اآلخرين‬
‫عا ًما كان قد أزيل البروستاتا قبل ‪ 65‬خالل الجولة‪ ،‬طلب من إحدى المتدربات في وحدته فحص مريض يبلغ من العمر "‪.‬المسعفين"باسم‬
‫أوقفها الجراح عندما حاولت سحب الستائر‪ ،‬حيث كان هناك عدد قليل ‪ .‬شعر كل من المتدرب والمريض بالحرج‪ .‬سألها بصوت عال‪.‬يومين‬
‫إنه مريض في ‪ .‬ال شيء تخجل منه‪ :‬قال‪.‬من المرضى اآلخرين بجانب هذا المريض الذين سيرون فحصه إذا لم يتم سحب الستائر‬
‫طلب المتدرب ‪ .‬التحدث مع المريض" ؟ ‪ X‬أليس هذا صحي ًحا يا سيد"‪ ،‬إذن‪"،‬مستشفى تعليمي لذا يتوقع أن تقوموا جمي ًعا بفحصه‬
‫إذن‬
‫كالعادة في جولته‪ ،‬تم االستهزاء‪ .‬اآلخرين بعض األسئلة"األطباء" ثم سألها الجراح وعلى ‪.‬المريض ثم قام بفحصه وموقع العملية‬
‫على المسعفين أن يأخذوا التعليمات التي يقدمها " ‪ :‬كان يقول دائ ًما‪ .‬أب ًدا فرصة لإلجابة"المسعفون"منح‬ ‫باإلجابات الخاطئة‪ ،‬ولم‬
‫لهم‬ ‫‪".‬األطباء‬

‫‪ .‬لقد حددنا القيم المهنية التي تنطوي عليها مهنتنا كطبيب)ج‬


‫فلنعد إلى السيد‪.‬من األهمية بمكان أن نفهم أن هذه القيم تنطوي على واجبات كل منا تجاه أنفسنا وزمالئنا ومهنتنا ومجتمعاتنا بشكل عام‬
‫من غير المقبول‪ :‬وإليك بعضا منها ‪ .‬يمكننا بسهولة إظهار العديد من السلوكيات غير المهنية التي شارك فيها‪.‬بوتشر‪ ،‬الجراح المعروف‬
‫إنه ألمر أكثر‪.‬اإلشارة إلى مقدمي الرعاية الصحية من غير األطباء بوصفهم مسعفين‪ ،‬على األقل بالطريقة التي يستخدم بها المصطلح‬
‫احترا ًما لزمالئك أن تسميهم من خالل مسمياتهم الوظيفية‬
‫ليتم وضعه في موقف حيث‪ -‬إن لم يكن هناك حسن معاملة ‪-‬يشعر المريض المسن‪ ،‬وخاصة الرجل في مجتمعنا‪ ،‬باالكتئاب الشديد ‪ .‬‬
‫تقوم أنثى في سن ابنته بفحص مناطقه الخاصة‬

‫عدم قبول طلب اإلذن من المريض لفحصه‪.‬من خالل عدم سحب الستائر‪ ،‬يحرم الطبيب مريضه من أدنى مستوى من الخصوصية ‪ .‬‬
‫هذا ينطبق حتى في المستشفيات الجامعية التعليمية‪.‬على اإلطالق‪ ،‬ما لم تكن حالة طارئة أو كان المريض غير قادر على إعطاء اإلذن‬

‫إنه أمر مسيء وغير مناسب تما ًما كنموذج يحتذى به‪.‬مرة أخرى‪ ،‬إذالل زمالئك‪ ،‬وخاصة الصغار منهم‪ ،‬هو أمر غير أخالقي تما ًما‪ .‬‬
‫واألسوأ من ذلك‪ ،‬أنه يوسع الفجوة بين أعضاء فريق الرعاية الصحية‪ ،‬والتي بدونها ال يستطيع حتى الطبيب‪.‬أمام الممارسين الصغار‬
‫األكثر مهارة تحقيق أهداف اإلدارة‬

‫يمكنك أن تتعلم الكثير‪ .‬يتمتع جميع أعضاء فريق الرعاية الصحية بقيمة متساوية ويجب منحهم الفرصة لمشاركة المعرفة التي لديهم‪ .‬‬
‫‪.‬منهم‪ ،‬وكذلك الكثير من زمالئك من األطباء‬

‫الشابتر الرابع‬
‫‪1‬الحالة‬

‫عا ًما مرا ًرا وتكرا ًرا في وحدة العناية المركزة بعد إصابته بسرطان الرئة ‪90‬تم تنويم رجل أعمال مشهور للغاية يبلغ من العمر‬
‫في‬
‫المرحلة النهائية‬
‫حقيقة )وعن أفراد األسرة اآلخرين(كان برفقته أحد أبنائه األربعة عشر أخبرك أنهم يخبئون عنه ‪.‬في قبوله األخير‪ ،‬أنت كان الطبيب المسؤول‬

‫‪PLEAZE, ANY NOTE OR CORRECTION OR ADDITIN SEND:‬‬ ‫‪[email protected]‬‬


‫نسألكم الدعاء بظهر الغيب‬
‫‪P a g e | 212‬‬
‫طلبوا منك عدم ‪.‬أنه مصاب بالسرطان‪ ،‬وأنهم أخبروه بأنه مصاب بمرض مزمن التهاب الصدر الذي يتم عالجه بالراحة والمضادات الحيوية‬
‫هذا‪.‬إخباره‪ ،‬وإال فقد يموت من الصدمة‪ ،‬وقد يفعل منافسيه في السوق إساءة استخدام هذه المعلومات إللضرار بوضعه المالي‬

‫‪PLEAZE, ANY NOTE OR CORRECTION OR ADDITIN SEND:‬‬ ‫‪[email protected]‬‬


‫نسألكم الدعاء بظهر الغيب‬
‫‪P a g e | 213‬‬
‫في وقت الحق‪ ،‬ابن آخر‪.‬يمكن أن يؤدي بعد ذلك إلى خسارة معظم ثروته‪ ،‬والتي ينفق معظمها على الصدقات للمساعدة المحتاج‬
‫لحسن الحظ‪ ،‬اتصل به األب عندما كنت‪.‬للمريض‪ ،‬من زوجة مختلفة لزوجة اقترب منك االبن األكبر وسألك عن تشخيص حالته اآلب‬
‫ماذا كنت ستفعل؟‪.‬على وشك التحدث معه‬

‫‪ .‬من حيث المبدأ‪ ،‬يحق للمريض معرفة حالته أ لسباب عديدة)ج‬


‫‪ ،.‬فعندئ ًذ لديه الحق في اطالعه على حالته‪ ،‬بما في ذلك تشخيصها وعالجها)راجع األقسام األولى في الكفاءة(أوالً‪ ،‬إذا كان مؤهالً‬
‫هذا‬
‫‪ .‬هذا هو احترام استقالليته‪.‬يمكن أن يحسن امتثاله‪ ،‬والذي قد يتحسن أو على األقل يحافظ على حالته‬
‫ًا‪ ،‬مع وجود ملف أسرة كبيرة وكثير من الناس يتوقعون أن يرثوا منه‪ ،‬ينبغي أن يكون أعطيت الفرصة لترتيب أولوياته بنفسه‬
‫هو‪.‬ثان ي‬
‫‪.‬األكثر أعلم من أوالده يحتاج أكثر ومن هو أفضل حاالً‬

‫أخي ًرا‪ ،‬كرجل أعمال‪ ،‬من المعقول أن نتوقع أنه يمكن أن يكون لديه الكثير القضايا المالية المعلقة‪ ،‬بما في ذلك الديون المستحقة عليه أو‬
‫ًا( ال تخبره عن حالته يحرمه من فرصة إعطاء اآلخرين ‪.‬المستحقة عليه‬ ‫قد يكون على استعداد لتخصيص بعض أمواله‪ .‬حقوق‪) .‬مال ي‬
‫‪ .‬إن الحجة القائلة بأن سوق األسهم ستتأثر ليست حجة صحيحة‪ .‬ال إخباره بحالته الحقيقية قد يضره هو واآلخرين أيضا‪.‬لألعمال الخيرية‬
‫‪ .‬فقط المريض هو من سيتم إبالغه ‪.‬في األساس أنتم ال تكشفون عن حالته للصحافة‬

‫أنت يجب أن ‪ .‬معرفة حالته)أو غيرهم من أفراد األسرة(الشيء الذي يجب مناقشته مع المريض في وقت مبكر هو أي منها يحق لألبناء‬
‫في هذه ‪ .‬حالتها لتدهور أي من أفراد أسرته يجب استشارتهم وعلى علم بالحالة الصحية‪/‬تناقش بصراحة مع أي مريض تتوقع حالته‬
‫الحالة بالذات‪ ،‬إذا كان المريض مختص‪ ،‬ثم يسأل مباشرة من يريد أن يعرف عن حالته؟ إذا لم يكن كذلك‪ ،‬فتحقق من الموافقة التي وقعها‬
‫إذا لم يتم العثور على اسم‪ ،‬فسيكون كذلك من الحكمة إحالة القضية إلى لجنة األخالقيات في المستشفى التي ستفعل ذلك ‪.‬يذكر صانع قرار بالوكالة‬
‫خالصة القول هي أن الطبيب ال ينبغي أن يضع افتراضات‪.‬من المحتمل أن تدعو لعقد اجتماع عائلي لمناقشة حالة المريض‬
‫‪.‬حول ما هو موجود مصلحة المريض أو األسرة‬

‫‪".‬ال تخبر"عندما تقول األسرة‪،‬‬


‫المسؤولية المهنية واألسرة التفضيالت وحقوق المريض في كثير من األحيان‪ ،‬سيطلب أفراد األسرة من الطبيب عدم إخبار المريض‬
‫بذلك التشخيص أو معلومات مهمة أخرى‬
‫التزام قانوني بالحصول على موافقة مستنيرة من المريض ‪ ،‬والحفاظ على‪:‬في هذه الحاالت ‪ ،‬قد يشعر األطباء بأنهم محاصرون بين ‪• .‬‬
‫تحالف متجانس مع األسرة من أجل ضمان عالقة عالجية ناجحة‬
‫ال"بدالً من مواجهة طلبهم بـ ‪• .‬‬

‫ي أن أقول؟‪ ،:‬اسألهم "بد ا ل من إخبار المريض‬


‫لماذا تشعر أنه ا ل ينبغي عل ً‬
‫ماذا تخشى أن أقول؟‪" .‬ثانيا"‬

‫ما هي تجاربك السابقة مع األخبار السيئة؟‪" .‬ثالثا"‬

‫هل هناك سياق شخصي أو ثقافي أو ديني لك القلق الذي يجب أن أحاول فهمه؟‪" .‬رابعا"‬

‫ًا لنسأل كم هو أو تريد أن تعرف عن صحته وما هي األسئلة قد يكون هناك‪ " :‬يقترح" •‬
‫"‪.‬لماذا ال نذهب إلى المريض م ع‬
‫الشابتر الخامس‬
‫‪1‬الحالة‬

‫طلبت طبيبة من زوجها ‪ .‬تأتي إلى قسم الطوارئ مصابة بنزيف حاد في المهبل‪ .10‬عا ًما وحامل في األسبوع ‪35‬نورا تبلغ من العمر‬
‫كيف يتم التعامل مع حقوق المريض في هذه‪ .‬المريضة غير قادرة على قبول طريقة عالجها دون استشارة‪.‬التوقيع على إنهاء الحمل‬
‫الحالة؟‬
‫‪ :‬تتضمن هذه القضية العديد من القضايا األخالقية)ج‬

‫‪PLEAZE, ANY NOTE OR CORRECTION OR ADDITIN SEND:‬‬ ‫‪[email protected]‬‬


‫نسألكم الدعاء بظهر الغيب‬
‫‪P a g e | 214‬‬
‫يريد زوجها التوقيع على استمارة الموافقة بدالً‪.‬تأتي مع نزيف حاد في المهبل ‪ .‬المرأة حامل لكنها ال تزال في الثلث األول من الحمل ‪1.‬‬
‫ما هي االستنتاجات التي يمكن استخالصها؟‪.‬من زوجته لكنها ترفض‬

‫المريضة سليم عقليا ً ومن حقها التوقيع على إعالن الموافقة على أي إجراء يتعلق ‪2.‬‬

‫إذا لم تكن على دراية بمزايا السالمة والمخاطر المرتبطة بموافقتها ‪ ،‬فمن واجب العاملين في مجال الرعاية الصحية تثقيفها ‪. 3. ،‬بجسدها‬

‫وفي‬
‫رفض العالج حق من حقوق المريضة ويجب السماح لها بذلك التوقيع على استمارة الموافقة‪.‬حالة الطوارئ اتخاذ القرار لحمايتها‬
‫ومع ذلك‪ ،‬في حالة طوارئ حقيقية‪ ،‬يجب االستماع إلى رأيها؛ إذا رفضت التوقيع على استمارة الموافقة المستنيرة‪.،‬المستنيرة بنفسها‬
‫‪.‬في هذه الحاالت‪ ،‬يمكن لطبيب الطوارئ التوقيع على شكل لحماية حياة المريض‪.‬زوجها أو قريب آخر قد يوقع‬

‫‪2‬الحالة‬

‫يتم ‪.‬تفضل عدم رؤية األطباء ‪.‬إنها مستقلة تما ًما ولم تصاب بمرض خطير ‪ .‬عا ًما وتعيش وحيدة في شقتها ‪80‬هوسا تبلغ من العمر‬
‫يقوم استشاري الطب الباطني بتشخيص تضيق األبهر ‪.‬إدخالها اآلن إلى المستشفى بعد سقوطها على السلم وإصابتها بكسر في عنق الفخذ‬
‫عندما قال إن ‪.‬يزور طبيب التخدير هوسا لمناقشة الجراحة والتخدير ‪ .‬والذي يؤكده تخطيط القلب ‪stenosis aortic critical‬الحرج‬
‫إنها تريد إصالح كسرها ألنها ببساطة ال تستطيع العيش مع ‪.‬الجراحة ترتبط بمخاطر جسيمة‪ ،‬قالت هوسا إنها ال تريد أن تعرف عنها‬
‫ضعف‬
‫‪.‬يشعر طبيب التخدير أنه من واجبه الكشف عن مخاطر التخدير‪.‬الحركة‬

‫تقول إن هدفها هو أن تكون‪ .‬طلبت السيدة هوسا من طبيب التخدير عدم الكشف عن المزيد من المخاطر المرتبطة بجراحة الورك)ج‬
‫‪.‬قادرة على المشي وأن معاناة المزيد من األلم وعدم الحركة أمر غير مقبول‬
‫‪.‬أيقن طبيب التخدير أن أي نقاش إضافي حول المخاطر لن يغير رأيها‪ ،‬بل قد يزعجها‬
‫كما يسألها عما ‪.‬يحترم طبيب التخدير طلب السيدة هوسا ولكنه يخبرها أنه يمكنها تغيير رأيها فيما يتعلق بمناقشة المخاطر في أي وقت‬
‫ترغب السيدة هوسا في مشاركة بناتها في ‪.‬إذا كان هناك أي أفراد من العائلة ترغب السيدة هوسا في إشراكهم في عملية صنع القرار‬
‫تم توثيق المناقشة بأكملها‪ ،‬بما في ذلك أسباب السيدة هوسا ‪.‬القرار‪ ،‬وبالتالي يتم الكشف عن الجراحة المجدولة ومخاطرها المحتملة‬
‫تخضع السيدة هوسا لعملية إصالح غير معقدة لكسر الورك وتعود إلى المنزل‪ .‬المزيد من الكشف عن مخاطر الجراحة)رفض(للتنازل‬
‫تتضمن هذه الحالة العديد من القضايا األخالقية‪ ،‬مثل سالمة المريض‪ ،‬والموافقة المستنيرة‪ ،‬وحقوق المرضى‪.،‬لتعيش بشكل مستقل‬
‫‪.‬وعملية صنع القرار‬

‫الشابتر التاسع‬
‫‪1‬الحالة‬

‫ًا بالسرطان المنتشر على نطاق واسع في المرحلة ‪90‬أخبر األطباء رجال يبلغ من العمر‬ ‫‪ 4‬عا ًما في وحدة العناية المركزة مصا ب‬
‫طلب أن ‪.‬وفشل متعدد األعضاء أنه ال يوجد شيء يمكنهم فعله لعكس مسار المرض‪ ،‬وأنهم ال يستطيعون سوى تقديم عالج لألعراض‬
‫يخرج‬
‫تم قبوله أخي ًرا في دار العجزة‪.‬اعترض أطفاله قائلين إنه بحاجة إلى تمريض معقد ال يمكنهم توفيره في المنزل ‪.‬ليموت في المنزل‬
‫‪ .‬التي قدمت الرعاية التلطيفية بتكلفة كبيرة‪hospice private‬الخاصة‬

‫‪ .‬أهمية التخفيف‪:‬تلميح‬

‫‪2‬الحالة‬

‫سنوات‪ ،‬وقد عينت زوجها باعتباره صانع‪ 5‬عا ًما مصابة بالتصلب المتعدد تتمتع بصحة جيدة لمدة ‪30‬كانت مريضة تبلغ من العمر‬
‫الزوج‪ ،‬الذي تزوج في ذلك‪ .‬عندما فقدت وعيها‪ ،‬احتاج األطباء إلى اتخاذ قرار بوضعها على أجهزة اإلنعاش القلبي الرئوي ‪.‬القرار‬
‫تدخل والدها وقرر دعم حياتها ألن ذلك‪.‬الوقت وعاش في منزل منفصل‪ ،‬قرر عدم استخدام أجهزة دعم الحياة ألنها ستطيل من معاناتها‬
‫‪.‬سيكون في مصلحتها‬

‫‪ .‬الحاجة إلى سياسة الختيار صانع قرار وكيل العائلة‪:‬تلميح‬


‫‪PLEAZE, ANY NOTE OR CORRECTION OR ADDITIN SEND:‬‬ ‫‪[email protected]‬‬
‫نسألكم الدعاء بظهر الغيب‬
‫‪P a g e | 215‬‬
‫‪3‬الحالة‬

‫أراد جراح الصدر إجراء عملية إزالة االنتفاخ لتقليل كتلة سرطان الرئة لتمكين المريض من التنفس بسهولة؛ أخبر المريض بارتفاع خطر‬
‫لم يكن الطبيب‪ .‬عا ًما يشعر بالنعاس بسبب األدوية ويشتبه في إصابته بالخرف ‪85‬كان المريض البالغ من العمر ‪.‬الوفاة بسبب النزيف‬
‫‪.‬متأك ًدا مما إذا كان المريض قاد ًرا على فهم التفسيرات المقدمة واتخاذ قرارات جادة بشأن العملية‪ ،‬ولم يكن للمريض أقرباء في الجوار‬

‫‪ .‬ضعف القدرة على اتخاذ القرار الذي يتطلب اختبار الكفاءة‪:‬تلميح‬

‫الظروف المتوقعة مقابل الظروف الواقعية‪:4‬الحالة‬

‫بعد سنوات‪ . ،‬كتب أستاذ جامعي نوبات سابقة من سكتة دماغية عابرة توجي ًها وشهد أنه إذا فقد وعيه فلن يرغب في إنعاشه‪:‬السيناريو‬
‫قرر األطباء الذين قرأوا توجيهاته عدم إنقاذه‪ ،‬لكن‪.‬تم نقله إلى المستشفى فاقدا للوعي من إصابات في الرأس أصيب بها في حادث سيارة‬
‫‪.‬زوجته أصرت على إنعاشه‬

‫‪ .‬مشكلة التوجيهات المسبقة التي يتم تطبيقها على المواقف غير المتوقعة‪:‬تلميح‬

‫مقابل األسرة‪ DNR‬أطباء ‪:5‬الحالة‬

‫‪ .‬عا ًما مصابة بسرطان المبيض المنتشر غير القابل للعالج‪ 80‬لجدة تبلغ من العمر )‪ (DNR‬كتب األطباء طلب عدم اإلنعاش ‪:‬السيناريو‬
‫قبل حل الخالف‪ ،‬انهارت المريضة بعد نوبة من االلتهاب الرئوي الحاد‪.‬اعترضت أسرتها بشدة عندما أ ًبلغت بهذا القرار وطالبت بإلغائه‬
‫‪ .‬باالتصال بفريق اإلنعاش ‪ DNR‬لم تقم الممرضات الالئي اتبعن أمر ‪.‬ال عالقة لها بحالتها األصلية‬

‫‪.‬هو قرار طبيب وليس عائلة ‪DNR :‬تلميح‬

‫القتل الرحيم‪:6‬الحالة‬

‫عا ًما‪ ،‬أصيب بسرطان متقدم مع ألم شديد ال يستجيب للمورفين‪ ،‬طلب من الطبيب قتله وإنقاذه ‪ 70‬رجل يبلغ من العمر ‪:‬السيناريو‬
‫كما رفض الطبيب إعطاء المريض أي نصيحة ‪ .‬رفض الطبيب‪ ،‬مدعيا أنه ال يستطيع أن يرتكب جريمة قتل غير شرعية‪.‬من المعانا ة‬
‫القتل الرحيم السلبي عن طريق ‪:‬تلميح‪ .‬والتغذية حتى الموت البطيء ‪ IVF‬وإلحاحا من المريض وافق الطبيب على وقف ‪.‬بشأن االنتحار‬
‫منع‬
‫‪.‬الطعام والماء غير قانوني‬

‫حجب دعم الحياة غير المجدي‪:7‬الحالة‬

‫أظهر تخطيط القلب ‪ .‬ذهبت إلى غرفة الطوارئ مع ضغط الدم أو النبض غير القابل للتسجيل‪ SHOCK‬ضحية حادث سيارة في حالة ‪:‬السيناريو‬
‫ألنه‪ -‬رغم إصرار أفراد األسرة ‪ -‬لم يعلن الطبيب عن وفاته‪ ،‬لكنه رفض تقديم أجهزة اإلنعاش ‪.‬موجات بطيئة ذات سعة منخفضة‬
‫‪ .‬هددت األسرة بمقاضاة الطبيب‪ .‬تم إعالن وفاة المريض بعد ساعة واحدة‪.‬رأى أنه ال أمل‬

‫‪ .‬حكم الطبيب مقابل المشاعر العائلية‪:‬تلميح‬

‫دعم الحياة مع موت جذع الدماغ‪:8‬الحالة‬

‫ًا بفشل في عدة أعضاء وعالمات سريرية لموت جذع الدماغ كان قيد دعم الحياة‪ 90‬رجل يبلغ من العمر ‪:‬السيناريو‬ ‫‪ .‬عا ًما مصا ب‬
‫كان يحتل آخر سرير متاح في وحدة العناية المركزة ألن األطباء كانوا يخشون الكشف عن الوفاة لألسرة‪ ،‬التي كان لديها الكثير من األعضاء‬
‫ناجا من موقع تحطم طائرة‪ ،‬قرر األطباء سحب أجهزة دعم الحياة من الرجل العجوز لتحرير‪ 50‬ومع ذلك‪ ،‬عندما تم إحضار ‪.‬الغاضبين‬
‫‪.‬سرير واحد على األقل من وحدة العناية المركزة‬

‫‪ .‬دعم الحياة غير الضروري بإصرار األسرة‪:‬تلميح‬

‫التهوية ألغراض حصاد األعضاء‪:9‬الحالة‬

‫أبقى طبيب وحدة العناية المركزة مريضا ميتًا في جذع الدماغ على أجهزة دعم الحياة االصطناعية للحفاظ على حيوية أعضائه‪:‬السيناريو‬
‫ًا لصالح ابن عمه الذي ولد‬‫حتى وصول فريق الزرع؛ كانوا يعتزمون حصاد القلب والرئتين التي تبرع بها المريض بينما ال يزال واع ي‬
‫‪.‬بتشوهات خلقية شديدة ويموت بدون الزرع‬

‫‪PLEAZE, ANY NOTE OR CORRECTION OR ADDITIN SEND:‬‬ ‫‪[email protected]‬‬


‫نسألكم الدعاء بظهر الغيب‬
‫‪P a g e | 216‬‬
‫‪ .‬تأخير تحديد الوفاة لمصالح أخرى ‪:‬تلميح‬

‫الشرطة‪ VS‬عائلة بعد الوفاة ‪:10‬الحالة‬

‫أرادت سلطات الشرطة إجراء تشريح للوفاة لتحديد سبب الوفاة‪ .‬توفي شرطي فجأة أثناء شجار مع مجرمين تم اعتقالهم الحقًا‪:‬السيناريو‬
‫‪ .‬اعترض بعض أفراد األسرة على تشريح الجثة على أساس أنه مخالف للشريعة‪.‬من أجل توجيه االتهام إلى المجرمين ومعاقبتهم بالقتل‬
‫‪.‬أيد أعضاء آخرون تشريح الجثة بسبب أغراض تعويض التأمين‬

‫‪ .‬التوازن بين الفوائد والمضار من فحص ما بعد الوفاة‪:‬تلميح‬

‫القدرة على اتخاذ القرار‬

‫قد تؤثر آثار المرض أو العالج على ‪.‬يجب على المريض النهائي المختص اتخاذ جميع القرارات المتعلقة برعايته وفا ًء لمبدأ االستقاللية‬
‫سيتعين على العاملين في مجال الرعاية الصحية اتخاذ قرار بشأن ما إذا كان المريض‪.‬قدرة المريض على اتخاذ القرار بدرجات متفاوتة‬
‫في معظم الحاالت‪ ،‬يكون الموقف واض ‪ .‬المعرفية لفهم المعلومات التي يتم تقديمها إليه والتصرف بنا ًء عليها‪/‬لديه القدرة الفكرية‬
‫على سبيل المثال‪ ،‬قد يكون المريض ‪ً .‬حا‪ ،‬ولكن في حاالت أخرى‪ ،‬قد يتعين إجراء اختبارات خاصة للكفاءة من قبل طبيب نفساني‬
‫قد يعاني المريض ذو القدرة الفكرية من ضعف في الذاكرة دون ‪.‬النهائي مؤهالً في بعض األمور‪ ،‬ولكن ليس في أمور أخرى‬
‫إذا كان المريض مؤهالً‪ ،‬فسوف يوقع على بيان بما يجب القيام به بعد فقدان ‪.‬القدرة على االحتفاظ بالكثير من المعلومات التخاذ القرار‬
‫يفضل‪.‬الوعي‬
‫شا مطو ًال‪ ،‬يتفحصون خالله عقل المريض الكتشاف ما‬ ‫أن يجلس العاملون في مجال الرعاية الصحية مع المريض ويخوضون نقا‬
‫يعتقده‬
‫يمكن للعاملين في مجال الرعاية الصحية بعد ذلك كتابة التعليمات الضرورية‪ ،‬والتي ستكون لها ميزة كونها مصاغة في‪.‬أو يرغب فيه‬
‫‪.‬المصطلحات الطبية‬

‫بالوكالة‪/‬صناع القرار البديل‬


‫سيتعين على صانع القرار‪.‬عندما نظر إلى المريض النهائي بأنه يفتقر إلى القدرة على اتخاذ القرار‪ ،‬فإنه يفقد الحق في االستقالل الذاتي‬
‫إذا لم يتم تعيين صانع قرار بديل ‪ .‬قد يكون صانع القرار هذا قد تم تعيينه مسبقًا من قبل المريض ‪.‬البديل اتخاذ القرارات الالزمة‬
‫تنشأ مشكلة عملية عندما يكون هناك العديد من أفراد األسرة الذين لديهم وجهات ‪.‬مسبقًا يمكن أن يكون أحد أفراد األسرة هو صانع القرار‬
‫بشكل عام ال ينبغي أن يتورط األطباء في الخالفات األسرية؛ يجب إخبار األسرة بالمناقشة فيما بينها والعودة بقرار ‪.‬نظر مختلفة‬
‫واحد‬
‫‪ .‬إذا فشل إجماع األسرة يمكن استخدام بعض ترتيب األسبقية بين أفراد األسرة بنا ًء على نقاط القوة الخاصة بهم بوصفهم ورثة‪.‬باإلجماع‬
‫ما يعتقد أن المريض كان )أ(يقرر صانع القرار بالوكالة بطريقتين‪ ،‬بنا ًء على ‪.‬على سبيل المثال‪ ،‬قرار االبن له األسبقية على قرار األخ‬
‫بالنسبة لقرارات ‪.‬عندما ال يكون أفراد األسرة متاحين‪ ،‬فإننا نتقدم بطريقتين‪ .‬المصلحة الفضلى للمريض )ب(سيقرره إذا كان مؤهالً‪ ،‬و‬
‫في الحاالت غير الطارئة التي تنطوي على جراحة‪ ،‬يمكن طلب قرار من‪.‬الطوارئ‪ ،‬يقرر الطبيب ما يعتقد أنه في مصلحة المريض‬
‫‪ .‬إذا لم تكن المحكمة متاحة بسهولة‪ ،‬يمكن لمدير المستشفى اتخاذ قرارات بشأن المرضى كممثل للسلطة الحكومية‪.‬المحكمة‬

‫توجيهات مسبقة‬
‫تمكن ‪.‬التوجيهات المسبقة هي مستندات مكتوبة خالل الفترة التي يكون فيها المريض مؤهالً‪ ،‬وهي جزء من االستقاللية المرتقبة‬
‫ًا ما يستخدم المصطلح الشائع ‪.‬المريض من التحكم فيما يحدث له بعد فقدان الوعي‪ ،‬أو حتى بعد الوفاة‬ ‫لإلشارة "اإلرادة الحية" غال ب‬
‫إلى البيان‬
‫عفى‬ ‫‪ .‬يتأكد المريض من استقالليته المستقبلية‪ ،‬حيث تتم رعايته وفقًا لرغباته‪ .‬البيان المسبق له فوائد للمريض واألطباء واألسرة‪.‬المسبق‬
‫تتحرر األسرة من توتر البحث عن اإلجماع ‪.‬األطباء من عبء البحث عن صانع القرار واتخاذ القرار بأنفسهم في غياب صانع القرار‬
‫يمكن أن يغطي التوجيه المسبق أي ‪.‬واتخاذ القرارات الصعبة عندما ال تكون حالتهم الذهنية في أفضل حاالتها بسبب مرض المريض‬
‫تحترم قرارات المريض حتى لو لم‪.‬جانب من جوانب الرعاية التي يحق للمريض اتخاذ قرار بشأنها أثناء المرض العضال وبعد الوفاة‬
‫يفضل كتابة‬ ‫‪DNR.‬‬ ‫التوجيه المسبق يتعامل مع القرارات الرئيسية مثل‪.‬تكن منطقية ولكن يجب أال تتعارض مع الشريعة اإلسالمية‬
‫التوجيه الشفهي‪ .‬من األفضل أن تقوم كل مؤسسة بتطوير تنسيق معين للتأكد من استيفاء جميع المتطلبات القانونية‪.‬التوجيه المسبق وشهده‬
‫‪.‬الذي يتم مشاهدته بشكل صحيح فعال‪ ،‬ولكن يجب تجنبه ألن الشكوك قد تنشأ حول صحتها‬

‫االمتناع عن دعم الحياة االصطناعية‬

‫‪PLEAZE, ANY NOTE OR CORRECTION OR ADDITIN SEND:‬‬ ‫‪[email protected]‬‬


‫نسألكم الدعاء بظهر الغيب‬
‫‪P a g e | 217‬‬
‫ً‬
‫يتم اتخاذ قرار االمتناع عن أجهزة دعم الحياة عندما يتم العثور على المريض ميت ا بالفعل أو عندما يكون هناك دليل واضح على أن هذا‬
‫في ‪ .‬ومع ذلك‪ ،‬فإن قول هذا أسهل من فعله ألن الحقائق العملية تحدد سلوك األطباء المعالجين‪.‬الدعم سيكون عديم الجدوى من الناحية الطبية‬
‫مو اجهة مريض في حالة حرجة مع أفراد أسرته القلقين الذين يتطلعون إلى الطبيب إلنقاذ المريض‪ ،‬سيجد الطبيب صعوبة نفسية في‬

‫‪PLEAZE, ANY NOTE OR CORRECTION OR ADDITIN SEND:‬‬ ‫‪[email protected]‬‬


‫نسألكم الدعاء بظهر الغيب‬
‫‪P a g e | 218‬‬
‫باإلضافة إلى ذلك‪ ،‬من الصعب على األطباء أخذ مثل هذا قرار جاد ال‪.‬حجب دعم الحياة الذي في حكمه األفضل ال طائل من ورائه‬
‫ولذلك فإن العديد من األطباء يلعبون بأمان‪.‬رجوع فيه بسبب عدم اليقين في التقييم السريري وعدم وجود وقت كاف الستيعاب الحقائق‬
‫من خالل بدء دعم الحياة؛ ومع ذلك‪ ،‬فإن هذا يخلق مشكلة جديدة تتعلق بموعد إنهاء ذلك‪ ،‬وهو قرار آخر مليء بالمشاعر تقاومه العائالت‬
‫‪.‬عادة‬

‫سحب أجهزة دعم الحياة االصطناعية‬


‫غا‪ ،‬أو عندما يكون من الواضح أنه ال طائل من ورائه‬ ‫يمكن االعتماد على‪.‬يجب إيقاف دعم الحياة نظر ًيا بمجرد موت المريض دما ً‬
‫‪.‬العالمات السريرية للموت الدماغي في هذه المسألة ويمكن تأكيدها عن طريق تخطيط الدماغ والتصوير‪ ،‬وكذلك االختبارات المعملية‬
‫يتبع سحب أجهزة دعم الحياة الموت فو ًرا في كثير من الحاالت‪ . ،‬ساعة للتأكيد‪12-6‬للتأكد‪ ،‬يجب تكرار اختبار موت جذع الدماغ بعد‬
‫ًنظر إلى الطبيب على أنه‬ ‫ًا "‪.‬يسحب القابس"و ي‬ ‫في كثير من األحيان‪ ،‬تعارض العائالت سحب القابس ويقبل األطباء أحيا ن‬
‫يمكن أن تتأثر قرارات االنسحاب بتوافر األسرة في وحدة ‪.‬وينتظرون بعض الوقت لمنح األسرة الوقت للتصالح مع حقيقة الموت ونهايته‬
‫العناية‬
‫‪ .‬في حاالت نقص األسرة‪ ،‬هناك جهود أكثر عدوانية ومتكررة الختبار موت جذع الدماغ‪.‬المركزة‬

‫‪ DNT‬و‪DNR‬أوامر‬

‫يجب إبالغ األسرة بالقرار‪ . ،‬هو قرار يتخذه ثالثة أطباء‪ ،‬بما في ذلك أخصائي األمراض وطبيب المريض األساسي)‪(DNR‬أمرعدم اإلنعاش‬
‫بشكل أساسي للحاالت التي يكون فيها اإلنعاش غير مجد وليس بالضرورة لمرض ‪ DNR‬يتم وضع أمر ‪.‬لكن ا ل يمكنهم التدخل في القرار‬
‫التنبيب والتهوية‪ ،‬والضغط على الصدر‪ ،‬والعقاقير المؤثرة في ‪ :‬يجب أن يحدد الطلب اإلجراءات التي يتم تضمينها‪.‬عضال في حد ذاته‬
‫بـ عالج الحالة المرضية األولية‪ ،‬مثل السرطان‪ ،‬عند ذلك)‪ (DNT‬يتعلق أمر عدم المعالجة ‪.‬التباين‪ ،‬وقناع الغاز‪ ،‬وما إلى ذلك‬
‫يجب إجراء‪ .‬ليعني أن اإلنعاش ال يتم تنفيذه في حاالت السكتة القلبية والتنفسية‪ DNT‬أحيانًا يساء فهم أمر ‪.‬يعتبر العالج غير مجدي‬
‫‪.‬إنعاش مع فائدة صافية تدوم لفترة معقولة للمرضى في نهاية المطاف بغض النظر عن شدة مرضهم‬

‫القتل الرحيم‬
‫القتل الرحيم النشط هو فعل من ‪ ،.‬يتسبب في وفاة مريض بمرض مميت إلنقاذه من مزيد من األلم والمعاناة"الموت الجيد"وهو حرفيا‬
‫القتل الرحيم السلبي هو فعل إغفال يفشل فيه الطبيب في اتخاذ اإلجراءات الالزمة للحفاظ ‪.‬صنع الطبيب يقوم فيه بعمل يؤدي إلى وفاة المريض‬
‫القتل الرحيم اإليجابي والسلبي كالهما غير قانوني وعاملين في مجال الرعاية الصحية يشاركون في نفوسهم يمكن ‪.‬على حياة المريض النهائي‬
‫السمة المميزة للقتل الرحيم هي ‪ .‬القتل الرحيم بنا ًء على طلب المريض وبموافقته المستنيرة ا ل يزال يعتبر غير قانوني‪.‬مقاضاتهم بتهمة القتل‬
‫ًا إذا كانت النية وراءه ‪.‬القصد من الفعل‪ ،‬وهو تجنيب المريض المزيد من المعاناة‬ ‫يمكن اعتبار اإلجراء الذي يعتبر القتل الرحيم قانون ي‬
‫االمتناع عن العالج ألنه ال طائل من ورائه مقبول‪ ،‬ولكن منعه إللسراع بوفاة المريض لتجنب المزيد من‪.‬مختلفة‬
‫‪.‬المعاناة هو القتل الرحيم السلبي‬

‫قضايا ما بعد الوفاة وتشريح الجثة والتحنيط‬


‫يمكن للمريض المختص إبداء رغباته بشأن فحص ما بعد ‪.‬ف ي سياق المرض العضال‪ ،‬قد يتعين اتخاذ قرارات بشأن إجراءات ما بعد الوفاة‬
‫الممارسة المعتادة والمقبولة هي الدفن )‪.‬على سبيل المثال‪ ،‬في حاالت تنفيذ العدالة( الوفاة وسيتم احترامها مع استثناءات قليلة‬
‫الفوري‪ ،‬ولكن إذا مات المريض في بلد أجنبي حيث ال يمكن ضمان عمليات الدفن وفقًا للشريعة اإلسالمية‪ ،‬فيمكن تحنيط الجثة لنقلها إلى‬
‫‪ .‬يمكن لصانعي القرار بالوكالة عن األسرة اتخاذ قرارات بشأن تشريح الجثة والتحنيط‪.‬بلد مسلم‬

‫القضايا األخالقية في رعاية المرضى الميؤوس من شفائهم‬

‫قبل الوصول إلى مرحلة العبث الطبي‪ ،‬يجب أن يتلقوا‪.‬يحتاج المرضى الميؤوس من شفائهم إلى دعم جسدي ونفسي اجتماعي وروحي‬
‫قد نصح بالعالج األقل شدة إذا كانت‪.‬العالج لمرضهم األصلي‪ ،‬مع الحرص على تحقيق التوازن الصحيح بين الفوائد واآلثار الجانبية‬
‫بعد مرحلة العبث الطبي‪ ،‬يتم تقديم الرعاية التلطيفية وعالج األعراض‪.‬الفوائد من حيث النتيج ة الصحية العامة ال تستحق اآلثار الجانبية‬
‫إن جوهر الرعاية التلطيفية هو السيطرة على األلم‪ ،‬ولكن يمكن أن تشمل الجراحة الملطفة والعالج اإلشعاعي الملطفة التي ال توقع‪.‬فقط‬
‫ًا؛ على سبيل المثال‪ ،‬يتم ‪.‬أن تعالج المرض ولكن للسيطرة على األعراض وتحسين نوعية الحياة‬ ‫يمكن أن يكون عالج األعراض عدوان ي‬
‫يستمر المرضى النهائيون في تلقي التغذية والترطيب والرعاية العامة الداعمة ‪.‬عالج االلتهاب الرئوي بالمضادات الحيوية من الخط األول‬
‫يمكن للعاملين في مجال الرعاية الصحية البدء بلباقة‪ .‬كما أنهم يحتاجون إلى دع م نفسي وروحاني للتخفيف من قلقهم‪.‬دون تمييز‬

‫‪PLEAZE, ANY NOTE OR CORRECTION OR ADDITIN SEND:‬‬ ‫‪[email protected]‬‬


‫نسألكم الدعاء بظهر الغيب‬
‫‪P a g e | 219‬‬
‫يجب أيضا تذكير المرضى المحتضرين بواجباتهم الدينية‪ ،‬مثل‪.‬في مناقشة القضايا القانونية‪ ،‬مثل التوجيهات المسبقة والتبرع باألعضاء‬
‫‪ .‬قد يذكرهم العاملون في مجال الرعاية الصحية بإبرام وصاياهم‪.‬دفع الزكاة‪ ،‬والتزاماتهم‪ ،‬مثل تسوية الديون‬

‫قرارات للمرضى الميؤوس من شفائهم‬


‫قد تكون القرارات الجادة التي لها عواقب ال رجعة فيها قد اتخذت من قبل أو نيابة عن المرضى في مراحله األخيرة األول واألهم هو‬
‫‪.‬قرار حجب أو سحب العالج القوي الذي ليس له فائدة صافية من شأنها أن تستمر لفترة زمنية معقولة‬
‫والثاني هو قرار منع اإلنعاش في حالة توقف القلب والجهاز التنفسي للمرضى الذين ال يستطيعون الحصول على فائدة صافية من‬
‫‪.‬اإلنعاش القلبي الرئوي والذين يستسلمون مرة أخرى ويضطرون إلى الخضوع لإلنعاش‬
‫هذا اإلنعاش المتكرر غير مجدي ويجب حجبه بأمر من الطبيب‪ ،‬مشي ًرا إلى أنه في حالة االنهيار‪ ،‬لن يتم اتخاذ تدابير اإلنعاش المحددة‬
‫بالنسبة للمرضى الذين )‪.‬دون السعي إلى إشراكهم في القرار( هذا ما يسمى بأمر عدم اإلنعاش هو قرار الطبيب‪ ،‬ولكن يجب إبالغ األسرة ‪.‬‬
‫ًا للحياة‪ ،‬يجب اتخاذ قرار بشأن موعد سحب الدعم‬ ‫إذا كان من الممكن التأكد من موت جذع الدماغ‪ ،‬فإن قرار سحب‪.‬يتلقون دع ًما صناع ي‬
‫إذا كان المريض في‪ .‬مقبول كتعريف للموت القانوني‪DEATH STEM BRAIN‬أجهزة دعم الحياة يكون سهالً ألن موت جذع الدماغ‬
‫االنسحاب على أساس تدني‪.‬غيبوبة ال رجعة فيها مع سالمة وظيفة جذع الدماغ‪ ،‬فإن قرار سحب أجهزة دعم الحياة يكون أكثر تعقي ًدا‬
‫ًا مقبولة أخالقيا ً عادة ً بسبب االهتمام الغالب بالحفاظ على الحياة‬ ‫حفظ (جودة الحياة واستمرار نفقات العناية المركزة ليست أسبا ب‬
‫ًا للخالف )النفس‬ ‫ً‬
‫يمكن سحب دعم الحياة في الحاالت التي ال جدوى منها بالتأكيد‪ ،‬لكن هذا ليس قرا ًرا سهال وعادة ما يكون سب ب‬
‫بين األسرة‬
‫‪.‬والعاملين في مجال الرعاية الصحية‬
‫القرار المسبق الذي‪.‬قد يتم االتصال بأسر المرضى في مراحله النهائية للموافقة على حصاد أعضائهم بمجرد التأكد من الوفاة السريرية‬
‫‪.‬يتخذه المريض النهائي وهو ال يزال مؤهالً سيجعل عمل فريق زراعة األعضاء أسهل‬
‫استمرارية الرعاية‬

‫مع تقدم العمر‪ ،‬تظهر مشاكل جديدة تتطلب رعاية المسنين‪ ،‬وهو نظام جديد يلبي ‪.‬عادة ما يتمتع الشباب في مقتبل العمر بصحة جيدة‬
‫يتم عالج المرض القابل للشفاء بسرعة‪ ،‬بينما ال يمكن عكس ‪ .‬يمكن أن يمرض الشخص السليم ويحتاج إلى رعاية طبية‪.‬احتياجات كبار السن‬
‫إذا لم يتم احتواء المرض‪ ،‬يدخل المريض مرحلة نهائية عادة ما يكون ‪.‬المرض المزمن‪ ،‬على الرغم من أنه يمكن التخفيف من آثاره الضارة‬
‫في المرحلة النهائية الممتدة‪ ،‬قد يتم تحديد ‪.‬لها تدخل تكنولوجي مكثف؛ ومع ذلك‪ ،‬بدون نتيجة جيدة‪ ،‬عادة ما يترتب على ذلك الموت‬
‫العالج الفعال للمرض على أنه عديم الجدوى من الناحية الطبية ويتم نقل المرضى إلى الرعاية التلطيفية حيث يتلقون التغذية‬
‫‪.‬والترطيب والسيطرة على األلم‪ ،‬فضالً عن الدعم االجتماعي والنفسي‬
‫القضايا األخالقية في رعاية المسنين‬
‫إذا لم‪ .‬واألخطر هو التدهور العقلي الذي يضعف القدرة على اتخاذ قرارات مستنيرة‪.‬يواجه اختصاصيو الشيخوخة العديد من المشاكل‬
‫من األفضل أن يعين المريض صانع القرار بالوكالة في وقت مبكر‪.،‬يقدم المريض المسن بيا ًنا مسبقًا‪ ،‬فيجب استخدام صانع قرار بالوكالة‬
‫‪ .‬إذا لم يتم تعيين صانع قرار بالوكالة‪ ،‬فيمكن أن يكون أحد أفراد األسرة هو صانع القرار‪ .‬لديها القدرة العقلية الكاملة‪/‬عندما يكون لديه‬
‫‪.‬في الحالة القصوى التي ال يوجد فيها قريب‪ ،‬سيقرر العاملون في مجال الرعاية الصحية وفقًا لما يعتقدون أنه في مصلحة المريض‬
‫القضايا األخالقية في الرعاية التلطيفية‬
‫تنشأ القضايا األخالقية فيما يتعلق‪.‬يتشارك مرضى الرعاية التلطيفية مع مرضى المسنين مشكلة ضعف القدرة على اتخاذ القرار‬
‫يعد‪.‬جا للعدوى التي تظهر‬ ‫قد تشمل أي‪ .‬قد تقتصر هذه على التغذية‪ ،‬والترطيب‪ ،‬والتحكم في األلم ‪.‬بالتدخالت التي يمكن القيام بها ضا عال‬
‫ضا بخصوص مستوى التخدير‬ ‫من ناحية‪.‬‬ ‫ًا‬
‫تثار أسئلةط‪.‬ا تنفس ي‬ ‫التحكم في األلم مشكلة ألن بعض المسكنات مثل المورفين قد تسبب تثبي‬
‫أي‬
‫سيترك التسكين األقل للمريض نشطا‪.‬ترك شبه واعي وغير قادر على التفاعل مع األسرة‬ ‫يمكن القضاء على األلم تما ًما‪ ،‬لكن المريض‬
‫ًا‪ ،‬ولكن مع مستوى معين من األلم‬ ‫‪.‬اجتماع ي‬
‫القضايا األخالقية في رعاية اإلعاقة‬
‫سنت العديد‪.‬قد يتعرض المرضى ذوو اإلعاقة للتمييز إذا لم يتم تزويدهم بخدمات خاصة لتمكينهم من العيش بشكل طبيعي قدر اإلمكان‬
‫‪ .‬تتطلب هذه القوانين توفير خدمات خاصة للمعاقين في المستشفيات ومنازلهم‪.‬من الواليات قوانين خاصة لحماية حقوق ومصالح المعاقين‬
‫‪.‬يتلقى العاملون في مجال الرعاية الصحية تدريبا ً على الحساسية في التعامل مع المعاقين واحتياجاتهم‬

‫‪PLEAZE, ANY NOTE OR CORRECTION OR ADDITIN SEND:‬‬ ‫‪[email protected]‬‬


‫نسألكم الدعاء بظهر الغيب‬
‫‪P a g e | 220‬‬
‫الشابتر العاشر‬
‫‪1‬الحالة‬

‫يتمثل دور الممرضة في ‪.‬عرضت الشركة المنتجة ألدوية لمعالجة ارتفاع ضغط الدم دفع راتب بدوام جزئي لممرضة في عيادة الطبيب‬
‫يعتقد الطبيب أن هذا ‪.‬تدقيق سجالت المرضى‪ ،‬والتأكد من أن المصابين بارتفاع ضغط الدم يتم فحصهم بانتظام والحصول على أحدث األدوية‬
‫يتم توفير معلومات‪ .‬تقدم الممرضة بيانات المريض المجهولة للشركة‪ ،‬لكنها ممنوعة من الترويج لمنتجاتها ‪.‬يعزز رعاية المرضى‬
‫توجد عالقة جيدة‪ ،‬وتقدم‪.‬‬ ‫حول أدوية الشركة بانتظام من قبل فريق المبيعات الذي يزور العيادة ويدفع مقابل غداء العمل مع الطبيب‬
‫‪.‬الشركة هدايا من حين آلخر وتدعو طاقم األطباء لتناول العشاء‬

‫في الحالة األولى‪ . ،‬تم اختيار هذه الحاالت ألنها شائعة وتمثل الحد الفاصل بين ما هو محظور بوضوح وما يعتبر مقبوالً أخالقيا ً )ج‬
‫مقابل خدمات الممرضة للتدقيق‪ ،‬وعلى الرغم من عدم حظرها بوضوح‪. ،‬ليس من الحكمة أن يقبل الطبيب هدايا أو وجبات غالية الثمن‬
‫على الرغم من أن الممرضات الذين ترعاهم الشركة ال يروجون‪.‬يجب مراجعة أي قيود مرتبطة بها بعناية‪ ،‬إذا كان هناك سجل عام محلي‬
‫قد تحصل الممرضات أيضا على‪.‬أللدوية‪ ،‬فإن البيانات التي يبحثون عنها يستخدمها فريق مبيعات الشركة لتقييم سوق العيادة المحتمل‬
‫‪.‬مكافآت عن طريق تحديد المرضى الذين يمكن نقلهم إلى مخططات األدوية الجديدة المكلفة‪ ،‬وبالتالي فإن الضغط لنقلهم يتراكم على الطبيب‬
‫يجب أن يكون وعي ‪.‬يجب التعامل مع أي شكل من أشكال الخدمة التي ترعاها الشركة بعناية‪ ،‬ومع مراعاة تصورات المرضى والجمهور‬
‫األطباء بأنفسهم وبزمالئهم أنماط وصف األدوية عالية ومتماشية مع ممارسة األقران‪ ،‬ومراجعتها إذا تغيرت األنماط نتيجة‬
‫‪.‬لقبول الخدمة‬
‫‪2‬الحالة‬

‫لقد كان يعمل بسعادة في‪ .‬عا ًما ويعمل في أحد مستشفيات الرياض ‪46‬الدكتور أحمد استشاري أمراض الجهاز الهضمي يبلغ من العمر‬
‫يخبر صالح‪.‬ذات يوم‪ ،‬اتصل به مندوب يدعى صالح من شركة أدوية كبرى ‪ .‬عا ًما‪ ،‬ولم يواجه أي مشاكل ‪13‬نفس المستشفى منذ‬
‫‪.‬الدكتور أحمد عن عقار جديد صنعته الشركة يمكنه عالج اإلسهال بشكل فعال‪ ،‬وأحمد مدعو لمشاهدة عرض تقديمي عن الدواء في دبي‬
‫يقبل أحمد ‪.‬تدفع الشركة جميع تكاليف رحلة أحمد التي تستغرق ثالثة أيام‪ ،‬على الرغم من أن العرض التقديمي سيكون في يوم واحد فقط‬
‫عند عودته‪ ،‬يزوره صالح مرة أخرى‪ ،‬ويقول إن الشركة كانت ‪.‬الدعوة‪ ،‬وبعد حضور العرض يستمتع ببقية األيام الثالثة في دبي كإجازة‬
‫رياالت له في كل مرة يصف فيها دواءهم‪10‬ممتنة له لحضور الع رض التقديمي‪ ،‬وكتعبير عن امتنانهم‪ ،‬فإنهم على استعداد لتقديم‬
‫‪.‬لمريض‬
‫في هذه الحالة‪ ،‬ناشدت شركة األدوية الدكتور أحمد‪ ،‬الذي يحظى بالثقة‪ . ،‬يقدم سيناريو الحالة هذا مثاالً ممتا ًزا على الرشوة)ج‬
‫بإرساله في رحلة فاخرة إلى دبي؛ ثم عرضوا عليه المال مقابل كل وصفة طبية لعقارهم‪ ،‬في محاولة للتأثير على حكمه وسلوكه‬
‫لمصلحتهم الخاصة بالطبع‪ ،‬بينما يستفيد كل من الطبيب وشركة األدوية‪ ،‬فإن المرضى هم الذين قد يتعرضون لألذى‪ ،‬حيث أن الدكتور أحمد‬
‫مؤتمن‬
‫‪.‬على القيام بواجبه‪ ،‬وهو اختيار أفضل دواء لكل مريض وفقًا له أو لها‬
‫من الناحية‪.‬ومن الناحية اإلسالمية فإن تصرفات الدكتور أحمد وصالح وشركة األدوية حرام‪ ،‬وسيعاقبون جميعا ً على هذه األفعال‬
‫لم يتبع الدكتور أحمد في هذا السيناريو األخالق الطبية المتمثلة في اإلحسان وعدم ‪.‬األخالقية‪ ،‬نعلم أن الرشوة تعتبر غير صحيحة‬
‫باإلضافة إلى ذلك‪ ،‬من وجهة نظر نفعية‪ ،‬ال يمكن تبرير تصرفات الطبيب وشركة‪.‬المخالفات‪ ،‬وربما أضر بمرضاه لمصلحته الشخصية‬
‫من ‪.‬األدوية‪ ،‬ألنه في الوقت الذي استفاد فيه كالهما‪ ،‬فإن عواقب أفعالهما قد تسبب األلم واألذى لكثير من الناس‬
‫في مراجعة األدبيات‪ ،‬ناقشنا التقارير اإلخبارية المختلفة لشركات ‪.‬وجهة نظر قانونية‪ ،‬هذا الشكل من الرشوة محظور في معظم دول العالم‬
‫ال تسمح قوانين المملكة العربية السعودية‪ ،‬وخاصة قانون‪.‬األدوية والعاملين الطبيين الذين تم القبض عليهم لتورطهم في الرشوة‬
‫كان من الممكن أي‪.‬عاقب الدكتور أحمد بالحبس والغرامة ضا تغريم شركة األدوية‬ ‫مكافحة الرشوة‪ ،‬بمثل هذا الفعل‪ ،‬وكان من الممكن أن‬
‫‪.‬بسبب أفعالها‬

‫الشابتر الحادي عشر‬


‫‪1‬الحالة‬

‫قام الطبيب بسحب الدم من‪.‬طلب قائد لواء من الجيش من طبيب اللواء إجراء بحث حول أسباب اإلجازات المرضية المرتفعة للغاية‬
‫‪ .‬عندما احتج بعض الجنود على عدم مطالبتهم بالموافقة‪ ،‬أخبرهم أنه ينفذ األوامر العسكرية ‪.‬جميع الجنود للبحث عن خصائصهم المناعية‬

‫‪ .‬الموافقة الطوعية ‪:‬تلميح‬

‫‪PLEAZE, ANY NOTE OR CORRECTION OR ADDITIN SEND:‬‬ ‫‪[email protected]‬‬


‫نسألكم الدعاء بظهر الغيب‬
‫‪P a g e | 221‬‬
‫‪2‬الحالة‬

‫‪ 10‬كانت ورقة الموافقة المستنيرة بطول ‪.‬كان أحد األطباء يجند المرضى إلجراء دراسة كبيرة متعددة المراكز حول احتشاء عضلة القلب‬
‫شعر معظم األشخاص بالملل من قراءة التفاصيل ‪.‬صفحات نظ ًرا لوجود العديد من التفاصيل اإلجرائية واآلثار السلبية للكشف عنها‬
‫أو‬
‫‪.‬االستماع إليها‪ ،‬وكانوا مستعدين للتوقيع ألنهم وثقوا في األطباء الذين يقومون بالدراسة‬
‫‪ .‬الموافقة المستنيرة بعد اإلفصاح الكامل ‪:‬تلميح‬

‫‪3‬الحالة‬

‫لم يكن ملف المخاطر ‪.‬تم تقديم عقار جديد أثبت فعاليته ضد اللوكيميا في التجارب على الحيوانات وفي المختبر والمرحلة الثانية للتجارب البشرية‬
‫لم يكن هناك عالج فعال معروف لهذا‪ .‬كان من المقرر اختباره ضد دواء وهمي‪.‬الخاص به مفهوما جيدا من الدراسات السابقة‬
‫‪.‬المرض‬

‫‪ .‬فائدة عالية ‪:‬تلميح‬

‫‪4‬الحالة‬

‫أكمل بانتظام نماذج ‪.‬وافق طبيب في مستشفى محلي على أن يكون محققًا في تجربة دولية متعددة المراكز برعاية شركة أدوية‬
‫في إحدى المرات‪ ،‬شك الراعى في البيانات المقدمة وأصر على إرسال مخطط المريض األصلي ‪ .‬وأرسلها إلى الراعي في الخارج‪CRF‬‬
‫إليه‬
‫‪.‬للفحص والتحقق‬

‫السرية‪:‬تلميح‬
‫‪5‬الحالة‬

‫‪5‬تم إجراء تجربة متعددة المراكز لدواء جديد في مستشفى محلي؛ كانت المستشفى من آخر من انضموا إلى التجربة التي استمرت‬
‫أظهر التحليل المؤقت للبيانات من قبل الجهة الراعية ‪ .‬أظهرت النتائج المحلية أن الدواء كان فعاالً وأن المرضى راضون‪.‬سنوات‬
‫لذلك قرر إنهاء الدراسة‪ .‬كما الحظ الراعي إذا تم حذف نتائج المستشفى المحلي‪ ،‬فسيكون حجم عينة المواقع المتبقية كافيا ً ‪.‬تفوق الدواء الجديد‬
‫‪.‬في المستشفى قبل األوان وقطع اإلمداد بالدواء‬

‫‪ .‬مسؤولية المحقق ‪:‬تلميح‬

‫‪6‬الحالة‬

‫بعد بدء التجربة‪ ،‬تم اكتشاف أن األطباء في ‪.‬تلقى مستشفى منحة كبيرة من شركة أدوية إلجراء مسح ما بعد التسويق على مسكن جديد‬
‫دعا الباحث الرئيسي إلى اجتماع لجميع األطباء في العيادة الخارجية‪ ،‬وطلب منهم البدء في وصف‪.‬المستشفى نادرا ما يصفون الدواء‬
‫‪ .‬وأوضح أن العقار آمن‪ ،‬وقد ثبت بالفعل فعاليته ‪.‬الدواء حتى ال يفقد المستشفى المنحة‬

‫‪ .‬األبحاث السريرية ‪ vs‬الرعاية السريرية ‪:‬تلميح‬

‫‪7‬الحالة‬

‫ًا ‪ .‬قدم الراعي بروتوكول مكتوب بشكل جيد ومفصل‪.‬حصل طبيب على منحة عالية لدراسة عقار جديد‬ ‫كان تنفيذ البروتوكول صع ب‬
‫وجد األشخاص أن معلومات الموافقة المستنيرة متداخلة‪ ،‬ولم يتمكنوا من االلتزام بمواعيد الزيارة المحددة في ‪.‬في الظروف المحلية‬
‫قام الطبيب بإجراء تعديالت على البروتوكول اعتقد أنها بسيطة وال تؤثر على صحة الدراسة أو سالمة المريض‪ ،‬ولم ير أي سبب ‪.‬البروتوكول‬
‫تجاهل الطبيب مالحظاتهم واستمر في الدراسة دون توثيق التغييرات‪ .‬أرسل الراعي مراقبين اكتشفوا التناقض‪REC. / IRB‬إلبالغ‬
‫التي أجراها‬
‫‪ .‬التقيد بالبروتوكول ‪:‬تلميح‪.‬‬

‫‪PLEAZE, ANY NOTE OR CORRECTION OR ADDITIN SEND:‬‬ ‫‪[email protected]‬‬


‫نسألكم الدعاء بظهر الغيب‬
‫‪P a g e | 222‬‬
‫‪8‬الحالة‬

‫أراد أستاذ الجراحة المتمرس ج ًدا إجراء بحث يقارن بين نهجين جراحيين كان يستخدمهما بالتناوب على مدار الخمسة عشر‬
‫لقد تأكد من استيفاء ‪ .‬واللوائح السعودية الخاصة بالبحوث‪ ICH-GCP‬واستعرض إعالني نورمبرج وهلسنكي‪ ،‬وكذلك إرشادات ‪.‬عا ًما الماضية‬
‫جميع شروط هذه الوثائق في بحثه‪ ،‬ولم ير أي سبب لطلب موافقة مجلس الهجرة والالجئين المحلي‪ ،‬والذي كان في رأيه يتألف من‬
‫‪.‬أعضاء شباب عديمي الخبرة‪ ،‬وكان معظمهم من طالبه‬

‫‪ .‬حوكمة البحث‪:‬تلميح‬

‫‪9‬الحالة‬

‫أجرى أستاذ في أمراض القلب دراسة استقصائية جيدة التصميم بعد التسويق لعقار تم تسويقه مؤخ ًرا في المملكة العربية السعودية‪ ،‬ولكن‬
‫كانت النتائج األولية ضد ما نشره العديد من الباحثين‪ ،‬وحتى بدت ‪ .‬سنوات في الواليات المتحدة واالتحاد األوروبي‪10‬تم تسويقه ألكثر من‬
‫أكد تحليل البيانات الكاملة التحليل ‪ .‬وطلب من فريق الباحثين إبقاء هذه المعلومات سرية لحين االنتهاء من الدراسة‪.‬غير منطقية بالنسبة له‬
‫وقرر األستاذ عدم تقديم النتائج للنشر خوفا على سمعته وتجنب إزعاج أطباء القلب اآلخرين في البلد الذين كانوا راضين‪.‬األولي‬
‫‪.‬عن العقار‬

‫‪ .‬أخالقيات النشر البحثي‪:‬تلميح‬

‫‪10‬الحالة‬

‫لمستشفى كبير للنظر في اقتراح ترعاه شركة أدوية متعددة الجنسيات‪ ،‬وأعلن جميع األعضاء‬ ‫جلس مجلس الهجرة والالجئين )‪(IRB‬‬
‫تم تعيين رئيس مجلس اإلدارة كمستشار مؤقت للشركة على مدى السنوات الخمس الماضية‪ ،‬وذلك ‪ .‬عن مصالحهم‪6‬الخمسة من أصل‬
‫وكان لصهر نائب رئيس مجلس اإلدارة أسهم في‪.‬بشكل أساسي إللقاء محاضرات حول عمليات تطوير األدوية للباحثين المحتملين‬
‫سنوات‪ 6،‬لم يحضر المحقق الرئيسي‪ ،‬وهو عضو لمدة ‪ .‬كان أحد األعضاء صهر المحقق الرئيسي‪ .‬الير سعودي‪1000‬الشركة بقيمة‬
‫‪ .‬شرعت اللجنة في النظر في االقتراح ألن مصالح الجميع أصبحت معروفة اآلن ‪ .‬عضو واحد فقط ليس لديه مصلحة لإلعالن‪.‬االجتماع‬

‫‪ .‬تضارب المصالح‪:‬تلميح‬

‫شرح مهم‬
‫الموافقة المستنيرة على البحث‬
‫بعد الكشف الكامل عن تفاصيل الدراسة لتمكين الموضوع من اتخاذ قرار مستنير‪ ،‬يجب عليه أو عليها الموافقة طواعية وبحرية على‬
‫‪ REC. / IRB‬يجب أن تكون المعلومات المقدمة للموضوع مكتوبة ويجب أن تكون قد تمت الموافقة عليها من قبل ‪.‬المشاركة في الدراسة‬
‫ًا لطرح األسئلة‬ ‫يجب ‪ .‬يجب أن يتم توقيع نموذج الموافقة من قبل الموضوع‪ ،‬ومؤرخا‪ ،‬وشاه ًدا‪.‬يجب إعطاء الموضوع وق ت‬
‫إعطاء‬
‫‪/‬الغرض من التجربة‪ ،‬أن تكون العالجات ‪:‬يجب تضمين المعلومات التالية في معلومات الموافقة ‪.‬الموضوع نسخة من نموذج الموافقة‬
‫المضايقات المتوقعة‪ ،‬والعالجات واإلجراءات البديلة‪/ ،‬اإلجراءات لألبحاث‪ ،‬واإلجراءات المتضمنة‪ ،‬والفوائد المتوقعة‪ ،‬والمخاطر‬
‫والتعويض عن اإلصابة التجريبية‪ ،‬وحرية الموضوع في االنسحاب من الدراسة دون الحاجة إلى إبداء األسباب‪ ،‬والسرية‪ ،‬وإذن‬
‫يجب ‪.‬الموضوع للوصول المباشر إلى سجالته‪ ،‬وشخص لالتصال به في حالة اإلصابة أو األسئلة‪ ،‬ومدة الدراسة‪ ،‬وعدد الموضوعات المعنية‬
‫اتخاذ احتياطات خاصة للتأكد من مصالح األشخاص الضعفاء محميون ألنهم قد ال يكونون قادرين على ممارسة استقالليتهم الكاملة بسبب قي ود‬
‫تشمل الموضوعات المعرضة للخطر القاصرين والمرضى العقليين واألشخاص العاجزين الذين تكون موافقتهم قانونية‪.‬معينة‬
‫في ظروف استثنائية‪ ،‬يمكن‪ .‬الطالب والموظفون الصغار معرضون للخطر أيضا ألنهم قد يوافقون على البحث تحت اإلكراه‪.‬وكيل‬
‫يمكن أن يحدث هذا‪ ،‬على سبيل المثال‪ ،‬في غرفة الطوارئ‪ ،‬عندما يكون المريض غير قادر على إعطاء‪.‬إجراء البحث دون موافقة‬
‫ًا لتحسين تقديم الخدمة‬ ‫تدرس‪ .‬في بعض أنواع البحث النفسي‪ ،‬قد يؤدي طلب الموافقة إلى تحيز المستفتي‪.‬الموافقة ويكون البحث ضرور ي‬
‫‪.‬الوضع بعناية فائقة قبل الموافقة على البحث دون موافقة‪RECs / IRB‬‬

‫اعتبارات المخاطر‪ vs‬الفوائد ‪:‬سالمة المريض‬

‫قد يستفيد األشخاص المشاركون في الدراسة من العالج الجديد‪ ،‬ويمكن أن ‪.‬تعتمد الموافقة على الدراسة على دراسة متأنية للفوائد والمخاطر‬
‫قد ال تحصل بعض األشخاص في ذراع التحكم على أي فوائد؛ في هذه الحاالت‪ ،‬علينا النظر في‪.‬يتحسنوا بعد فشل العالجات التقليدية‬

‫‪PLEAZE, ANY NOTE OR CORRECTION OR ADDITIN SEND:‬‬ ‫‪[email protected]‬‬


‫نسألكم الدعاء بظهر الغيب‬
‫‪P a g e | 223‬‬
‫جميع ‪ .‬يجب مراعاة فوائد الدراسة‪ ،‬سواء كانت فردية أو مجتمعية‪ ،‬مقابل المخاطر التي يواجهها موضوع الدراسة‪.‬فائدة الدراسة للمجتمع‬
‫ومع ذلك‪ ،‬ليس من الممكن تصور جميع المخاطر مقد ًما‪ ،‬لذلك نحن بحاجة إلى أنظمة لضمان سالمة‪.‬الدراسات تخطط لتقليل هذا الخطر‬
‫يتم تحديد األحداث السلبية المحتملة في‪.‬‬ ‫المرضى يتم ضمان سالمة المرضى من خالل مراقبة األحداث السلبية واإلبالغ عنها‪.‬‬
‫يجب ‪ .‬التفاعل الدوائي الضار هو أي استجابة سيئة وغير مقصودة للمنتج قيد البحث‪.‬البروتوكول ويتم اإلبالغ عنها في نموذج اإلبالغ الموحد‬
‫يجب ‪ .‬أيام‪ 7‬يتم اإلبالغ عن الوفاة والحاالت التي تهدد الحياة في غضون ‪ .‬والجهة الراعية‪REC / IRB‬اإلبالغ عن األحداث السلبية الخطيرة إلى‬
‫بالتحقيق في التقرير‪ ،‬وقد يتخذ قرارات بشأن تعليق الدراسة أو إيقافها ‪ REC / IRB‬سيقوم ‪ .‬يو ًما‪15‬اإلبالغ عن اآلخرين في غضون‬
‫المجموعات االقتصادية اإلقليمية ولكن يبدو أن هذا‪ /‬يتم النظر في الفوائد مقابل المخاطر من قبل مجالس المراجعة المؤسسية ‪.‬‬
‫‪ .‬قد يكون من الضروري استخدام التقنيات المستخدمة في تخصصات أخرى‪ ،‬مثل علوم صنع القرار ‪.‬ال يتم بطريقة منهجية‬

‫‪ ،REC / IRB‬لوائح البحث السعودية ‪REC / IRB : ،GCP‬مراجعة الحوكمة‬

‫تقوم بمراجعة واعتماد بروتوكول البحث وجميع وثائق‪.‬هي هيئة مستقلة وظيفتها الرئيسية حماية سالمة المريض ورفاهيته وحقوقه‬
‫كما يتعين عليها أي‪.‬التجارب‪ ،‬والتأكد من أن الباحثين مؤهلين وأن نسبة المخاطر إلى الفوائد مواتية ضا مراقبة طريقة إجراء الدراسة‪،‬‬
‫‪.‬للتأكد من التزامها بالبروتوكول‬

‫الشابتر الثالث عشر‬


‫ثنائية العقل والجسم‪:1‬الحالة‬

‫كشف الفحص والتحقيقات الجسدية عن عدم وجود ‪ .‬مريض قدم إلى العيادة مع شكاوى غامضة في البطن ومخاوف بشأن السرطان ‪:‬السيناريو‬
‫ًا على المريض إلضاعة وقته في العيادة وهو بصحة جيدة ‪.‬أمراض‬ ‫عندما كان المريض يغادر‪ ،‬أخبر الطبيب‪.‬كان الطبيب غاض ب‬
‫‪.‬لم يستجب الطبيب‪.‬أن عمه توفي في األسبوع السابق إلصابته بسرطان المعدة‬

‫‪ .‬لقد أخطأ الطبيب بتجاهل مشاعر وقلق المريض‪ ،‬والتي يمكن أن تحدث في حالة عدم وجود مرض جسدي وتحتاج إلى معالجة‪:‬تلميح‬

‫التعب النفسي‪ vs‬المرض ‪:2‬الحالة‬

‫كشف الفحص البدني‪ .‬شاب أرسل لفحص ما قبل التوظيف قام بملء استبيان صحي ولم يذكر أي مشاكل صحية على اإلطالق‪:‬السيناريو‬
‫وعند سؤاله عنهم‪ ،‬اعترف بأنهم تسببوا له في األلم من حين آلخر‪ ،‬لكنه كان صبو ًرا‪.،‬عن خلع شديد في الكتف وكسر أخرم غير ملتئم‬
‫‪.‬وال يقلق كثي ًرا بشأن هذه المشاكل‬
‫قد يشعر الشخص بالمرض ولكن ليس)‪.‬شعور شخصي( ومرض )الجسدي الباثولوجي( هذه الحالة توضح الفرق بين المرض ‪:‬تلميح‬
‫‪.‬لديه اضطراب جسدي‪ ،‬تما ًما كما قد ال يشتكي الشخص المصاب باضطراب جسدي من أي مرض‬
‫األساس العقلي للمرض‪:3‬الحالة‬

‫‪ .‬كانت مضطربة ورفضت التحدث إلى الطبيب‪ .‬تم إحضار مراهقة قسرا ً من قبل والدها إلى العيادة بسبب فقدان الشهية الشديد‪:‬السيناريو‬
‫انفتحت الحقًا على األخصائية االجتماعية‪ ،‬وكشفت أنها كانت حزينة وفقدت شهيتها بسبب النزاعات المتعددة في األسرة بين والديها وبين‬
‫‪.‬شقيقاتها الثالث‬
‫‪ .‬توضح هذه الحالة كيف يمكن أن تؤدي العوامل العقلية إلى مرض جسدي‪ ،‬وتؤكد أهمية النظرة الشاملة للصحة‪:‬تلميح‬
‫األساس العقلي واالجتماعي للمرض الجسدي‪:4‬الحالة‬

‫كان مجتمع الضواحي من رجال األعمال األثرياء مع سكن جيد ورعاية صحية ومرافق تعليمية وترفيهية أعلى معدالت ‪:‬السيناريو‬
‫كشفت دراسة استقصائية أجرتها وزارة الصحة عن عدم وجود مشاكل‪.‬االنتحار وتعاطي المخدرات غير المشروع في المنطقة بأكملها‬
‫‪.‬صحية أخرى‪ ،‬لكنها أكدت وجود ميول انتحارية وتعاطي المخدرات غير المشروع‬
‫‪ .‬توضح هذه الحالة أن العوامل االجتماعية يمكن أن تضر بالصحة الجيدة‪:‬تلميح‬
‫العالج‪ vs‬الطب الوقائي ‪:5‬الحالة‬

‫‪PLEAZE, ANY NOTE OR CORRECTION OR ADDITIN SEND:‬‬ ‫‪[email protected]‬‬


‫نسألكم الدعاء بظهر الغيب‬
‫‪P a g e | 224‬‬
‫نشب جدال حاد في مجلس المدينة بين مسؤول الصحة العامة‪ ،‬الذي أراد الحصول على تمويل لبرنامج التوعية بأمراض القلب ‪:‬السيناريو‬
‫ًا‪ ،‬وأعضاء المجلس الذين أرادوا تخصيص ميزانية أعلى إلنشاء وحدة رعاية الشريان ‪000،50‬المجتمعية بتكلفة‬ ‫الير سعودي سنو ي‬
‫في النهاية‪ ،‬تم اتخاذ قرار إنشاء وحدة رعاية الشريان التاجي بسبب أصوات غالبية أعضاء المجلس‪ .‬مليون الير سعودي‪5‬التاجي بتكلفة‬
‫‪.‬من كبار السن‪ ،‬وبعضهم مصاب بأمراض القلب‬
‫ضا أن الطب العالجي يفوز‬ ‫كما يوضح أي‪ .‬توضح هذه الحالة معضلة االختيار بين الوقاية والعالج عند تخصيص الموارد الصحية‪:‬تلميح‬
‫‪.‬عادة بسبب تفضيالت أعضاء المجتمع األقوياء‬
‫تعزيز الصحة‪:6‬الحالة‬

‫أقنع عدد كبير من المواطنين الذين أرادوا ممارسة الرياضة البدنية عن طريق المشي وركوب الدراجات‪ ،‬المدينة بتخصيص ميزانية ‪:‬السيناريو‬
‫ولم يكن بعض المواطنين سعداء بالتخصيص الضخم للميزانية‪ ،‬قائلين إن الفائدة‪.‬كبيرة لبناء ممرات لراكبي الدراجات والمشاة‬
‫‪.‬غير واضحة‬
‫توضح هذه الحالة الشكوك المعتادة لدى الجمهور فيما يتعلق ببرامج تعزيز الصحة ألن نتائجها اإليجابية ليست فورية وال يمكن‪:‬تلم يح‬
‫قياسها‬
‫الجدل حول تمنيع ‪ :7‬الحالة ‪.‬بسهولة‬

‫األطفال‬
‫اصطحاب أطفالهم لتلقيح ضد الحصبة‪ ،‬بحجة أن اآلثار الجانبية للقاح )بما في ذلك األطباء( في حي غني بالمدينة‪ ،‬رفض المهنيون ‪:‬السيناريو‬
‫تم تشخيصهم في مجتمعهم ‪ .‬كانت أعلى بكثير من مخاطر اإلصابة بالحصبة‪ ،‬والتي لم تكن كذلك‪ -‬على الرغم من انخفاضها بشكل عام ‪-‬‬
‫كما جادلوا بأن أطفالهم ذهبوا إلى مدارس األحياء‪ ،‬ولم يختلطوا مع أطفال من مناطق كانت‪.‬خالل السنوات العشر الماضية‬
‫‪.‬الحصبة متوطنة فيها‬
‫إذا كان الجميع يفكر فقط في مصالحه ‪ .‬توضح هذه الحالة الموقف غير األخالقي المتمثل في وضع المصالح الشخصية على المصالح العامة‪:‬تلميح‬
‫في هذه الحالة‪ ،‬قد تكون النتيجة انخفاض كبير في مناعة القطيع مما يؤدي في النهاية إلى‪.‬الخاصة‪ ،‬فإن المجتمع بأسره سيعاني‬
‫‪.‬انتشار وبائي للعدوى‬
‫المنفعة‪ vs‬االستقاللية ‪:8‬الحالة‬

‫أضعاف في تشخيص سرطان القولون في المرحلة المتأخرة التي كانت قاتلة في غضون عام‪ ،‬أصدر مجلس ‪ 3‬مع مالحظة زيادة ‪:‬السيناريو‬
‫سنوات‪ ،‬مع تحذير‪5‬المدينة قرا ًرا يطالب جميع الرجال والنساء فوق سن األربعين بالخضوع لفحص القولون بالمنظار مرة كل‬
‫ًا إذا أصيبوا بالسرطان‬‫‪.‬أولئك الذين لن يتم عالج رفض الفحص مجا ن‬
‫توضح هذه الحالة إحباط مسؤولي الصحة العامة‪ ،‬الذين ال يستطيعون تنفيذ تدابير وقائية فعالة قائمة على األدلة بسبب رفض‪:‬تلميح‬
‫‪ .‬اإلكراه ليس أخالقيا في هذه الحالة‪.‬السكان‬
‫المعاناة من النتائج السلبية الخاطئة‪ vs‬فوائد الفحص المبكر للمرض ‪:9‬الحالة‬

‫خالل العام‪ ،‬تم العثور ‪ smear .pap‬وما فوق ‪ 20‬أدخل مجلس المدينة الفحص الشامل اإلجباري لعنق الرحم لجميع النساء في سن ‪:‬السيناريو‬
‫ولكنهن سلبيات في‪smear pap‬أي النساء الالئي كن إيجابيات في اختبار عنق الرحم (‪20 ٪‬على معدل إيجابي كاذب بنسبة‬
‫‪ .‬قادت وسائل اإلعالم حملة لوقف برنامج الفحص ألنه وجد أن عينة عنق الرحم مرتبطة بالعديد من المشاكل األخرى)‪.‬العينة التوكيدية‬
‫هناك عدد كبير جدا من النتائج السلبية الخاطئة تفوق فوائد الفحص‪ .‬النتائج السلبية الكاذبة أمر ال مفر منه‪ ،‬ولكن يجب التقليل منه‪:‬تلميح‬
‫وستؤدي إلى حدوث ذلك‬
‫الفردية واالستقاللية‪ vs‬المصالح العامة ‪:10‬الحالة‬

‫اعترض المواطنون‪ .‬أمرت وزارة الصحة بتلقيح جماعي ضد وباء فيروس األنفلونزا الجديد الذي انتشر للتو من دولة مجاورة‪:‬السيناريو‬
‫‪.‬على التطعيم دون موافقتهم‬
‫في هذه الحالة‪ ،‬يكون التطعيم اإلجباري مبر ًرا‪ .‬المصلحة العامة لها األسبقية على المصلحة الخاصة‪ ،‬وستتجاوز االستقاللية الفردية‪:‬تلميح‬

‫‪PLEAZE, ANY NOTE OR CORRECTION OR ADDITIN SEND:‬‬ ‫‪[email protected]‬‬


‫نسألكم الدعاء بظهر الغيب‬
P a g e | 225
‫إذا كان هناك دليل علمي قوي على فعاليته‬.

PLEAZE, ANY NOTE OR CORRECTION OR ADDITIN SEND: [email protected]


‫نسألكم الدعاء بظهر الغيب‬
‫‪P a g e | 226‬‬

‫حق المرضى في الخصوصية والسرية‪ vs‬اإلفصاح لحماية اآلخرين ‪:11‬الحالة‬

‫بحث الطبيب على الفور عن رقم هاتف‪ .‬وجد طبيب يفحص مواط ًنا عائ ًدا من الخارج إصابته بفيروس نقص المناعة البشرية‪:‬السيناريو‬
‫‪ .‬ذهبت الزوجة إلى المحكمة وطلبت الطالق‪.‬منزل الرجل في دليل الهاتف وأبلغ الزوجة باتخاذ اإلجراءات لتجنب العدوى‬
‫خطأ الطبيب هو الطريقة المتبعة والتي ‪ .‬على الطبيب واجب حماية الزوجة واألسرة وهذا الواجب ينطوي على اإلخالل بالسرية‪:‬تلميح‬
‫ذات الخبرة في تعقب المخالطين لألمراض‪ authorities‬كان من المفترض أن يخطر السلطات‪.‬انتهى بها األمر إلى طالق ال داعي له‬
‫‪.‬المعدية حتى يتمكنوا من التعامل مع األمر بشكل احترافي‬
‫االستقالل الفردي‪ vs‬المصلحة العامة ‪:12‬الحالة‬

‫سيمنع إدمان السجائر عن طريق إنتاج أجسام مضادة تمنع‬ ‫طور مركز أبحاث جامعي لقا‪:‬السيناريو حا‪ ،‬إذا تم إعطاؤه للمراهقين‪،‬‬
‫ًا لجميع األوالد‪.،‬النيكوتين من عبور الحاجز الدموي الدماغي‬ ‫ًا للفائدة المحتملة‪ ،‬أصدرت وزارة الصحة قانو ن‬
‫ًا يجعل التطعيم إلزام ي‬ ‫إدرا ك‬
‫‪.‬ولكن ليس للفتيات‬
‫على الرغم من أن التطعيم يفيد الجمهور من خالل وقف اإلدمان‪ ،‬إال أنه ينتهك االستقاللية الفردية‪ ،‬وخاصة أولئك الذين لن‪:‬تلميح‬
‫‪ .‬تشكل األجسام المضادة تغي ًرا دائ ًما في الجسم يمكن أن يكون له تأثيرات أخرى غير معروفة حتى اآلن‪.‬يتعرضوا للمخدرات‬
‫المنفعة االجتماعية‪ vs‬الضعف ‪ :‬تحديد أولويات الخدمات في حالة الطوارئ‪:13‬الحالة‬

‫عقد مجلس المدينة اجتماعا لتحديد األهداف‪ .‬تسرب غاز سام من مصنع في المدينة ولم تتوفر سوى كميات محدودة من الترياق‪:‬السيناريو‬
‫‪ .‬تم اتخاذ القرار بإعطاء الجرعات المتاحة للعاملين الصحيين في حاالت الطوارئ فقط‪.‬ذات األولوية للترياق‬
‫األطفال ( هل نعطي األولوية ألضعف أفراد المجتمع وأكثرهم ضعفًا ‪ :‬توضح هذه الحالة معضلة مشتركة دون حل سهل‪:‬تلميح‬
‫إن )‪.‬العاملون في مجال الرعاية الصحية واألمن وما إلى ذلك( أو أولئك الذين يحتاجون إلى العمل في حاالت الطوارئ )والحوامل وكبار السن‬
‫رد‬
‫‪.‬الفعل العاطفي سيفضل الضعف‪ ،‬في حين أن رد الفعل العقالني سيفضل المنفعة‬
‫الدفع ألنماط الحياة غير الصحية المسببة لألمراض‪:14‬الحالة‬

‫اعترض المدافعون عن مكافحة التدخين على تغطية األمراض ‪ .‬أدخلت الحكومة التأمين الصحي الشامل المجاني لجميع المواطنين‪:‬السيناريو‬
‫المرتبطة بالتدخين‪ ،‬قائلين إن على المدخنين دفع تكاليف رعايتهم الصحية ألنهم ع ًرضوا أنفسهم عم ًدا ألسلوب حياة محفوف‬
‫‪.‬بالمخاطر‬
‫ال يمكن الدفاع عنه أخالقيا ً بسبب تطبيقه االنتقائي على المدخنين وليس على "خطاياهم" إن مبدأ معاقبة ضحايا المرض على ‪:‬تلميح‬
‫النهج األخالقي‪ .‬على سبيل المثال‪ ،‬اإلفراط في تناول الطعام‪ ،‬وعدم ممارسة الرياضة‪ ،‬وسوء نظافة الفم‪ ،‬وما إلى ذلك‪:‬الفئات األخرى‬
‫‪.‬سيكون لخطة التأمين أن تعطي حوافز أ لولئك الذين أقلعوا عن التدخين عن طريق تخفيض أقساطهم‬
‫الشابتر الرابع عشر‬
‫‪1‬الحالة‬

‫عا ًما إلى قسم الطوارئ في منتصف الليل ولديها تاريخ من آالم أسفل البطن اليمنى الشديدة لمدة ثالثة ‪30‬تأتي امرأة تبلغ من العمر‬
‫تطلب أن تتم رؤيتها من قبل طبيبة في قسم الطوارئ حيث ال توجد إناث؛ تصرخ المريضة أن ‪.‬أيام مع قيء وحمى منخفضة الدرجة‬
‫تفحصها‬
‫كيف نتعامل مع هذا األمر؟‪.‬طبيبة فقط‬
‫‪ :‬هذه حالة طوارئ حقيقية مع قضايا أخالقية)ج‬
‫‪.‬هذا ألم شديد في البطن مصحوب بالقيء‪ ،‬لذا فهو حالة طارئة حقيقية ويحتاج إلى تدخل طارئ عاجل‬
‫‪ :‬هناك قضيتان أخالقيتان طارئتان‪ ،‬وهما كالتالي‪-‬ب‬
‫‪ .‬تحتاج إلى فحصها من قبل طبيبات ولكن ال يوجد‪.1‬‬
‫‪PLEAZE, ANY NOTE OR CORRECTION OR ADDITIN SEND:‬‬ ‫‪[email protected]‬‬
‫نسألكم الدعاء بظهر الغيب‬
‫‪P a g e | 227‬‬
‫‪ .‬تصرخ وال تحترم طاقم الطوارئ‪.2‬‬

‫يجب على طاقم الطوارئ أن يشرح للمريضة فيما يتعلق أن حالتها هي حالة طوارئ حقيقية‪ ،‬وهي منتصف الليل‪ ،‬وال توجد موظفات‬
‫المسألة األخرى هي أنه يجب على طاقم قسم الطوارئ أن ‪ .‬لذلك‪ ،‬يجب فحصها من قبل طبيب متمرس من أجلها‪ ،‬ألنها حالة طارئة‪.‬متاحات‬
‫يجب أن يحترم طاقم قسم الطوارئ‪.‬يشرح لها ولقريبها أن الدين اإلسالمي يقبل أن الذكور يمكنهم فحص اإلناث في حالة الطوارئ‬
‫يجب أن يخبرها أطباء قسم الطوارئ الطبية أنه إذا أصبحت حالتها مستقرة‪.،‬أيضا المريضة عندما تصرخ ألنها تعاني من األلم والقلق‬
‫‪.‬فيمكنها الذهاب إلى مستشفى آخر حيث يمكن أن تعالج من قبل موظفات‬
‫‪2‬الحالة‬

‫اثنان منهم في ‪.‬تعرض ثالثة رجال لحادث مروري مؤخ ًرا وتم إحضارهم إلى منطقة الفرز في قسم الطوارئ‪ ،‬بدون مالبس للخصوصية‬
‫عا ًما يعاني من صعوبة في التنفس ونزيف حاد‪25‬رجل يبلغ من العمر (حالة مستقرة مع إصابات طفيفة وكدمات‪ ،‬والثالث في حالة سيئة‬
‫ظا بالمرضى اآلخرين ولم تكن هناك أسرة متاحة‬ ‫كيف‪.‬‬ ‫عندما تم إحضار الرجال إلى قسم الطوارئ‪ ،‬كان مكت)‪.‬وانخفاض ضغط الدم‬
‫يمكنك التعامل مع هذا الوضع؟‬
‫‪ .‬كان قسم الطوارئ مزدح ًما‪ ،‬ولم تكن هناك أسرة متاحة‪ ،‬ولم تكن هناك خصوصية للرجال‪ .‬هذه الحالة تمثل سيناريو طارئ حقيقي)ج‬
‫‪: .1‬هناك بعض القضايا األخالقية وهي كالتالي‬
‫توافر األسرة لحاالت الطوارئ‬
‫أولوية فرز الحالة ‪.2‬‬
‫خصوصية هؤالء ‪.3‬‬
‫المرضى‬
‫‪:‬على طبيب الطوارئ المسؤول وموظفيه التعامل بسرعة مع هذا الموقف بطريقة مهنية‪ ،‬باتباع الخطوات التالية‬
‫‪ .‬يجب عليهم أيضا طمأنتهم بشأن أوضاعهم وخصوصياتهم‪ .‬يجب على طاقم الطوارئ فرز المرضى بسرعة وبخصوصية تامة‪.1‬‬
‫ضا إعادة ترتيب موارد األسرة‬ ‫يجب على طاقم قسم الطوارئ أي‪ .‬يجب أن تعالج الحالة الثالثة بسرعة كبيرة ويتم قبولها بعناية كاملة‪.2‬‬
‫‪.‬في قسم الطوارئ‬
‫‪ .‬يجب أن يحتوي قسم الطوارئ على سرير خاص محجوز‪ ،‬والذي يبقى شاغرا ً في حاالت الطوارئ الخطيرة‪.3‬‬
‫الشابتر الخامس عشر‬
‫‪1‬الحالة‬

‫سنوات‪ ،‬وهي اآلن حامل في األسبوع الخامس ‪ 10‬و‪ 8‬وهي أم لطفلين يبلغان من العمر ‪ .‬عا ًما‪36‬بتول سيدة سعودية تبلغ من العمر‬
‫‪ ،‬ووجد أنها) سم‪ 19( × 12‬قبل أسبوعين‪ ،‬تم اكتشاف كتلة ضخمة في المبيض ‪.‬عشر من الحمل بجنين طبيعي وقادر ع لى الحياة‬
‫نظ ًرا ألن المريضة مرشحة للعالج الكيميائي‪ ،‬أوصى مجلس األورام في المستشفى بإنهاء‪.‬سرطانية مع مالمح ثانويات ورم خبيث‬
‫‪ .‬ومع ذلك‪ ،‬لم تقبل المريضة إنهاء الحمل‪ .‬وافق ثالثة استشاريين‪ ،‬بمن فيهم طبيب التوليد واألورام التالي لها‪ ،‬على هذه التوصية‪.‬الحمل‬
‫‪ .‬شعرت بتول بضيق شديد لما حدث‪ ،‬ورفضت بدء العالج الكيميائي‪.‬وبنا ًء عليه‪ ،‬تم االقتراب من الزوج؛ وافق ووقع الموافقة نيابة عنها‬
‫تم رفع الحالة إلى لجنة األخالقيات بالمستشفى‬
‫‪ .‬على مستوى أهداف الشريعة‪ ،‬يبدو إنهاء الحمل مواتيا ً‪ ،‬إذا تم منح موافقتها طواعية )ج‬
‫‪.‬ما لم يتم إعطاء الموافقة الطوعية‪ ،‬ال ينبغي تنفيذ اإلجراء‬
‫أي تركها دون عالج حتى الوالدة‪ ،‬أو إنهاء في هذه(يحتاج الفريق المعالج إلى توفير المزيد من االحتماالت الصحيحة لكال الخيارين‬
‫من ضرر ترك األم دون عالج )التسامح(إن ضرر اإلنهاء أكثر تبري ًرا ‪) .‬المرحلة‬
‫يرجع رفض المريض على األرجح إلى الطريقة التي تم بها‪ .‬نتوصل إلى نتيجة مفادها أن إنهاء الحمل في هذا العمر الحملي مسموح به‪.‬‬
‫‪.‬اتخاذ القرار‪ ،‬وليس القرار نفسه‬
‫ًا إلى مهارات االتصال الالزمة لنقل خيارات العالج إلى مرضاهم‬ ‫بما أن المريضة مؤهلة التخاذ القرار‪ ،‬فإن تجاوزها‪.‬يفتقر األطباء أحيا ن‬
‫‪.‬للحصول على موافقة زوجها أمر غير مقبول أخالقيا ً ومهنيا ً‬

‫‪PLEAZE, ANY NOTE OR CORRECTION OR ADDITIN SEND:‬‬ ‫‪[email protected]‬‬


‫نسألكم الدعاء بظهر الغيب‬
‫‪P a g e | 228‬‬
‫‪ .‬قد يكون للزوج أسباب أخرى إلنهاء هذا الحمل غير طلب عالج زوجته‪.‬عالوة على ذلك‪ ،‬قد يكون األمر مزعجا من الناحية القانونية‬
‫‪ .‬وطلب المساعدة المناسبة إذا شعرنا أننا نفشل في التواصل مع المريض)التحمل(وبالتالي‪ ،‬نحتاج إلى توخي الحذر والصبر‬

‫الشابتر السادس عشر‬

‫‪1‬حالة‬

‫أرادت‪.‬كان زوجان يعانيان من العقم في خضم إجراء التلقيح االصطناعي عندما توفي الزوج بعد فترة وجيزة من تجميد السائل المنوي‬
‫أرادت الزوجة السابقة أي‪.‬الزوجة الحصول على السائل المنوي وإنجاب طفل من أم بديلة ضا السائل المنوي ألن لديها فتاة مصابة‬
‫‪.‬بسرطان الدم وتحتاج إلى متبرع بنخاع عظم متوافق‪ ،‬ويفضل أن تكون أختًا‬
‫‪ .‬األبوة خارج إطار الزواج غير مسموح بها‪:‬تلميح‬
‫‪2‬الحالة‬

‫أراد زوجها أن تتوقف زوجته عن‪.‬واصلت امرأة متزوجة حديثًا تناول موانع الحمل الفموية الموصوفة لعدم انتظام الدورة الشهرية‬
‫‪.‬استخدام وسائل منع الحمل ألنه أراد تكوين أسرة على الفور‪ ،‬لكن الزوجة رفضت‬
‫‪ .‬الموافقة المتبادلة مطلوبة لمنع الحمل‪:‬تلميح‬
‫‪3‬الحالة‬

‫كانت‪ .‬كانت حامالً في شهرين مما زعمت أنه اغتصاب‪ .‬عا ًما إلى المستشفى إلجراء عملية إجهاض‪14‬تم إدخال فتاة تبلغ من العمر‬
‫‪ .‬كان األطباء مترددين لعدم وجود سبب طبي‪.‬األسرة في حالة ذهول وأرادت من األطباء إجراء اإلجهاض على الفور‬
‫‪ .‬الحفاظ على الحياة له األسبقية على االعتبارات األخرى‪:‬تلميح‬
‫‪4‬الحالة‬

‫قرروا تجربة التلقيح االصطناعي مع‪.‬كان الزوجان اللذان أنجبا ثماني فتيات في حاالت الحمل المتتالية يتمنون بشدة إنجاب صبي‬
‫‪ .‬رفض أطباء التوليد لعدم وجود مؤشر طبي‪ ،‬حيث لم يكن للزوجين مشكلة في الحمل‪.‬مجموعة مختارة من األمشاج الذكور‬
‫‪ .‬فائدة الجنس المرغوب‪ VS‬قارن مخاطر التلقيح االصطناعي ‪:‬تلم يح‬

‫‪5‬الحالة‬

‫كانت الغدد‪ .‬عا ًما بسبب تأخر الدورة الشهرية‪14‬تم نقل الطفل الذي كان مظهره الخارجي أنثى والذي نشأ كفتاة إلى المستشفى في سن‬
‫‪ .‬أراد الوالدان إجراء عملية تغيير الجنس لتتوافق مع الملف الجيني‪.‬التناسلية الداخلية واألنماط الصبغية من الذكور‬
‫‪.‬رفضت الطفلة تغيير هويتها األنثوية المألوفة‬
‫حق االستقاللية واالختيار للقاصر المختص‪:‬تلميح‬
‫‪6‬الحالة‬
‫رفض الطبيب ألنه رأى أن‪.‬طلبت امرأة في منتصف العمر ال تعاني من أي حالة طبية من طبيبها العالج الهرموني ليبدو أصغر سنا ً‬
‫‪.‬مخاطر اإلصابة بمضاعفات القلب واألوعية الدموية والسرطان تفوق الفوائد‬
‫‪ .‬ضع في اعتبارك المخاطرة مقابل الفائدة‪:‬تلميح‬
‫‪7‬الحالة‬

‫ًا بإزالة بويضاتها ووضعها في مخزن بارد طوال ‪14‬نصحت فتاة تبلغ من العمر‬ ‫عا ًما مصابة بالسرطان وتتطلب عال ًجا كيميائ ي‬
‫فترة‬
‫‪ .‬رفض والداها اإلجراء ألنهما ال يؤمنان بالتلقيح االصطناعي ولم تكن متزوجة بعد‪.‬العالج‬
‫‪ .‬ضع في اعتبارك المخاطرة مقابل الفائدة‪:‬تلميح‬

‫‪PLEAZE, ANY NOTE OR CORRECTION OR ADDITIN SEND:‬‬ ‫‪[email protected]‬‬


‫نسألكم الدعاء بظهر الغيب‬
‫‪P a g e | 229‬‬
‫‪MODULE 8 - PRIVACY AND CONFIDENTIALITY‬‬

‫‪ .‬عاملهم كما تحب األم أو األخت أو األب في العالج‪.‬في الختام‪ ،‬عليك أن تتذكر أن المرضى الذين تعالجهم هم أمهاتكم وآباؤكم وأخواتكم‬
‫تحتوي ‪ .‬لقد كان هذا وستكون دائ ًما قضية نزاهة وثقة‪ .‬ال تجعل تعرضك المتكرر ألجساد الناس عادة‪.‬احترم أجساد المرضى‬
‫ال تختلق االعذار بالقول إن المستشفى الذي تعمل فيه ليس لديه سياسات لهذه ‪.‬المبادئ التوجيهية على تدابير معقولة ومجدية لمعالجة هذه المسألة‬
‫عدم االلتزام بهذه ‪ .‬يمكن التحكم فيها ويمكن تنفيذها حتى من قبل األطباء األفراد ‪ -‬إن لم يكن معظمها ‪-‬األمور القضايا؛ العديد من التدابير‬
‫ًا ألن جز ًءا من واجباتك هو أوال وقبل كل شيء تفعل الخير من قبل مرضاك‪ ،‬وحمايتهم منها ضرر‬ ‫باإلضافة إلى‪.‬اإلجراءات خطأ أخالق ي‬
‫ذلك‪ ،‬فإنه يعرضك لخطر الوقوع في مشكلة قانونية‬

‫تتعلق الخصوصية باحترام جسد المريض‪ ،‬بينما تتعلق السرية حول حماية المعلومات الطبية لمريضك أنت‬ ‫‪‬‬
‫بحاجة إلى المعرفة والممارسة وفقًا لمدونة تساعدك حماية خصوصية المريض وسريته‬ ‫‪‬‬
‫االلتزام بهذه اإلجر اءات سوف يساعد في بناء ثقة المجتمع في نظام الرعاية الصحية والتزاماته األخالقية والدينية وال‬ ‫‪‬‬
‫حاالت استثنائية يمكنك فيها اإلفصاح ضمن حدود بعض أو كل المعلومات الطبية السرية‬ ‫‪‬‬
‫التعرف على السياسات واإلجراءات ذات الصلة في مجال الصحة مؤسسة الرعاية التي تعمل فيها ‪ ،‬باإلضافة إلى االعتبارات‬ ‫‪‬‬
‫‪.‬الثقافية‬ ‫‪‬‬
‫‪1‬الحالة‬

‫عند عودته إلى المملكة العربية السعودية‪ ،‬فضل أن يتدرب في منطقة ‪.‬الدكتور مان طبيب توليد أنهى تدريبه مؤخ ارا في كندا‬
‫عا اما بريما حبيدا أتت إلى عيادته بحجابها ‪ 18‬في أسبوعه األول‪ ،‬كانت امرأة تبلغ من العمر ‪.‬هامشية بالقرب من مسقط رأسه‬
‫كانت المرأة الحامل في الثلث األول من حملها‪ ،‬واشتكت آالم أسفل البطن ونزيف‪ .‬فوق عبايتها برفقة امها)خمار(الكامل‬
‫سألت األم عليه أن تعرض ابنتها على طبيبة‪ ،‬ويطلب منه‪ .‬أخذ الطبيب تاريخا قصي ارا من األم‪ ،‬ثم أراد أن يبدأ فحصه‪.‬مهبلي‬
‫كان الطبيب جدا قلقاا من أن الحالة قد تكون خطيرة وقد ال يكون لديه الوقت الكافي لذلك ‪.‬ذلك انتظر الزوج قبل أن تلمس ابنتها‬
‫ثم طلب من المرأة ‪ .‬أخبر األم أنه الطبيب المختص الوحيد متوفرة‪ .‬تحت تدريبه)‪ (R3‬الطبيبة الوحيدة المتوافرة كانت مقيمة ‪.‬انتظر الزوج‬
‫هو طلب ‪ .‬كانت تتألم وقالت شيء باللهجة المحلية ألمها لم يفهمه جيداا"‪.‬فحص خاص"الحامل أن تكشف وجهها وسأل إذنها إلجراء‬
‫غادرت األم العيادة وعادت خمس دقائق الحقاا ‪.‬من األم االتصال بالممرضة من مكتب الممرضات حيث كان مشغوال وقف النزيف‬
‫بعد بضع دقائق‪ ،‬وصل الزوج وكان مستا اءا ‪ .‬تمكن الطبيب من الحفاظ على العالمات الحيوية والتوقف النزيف ‪.‬مع الممرضة‬
‫لحسن ‪".‬سيتقدم بشكوى ضد المستشفى "جداا من زوجته تم فحصه من قبل طبيب ذكر؛ بدأ بالصراخ وهدد قال المدير الطبي إنه‬
‫الحظ‪ ،‬أوقفه أمن المستشفى قبل أن يتمكن من الوصول إلى العيادة‪ ،‬حيث قد يكون قد أضر بالطبيب‬
‫‪.‬جسد ياا‬

‫الحل‬
‫‪.‬حالة الدكتور مان ليست نادرة‪ ،‬ولكن أيضا القضايا الثقافية والدينية والقانونية المتعلقة بالسيناريو‬
‫ومع ذلك‪ ،‬هذا ال يكفي عادة لتبرير المزيد من‪.‬أوال‪ ،‬على ما يبدو لدى الطبيب نوايا حسنة لمساعدة المريض وحمايته منها ضرر‬
‫فشل الطبيب في الحصول على هذا اإلذن‪ ،‬ربما ‪.‬التدخالت دون موافقة واضحة‪ ،‬أو على األقل إذن‪ ،‬من المريض أو منها بديل صانع القرار‬
‫قد يكون هذا مبر ًرا في الظروف التي تهدد الحياة حيث ال‪.‬ألنه أعطى األولوية إلنقاذ حياة المريض بدالً من الحصول على الموافقة‬
‫كان المريض على ما يبدو مؤهالً؛‪.‬يوجد أحد يمكن الوصول إليه بسهولة وبسرعة التخاذ القرار بشأنه نيابة عن المريض فاقد الوعي‬
‫يجب ‪.‬لذلك‪ ،‬كان من الممكن أن ينتظر الطبيب وقتًا معقوالً بعد تثبيت الحالة العامة قبل متابعة المزيد من التدخالت غير المنقذة للحياة‬
‫ًا ‪ ،.‬بدون وجود المربية أو األم"خاص"أال يكون الطبيب قد بدأ في أي فحص جسدي‪ ،‬خاصةً أ‬ ‫هذا من الناحية القانونية‪ ،‬دين ي‬
‫ًا‬
‫هو كما أساء تقديم ‪ .‬من ا لناحية القانونية‪ ،‬كشف نفسه مزاعم سوء المعاملة أو المضايقة‪ ،‬حيث لم يكن هناك شاهد في الغرفة‪.‬وثقاف ي‬
‫المعلومات‬
‫من الناحية ‪.‬المتعلقة بوجود أنثى أخرى طبيب‪ ،‬حيث كان هناك واحد متاح مؤهل بما يكفي لإلدارة القضية‪ ،‬حتى لو كانت تحت إشرافه‬
‫الدينية‪ ،‬في اإلسال م هذا الشرط ال يوفر عذرا مطلقا للطبيب للبقاء بمفردها مع مريضة غير مصحوبة بفضح العورة فيها يعتبر السؤال خيانة‬
‫هذا‪ .‬أخي ًرا‪ ،‬ما فعله كان غير حساس للثقافة وأعراف المجتمع الذي يعمل فيه ‪ .‬عذر الحاجة ال يبدو أنه يساعده هنا‪.‬كبيرة للثقة‬
‫السلوك لن يؤثر فقط على سمعته كطبيب فردي‪ ،‬ولكن قد تتسبب أيضا في انتشار الشائعات بأن المستشفى يجبر النساء على ذلك فحص‬
‫قد يؤثر ذلك على عنصر الثقة؛ وهكذا‪ ،‬كثير يجوز لألزواج منع زوجاتهم من الذهاب إلى المستشفى أو البحث المشورة ‪.‬من قبل الرجال‬
‫الجمهور اآلثار الصحية المترتبة على مثل هذه المواقف ال يتم عالجها‪.‬الطبية‪ ،‬حتى لزيارات الرعاية السابقة للوالدة أو التطعيمات‬
‫‪.‬بسهولة أو بسرعة‪ ،‬ويجب أخذها في االعتبار قبل االنخراط في أي من هذه السلوكيات‬

‫‪PLEAZE, ANY NOTE OR CORRECTION OR ADDITIN SEND:‬‬ ‫‪[email protected]‬‬


‫نسألكم الدعاء بظهر الغيب‬
‫‪P a g e | 230‬‬
‫‪How privacy and confidentiality differ1‬‬

‫ًا في كثير من البلدان التعليم الذي تلقيناه"السرية" و "الخصوصية"تم استخدام مصطلحي‬ ‫‪ .‬وبالتالي‪ ،‬قد يعتقد الكثيرون أنها مترادفة ‪ .‬م ع‬
‫أي ( من الناحية العملية‪ ،‬تتعلق الخصوصية بالمكان الذي يتم من خالله أخذ المعلومات الطبية الخاصة بالمريض ‪.‬في الواقع‪ ،‬انهم ليسوا كذلك‬
‫تشمل السرية جميع ‪( .‬أي المريض المعلومات( حول المريض ‪ ،/‬بينما تتعلق السرية بالمعلومات التي يتم جمعها من )جسم المريض‬
‫سواء كانت مكتوبة‪ ،‬محوسب أو مرئي أو صوتي أو مسجل أو محفوظ ببساطة في ذاكرة المهنيين‪.‬معلومات المريض التي يمكن تحديدها‬
‫صحة الفرد في الماضي أو الحاضر أو المستقبل الجسدية أو العقلية أو شرط؛‪:‬يغطي ‪.‬الصحيين‪ ،‬هذه المعلومات تخضع لواجب السرية‬
‫الصور أو الصور الفوتوغرافية أو مقاطع الفيديو أو األشرطة الصوتية أو‪‬أي معلومات إكلينيكية عن التشخيص أو العالج للفرد؛‬
‫أي شيء آخر يمكن‪.‬غيرها من المواد الخاصة بـ مريض؛ من هو طبيب المريض وما هي العيادات التي يحضرها المرضى ومتى‬
‫استخدامه لتحديد هوية المرضى بشكل مباشر أو بشكل غير مباشر؛ المدفوعات السابقة أو الحالية أو المستقبلية لتوفير الرعاية الصحية‬
‫للفرد‬
‫‪Measures to protect patients‟ privacy‬‬

‫فيما يلي ملخص للتدابير التي يتعين على مقدمي الرعاية الصحية‪.‬من المهم فهم التدابير المختلفة الالزمة لحماية خصوصية المرضى‬
‫‪:‬اتباعه من أجل حماية خصوصية مرضاهم‬

‫أو الموظفين‪/‬تأكد من إجراء جميع الفحوصات البدنية بمعزل عن اآلخرين المرضى وأفراد األسرة غير المصرح لهم و ‪‬‬

‫توفير غرف انتظار وفحص تراعي الفوارق بين الجنسين توفير المالبس المناسبة للمرضى المنومين‪‬‬

‫تأكد من تغطية المرضى جي ًدا عند نقلهم من مكان واحد آلخر في المستشفى‪‬‬

‫تأكد من أن جسم مريضك مكشوف فقط بالقدر الذي تحتاجه الفحص أو التحقيق‪‬‬

‫يجب أن يكون لدى المرضى مصاعد منفصلة وأن تعطى األولوية‪‬‬

‫من نفس الجنس مثل حاضر المريض طوال أي فحص)ممرضة(تأكد من وجود شخص آخر ‪‬‬

‫خذ دائ ًما اإلذن من المريض قبل البدء في أي منها فحص‪‬‬


‫تجنب إبقاء المرضى لفترات أطول مما تتطلبه إجراء‪-‬‬ ‫ضمان الخصوصية عند أخذ المعلومات من المرضى‪‬‬

‫يمنع فحص المريض في الممرات أو مناطق االنتظار‪‬‬

‫ًا لفضح الجزء‪‬‬


‫أثناء الفحص‪ ،‬يجب أال يكون هناك أي شخص غير ذي صلة بالمستشفى يسمح لها بالحضور امنح المريض وقتًا كاف ي‬
‫المصاب باأللم‬

‫يسمح فقط للموظفين المعنيين بالدخول إلى غرفة الفحص في في أي وقت أثناء الفحص‪‬‬
‫‪‬‬

‫‪Measures to protect the confidentiality of your patients‟ information‬‬

‫أو فريق( أي معلومات يمتلكها يعرف الطبيب )الطبي( يتضمن سر الكفاءة ‪.‬حا بشأن ماهية سرية مرضاك معلومة‬ ‫يجب أن تكون واض‬
‫‪ ،‬بشكل مباشر أو بشكل غير مباشر من خالل امتياز مكانتهم المهنية‪ ،‬واإلفصاح عنها أ قد يعتبر)حيا أو ميتا( عن المريض )العالج‬
‫يتضمن أي معلومات حول هوية‪ .‬سمعتها‪ ،‬الوضع المالي أو االجتماعي أو المهني ‪/‬المريض غير مرغوب فيه أو ضار بصحته‬
‫فيما يلي بعض التدابير ‪) .‬سواء كانت فرص الشفاء أو العجز أو الوفاة( أو التكهن ‪/‬المريض‪ ،‬الحالة‪ ،‬التشخيص‪ ،‬التحقيقات‪ ،‬النتائج‬
‫تحتاج إلى اتباعها عند التعامل مع المرضى المعلومات)بمساعدة من مؤسسة(الرئيسية التي يمكنك‬
‫‪Conditions to disclose identifiable medical information‬‬

‫المبادئ التوجيهية السعودية‪ ،‬والتي تشبه إلى حد بعيد المبادئ التوجيهية لكثير من اآلخرين وقد وضعت الدول العربية واإلسالمية‬
‫فيما يلي الشروط‪.‬بعض الشروط التي يجوز بها ذلك للكشف عن معلومات سرية حول مريضك‪ ،‬حتى ضده سوف‬
‫‪:‬الرئيسية؛ سيتم استكشافها وتمت مناقشته بمزيد من التفصيل في قسم الحق من هذه الوحدة‬

‫‪PLEAZE, ANY NOTE OR CORRECTION OR ADDITIN SEND:‬‬ ‫‪[email protected]‬‬


‫نسألكم الدعاء بظهر الغيب‬
‫‪P a g e | 231‬‬
‫موافقة المريض أو من ينوب عنه في القرار الحد المعطى في الموافقة‪‬‬
‫إذا كانت المعلومات مطلوبة من قبل القضاء‪‬‬
‫استشارة أو رأي ثان‪‬‬
‫الوالدة‪ ،‬الوفاة‪ ،‬األمراض التي يجب اإلبالغ عنها‪ ،‬وما إلى( التهديدات ‪ /‬اإلخطار باألحداث التي تهم الصحة العامة ‪‬‬
‫ذلك‬
‫‪ ،‬إلخ)‪ ،(STIs‬الجرائم الجنسية العدوى المنقولة)على سبيل المثال( الشخصية ‪ /‬لمنع التهديدات الفردية ‪‬‬
‫إذا احتاج الطبيب للدفاع عن نفسه أمام القضاة‪ ،‬أو أ لجنة االنضباط‪‬‬
‫الهيئة السعودية للصحة( ” أن يفشي الطبيب بعض السر أو السر كله إذا رأت هذا ضروري لشفاء المريض‪‬‬
‫‪ ،.‬أو بطريقة تصب في مصلحة المريض‪)2003‬التخصصات‬

‫‪MODULE 7 – PATIENT AUTONOMY AND CONSENT TO TREATMENT‬‬

‫‪1‬الحالة‬

‫اعترضت ‪ .‬عا اما‪ ،‬واعي تما اما‪ ،‬وكفؤ مع تقدم ال شفاء منه يحتاج السرطان إلى عالج كيميائي ملطف‪80‬رجل يبلغ من العمر‬
‫األسرة عند الطبيب أراد الحصول على موافقة مستنيرة من المريض ألن ذلك من شأنه ينطوي على الكشف عن التشخيص‪ ،‬مما يجعل‬
‫ماذا يجب أن يفعل الطبيب؟ قدم تفكيرك‪ .‬أرادت األسرة اتخاذ القرار دون إبالغ مريض‪.‬المريض حزيناا جداا ومحبط‬
‫)ج‪.‬األخالقي‬
‫يجب أن يسأل أوال المريض سواء أراد شخصيا ا تلقي معلومات عنه من أجل ‪.‬يجب على الطبيب احترام استقاللية المريض‬
‫اتخاذ قرارات بشأن عالجه‪ ،‬أو ما إذا كان سيفعل ذلك وإن كان هو يفضل أن يتم الكشف عن المعلومات لعائلته واألسرة مخول التخاذ‬
‫إذا أصر على اتخاذ القرارات لنفسه‪ ،‬يجب أن يحصل على الكشف الكامل وممارسة حقه المستقل‪.‬القرارات نيابة عنه‬
‫‪ .‬إذا اختار ترك كل شيء للعائلة‪ ،‬فإن يمكن للطبيب التعامل مع األسرة وفقاا لذلك ‪.‬للموافقة المستنيرة‬

‫‪2‬الحالة‬

‫هي وافق على إجراء عملية فتح البطن‪ .‬عا ًما ظهرت عليها عالمات كالسيكية اللتهاب الزائدة الدودية الحاد ‪30‬امرأة تبلغ من العمر‬
‫ًا لم يكن ليؤدي‪.‬وجد الجراح كيس مبيض لم يتم تشخيصه من قبل وقررت إزالته ‪.‬وإزالة الملتهب الملحق‬ ‫كانت اإلزالة إجرا ًء بسيطا وآم ن‬
‫‪.‬ماذا يجب على الجراح فعل؟ قدم تفكيرك األخالقي ‪.‬الممرضة الرئيسية رفض ألن المريض لم يوافق ‪.‬إلى زيادة مدة العملية‬

‫في هذه‪ .‬ال ينبغي للطبيب أن يمضي قدما في إزالة الكيس بسبب ذلك سيكون خارج نطاق الموافقة المستنيرة التي تم الحصول عليها )ج‬
‫‪.‬الحالة‪ ،‬ال توجد حاجة طارئة إلنقاذ األرواح للعمل دون موافقة‬
‫‪3‬الحالة‬

‫عا اما مصا باا بمرض السكر‪ ،‬وتوفي ابنه العام الماضي بسبب عملية نقل الدم من الدم غير المتطابق‪ ،‬تم ‪80‬رجل يبلغ من العمر‬
‫وأصر على عدم تنفيذ أي إجراء دون كتابة بموافقة ابنه الطبيب ‪.‬إدخاله إلى نفس المستشفى للمالحظة بعد ذلك الوقوع في المنزل‬
‫تم إزعاج الممرضات من خالل االضطرار إلى الحصول على إذن كتابي لـ ‪.‬الذي أراد أن يجلس بجانب سريره جمي اعا الوقت‬
‫رفضت الممرضات امتثل لرغباته ورفضه التعاون‪ ،‬مما أدى إلى‪.‬المراقبة الروتينية للعالمات الحيوية وحقن األنسولين‬
‫‪.‬ماذا يجب أن يفعل الطبيب المسؤول؟ قدم تفكيرك األخالقي ‪.‬مواجهة‬
‫ومع ذلك‪ ،‬إذا وجدوا أن االمتثال ال يمكن ‪ .‬يجب على األطباء احترام استقاللية المريض والتكيف معه يحتاج إلى أكبر قدر ممكن)ج‬
‫تحقيق رغبات المريض دون تعطيل عمل الجناح يمكن أن تتبع إجراءات رفض العالج من قبل المريض‪ ،‬والتي قد تؤدي‬
‫‪.‬الحقاا إلى الخروج إلى مؤسسة أخرى لديها القدرة على التعامل مع احتياجات المريض‬
‫‪4‬الحالة‬

‫جراح أعصاب شاب يعتزم إجراء عملية جراحية لمريض مصاب بالعمود الفقري القطني إصابة كانت فرصة نجاحها تتراوح بين‬
‫إذا أبلغ المريض بإمكانية إجراء العملية خاطئة وتؤدي إلى الشلل‪.‬شعر بعدم اليقين بشأن تناوله موافقة مسبقة ‪ 10 ٪.‬و‪5‬‬

‫‪PLEAZE, ANY NOTE OR CORRECTION OR ADDITIN SEND:‬‬ ‫‪[email protected]‬‬


‫نسألكم الدعاء بظهر الغيب‬
‫‪P a g e | 232‬‬
‫‪ ٪‬فرصة لمزيد من ‪95‬إذا لم يتم تنفيذ العملية كان هناك ‪ ٪ .‬أن المريض رفض العملية ‪90‬النصفي‪ ،‬كانت هناك فرصة بنسبة‬
‫ماذا يجب أن يفعل جراح األعصاب؟ قدم تفكيرك األخالقي‪.‬التدهور‪ ،‬مما يؤدي إلى الشلل النصفي بعد بضعة أشهر‬
‫الخوف من رفض العالج الضروري ليس مبر ارا‪ .‬للمريض الحق في اإلفصاح الكامل حتى لو أدى ذلك إلى الرفض عالج )ج‬
‫‪.‬لذلك انتهاك استقاللية المريض‬

‫‪5‬الحالة‬
‫الجراح قرر أن مريضه ضعيف التعليم ال يستطيع فهم المعلومات حتى ‪ .‬صفحات ‪3‬تحتوي عملية الدماغ المعقدة على ورقة كشف مخاطر من‬
‫كانت العملية ضرورية لتحرير ورم دموي وكسر عظم متفتت يضغط على ‪.‬مع أفضل الترجمات‪ ،‬وقد ترفض حتى عملية إنقاذ الحياة‬
‫ًا يؤدي إلى فقدان الوعي بسبب زيادة الضغط داخل الجمجمة‬ ‫هو أعطى معلومات بسيطة للمريض تساعده ‪.‬المخ‪ ،‬والذي سيحدث قري ب‬
‫ماذا يجب أن يفعل الجراح؟ قدم تفكيرك األخالقي‪.‬العملية على التعافي منها آثار الصدمة وان لها بعض المخاطر التي لم يذكرها‬
‫يحق للمريض اإلفصاح الكامل‪ ،‬لكن يكفي الملخص إذا كان ذلك يستثني التفاصيل الفنية ولكنه يغطي الفوائد الرئيسية‪ ،‬وخاصة)ج‬
‫‪.‬هذا له ما يبرره ألنه يحافظ احترام حق المريض في المعرفة‪.‬مخاطر العملية بلغة بسيطة‬

‫‪6‬الحالة‬
‫بينما ‪.‬تم حجز أستاذ جامعي بجلطة دماغية رفض حتى العالج المنقذ للحياة بعد شرح شامل من ابنه الذي كان جراح مخ وأعصاب‬
‫ومع ذلك‪. ،‬في المستشفى‪ ،‬بدا وكأنه نسي معلومات أساسية عن مرضه عمره واسم زوجته‪ ،‬وكان مرتب اكا بشأن يوم األسبوع‬
‫ماذا يجب أن يفعل الطبيب؟ يمنح تفكيرك‪.‬كان على اتصال هاتفي مستمر مع مختبره في الجامعة‪ ،‬مرشدا للباحثين الشباب‬
‫األخالقي‬
‫إذا وجد األستاذ‪.‬‬ ‫من الضروري إجراء اختبار رسمي للكفاءة من قبل طبيب أو طبيب نفساني هذه القضية)ج‬
‫‪ .‬فيجب احترام رفضه للعالج ‪COMPETENCE‬‬

‫‪7‬الحالة‬
‫عا ًما في غيبوبة عميقة‪ ،‬مع وجود بعض عالمات على وظيفة جذع الدماغ‪ ،‬وتم وضعه على‪30‬ضحية حادث مروري يبلغ من العمر‬
‫وكان قد أخبر زوجته قبل وقوع الحادث أنه يريد أن ترك ليموت بكرامة بدالً من العيش‪.‬أجهزة دعم الحياة في وحدة العناية المركزة‬
‫‪.‬كما أنه أذن لزوجته كتابة بذلك‪ ،‬التخاذ قرارات بشأن عالجه إذا فقد الوعي ‪.‬بمساعدة اآاللت‬
‫‪.‬ماذا يجب أن يفعل الطبيب؟ قدم تفكيرك األخالقي ‪.‬ولكن رفض والده وعائلته ذلك وأصروا على استمرار دعم الحياة حتى التعافي‬
‫‪.‬ومع ذلك‪ ،‬يمكن لألب أن يتجاوزها بناء على اعتبارات من الشريعة‪ .‬يحترم قرار الزوجة بنا ًء على رغبة المريض )ج‬

‫‪8‬الحالة‬

‫‪ ،‬على دراية بالمراحل األخيرة من مرضه‪ ،‬وقع على توجيه مسبق ‪ MS‬عا ًما ضحية لمرض التصلب العصبي المتعدد ‪40‬يبلغ من العمر‬
‫أصيب بالتهاب رئوي حادا بعد فترة وجيزة من كتابة ‪ .‬إذا توقف عن التنفس من تلقاء نفسه ‪SUPPORT LIFE‬يسمح لألطباء بعدم بدء‬
‫ماذا‪.‬كان أفراد األسرة منقسمون في آرائهم ‪.‬لم يكن األطباء متأكدين مما يجب عليهم فعله ‪.‬التوجيه‪ ،‬عانى من أمراض تنفسية حادة محنة‬
‫‪.‬يجب أن يفعل األطباء؟ قدم معنوياتك منطق‬

‫طا بفشل الجهاز التنفسي بسبب عدة التصلب المرض األساسي وال يمكن تطبيقه على فشل الجهاز التنفسي‬ ‫كان التوجيه المسبق مرتب)ج‬
‫‪.‬بسبب االلتهاب الرئوي الحاد‬

‫‪9‬الحالة‬

‫عا ًما مصاب بسرطان العظام محصور في عظمة الساق ورفض عملية البتر والتى من شأنها تمنع انتشار ‪14‬طفل يبلغ من العمر‬
‫ماذا‪.‬والده ووالدته عارض قراره وأذن للجراحين بإجراء البتر ‪.‬هو فهم العواقب السلبية لقراره ‪.‬السرطان إلى أجزاء أخرى من الجسم‬
‫يجب أن يفعل الطبيب؟ قدم تفكيرك األخالقي‬
‫‪ .‬عا ًما اتخاذ قرار برفض العالج لذلك في هذه الحالة قرار الوالدين هو الذي يؤيد‪ 14‬ال يستطيع الشاب البالغ من العمر )ج‬

‫‪PLEAZE, ANY NOTE OR CORRECTION OR ADDITIN SEND:‬‬ ‫‪[email protected]‬‬


‫نسألكم الدعاء بظهر الغيب‬
‫‪P a g e | 233‬‬
‫‪10‬الحالة‬

‫أخذوها إلى‪.‬‬ ‫اعتادت فتاة في الرابعة عشرة من عمرها على الخروج من منزلها ويخشى الوالدان أن تتعرض لالغتصاب وتحمل‬
‫ماذا يجب أن يفعل الطبيب؟ قدم تفكيرك‪.‬‬ ‫‪.‬المستشفى وطلبوا من األطباء جعلها عقيمة تحدث األطباء معها وعارضت العملية‬
‫‪.‬األخالقي بشدة‬

‫ًل التخاذ القرار ‪ 14‬الشاب البالغ من العمر )ج‬


‫نظرا لطبيعة العملية انه ال رجوع فيها بعد اجرائها ‪ ،‬يجب طلب المشورة‪ .‬عا ًما ليس مؤه ا‬
‫‪court.‬من محكمة قانونية‬

‫‪11‬الحالة‬

‫‪ .‬عا ًما إلى المستشفى في غيبوبة بسبب مرض السكري الحماض الكيتوني والقدم الغرغرينا ‪60‬تم إدخال مريض سكري يبلغ من العمر‬
‫رفض أبناء وبنات المريض العملية حتى بعد ذلك تفسيرات ‪.‬قرر األطباء بتر القدم حالما تستقر الحالة العامة بدرجة كافية لتحملها تخدير‬
‫واستنتجوا أنه من األفضل أن يموت ويدفن مع كل أجزاء جسده من العيش بطرف‪.‬أن الغرغرينا قد تنتشر وتؤدي إلى تسمم دم قاتل‬
‫‪.‬ماذا يجب أن يفعل الطبيب؟ قدم تفكيرك األخالقي ‪.‬مبتور‬

‫‪ .‬استبدال صناع القرار‪ VALID‬يتم تأييد قرار األسرة في هذه الحالة ألنها صحيحة )ج‬

‫مصطلحات‬
‫‪JUSTICE‬‬

‫مع اآلخرين"عادلين"إخبار مرضانا بالح قيقة حول ظروفهم من شأنه أن يجعلهم قرارات أكثر استنارة‪ ،‬وستساعدهم على أن يكونوا‬
‫‪ .‬إنها من المهم التأكيد على أن العدالة هنا ال تتعلق فقط بإنصافنا المرضى‪ ،‬ولكن يتضمن أيضا جعل مرضانا عادلين لآلخرين‪.‬أيضا‬
‫سنقدم واحدة مثال‪ ،‬إذا تم تضليل المر يضة بشأن حالتها‪ ،‬على سبيل المثال‪ ،‬قيل لهم أن حالتها لديها تشخيص جيد للغاية‪ ،‬بينما في الواقع‬
‫هي تتوقع أن ‪.‬معظمها تشير التجارب السابقة من ممارستنا وأدبنا إلى على العكس من ذلك‪ ،‬قد تؤخر زفاف ابنتها حتى تتحسن حالتها‬
‫في هذه الحالة‪ ،‬كانت األم غير عادلة‪.‬تتعافى في غضون أسابيع‪ ،‬ولكن إذا لم تفعل ذلك‪ ،‬فحينئ ًذ في نهاية المطاف الزواج قد ال يتم أبدا‬
‫‪.‬يمكن رؤية األمثلة األكثر تطرفًا في الممارسة العملية‪.‬البنتها عن غير قصد ألنه لم يتم إخبارها بالحقيقة‬

‫‪PETRNALISM‬‬

‫ًا بين األطباء وقد اختفى اآلن تقريبا‬


‫يفترض الطبيب األبوي أنه يعرف ما هو األفضل للمريض ويتعين عليه‪.‬هو موقف سلبي كان شائ ع‬
‫‪.‬األبوة هي انتهاك للمريض حقوق الحكم الذاتي‪.‬اتخاذ قرارات العالج دون الرجوع إلى المريض‬

‫‪Capacity/competence to consent‬‬

‫لكي تكون الموافقة المستنيرة صالحة قانو ًنا‪ ،‬يجب على المريض الذي يتخذ القرار أن يحكم على أهليته القانونية؛ بعبارة أخرى‪ ،‬أن يكون‬
‫األطفال دون سن ‪ .‬يتم الحكم على الكفاءة من خالل القدرة الفكرية على الفهم‪ ،‬االحتفاظ بالمعلومات والحكم عليها‪.‬لديك القدرة على صناعة القرار‬
‫إذا كان‪ .‬يتم الحكم على شخص بالغ عادي بأهلية قانونية ما لم يكن هناك سبب للشك بخالف ذلك‪.‬الرشد هم تعتبر غير مختصة‬
‫في الحاالت البسيطة‪. ،‬هناك مثل هذا الشك‪ ،‬أو إذا كان طبيعة المرض تؤثر على القدرة العقلية‪ ،‬اختبارات محددة للكفاءة يجب تنفيذها‬
‫يمكن للطبيب الذي يعتني بالمريض اختبار الكفاءة من خالل طرح أسئلة بسيطة حول‪ ،‬على سبيل المثال‪ ،‬اسم المريض وعنوانه واتجاهه في‬
‫في المزيد في الحاالت المعقدة‪ ،‬قد تتم دعوة طبيب نفساني ‪.‬الزمان والمكان وقدرة المريض لفهم المعلومات واالحتفاظ بها‪ ،‬وإصدار األحكام‬
‫يفضل أن يتضمن السجل‪ .‬يجب تسجيل اختبار الكفاءة بشكل واضح في مخطط المريض‪.‬سريري اللختبار الكفاءة بطريقة رسمية‬
‫‪.‬البنود تستخدم اللختبار‬

‫‪Proxy consent and substitute decision maker‬‬

‫ًا اتخاذ قرارات بشأنه عالجه أو معالجتها‬ ‫يجب العثور على صانع قرار‪.‬ال يمكن للمريض الذي يتم الحكم عليه بأنه غير مؤهل قانون ي‬
‫‪ .‬إذا كان أفراد األسرة غير متوفر‪ ،‬يمكن العثور على وكالء آخرين‪ .‬ال عادة ما يكون الوكيل أو البديل أحد أفراد األسرة‪.‬وكيل أو بديل‬
‫في بعض الحاالت‪. ،‬المشكلة التي تواجهها عادة الممارسون هو عند وجود عدة أفراد من األسرة وواحد من عليهم أن يكونوا صانع القرار‬
‫ًن أحدهم كصانع قرار‪ ،‬مما يجعل العملية غاية في األهمية سهل‬ ‫إذا لم يعين المريض صانع قرار ولكن هناك أ ‪.‬قد يكون لدى المريض ع ي‬
‫اتفاق باإلجماع بين أفراد األسرة حول من يجب أن يقرر‪ ،‬ومن السهل التعامل مع األمر مرة أخرى؛ عادة األب أو أكبر عضو من األسرة‬

‫‪PLEAZE, ANY NOTE OR CORRECTION OR ADDITIN SEND:‬‬ ‫‪[email protected]‬‬


‫نسألكم الدعاء بظهر الغيب‬
‫‪P a g e | 234‬‬
‫إذا أشار المريض في وقت ‪ .‬تنشأ المشاكل عندما يكون هناك الخالف بين أفراد األسرة واألطباء ال يعرفون لمن استمع إلى‪.‬يتحمل المسؤولية‬
‫إذا كان المريض يفعل أال يرشح أحد حسب األعراف السعودية ‪.‬القبول إلى أي فرد من أفراد األسرة يجب أن يمثله‪ ،‬ثم نتبع رغبات المريض‬
‫في بعض الحاالت‪ ،‬يتصرف األب بصفته صانع القرار حتى لو كان قام المريض بتعيين شخص‪.‬لألب الحق في ذلك يقرر‬
‫‪.‬آخر‬

‫‪( directives advance and consent Prospective‬الموافقة المحتملة والتوجيهات المسبقة(‬

‫يجوز للمريض أن يصدر توجيهات مسبقة ‪.‬كجزء من استقاللية المريض المرتقبة‪ ،‬يتم اتخاذ القرارات مسبقًا احترام فقدان الكفاءة‬
‫ال قد يكون التوجيه حول سحب أجهزة دعم الحياة في ظروف غير مجدية‪ ،‬أو عن ‪.‬عن الكيفية التي يريد أن يعامل بها في حالة فقد األهلية‬
‫األطباء‪.‬أفراد األسرة مرتاحون من الصعب والمرهقة واجب اتخاذ القرار ‪.‬التوجيه المسبق هو راحة للجميع متورط ‪.‬أي إجراء طبي آخر‬
‫ض الطعن في التوجيه المسبق إذا كانت ظروف‪.‬‬ ‫مرميكن‬
‫أيضا مرتاحون ألنهم يعرفون أن لديهم موافقة مسبقة على عالجهم اللل ي‬
‫المريض لديها تغيرت بشكل جوهري منذ أن أصدروا التوجيه‬
‫‪Consent for children‬‬

‫إذا رفض كال الوالدين‪ .،‬سنة في المملكة العربية السعودية‪18‬للوالدين الحق العام في اتخاذ القرار بشأن األطفال دون سن الرشد‪ ،‬وهي‬
‫‪.‬يمكن للطبيب الذهاب إلى األمام وتقديم العالج المنقذ للحياة في حاالت الطوارئ دون موافقة في حالو إنقاذ الحياة‬
‫‪.‬إذا اختلف الوالدان‪ ،‬يمكن للطبيب البدء فى العالج المنقذ للحياة بنا ًء على موافقة أحد الوالدين‬
‫غير العاجل من الطبيب تعتبر ضرورية إلنقاذ األرواح يمكن حلها بالرجوع إلى القانون‬ ‫رفض أحد الوالدين أو كليهما العالج‬
‫‪COURT.‬المحاكم‬

‫ً ًاس وهي سبع سنوات‪ ،‬ليس لها رأي على اإلطالق في القرارات المتعلقة بالعالج؛ كل شيء (األطفال دون سن التمييز‪ ،‬سن التميز‬ ‫ي‬
‫من ‪ .‬األطفال فوق سن السابعة ولكن تحت سن البلوغ‪ ،‬لديهم قدرة فكرية متزايدة على الفهم والمشاركة في صنع القرار‪.‬في أيدي الوالدين‬
‫األطفال فوق سن البلوغ ‪.‬الحكمة االستماع إليهم والتفكير آرائهم مع العلم الكامل بأن الوالدين هم من سيوافقون؛ ال يمكن لألطفال الموافقة فقط‬
‫ومع ذلك‪ ،‬منذ‪.‬هم في الواقع بالغون وما لم يكن هناك دليل على عدم كفاءتهم‪ ،‬ينبغي السماح لهم بذلك اتخاذ قرارات بشأن عالجهم‬
‫عا ًما‪ ،‬يجب على اآلباء الموافقة على هذه القرارات قبل تنفيذها ‪18‬‬ ‫سن الرشد‬
‫‪.‬‬
‫‪Consent for the mentally impaired‬‬

‫البعض لديه المجموع ضعف القدرة الفكرية‪ ،‬وعدم القدرة على الفهم ‪.‬المرضى الذين يعانون من مرض عقلي لديهم درجات متفاوتة من القدرات‬
‫لذلك لدينا طيف من الضعف ‪ .‬البعض لديه انتقائية وقادرة على اتخاذ بعض القرارات‪.‬واالحتفاظ‪ ،‬تحليل حالة المرض وإصدار أحكام بشأنها‬
‫يتم ‪ -‬وحيث يوجد خطر ل المريض والجمهور ‪ -‬في حاالت الفقدان التام للقدرة العقلية ‪.‬الكلي إلى الضعف الجزئي أو االنتقائي االهلية‬
‫ًا وفقًا لما تنص عليه تشريعات الصحة النفسية يمنح األطباء الحق في اتخاذ القرارات‬ ‫قبول المرضى وتقييدهم ومعالجتهم ال إراد ي‬
‫اإلجراءات المتخذة يجب أن يكون من قبل األطباء ‪.‬ضمن الضمانات التي تحددها القانون‪ ،‬والذي يتضمن مراجعات منتظمة من قبل المحاكم‬
‫بعض المرضى الذين يعانون من مشاكل نفسية يدخلون العالج طواعية أو يتم ‪ .‬في مصلحة حماية رفاهية المريض ورفاهية الجمهور‬
‫في الحالة األخيرة‪ ،‬يجب السماح ألولئك الذين يعانون من ضعف انتقائي أو جزئي للقيام‪.‬إحضارهم من قبل عائالتهم‬
‫‪.‬ببعض القرارات ذات الصلة‪ ،‬بينما يمكن ألفراد األسرة اتخاذ قرارات لهم مع فقدان كامل للقدرة الفكرية‬

‫‪Consent for the unconscious‬‬

‫إذا كان أقرب األقارب غير متوفر‪.،‬‬ ‫يعتبر المرضى الذين يعانون من فقدان كامل للوعي غير أكفاء‪ ،‬ويتخذ األقارب قرارات العالج‬
‫إذا ترك المريض توجي ًها مسبقًا‪ ،‬فسيتم ذلك يجب احترامه ما ‪.‬سيقوم الطبيب بإجراء عالج عاجل يتم النظر فيه في مصلحة المريض‬
‫أي يجب أن تكون القرارات التي يتخذها ‪.‬لم يكن غير ذي صلة بالظروف السريرية الفعلية التي لم يكن المريض يتوقع وقت إصدار التوجيه‬
‫ًا بشكل عاجل ‪.‬صانع القرار بالوكالة أو الطبيب تم تأكيده أو عكسه عندما يستعيد المريض وعيه‬
‫إذا كان العالج في االعتبار ليس مطلو ب‬
‫إلنقاذ األرواح‪ ،‬يجب أن يكون األمر كذلك تمت إحالته إلى محكمة قانونية التخاذ قرار إذا كان اإلجراء كبي ًرا‪ ،‬مثل البتر أو الزرع‬
‫إذا‪.‬‬
‫‪.‬كان اإلجراء بسيطا‪ ،‬فقم بتعيين مسؤول في المستشفى يمكن أن يكون صانع القرار‬

‫‪PLEAZE, ANY NOTE OR CORRECTION OR ADDITIN SEND:‬‬ ‫‪[email protected]‬‬


‫نسألكم الدعاء بظهر الغيب‬
‫‪P a g e | 235‬‬

‫‪MODULE 6 - MEDICAL MALPRACTICE AND MEDICAL ERRORS‬‬


‫‪1‬الحالة‬

‫تم استدعاء معلمه من‪.‬طلب منه إجراء عملية لم يقم بها من قبل ‪ .‬عا ًما كان في سنته األخيرة في كلية الطب ‪23‬طالب طب يبلغ من العمر‬
‫‪.‬غرفة العمليات لسبب طارئ وارتكب الطالب الشاب خطأ أدى إلى تعقيد أدى إلى فقدان المريضة لحياتها‬

‫الطالب الذي أخطأ‬

‫أال يترك المتقدم الطالب بمفرده وأن يتركه أخبره أن ينتظر عودته‬ ‫‪‬‬
‫‪senior‬يجب عليه اتصلوا بكبار آخر‪.‬أال يكون الطالب قد أكمل بنفسه‬ ‫‪‬‬
‫يجب أن نثقف طالبنا لمعرفة حدودهم‪ ،‬في كليهما المعرفة والمهارات‬ ‫‪‬‬
‫يجب أن نكشف عن أي حالة على الفور للسلطات العليا تعويضات‬ ‫‪‬‬
‫‪.‬يجب إبالغ أسرة المريض فو ًرا ووفقًا يجب تعويضهم بموجب القانون‬ ‫‪‬‬
‫‪2‬الحالة‬

‫‪ .‬عا ًما لجلسة عالج كيماوي عنيفة واستئصال الرحم بعد تشخيص إصابتها بنوع نادر من السرطان ‪30‬خضعت مريضة تبلغ من العمر‬
‫‪.‬في وقت الحق‪ ،‬أخبرها طبيب األورام أنها شخصت بالخطأ‬

‫المرأة التي أجريت لها عملية استئصال الرحم خطأ وتم تشخيصه بشكل خاطئ‬

‫يجب أن ندعم حق المريض في التعويض واعتذار‬ ‫‪‬‬


‫يجب أن نشرح بشكل شامل ما حدث‬ ‫‪‬‬
‫إعادة مراجعة النتيجة مرا ًرا وتكرا ًرا قبل أن يأخذوها أي عمل)الطبيب والمختبر األخصائي(وجود مشكلة في التشخيص لذلك‬ ‫‪‬‬
‫مع المريض‬ ‫‪‬‬
‫‪.‬حسب القانون‪ ،‬يجب سؤال الطبيب عن ذلك قضية‬ ‫‪‬‬
‫‪MODULE 17 – ORGAN TRANSPLANT AND DONATION‬‬ ‫‪‬‬
‫‪1‬الحالة‬

‫على الرغم من أن وقت التعافي الالزم ‪ .‬عا ًما التبرع بكليتها لزميل في العمل تعرفه عن بعد‪53‬عرضت أم عزباء تبلغ من العمر‬
‫‪.‬بعي ًدا عن العمل بعد التبرع سيؤدي إلى إجهاد دخلها المتواضع‪ ،‬تخبر المرأة فريق الزرع أنها تتفهم ذلك وأنها مستعدة للمضي قد ًما‬
‫توضح أن‬
‫‪.‬دافعها للتبرع هو مجرد مساعدة إنسان آخر‬
‫قد يكون من المفيد أن يكون لديك سياسات وحدة الزرع على المعيشة التبرعات من غير األقارب‪ ،‬خاصة وأنهم يزدادون بشكل متزايد‬
‫‪.‬مشترك‬

‫في الحالة األولى‪ ،‬يجب أن توازن وحدات الزراعة بين الحاجة إلى ذلك استكشاف الدوافع الخفية‪ ،‬مثل المدفوعات السرية‪ ،‬مع الحاجة إلى االستجابة‬
‫ويفضل أن يكون ذلك من قبل خبير مستقل في التبرع باألعضاء الحية تدار‪ ،‬ولكن ‪ .‬تقييم نفسي اجتماعي ‪.‬لها متبرع بدافع اإليثار الحقيقي‬
‫باإلضافة‪ ،‬ال تستطيع وحدات الزرع التحكم في تبادل المكافآت المادية أو غيرها األحداث التي قد تحدث بعد ‪.‬من الصعب تحديد دوافع خفية‬
‫يجب أن يكون مركز الزرع قد خطط مسبقًا للتأكد من ‪ .‬هذه هي أيضا من األمور ذات االهتمام في تبرعات األقارب ‪.‬اكتمال عملية الزرع‬
‫كان المستلم‪.‬المتبرع حصلت على دعم كاف‪ ،‬بما في ذلك الدعم المالي المشروع‪ ،‬خالل فترة عملها الشفاء من العملية‬
‫ًا حقًا‬‫عا إيثار ي‬‫كان المتبرع على علم تام بمخاطر وفوائد هذه العملية ‪.‬‬ ‫زميل عمل بعيد‪ ،‬مما جعل هذا‪ ،‬في غياب أي إكراه‪ ،‬تبر‬
‫وفهمها‬
‫استنتج فريق الزرع ذلك كانت المرأة راغبة وواعية وذات دوافع إيثارية وترغب في ذلك للتبرع ألسباب منطقية‪.‬تبرعها وعملية التبرع‬
‫‪.‬كانت مهمة بالنسبة لها‪ ،‬وكانت لذلك‪ ،‬مانح مناسب‬

‫‪PLEAZE, ANY NOTE OR CORRECTION OR ADDITIN SEND:‬‬ ‫‪[email protected]‬‬


‫نسألكم الدعاء بظهر الغيب‬
‫‪P a g e | 236‬‬
‫‪2‬الحالة‬

‫عا ًما تعرض لحادث مروري خطير إلصابات خطيرة وتم وضعه على أجهزة اإلنعاش أثناء اكتمال ‪50‬تعرض رجل يبلغ من العمر‬
‫يفكر الطبيب المبتدئ الذي يعالج المريض في سحب‪ .‬أقاربه غير موجودين في المستشفى‪ .‬تشير النتائج إلى أنه لن ينجو‪.‬التحقيقات‬
‫‪ .‬يتساءل عما إذا كان المريض سيكون مرشحا للتبرع بقلب جثث بعد إعالن الموت القلبي‪.‬العالج الداعم‬

‫ًا في المؤسسات ذات‬ ‫إذا كان هناك إجماع على ضرورة إجراء عملية زرع جثة في الحالة الثانية الموصوفة‪ ،‬فيجب إجراؤها حصر ي‬
‫يقسم تصنيف ماستريخت أنواع التبرع الخمسة إلى مجموعات غير خاضعة للرقابة وخاضعة للرقابة‪. ،‬البروتوكوالت المحددة بوضوح‬
‫ًا أم ال‬
‫يجب اتخاذ العديد من القرارات في ظل ظروف الطوارئ عندما يتم إحضار ‪.‬اعتما ًدا على ما إذا كان الموت القلبي متوق ع‬
‫خاضع (‪ ،‬أو في الحاالت القصوى دون أمل في البقاء على قيد الحياة )غير المنضبط(المريض إما ميتًا‪ ،‬وفقًا لمعايير القلب والرئة‬
‫في)‪.‬للسيطرة‬
‫‪ ،‬إذا لم يكن أقارب المريض موجودين‪ ،‬فهل يجب المضي قد ًما في )كما هو الحال مع هذا المريض(المجموعة الخاضعة للرقابة‬
‫التحضير الستخراج األعضاء أثناء البحث عنهم؟ عادة ما يأخذ هذا شكل تبريد األعضاء وقد يشمل إدارة األدوية لحماية األعضاء‪ ،‬ولن‬
‫ًا ‪.‬يفيد أي منهما المريض المصاب والمحتضر‬ ‫إذا تعذر االتصال باألقارب‪ ،‬فهل ينبغي المضي قدما في استرجاع األعضاء؟ يتم حال ي‬
‫بافتراض وجود األقارب‪ ،‬توجد اختالفات طفيفة في إجراءات الموافقة ‪.‬استكشاف هذه القضايا الملحة باهتمام كبير في العديد من المراكز‬
‫المستخدمة‬
‫في الواقع‪ ،‬بدأت )‪.‬أي الموت الدماغي(حاليًا في الممارسة القياسية‪ ،‬حيث علن عن وفاة المتبرع المحتمل وفقًا للمعايير العصبية‬
‫زراعة‬
‫ً‬
‫‪.‬األعضاء المتوفين في األصل بالتبرع بعد الموت القلبي‪ ،‬وليس بتطبيق معايير الموت الدماغي‪ ،‬التي دخلت ممارسة الزراعة الحق ا‬
‫إذا كان القلب ال يزال ينبض‪ ،‬يبرز سؤال آخر‪ ،‬بنا ًء على التوتر بين ‪.‬ال تزال معايير الموت الدماغي مثيرة للجدل في بعض األماكن‬
‫بعد سحب أجهزة دعم الحياة‪ ،‬كم ‪:‬الرغبة في تأكيد الموت بشكل مطلق والرغبة في الحصول على أعضاء لم تتضرر بسبب نقص التروية‬
‫من‬
‫الوقت يجب أن ينتظر الجراح بعد ذلك؟ توقف القلب قبل إزالة األعضاء؟ في هذه الحالة‪ ،‬تم العثور على األقارب بسرعة ووافقوا أوالً‬
‫وشرع األطباء في تبريد الكلى عن طريق قسطرة في البطن‪ ،‬لكنهم اختاروا‪.‬على سحب أجهزة اإلنعاش وبعد ذلك على التبرع بالكلى فقط‬
‫‪)3 ،‬ماستريخت نوع ( تم نقل المريض إلى غرفة العمليات‪ ،‬وسحب أجهزة اإلنعاش ‪.‬عدم إعطاء أي أدوية للمساعدة في الحفاظ عليها‬
‫وانتظر الجراح عشر دقائق كاملة قبل استئصال الكليتين‪ ،‬والتي تم تقديمها الثنين من المتلقين مع علمهما الكامل أن الكلى جاءت من‬
‫عملت إحدى الكليتين على الفور‪ ،‬بينما أصيبت األخرى بأضرار إقفاريه خفيفة‪ ،‬لكنها بدأت تعمل بشكل‪ .‬متبرع ينبض القلب‪-‬شخص غير‬
‫‪ .‬كال المستفيدين على قيد الحياة مع الكلى تعمل بعد أربع سنوات‪.‬جيد بعد ثالثة أيام‬
‫القواعد االسالمية للتبرع‬
‫قرا ًرا)جزء من رابطة العالم اإلسالمي(نظ ًرا للتطور السريع لسوق األعضاء‪ ،‬أصدر علماء اإلسالم من مجلس مجمع الفقه اإلسالمي‬
‫‪1986‬بشأن الوفاة وزرع األعضاء في المؤتمر الدولي الثالث لفقهاء المسلمين المنعقد في عمان‪ ،‬األردن عام‬

‫ًا‪ ،‬ويمكن تطبيق جميع مبادئ الشريعة اإلسالمية‪ ،‬عند ثبوت إحدى‪ " :‬نص هذا القرار على ما يلي‪‬‬ ‫يعتبر الشخص ميتًا قانو ن‬
‫العالمات التالية‬
‫توقف القلب والتنفس نهائيا ً‪ ،‬ويقرر األطباء أنه ال رجوع فيه‪‬‬
‫لقد أمنت هذه المبادئ‪ .‬التوقف التام لجميع وظائف الدماغ الحيوية‪ ،‬يقرر األطباء أنه ال رجوع فيه‪ ،‬وبدأ الدماغ في التدهور ‪‬‬
‫التوجيهية طريقة للسماح بالتبرع من موت جذع الدماغ والمتبرعين بالقلب الجثث‪ ،‬وتم وضع متطلبات موافقة مختلفة في جميع‬
‫‪.‬أنحاء العالم اإلسالمي‬
‫‪:‬واألهم من ذلك أن علماء اإلسالم اتفقوا على جواز التبرع بشروط‬

‫بالتأكيد سيساعد المتلقي‪‬‬


‫ال يضر المتبرع‪‬‬
‫تبرع المتبرع بالعضو أو النسيج طواعية وبدون حافز مالي أو تعويض‪‬‬ ‫‪.‬‬

‫‪PLEAZE, ANY NOTE OR CORRECTION OR ADDITIN SEND:‬‬ ‫‪[email protected]‬‬


‫نسألكم الدعاء بظهر الغيب‬
P a g e | 237

PLEAZE, ANY NOTE OR CORRECTION OR ADDITIN SEND: [email protected]


‫نسألكم الدعاء بظهر الغيب‬
P a g e | 238

PLEAZE, ANY NOTE OR CORRECTION OR ADDITIN SEND: [email protected]


‫نسألكم الدعاء بظهر الغيب‬
P a g e | 239

PLEAZE, ANY NOTE OR CORRECTION OR ADDITIN SEND: [email protected]


‫نسألكم الدعاء بظهر الغيب‬
P a g e | 240

PLEAZE, ANY NOTE OR CORRECTION OR ADDITIN SEND: [email protected]


‫نسألكم الدعاء بظهر الغيب‬
P a g e | 241

PLEAZE, ANY NOTE OR CORRECTION OR ADDITIN SEND: [email protected]


‫نسألكم الدعاء بظهر الغيب‬
P a g e | 242

PLEAZE, ANY NOTE OR CORRECTION OR ADDITIN SEND: [email protected]


‫نسألكم الدعاء بظهر الغيب‬
P a g e | 243

PLEAZE, ANY NOTE OR CORRECTION OR ADDITIN SEND: [email protected]


‫نسألكم الدعاء بظهر الغيب‬
P a g e | 244

PLEAZE, ANY NOTE OR CORRECTION OR ADDITIN SEND: [email protected]


‫نسألكم الدعاء بظهر الغيب‬
P a g e | 245

PLEAZE, ANY NOTE OR CORRECTION OR ADDITIN SEND: [email protected]


‫نسألكم الدعاء بظهر الغيب‬
‫‪P a g e | 246‬‬

‫نسألكم الدعاء مراجعة‬


‫وتصحيح‬

‫د‪/‬محمد إبراهيم‬
‫د‪/‬محمد عيد‬

‫‪PLEAZE, ANY NOTE OR CORRECTION OR ADDITIN SEND:‬‬ ‫‪[email protected]‬‬


‫نسألكم الدعاء بظهر الغيب‬

You might also like